Vous êtes sur la page 1sur 428





Chairperson, CBSE
Contents
S NO TOPIC PAGE
NUMBER
1. Ellipse - I 1
Practice Questions 3
2. Ellipse - II 5
Practice Questions 9
3. Ellipse - III 11
Practice Questions 15
4. Ellipse - IV 17
Practice Questions 20
5. Ellipse - V 24
Practice Questions 28
6. Hyperbola - I 30
Practice Questions 34
7. Hyperbola - II 35
Practice Questions 38
8. Hyperbola - III 40
Practice Questions 44
9. Hyperbola - IV 45
Practice Questions 48
10. Hyperbola - V 50
Practice Questions 55
11. Hyperbola - VI 57
Practice Questions 61
12. Hyperbola - VII 63
Practice Questions 68
13. Hyperbola - VIII 70
Practice Questions 73
14. Complex Number - I 75
Practice Questions 78
15. Complex Number - II 80
Practice Questions 85
16. Complex Number - III 87
17. Complex Number - IV 90
Practice Questions 96
18. Complex Number - V 98
Practice Questions 104
19. Complex Number - VI 106
Practice Questions 112
20. Complex Number - VII 114
Practice Questions 122
21. Complex Number And Quadratic Equations - I 125
Practice Questions 131
22. Complex Number And Quadratic Equations - II 133
Practice Questions 135
23. Complex Number And Quadratic Equations - III 139
Practice Questions 141
24. Complex Number And Quadratic Equations - IV 144
Practice Questions 147
25. Sequences And Series - I 149
Practice Questions 154
26 Sequences And Series - II 156
Practice Questions 161
27. Sequences And Series - III 164
Practice Questions 169
28. Sequences And Series - IV 171
Practice Questions 176
29. Sequences And Series - V 179
Practice Questions 188
30. Sequences And Series - VI 191
Practice Questions 198
31. Sequences And Series - VII 200
Practice Questions 206
32. Sequences And Series - VIII 209
Practice Questions 215
33. Permutation And Combinations - I 217
Practice Questions 220
34. Permutation And Combinations -II 222
Practice Questions 225
35. Permutation And Combinations - III 227
Practice Questions 230
36. Permutation And Combinations - IV 233
Practice Questions 235
37. Binomial Theorem - I 237
Practice Questions 245
38. Binomial Theorem - II 247
Practice Questions 254
39. Binomial Theorem - III 256
Practice Questions 260
40. Binomial Theorem - IV 262
Practice Questions 266
41. Trigonometry - I 268
Practice Questions 276
42. Trigonometry - II 282
Practice Questions 286
43. Trigonometry - III 289
Practice Questions 296
44. Trigonometry Equations - I 300
Trigonometry Equations - II 308
45. Practice Questions 313
Trigonometry Equation - III 316
46. Practice Questions 323
Trigonometry Equations - IV 326
Practice Questions 329
47. Trigonometry Triangles - I 332
Practice Questions 340
48. Trigonometry Triangles - II 343
Practice Questions 347
49. Trigonometry Triangles - III 351
Practice Questions 361
50. Trigonometry Functions - I 364
Practice Questions 376
51. Trigonometry Functions - II 379
Practice Questions 384
52. Trigonometry Functions - III 386
Practice Questions 398
53. MathematicalReasoning 401
54 Statistics 409
Practice Questions 416
ELLIPSE - I

Topics covered
1. Definition of an Ellipse
2. Standard Equation of an ellipse
3 Basic terminology of ellipse
4 Comparison of standard equation of an ellipse when a>b and b<a.
P
1 Definition of an Ellipse
An ellipse is the set of all points in a plane, the sum of whose
distances from two fixed points in the plane is a constant. Fixed
points are called focus. F2 F1
PF1+PF2 = constant
2 Standard equation of an ellipse.
x2 y2
+ = 1 with a>b.
a2 b2
Y
Where b2=a2(1–e2) N M
(i) The line containing the two fixed points (foci)
is called focal axis (majoraxis) and points of
intersection of the curve with focal axis are L 1 B L
D
called the vertices of the ellipse ie. A(a,0) and
Ac (–a,0). F 2 F A
X
Ac 1

The distance between F1 and F2 is called the ( a ,0) E


(–ae,0) (ae, 0)
(a, 0)
focal length. F1F2 = 2ae. c Bc Lc x
a
L e
The distance between the vertices is A Ac = 1

2a is called major axis. Nc Mc


The distance BB1 = 2b is called minor axis
(ii) Point of intersection of the major and minor axis is called the centre of the ellipse. Any
chord of the ellipse passing through it gets bisected by it and is called diameter.
(iii) Any chord through focus is called a focal chord and any chord perpendicular to the focal
axis is called double ordinate DE.
(iv) A particular double ordinate through focus and perpendicular to focal axis is called its
2b 2
latus rectum (L L c ). Length of latus rectum =
a
a
(v) M M c and N Nc are two directrices of the ellipse and their equations are x = and x
e
a
=– respectively
e
(vi) A chord of the ellipse passing through its focus is called a focal chord.
(vii) An ellipse is the locus of point which moves in a plane such that the ratio of its distance
from a fixed point (focus) to the fixed line (directrix) is less than 1. This ratio is called
eccentricity and is denoted by e. For an ellipse e<1.

1
(viii) The equation to the ellipse whose focus is (h.k) and directrix is A x+my+n = 0 and whose
2
§ Ax  my  n ·
eccentricity e<1, is (x–h)2 +(y–k)2 = e2 ¨¨ 2 2 ¸
¸ .
© A m ¹
(ix) Special form If the centre of the ellipse is at point (h, k) and the directions of the axis are
(x – h) 2 (y – k) 2
parallel to the co-ordinate axes, then its equation is + =1
a2 b2
If we shift the origin at (h,k) rotating totaling co-ordinate axes then equation of the
X2 Y2
ellipse with respect to new origin becomes + =1
a2 b2
EXAMPLES
1 The eccentricity of the ellipse 9x +5y2 – 30y = 0 is equal to
2

(a) 1/3 (b) 2/3 (c) 3/4 (d) None of these.


Solution : Here equation of ellipse is
9x2+5y2–30y = 0
9x2+5(y2–6y+9–9) = 0
9x2+5(y–3)2– 45 = 0
9x2+5(y –3)2 = 45
x2 ( y – 3) 2
+ =1
5 9
9>5 ? b>a
a= 5 b=3

a2
Hence e = 1 –
b2

5
= 1–
9

4
=
9
= 2/3
2 P is any point on the ellipse 81x2+144y2=1944 whose foci are S and Sc . Then SP+ Sc P equals.
(a) 4 6 (b) 3 6 (c) 36 (d) 324
Solution : Here equation of ellipse is
81x2+144y2 =1944

x2 y2
1944 + 1944 = 1
81 144

2
y2
x2
+ 27 = 1
24 2
27
24>
2
? a>b a = 2 6
SP+ Sc P = 2a =4 6

PRACTICE QUESTIONS
1 The eccentricity of ellipse if length of latus rectum is one-third of major axis
4
2 5 §3·
(a) 2/3 (b) 3 (c) (d) ¨ ¸
6 ©4¹

2 The curve represented by x = 3(cost+sint) y = 4(cost–sint) is


(a) Ellipse (b) Parabola (c) Hyperbola (d) Circle

3 The foci of the ellipse 25(x+1)2+9(y+2)2 = 225 are at


(a) (–1,2)&(–1,6) (b) (–2,1)&(–2,6)
(c) (–1,–2)&(–2,–1) (d) (–1,–2)&(–1,–6)
x2 y2
4 The equation + +1 = 0 represents an ellipse, if
2–O O–5
(a) O <5 (b) O <2 (c) 2< O <5 (d) O <2or O >5

5 The sum of the focal distances of any point on the ellipse 9x2+16y2 = 144 is
(a) 32 (b) 18 (c) 16 (d) 8

6 The latus rectum of the conic 3x2+4y2–6x+8y–5 = 0


3 2
(a) 3 (b) (c) (d) None of these
2 3

( x  y – 2) 2 ( x – y) 2
7 The centre of the ellipse + = 1 is
9 16
(a) (0,0) (b) (1,1) (c) (1,0) (d) (0,1)

8 In an ellipse the distance between its foci is 6 and its minor axis 8. Then its eccentricity is
1
(a) 4/5 (b) (c) 3/5 (d) 1/2
52

3
9 For the ellipse x2+4y2 = 9
1 3
(a) The eccentricity is (b) The latus rectum is
2 2
–2
(c)
a focus is 3 3 ,0 (d) a directrix is x
3

10 The difference between the lengths of the major axis and the latus rectum of an ellipse is
(a) ae (b) 2ae (c) ae2 (d) 2ae2

Answers
1. b 2. a 3. a 4. c 5. d
6. a 7. b 8. c 9. b 10. d

4
ELLIPSE - II

x2 y2
1 The equation + = 1 represents an ellipse if
10 – a 4 – a
(a) a<4 (b) a>4 (c) 4<a<10 (d) a>10
Solution : Here equation of ellipse is
x2 y2
+ =1
10 – a 4–a
a2 = 10–a and b2 = 4–a
? 10–a>0 and 4–a>0
10>a and 4>a
? a<4.
x2 y2
2 The radius of the circle passing through the foci of the ellipse + = 1 and having its
16 9
centre (0,3) is
(a) 4 (b) 3 (c) 12 (d) 7/2
Solution ;
x2 y2
+ =1
16 9
a2 = 16 b2 = 9
a = 4 b = 3 a>b

b2
?e = 1–
a2

9
= 1–
16

7
=
16

7
=
4

Foci = ( r ae,0) ? foci is r 7 ,0
Radius of the circle through foci & centre (0,3) is ( 7 ) 2  32 = 7  9 = 16 = 4.

1
3 The equation of the ellipse whose focus is (1,–1) directrix x–y–3 = 0 and eccentricity is
2
(a) 7x2+2xy+7y2–10x+10y+7=0 (b) 7x2+2xy+7y2+7=0
(c) 7x2+2xy+7y2+10x–10y–7=0 (d) None of these

5
Solution : Let P(x,y) be any point on the ellipse
Then by definition SP = ePM.

2 2
1 x – y–3
( x – 1)  ( y  1) =
2 2
1 2 2
(x–1)2+(y+1)2 = (x +y +9–2xy+6y–6x)
8
8x2–16x+8+8y2+16y+8 = x2+y2–2xy+6y–6x+9
7x2+7y2+2xy–10x+10y+7 = 0

4 The equation (5x–1)2+(5y–2)2 = ( O 2–2 O +1) (3x+4y–1)2 represents an ellipse if O  .


(a) (0,1) (b) (0,2) –{1} (c) (1,2) (d) (–1,0)
Solution :
(5x–1)2+(5y–2)2 = ( O 2– O +1)(3x+4y–1)2
2 2
§ 1· § 2·
25 ¨ x – ¸ + 25 ¨ y – ¸ = ( O –1)2 (3x+4y–1)2
© 5¹ © 5¹
2 2
§ 1· § 2· (3x  4 y – 1) 2
¨ x – ¸ + ¨ y – ¸ = ( O –1)2
© 5¹ © 5¹ 25
Sp = e (PM)
2 2 2

? e = | O –1|
In ellipse 0<e<1
? 0<| O –1|<1
0< O <2 –{1}
? O  (0,2) –{1}
x2 y2
5 The eccentricity of an ellipse + = 1 whose latus rectum is half of its major axis is
a2 b2
1 2 3
(a) (b) (c) (d) None of these
2 3 2
Solution:
x2 y2
+ 2 = 1 a>b
a2 b
major axis = 2a
2b 2
latus rectum is = =a
a

6
2b 2
According to question ; =a
a
2b2 = a2

b2
eccentricity e = 1–
a2

1
= 1–
2
1
=
2

6 If (5,12) and (24,7) are the foci of an ellipse passing through the origin, then the eccentricity of the
conic is
386 386 386 386
(a) (b) (c) (d)
12 13 25 38
Solution : P(0,0)
S(5,12), Sc (24,7)
SP = 5 2  12 2 = 13 S Sc
(5,12) (24,7)

Sc P = 24 2  7 2 = 25
S Sc = 2ae = 19 2  5 2
= 361  25
= 386
386
ae =
2
SP+ Sc P = 2a = 13+25
2a= 38
a = 19
SSc 2ae
e= =
SP  ScP 2a
386
=
38

x2 y2
7 Locus of the point which divides double ordinate of the ellipse + = 1 in the ratio 1:2
a2 b2
internally is

7
x2 9y 2 x2 9y 2 1 9x 2 9y 2
(a) + = 1 (b) + (c) = + 2 = 1 (d) None of these.
a 2 b2 a 2 b2 9a2 b
Solution :
Let P(h,k) be a point divides double ordinate in the ratio 1:2 internally
Y

B (h1,y1)

P(h, k)
X
O h2

Let coordinates of ends of double ordinate (h,y1) and (h,–y1). (h1, –y1)
– y1  2 y1 y1
? By section formula k = =
3 3
y1 = 3k
Now the point (h,y1) = (h,3k) lies on the ellipse
h2 9k 2 x2 9y 2
? + = 1 or + =1 (' (h,k) are arbitrary)
a2 b2 a2 b2

8 If C is the centre of the ellipse 9x2+16y2 = 144 and S is one focus. The ratio of CS to semi major
axis is
(a) 7 :16 (b) 7:4 (c) 5: 7 (d) None of these
Solution : Here equation of ellipse is
9x2+16y2 = 144
x2 y2
+ =1
16 9
16>9 ? a>b

b2
e = 1–
a2

9
= 1–
16

7
=
16

7
e =
4

8
Foci is ( r ae,0) = ( r 7 ,0)
2a
Semi major axis is = =a=4
2
CS = 7 and semi major axis is 4.
? Required ratio is 7 :4.

PRACTICE QUESTIONS
x 2 y2
1 The equation of the circle drawn with the two foci of 2 + 2 = 1 as the end points of a
a b
diameter is
(a) x2+y2=a2+b2 (b) x2+y2 = a2 (c) x2+y2=2a2 (d) x2+y2 = a2–b2
x2 y2
2 The radius of the circle passing through the foci of the ellipse + = 1 and having its
16 7
centre (0,3) is
7
(a) 3 2 (b) 3 (c) 12 (d)
2
x2 y2
3 + 2 = 1 will represents the ellipse, if r lies in the interval
r2 – r – 6 r – 6r  5
(a) (– f ,2) (b) (3, f ) (c) (5, f ) (d) (1, f )

4 The semi latus rectum of an ellipse is


(a) The AM of the segments of its focal chord.
(b) The GM of the segments of its focal chord
(c) The HM of the segments of its focal chord
(d) None of these

5 The following equation represents an ellipse 25(x2–6x+9) + 16y2 = 400. How should the axes be
x2 y2
transformed so that the ellipse is represented by the equation + = 1______
16 25
x2 y2
6 Let P be a variable point on the ellipse + = 1 with foci S1 and S2. It A be area of the
16 25
triangle PS1S2 then the maximum value of A is __________

7 In an ellipse, if the lines joining a focus to the extremities of the minor axis make an equilateral
triangle with the minor axis, the eccentricity of the ellipse is
(a) 3/4 (b) 3/2 (c) 1/2 (d) 2/3

9
8 Column Matching :
x2 y2
For the ellipse + =1
5 4

Column I Column II
1 x=0 a a directrix
2 y=0 b a latus rectum
3 x=1 c minor axis
4 x=5 d major axis

9 The centre of the ellipse 14x2–4xy+11y2–44x–58y+71 = 0 is _______

Answer
1. d 2. a 3. c 4. c 5. (3,0) 6. 12
7. b 8. (1) c (2) d (3) b (4) a 9. (3,1)

10
ELLIPSE -III

Topics covered
1. Auxilliary circle
2. Eccentric angle
3 Equation of chord
4 Position of a point with respect to an ellipse.

Auxiliary Circle
The circle described on the major axis of an ellipse as diameter is called an auxiliary circle of
the ellipse
x2 y2
If 2 + 2 = 1 is an ellipse then its auxiliary circle is x +y =a
2 2 2
a b
Eccentric angle of a point
x2 y2
Let P be any point on the ellipse 2 + 2 = 1
a b
Y
Draw PM perpendicular to major axis from P and
produce MP to meet the auxiliary circle at Q. Join CQ.
‘ QCA = T is called eccentric angle of point P Q Auxiliary
x2+y2=a2 circle
Note that the angle ACP is not eccentric angle. B P
i.e. eccentric angle of P on an ellipse is the angle which P(x, y)

the radius through the corresponding point on, the Xc X


Ac OC M A
auxiliary circle makes with the major axis
? Q (acos T , asin T ) ellipse
Bc
? x-coordinate of P is acos T x 2 y2
 1
a2 b2
a 2 cos 2 T y2 Yc
+ =1
a2 b2
y2
= 1– cos2 T
b2
y2 = b2 sin2 T
y = b sin T
? Coordinate of P is (acos T , bsin T )
i.e. x = acos T and y = bsin T is the parameter equations of the ellipse.
(acos T ,bsin T ) is also called the point ‘ T ’

Equation of the chord

11
x2 y2
Let P (acos T ,bsin T ) and Q(acos I ,bsin I ) be any two points of the ellipse + = 1 then the
a2 b2
equation of the chord joining these two points is
b sin I – b sin T
y– bsin T = (x–acos T )
a cos I – a cos T
Simplifying the equation we get
x §TI· y §TI· T–I
cos¨ ¸  sin ¨ ¸ cos
a © 2 ¹ b © 2 ¹ 2
T & I are eccentric angle of points P and Q of ellipse
Position of a point (h,k) with respect to an ellipse
x2 y2
Let ellipse be 2 + 2 = 1
a b
x2 y2
Now P will lie outside, on or inside the ellipse 2 + 2 = 1 according as
a b
h2 k2
+ 2 –1>,=,<0
a2 b
EXAMPLES
1 Find the equation of the curve whose parametric equation are x=1+4cos T , y = 2+3sin T
T R
Solution: We havex=1+4cos T , y = 2+3sin T
x –1 y–2
? = cos T and = sin T
4 3
Squaring and adding we get
2 2
§ x –1· § y – 2·
¨ ¸ +¨ ¸ = cos2 T + sin2 T
© 4 ¹ © 3 ¹
( x – 1) 2 ( y – 2) 2
+ =1
16 9
Which is an ellipse.
x2 y2
2 Find the eccentric angle of a point on the ellipse + = 1 whose distance from the
6 2
centre of the ellipse is 5
Solution :
x2 y2
We have + =1
6 2

12
a2 = 6 b2 = 2
a= 6 b= 2
? any point on the ellipse with T as eccentric angle is P 6 cos T, 2 sin T
Here centre is origin
? CP = 6 cos2 T  2 sin 2 T = 5
Ÿ 6cos2 T +2sin2 T = 5
Ÿ 4cos2 T = 3
3
cos2 T =
4
3
cos T = r
2
S 5S 7S 11SS
? T = 6, 6 , 6 , 6
3 If D and E are the eccentric angles of the extremities of a focal chord of an ellipse, then the
eccentricity of the ellipse is
cos D  cos E sin D – sin E cos D – cos E sin D  sin E
(a) (b) (c) (d)
cos(D – E) sin(D – E) cos(D – E) sin(D  E)
Solution : Equation of chord joining points having eccentric angles D and E is
x §DE· y §DE· §D –E·
cos ¨ ¸ + sin ¨ ¸ = cos ¨ ¸
a © 2 ¹ b © 2 ¹ © 2 ¹
Since these points are extremities of focal chord so it passes through focus (ae,0) then
§DE· §D –E·
? e cos ¨ 2 ¸ = cos ¨ 2 ¸
© ¹ © ¹

§D –E·
cos¨ ¸
© 2 ¹
e = §D E·
cos¨ ¸
© 2 ¹

§DE·
Multiply & divide by 2sin ¨ ¸ on right side
© 2 ¹

§DE· §D –E·
2 sin ¨ ¸ cos¨ ¸
© 2 ¹ © 2 ¹
e = §D E· §DE·
2 sin ¨ ¸ cos¨ ¸
© 2 ¹ © 2 ¹

13
sin D  sin E
e =
sin(D  E)
4 An ellipse passes through the point (4,–1) and touches the line x+4y–10 = 0. Find its equation
of its axes coincide with coordinate axes.
x 2 y2
Solution: Let the equation of ellipse be + =1
a 2 b2
It passes through (4,–1)
16 1
? 2 + = 1 or a2+16b2 = a2b2......................(1)
a b2
x+4y–10 = 0 is a tangent to the ellipse.
1 10
y= – x+ Ÿ y= mx+c
4 4
1 10
m= – ,c=
4 4
c= a 2 m 2  b 2 is a condition for tangent
10 1
= a2 u  b2
4 16
100 a 2
= + b2
16 16
16b2 = 100–a2
a2+16b2 = 100
From (1) we get
100 = a2b2
100
b2 =
a2
1600
a2+ = 100
a2
a4–100a2+1600 = 0
a4–80a2–20a2+1600 = 0
a2(a2–80)–20(a2–80) = 0
(a2–80)(a2–20) = 0
10 5 100
b2 = = or b2 =5
8 4 20

14
x 2 4y2
? Equation of ellipse is + =1
80 5
x 2 y2
or + =1
20 5
x y x2 y2
5 If + = 2 touches the ellipse 2 + 2 = 1, then find its eccentric angle T of point of
a b a b
contact.
Solution : Let T be the eccentric angle of the point of contact :
? coordinates of the point is (acos T , bsin T )
Equation of tangent at this point is
x cos T y sin T
+ –1 = 0 ...................(1)
a b
x y
Given that + – 2 = 0 .....................(2) is tangent
a b
Comparing (1) and (2) as these two are identical, we get
cos T sin T
a –1
= b =
1 1 – 2
a b
1
cos T = = sin T
2
S
?T =
4
PRACTICE QUESTIONS
1 The sum of the squares of the reciprocals of two perpendicular diameter of an ellipse is
1 §1 1 · 1 §1 1 · 1 1
(a) ¨ 2  2¸ (b) ¨ 2  2¸ (c)  2 (d) None of these
4 ©a b ¹ 2 ©a b ¹ a 2
b
1
2 Prove that any point on the ellipse whose foci are (–1,0) and (7,0) and eccentcicity is
2
(3+8cos T , 4 3 sin T ), T  R. Also find the eqn of the ellipse

x2 y2
3 Let E be the ellipse + = 1 and C be the circle x2+y2 = 9. Let P and Q be the points
9 4
(1,2) and (2,1) respectively. Then
(a) Q lies inside C but outside E
(b) Q lies outside both C and E

15
(c) P lies inside both C and E
(d) P lies inside C but outside E
x2 y2
4 P is a variable on the ellipse 2 + 2 = 1 with A Ac as the major axis. Then the maximum
a b
area of the triangle AP Ac is
(a) ab (b) 2ab (c) ab/2 (d) None of these

5 A man running round a race course notes that the sum of the distances of two flag-posts from him
is always 10m and the distance between the flag-posts is 8m. The area of the path he encloses in
square meters is
(a) 15 S (b) 12 S (c) 18 S (d) 8 S
x2 y2
6 If the line A x+my+n = 0 cuts the ellipse + = 1 in points whose eccentric angles
a2 25
S a 2A 2  b 2 m 2
differ by then
2 n2
(a) 1 (b) 2 (c) 4 (d) 3/2

7 If PSQ is a focal chord if the ellipse 16x2+25y2 = 400 such that SP = 8, then SQ =
(a) 1 (b) 2 (c) 3 (d) 4

8 If equation of the ellipse is 2x2+3y2–8x+6y+5 = 0 then which of the following are true?
(a) equation of director circle is x2+y2–4x+2y = 10
(b) director circle will pass through (4, –1)
(c) equation of auxillary circle is x2+y2–4x+2y+2 = 0
(d) None of these
2 2
§x· §y·
9 The foci of ellipse ¨ ¸ + ¨ ¸ = 1 are S and Sc . P is a point on ellipse whose eccentric
©5¹ ©3¹
angle is S /3. The incentre of triangle SP Sc is

§ 2 · § 3·
(a) 2, 3 (b) ¨ 2,
©
¸

(c) ¨ 2,
¨ 2 ¸
©
¸
¹
(d) 3 ,2
Answers
( x – 3) 2 y2
1. a 2. + =1 3. d 4. a 5. a 6. b 7. b 8. c 9. b
64 48

16
ELLIPSE - IV

Intersection of a line and an ellipse


x2 y2
Line y = mx+c_______(1) and ellipse + = 1 _________(2)
a2 b2
Solving equations (1) & (2) we get
(a2m2+b2)x 2 +2a2|cmx+a2(c2–b2) = 0
If D>0 then y = mx+c is a secant
D = 0 then y = mx+c is a tangent
D<0 y = mx+c does not meet ellipse
Point form
Equation of tangent to the ellipse at point (x 1, y1)
x2 y2
Let the equation of ellipse be + =1
a2 b2
xx1 yy1
Then equation of tangent in point form is 2 + =1
a b2
Parametric form
x cos T y sin T
Equation of tangent at point (acos T ,bsin T ) to the ellipse is + =1
a b
Slope form
x2 y2
y = mx r 2
a m b 2 2 is a tangent to an ellipse + = 1 and point of contact is
a2 b2

§ a 2m b2 ·
¨r ,  ¸
¨ ¸
© a 2m2  b2 a 2m2  b2 ¹
Number of tangents through a given point P (h,k)
x2 y2
y = mx+ a m  b
2 2 2 is any tangent to the ellipse + =1
a2 b2
If it passes through P (h,k) then
k = mh+ a 2 m 2  b 2

k–mh = a 2m2  b2
(k–mh)2 = a2m2+b2
m2(h2–a2) – 2hkm+(k2–b2) = 0
It is a quadratic in m and will give two values of m hence there are two tangents.

17
Examples
1 If the line 3x+4y = 7 touches the ellipse 3x2+4y2 = 1, then the point of contact is

§ 1 1 · § 1 –1· § 1 –1 ·
(a) ¨¨ , ¸¸ (b) ¨¨ , ¸¸ (c) ¨¨ , ¸¸ (d) None of these
© 7 7¹ © 3 3¹ © 7 7¹
Solution : (a)
Let P (x 1,y1) be point of contact the equation of tangent to the ellipse

x2 y2 xx1 yy1
+ 1 = 1 is + =1
1 1 1
3 4 3 4
3xx 1+4yy–1 1=0 __________(1)
Given that 3x+4x– 7 = 0 ________________(2) touches the ellipse
? (1) and (2) are same
By comparing we get
3x 1 4 y1 –1
= =–
3 4 7
1 1
x= , y1 =
7 7
§ 1 1 ·
¨¨ , ¸¸ is the point of contact
© 7 7¹

x2
Example 2: The number of values of c such that the line y = 4x+c touches the curve + y2 = 1 is
4
(a) 0 (b) 1 (c) 2 (d) infinite.
x2 y2
Solution : Given ellipse is +
4 1
a2 = 4 b2 = 1
and a line y = 4x+c is a tangent
m=4
? c =r a 2m2  b2
= r 4 u 16  1
= r 65
? c has 2 values
c= 65 or – 65
x2 y2
3 If 3 bx+ay =2ab touches the ellipse + = 1 then the eccentric angle of the point of contact
a2 b2

18
is
S S S S
(a) (b) (c) (d)
6 4 3 2
Solution : (a)
x 3 y 1
Equation of tangent + = 1____________(1)
a 2 b 2
x y
and equation of tangent at the point (acos T , bsin T ) is cos T + sin T = 1________(2)
a b
comparing (1) & (2) we get
3 1
cos T = and sin T =
2 2
1 S
? tan T = = tan
3 6
S
? T=
6
–4 x2 y2
4 A tangent having slope of to the ellipse + = 1 intersects the major and minor axes
3 18 32
at points A and B respectively. If C is the centre of the ellipses, then the area of the triangle
ABC is
(a) 12sq.u (b) 24sq.u (c) 36sq.u (d) 48sq.u
Solution : (b)
x2 y2
Equation of tangent to the ellipse + = 1 is
a2 b2
y = mx+ a 2 m 2  b 2 (b>a)
–4 2
Here m = , a = 18, b2 = 32
3
–4 16
y= x + 18 u  32
3 9
–4
y= x+8
3
Then points on the axis where tangents meet are A(6,0) and B(0,8)
1
Then area of ' ABC is ×6×8 = 24sq.u
2

19
x2 y2
5 If the tangents to the ellipse + = 1 make angle D and E with the major axis such that
a2 b2
tan D +tan E = O , then the locus of their point intersection is
(a) x2+y2 = a2 (b) x2+y2 = b2 (c) x2–a2 = 2 O xy (d) O (x2–a2) = 2xy
Solution : (d)
Equation of tangent to the ellipse with slope m is
y = mx+ a 2 m 2  b 2
If it is passes through the point P (h,k) then
k = mh+ a 2 m 2  b 2

or (k–mh)2 = a 2m2  b2
k2+m2h2–2mkh = a2m2+b2
m2(h2–a2)–2mkh+k2–b2 = 0
It is a quadratic in m having two roots m1 & m2
2kh k 2 – b2
m1+m2 = and m1m2 =
h2 – a2 h2 – a2
Given that tan D +tan E = O
m1+m2 = O
2kh
=O
h – a22

2kh = O (h2–a2)
? locus of point P(h,k) is
O (x2–a2) = 2xy

PRACTICE QUESTIONS
1 If P(x,y), F1(3,0), F2(–3,0) and 16x2+25y2 = 400, then PF1+PF2 equals
(a) 8 (b) 6 (c) 10 (d) 12
2 The length of the major axis of the ellipse
(3x – 4 y  7) 2
(5x–10) + (5y+15) =
2 2
is
4
20 20
(a) 10 (b) (c) (d) 4
3 7
3 Angle subtended by common tangents of two ellipses 4(x–4)2+25y2 = 100 and 4(x+1)2+y2 = 4
at origin is
S S S S
(a) (b) (c) (d)
3 4 6 2

20
x2 y2
4 The distance of a point on the ellipse + = 1 from the centre is 2. Then the eccentric angle
6 2
of the point is
S 3S 5S S
(a) (b) (c) (d)
4 4 6 6
x2 y2
5 If the chord through the points whose eccentric angles are T and I on the ellipse + =1
25 9
T I
passes through a focus, then the value of tan tan is
2 2
1 –1
(a) (b) –9 (c) (d) 9
9 9
6 In an ellipse the distance between its foci is 6 and its minor axis is 8, the eccentricity of the
ellipse is
4 3 1 1
(a) (b) (c) (d)
5 5 52 2

x2
7 The number of values of C such that the straight line y = 4x+c touches the curve + y2 = 1,
4
is
(a) 0 (b) 2 (c) 1 (d) f
x2 y2
8 The line 3x+5y = 15 2 is a tangent to the ellipse + = 1, at a point whose eccentric
25 9
angle is
S S S 2S
(a) (b) (c) (d)
6 4 3 3
9 Tangents are drawn to the ellipse 3x +5y = 32 and 25x +9y = 450 passing through the point
2 2 2 2

(3,5). The number of such tangents are


(a) 2 (b) 3 (c) 4 (d) 0
x2 y2
10 Tangents are drawn to the ellipse + = 1 at ends of latus rectum. The area of quadrilateral
9 5
so formed is
27 27 27
(a) 27 (b) (c) (d)
2 4 55
11 An ellipse passes through the point (4,–1) and its axes are along the axes of coordinates. If the
line x+4y–10 = 0 is a tangent to it then its equation is
x2 y2 x2 y2 x2 y2
(a) + =1 (b) + =1 (c) + =1 (d) None of these
100 5 8 5/ 4 20 5

21
x2 y2
12 Prove that the line 2x+3y = 12 touches the ellipse + =2
9 4
§ 16 ·
13 The tangent at the point ¨ 4 cos I, sin I ¸ to the ellipse 16x2+11y2 = 256 is also a tangent to the
© 11 ¹
circle x2+y2 –2x = 15, find the value of I .
14 Find the equations of tangents to the ellipse 9x2+16y2 = 144 which pass through the point (2,3).
15 The angle between pair of tangents drawn to the ellipse 3x2+2y2 = 5 from the point (1,2) is tan1(12/
5)
16 Prove that the portion of the tangent to the ellipse intercepted between the curve and the directrix
subtends a right angle at the corresponding focus.
17 Linked Comprehension Type.
For all real p, the line 2px+y 1 – p 2 = 1 touches a fixed ellipse whose axes are coordinate
axes.
(i) The eccentricity of the ellipse is
2 3 1 1
(a) (b) (c) (d)
3 2 3 2
(ii) The foci of ellipse are

(a) 0,r 3 (b) 0,r 2 3 § r 3 ,0 ·


¨
(c)
© 2 ¸¹ (d) None of these
(iii) The locus of point of intersection of perpendicular tangents is
(a) x2+y2 = 5/4 (b) x2+y2 = 3/2 (c) x2+y2 = 2 (d) None of these
x2 y2
18 C1 : x2+y2 = r2 and C2 = + = 1 intersect at four distinct points A, B, C and D, Their
16 9
common tangents form a parallelogram Ac Bc Cc Dc .
(i) If ABCD is a square then r is equal to
12 12 12
(a) 2 (b) (c) (d) None of these
5 5 5 5
(ii) If Ac Bc Cc Dc is a square then r is equal to
(a) 20 (b) 12 (c) 15 (d) None of these
(iii) If Ac Bc Cc Dc is a square, then the ratio of area of the circle C1 to the area of the
circumcircle of ' Ac Bc Cc is
9 3 1
(a) (b) (c) (d) None of these
16 4 2

22
x2 y2
19 The ellipse + = 1 is such that it has the least area but contains the circle (x–1)2+y2 = 1
a2 b2
(i) The eccentricity of the ellipse is
2 1 1
(a) (b) (c) (d) None of these
3 3 2
(ii) Equation of auxilliary circle of ellipse is
(a) x2+y4 = 6.5 (b) x2+y4 = 5 (c) x2+y4 = 45 (d) None of these
(iii) Length of latus rectum of the ellipse is
(a) 2 units (b) 1unit (c) 3units (d) 2.5 units
x2 y2
20 The equation of the straight lines joining the foci of the ellipse + = 1 to the foci of the
25 16
x2 y2
ellipse + = 1 forms a parallelogram. Then the area of the figure formed by the foci of
24 49
these two ellipse.
(a) 15 (b) 30 (c) 20 (d) 18

ANSWERS

1. c 2. b 3. b 4. a,b 5. c,d 6. b 7. b 8. b
S
9. b 10. a 11. b,c 13. r 14. y = 3, x+y = 5
6
17. (i) a (ii) d (iii) a 18. (i) a (ii) d (iii) c 19. (i) a (ii) c (iii) b
20. b

23
ELLIPSE - V

Equations of Normals in different forms


(i) Point form
x2 y2
Let equation of ellipse be + =1
a2 b2

a 2x b2y
Equation of normal is – = a2–b2
x1 y1
(ii) Parametric form
x2 y2
Let equation of ellipse be + =1
a2 b2
Equation of normal is axsec I – bycosec I = a2–b2
(acos I , bsin I ) is parametric coordinates of P(x1,y1), i.e. P(acos I , bsin I )
(iii) Slope form
x2 y2
For an ellipse + 2 =1
a2 b
Equation of normal in terms of slope (m) is
m(a 2 – b 2 )
y = mx r
a 2  b 2m 2

x2 y2
Condition of normality when y = mx+c is the normal of + = 1 is
a2 b2

m 2 (a 2 – b 2 ) 2
c =
2
a 2  b 2m 2

m(a 2 – b 2 )
or c= r
a 2  b 2m 2
Examples
x2 y
1 If the normal at the point P( T ) to the ellipse + = 1 intersects it again at the point Q (2 T )
14 5
then cos T is equal to
2 2 3 3
(a) (b) – (c) (d) –
3 3 2 2
Solution :

24
ax by
The normal at (asin T , bsin T ) is – = a2 –b2
cos T sin T
Here a2 = 14 & b2 = 5
a = 14 b = 5
14 x 5y
– = 14 – 5
cos T sin T

14 x 5y
– = 9 _______________(1)
cos T sin T
? It meets the curve again at T (2 T ) (acos2 T ,bsin2 T )

i.e. 14 cos 2lj, 5 sin 2lj
? equation (1) satisfy this point
14 cos 2T 5 sin 2T
– =9
cos T sin T

14(2 cos 2 T – 1) 5(2 sinTcosT )


– =9
cos T sin T
28cos2 T – 14 –10cos2 T = 9cos T
18cos2 T –9cos T –14 = 0
(6cos T –7) (3cos T +2) = 0
7
cos T = > 1not possible (' cos T <1)
6
–2
cos T =
3
? option (b) is correct.

x2 y2
2 The equation of the normal to the ellipse + = 1 at the end of the latus rectum in the first
a2 b2
quadrant is
(a) x+ey–ae3 = 0 (b) x–ey+ae3 = 0 (c) x–ey–ae3 = 0 (d) None of these
Solution :

§ b2 ·
The end of the latus rectum in the first quadrantal is ¨¨ ae, ¸¸
© a ¹

§ b2 ·
Equation of normal at ¨¨ ae, ¸¸ is
© a ¹

25
b2 y
2
a x a 2x b2 y
– b 2
= a –b , x1 – y1
2 2 a 2 – b2
ae a
ax
– ay = a2–b2
e
ax–eay = ea2–eb2
= ea2–e(a2–a2e2)
=ea2–ea2+a2e3
? ax–aey–a2e3 = 0
x–ey–ae3 = 0
Correct option is c
x2 y2
3 The condition that the line xcos D +ysin D = p may be a normal to the ellipse + = 1then
a2 b2
(a) (a2–b2)2 = p2 (a2sec2 D +b2cosec2 D ) (b) (a2–b2)2 = p2 (a2cosec2 D +b2sec2 D )
(c) (a2+b2)2 = p2 (a2sec2 D +b2cosec2 D ) (d) None of these
Solution :
x2 y2
The equation of normal to + = 1 at ‘ T ’ is
a2 b2
a 2x b2 y
– – (a2–b2) = 0 ____________(1)
a cos T b sin T
Given that xcos D +ysin D –p = 0 is normal to the ellipse comparing (1) & (2) we get
a / cos T – b /sin T a 2 – b2
= =
cos D sin D p

a –b a 2 – b2
or = =
cos T cos D sin Tsin D p
cos T cos D sin T sin D p
or = – = 2
a b a – b2
ap , sin = – bp
cos T = T
(a – b ) cos D
2
2
(a – b 2 ) sin D
2

Squaring and adding we get


a 2p2 b2p2
cos T +sin T = 2
2 2
+ 2
(a – b 2 ) 2 cos2 D (a – b 2 ) 2 sin 2 D
(a2–b2)2 = p2(a2sec2 D +b2cosc2 D )
? correct option is a

4 If the normals at P(x1,y1), Q(x2,y2) and R(x3,y3) to the ellipse are concurrent, then

26
x2 y2 x 2 y2 x1 y1 x1 y1
x1 y1 x1 y1 x2 y2 x 2 y2
(a) = –1 (b) =0
x3 y3 x 3 y3 x3 y3 x 3 y3

x2 y2 x 2 y2
x3 y3 x 3 y3
(c) =1 (d) None of these
x1 y1 x1 y1
Solution :
x2 y2
The equations of the normals at P(x1,y1),Q(x2,y2) and R(x3,y3) to the ellipse 2 + 2 = 1 are
a b
a 2x b2 y
– = a2 – b2 ______________(1)
x1 y1

a 2x b2 y
– = a2 – b2 ______________(2)
x2 y2

a 2x b2 y
– = a2 – b2 ______________(3)
x3 y3
respectively
These lines are concurrent, if

a2 – b2
a 2 – b2
x1 y1
a2 – b2
a 2 – b2
x2 y2 =0
a2 – b2
a 2 – b2
x3 y3

1 –1
1
x1 y1
1 –1
1
a2b2(a2–b2) x 2 y2 =0
1 –1
1
x3 y3

R1 o x1y1R1 ; R2 o x2y2R2 ; R3 o x3y3 we get

27
y1 – x1 x1 y1
y2 – x2 x 2 y2
=0
y3 – x3 x 3 y3

x1 y1 x1 y1
x2 y2 x 2 y2
OR =0
x3 y3 x 3 y3
your correct option is ‘b’
PRACTICE QUESTIONS
x2 y2
1 In the normal at the end of latus rectum of the ellipse + = 1 with eccentricity e, passes
a2 b2
through one end of the minor axis, then :
(a) e2(1+e2) = 0 (b) e2(1+e2) = 1 (c) e2(1+e2) = – 1 (d) e2(1+e2) = 2
x2 y2
2 If the normals to + = 1 at the ends of the chords A 1 x+m1y =1 and A 2 x+m2y =1 are
a2 b2
concurrent, then :
(a) a2 A 1 A 2+b2m1m2 =1 (b) a2 A 1 A 2+b2m1m2 = –1
(c) a2 A 1 A 2–b2m1m2 = – 1 (d) None of these
3 If the normal at an end of a latus rectum of an ellipse passes through one extremity of the minor
axis, then the eccentricity of the ellipse is given by
(a) e4+e2–1 = 0 (b) e4+e2–5 = 0 (c) e3 = 5 (d) None of these
4 The number of normals that can be drawn from a point to a given ellipse is
(a) 2 (b) 3 (c) 4 (d) 1
x2 y2
5 If the normal at any point P on the ellipse + = 1 meets the axes in G and g respectively,,
a2 b2
then PG :Pg is equal to
(a) a : b (b) a2 : b2 (c) b2 : a2 (d) b:a

x2 y2 § b2 ·
6 If normal to ellipse 2 + 2 = 1 at ¨¨ ae, ¸¸ is passing through (0,–2b), then value of
a b © a ¹
eccentricity is
(a) 2 –1 (b) 2
2 –1 (c)
2 2 –1 (d) None of these

x2 y2 PM
7 If normal at any point P to the ellipse 2 + 2 = 1 , a>b meet the axes at M and N so that
a b PN

28
2
= , then value of eccentricity e is
3
1 2 1
(a) (b) (c) (d) None of these
2 3 3
8 If the tangent drown at point (t2,2t) on the parabola y2 = 4x is same as the normal drawn at point
( 5 cos T , 2sin T ) on the ellipse 4x2+5y2 = 20. Then the values of t and T are

§ –1 · –1 § 1 · 1
(a) T = cos–1 ¨ 5 ¸ & t = 5 (b) T = cos–1 ¨ 5 ¸ & t = 5
© ¹ © ¹
§ –2· –2
(c) T = cos–1 ¨ 5 ¸ & t = 5 (d) None of these
© ¹
x2 y2
9 The normals at four points on the ellipse + = 1 meet in the point (h,k). Then the mean
a2 b2
position of the four points is
§ a 2h b2k · § a 3h b 3k ·
(a) ¨¨ , 2 ¸
¸ (b) ¨¨ , 2 ¸
¸
© 2(a  b ) 2(a  b ) ¹ © 2(a  b ) 2(a  b ) ¹
2 2 2 2 2 2

§ ah bk · § a 2h b2k ·
(c) ¨¨ 2 2
, 2 2 ¸
¸ (d) ¨¨ 2 2
, 2 2 ¸
¸
© 2(a – b ) 2(a – b ) ¹ © 2(a – b ) 2(a – b ) ¹
10 The equation of the normal at the point (2,3) on the ellipse 9x2+16y2 = 180 is
(a) 3y = 8x–10 (b) 3y–8x+7 = 0 (c) 8y+3x+7 = 0 (d) 3x+2y+7 = 0
x2 y2
11 Number of distinct normal lines that can be drawn to the ellipse + = 1 from the point P
169 25
(0,6) is
(a) one (b) two (c) three (d) four
x2 y2
12 Any ordinate MP of the ellipse + = 1 meets the auxiliary circle at Q, then locus of the point
25 9
of intersection of normals at P and Q to the respective curve is
(a) x2+y2 = 8 (b) x2+y2 = 34 (c) x2+y2 = 64 (d) x2+y2 = 15
§S · x2 y2
13 If the normals at P ( T ) and Q ¨  T ¸ to the ellipse 2 + 2 = 1 meet the major axis at G and g
©2 ¹ a b
respectively, then PG +Qg =
2 2

(a) b2(1–e2)(2–e2) (b) a2(e4–e2+2) (c) a2(1+e2)(2+e2) (d) b2(1+e2)(2+e2)

Answers
1. b 2b 3. a 4. c 5. c 6. c 7. c 8. a
9. d 10. b 11. c 12. c 13. b

29
HYPERBOLA - I
Equation of Hyperbola

Definition 1 : The locus of a point in a plane, the difference of whose distance from two fixed points
in the plane is constant.
According to definition
~PF2–PF1~ = constant
Conjugate axis

Centre Transverse axis


A
F2 F1
(Focus) Vertex Vertex

Two fixed points are known as Foci of the hyperbola.


The mid point of the line segment joining the foci is called the centre. The line joining the vertices is
known as transverse axis and the line through the centre and perpendicular to transverse axis is
known as conjugate axis. The point at which the hyperbola intersect the transverse axis is known as
vertices of the hyperbola.
Let distance between two foci be 2c and distance between two vertices be 2a and a quantity ‘b’ be
b2=c2–a2.
Let a point P on the hyperbola be (x, y), then according to definition of hyperbola ( x  c) 2  y 2 –

( x – c) 2  y 2 =2a

( x  c) 2  y 2 =2a + ( x – c) 2  y 2

(x+c)2+y2 = 4a2 + (x–c)2+y2+4a ( x – c) 2  y 2 >2a  2c Ÿ a  c@


4cx–4a2=4a ( x – c) 2  y 2
P
cx (x, y)
–a= ( x – c) 2  y 2 Xc X
a F2 O F1
(–c, 0) ( c, 0)
c2 x 2 2
+a –2cx=x2+c2–2cx+y2
a2
c 2

– a2 2 2 2 2
x –y =(c –a )
a2
x2 y2 x 2 y2
– =1 Ÿ – =1
a 2 c2 – a 2 a 2 b2
x 2 y2
– =1
a 2 b2

30
b
Ÿ y= r x2 – a2
a
y is real when x 2 t a2
y is real when x t a or x d–a
Definition 2 : The locus of a point which moves in a plane such that the ratio of its distance from a
fixed point to its perpendicular distance from a fixed straight line (not passing through given fixed
point) is always constant and greater than 1.
PS P(x,y)
=e > 1 M
PM
The equation of hyperbola whose focus is the point (h, k) and ax+by+c=0 S(x1,y1)
directrix is ax+by+c=0 and whose eccentricity is e, is
ax  by  c 2
(x–h)2+(y–k)2=e2
a 2  b2
Example 1.
3
Equation of hyperbola whose focus is (1,2) equations of directrix is x+y+1=0 and eccentricity is is
2
(a) x2+y2–18xy–y–31=0
(b) x2+y2+18xy+34x+50y–31=0
(c) x2+y2–18xy–x+y–31=0
(d) x2+y2+18xy+x+y+31=0
Solution
Let variable point P(x,y) then PS=ePM
3 ­ x  y  1½
( x – 1) 2  ( y – 2) 2 = 2 ® ¾
¯ 2 ¿
9
x2+y2–2x–4y+5= (x2+y2+2xy+2x+2y+1)
8
x +y +18xy+34x+50y–31=0
2 2

Ans. b
Eccentricity :
Distance between foci 2c c
e= = =
Distance between vertices 2a a
c2 a 2  b2 b2
e = 2=
2
=1+ 2
a a2 a
a e =a +b
2 2 2 2

x2 y2
? 2 – =1
a a 2 (e 2 – 1)

31
Some terms related to Hyperbola
x 2 y2
Equation of hyperbola 2 – 2 =1
a b
1. Centre : All chords passing through a point and bisected at that point is known as centre of
hyperbola C(0,0)
Distance between foci
2. Eccentricity : e=
Distance between vertices
(conjugate axis) 2
e= 1
(transverse axis) 2
PS
e= , where S is focus, P is any point on the hyperbola, PM is distance from directrix.
PM
3. Foci : S and Sc are foci whose coordinates are S(ae,0) and Sc (–ae, 0)
Y
Z2 Z1

Xc X
Sc Ac C A S

c c
Z2 Z1
Yc

a a
4. Directrices : Z1 Z1c and Z2 Z 2c are the directrices whose equations are x= and x=–
e e
5. Vertices : A and A c are the vertices of hyperbola . A(a,0) and A c (–a,0).
6. Axes : The line A A’ is called transverse axis and the line perpendicular to its through the centre
of the hyperbola is called conjugate axis. Equations of transverse axis is y=0 and equation of
conjugate axis is x=0.
Length of transverse axis =2a
Length of conjugate axis =2b
7. Double Ordinate : A chord of hyperbola which is perpendicular to transverse axis is known as
double ordinate Q Qc , Q(h, k), Qc (h,–k).

Qc

8. Latus rectum : The double ordinates passing through focus is known as latus rectum.

32
b 2 Lc §¨ ae,– b ·¸
2

L1(ae, ), 1 ¨ Y
a © a ¸¹
L2 L1
b 2 § b2 ·
c
L2(–ae, ), L 2 ¨ – ae,– ¸¸
¨
a © a ¹ Xc
Sc S
X

9. Focal Chord : A chord passing through focus is c


L2 c
known as focal chord. L1
10. Focal Distance : Yc
a
PS1=ePM1=e(x1– )
e
Y
= ex 1–a Z2 Z1
a M2 M1
PS2=e(x1+ ) P(x1,y1)
e
Xc X
= ex 1+a S2 O S1(ae,o)
? PS2–PS1=2a
c c
Z2 Z1
Rectangular or Equilateral Hyperbola Yc
If a=b, then equation of hyperbola is
x2–y2=a2 known as rectangular or equilateral hyperbola.
The eccentricity of rectangular hyperbola is
b2
e =1+ 2 =1+1
2
a
e= 2
Equation of hyperbola if centre is (h, k) and axes are parallel to coordinate axes is
(x – h) 2 (y – k) 2
– =1
a2 b2

33
PRACTICE QUESTIONS
1. The eccentricity of the conic 9x2–16y2=144 is
4 4 5
a. b. c. d. 17
3 5 4
2. The equation of the conic with focus at (1, –1) directrix along x–y+1=0 and eccentricity 2 is
a. xy=1 b. 2xy+4x–4y–1=0
c. x –y =1
2 2
d. 2xy–4x+4y+1=0
3. Equation of the hyperbola whose vertices are ( r 3,0) and foci at ( r 5,0) is
a. 9x 2–25y2=144 b. 16x2–9y2=144 c. 9x2–16y2=144 d. 25x2–9y2=225
4. The eccentricity of the hyperbola whose latus rectum is half of its transverse axis, is
3 3 2 2
a. b. c. d.
2 2 3 3
5. The differences of the focal distances of any point on the hyperbola is equal to
a. eccentricity b. latus-rectum
c. length of conjugate axis d. length of transverse axis
6. The distance between the foci of a hyperbola is 16 and its eccentricity is 2 , then equation of
the hyperbola is
a. x2–y2=32 b. x2–y2=16 c. y2–x 2=16 d. y2–x 2=32
7. The length of latus rectum of the hyperbola 3x2– 6y2
a. 24 b. 2 3 c. 3 2 d. 4 3
8. The eccentricity of the hyperbola 9x –16y –18x+32y–151=0 is
2 2

5 5 5
a. b. c. d. 5
3 4 2
x 2 y2
9. The foci of a hyperbola coincide with the foci of the ellipse  =1. If its eccentricity is 2
25 9
then equation of hyperbola is
x 2 y2 x 2 y2 x 2 y2
a. – =1 b. – =1 c. – =1 d. x2–y2=4
12 4 8 12 4 12
10. The eccentricity of rectangular hyperbola is
a. 2 b. 1 c. 3 d. 2

Answers
1. c 2. d 3. b 4. a 5. d
6. a 7. d 8. b 9. c 10 d

34
HYPERBOLA - II
Conjugate Hyperbola
x 2 y2
Position of a point : Let a point P(x 1,y1) and equation of hyperbola be 2
– 2 –1=0
Ya b
2 2
x1 y R(x3,y3)
S1= 2
– 12 –1 Q(x2,y2)
a b
P(x1,y1)
If S1 > 0, point is inside the hyperbola. Xc X

If S1 = 0, point is on the hyperbola.


If S1 < 0, point is outside the hyperbola.
Yc

Conjugate Hyperbola : Corresponding to every hyperbola there exists a hyperbola such that the
transverse axis and conjugate axis of one is equal to the conjugate axis and transverse axis of the
other. Such hyperbolas are known as conjugate to each other. Y
(0, be)
x 2 y2 S3
Therefore for the hyperbola 2 – 2 =1
a b
x 2 y2 Xc
S2 O S1(ae,0)
X
Conjugate hyperbola is 2 – 2 (–ae,0)
a b
x 2 y2 S4
Let e1 be the eccentricity of 2 – 2 =1 and e2 be Yc
(0,–be)
a b
x 2 y2 b2 a 2  b2
the eccentricity of 2 – 2 =–1 then e 1=1+ 2
2
a b a a2
a2 b2  a 2
and e 2=1+ 2
2
b b2
1 1
? e 2  e 2 =1
1 2

The foci of a hyperbola and its conjugate hyperbola are concyclic and form the vertices of a square.
Auxiliary circle and eccentric angle
A circle drawn with centre O and transverse axis as diameter
is known as auxiliary circle. Equation of auxiliary circle is (acosT ,asinT )
x2+y2=a2 A P(asecTbtanT )
A is any point on the circle whose coordinates are (acosT, T
X
O B
a sinT), where Tis known as eccentric angle. Now, In
'OAB, OA=a
a
cosT= Ÿ OB=asecT
OB

35
a 2 sec 2 T y 2
P point lies on hyperbola, so – 2 =1 Ÿ y r b tan T
a2 b
P(asecT, btanT)
0dT< 2S
The equations x=asecT and y=btanTrepresents a hyperbola. So, the parametric form of the hyperbola
x 2 y2
– =1 can be represented as x=asecT,y=btanT
a 2 b2
(x – h )2 ( y – k )2
For the hyperbola – =1, parametric form is x=h+asecT; y=k+btanT.
a2 b2
Hyperbola Conjugate Hyperbola
x 2 y2 x 2 y2
Equation – 1 –  1
a 2 b2 a 2 b2
Centre (0, 0) (0, 0)
Vertice (a, 0) & (–a, 0) (0, b) & (0,–b)
Foci (ae, 0) & (–ae, 0) (o, be) & (0, –be)
Length of transverse axis 2a 2b
Length of conjugate axis 2b 2a
2b 2 2a 2
Length of latus rectum
a b
Equation of transverse axis y=0 x=0
Equation of conjugate axis x=0 y=0
a b
Equation of directrices x= r y= r
e e

b2 a2
Eccentricity e= 1 or b2=a2(e2–1) e= 1 or a2=b2(e2–1)
a2 b2

Example 1.
§ at a bt b ·
The point ¨  , – ¸ lies on the, [ for all values of t(t z 0)]
© 2 at 2 2 t ¹
(a) circle (b) parabola (c) ellipse (d) hyperbola
Solution
at a bt b
Let x=  and y= –
2 2t 2 2t
2x 1 2y 1
t  and t–
a t b t
Squaring and subtracting, we get

36
4x 2 4 y 2
– 2 =4
a2 b
x 2 y2
? – =1
a 2 b2
Ans. d
Example 2.
The position of the point (5, –4) relative to the hyperbola 9x 2–y2=1 is
(a) on the hyperbola (b) outside the hyperbola
(c) Inside the hyperbola (d) can not say
Solution
S=9x2–y2–1
S1=9(5)2–(–4)2–1=225–16–1=208>0
? point (5, –4) lies inside the hyperbola.
Ans c.
Example 3.
Two circles are given such that they neither intersect nor touch. The locus of centre of variable circle
which touches both the circles externally is
(a) a circle (b) a parabola
(c) an ellipse (d) a hyperbola
C C
Solution 1 2

Let radii of the fixed circles be r 1 and r2 and radius of C


variable circle be r
Let variable circles with centre C touches two fixed circles with centre C 1 and C2.
Then CC1=r+r1 and CC2=r+r2 CC1–CC2=r1–r2=constant
Locus of C is hyperbola whose foci are C 1 and C2
Ans. d
Example 4:
If the latus rectum subtends a right angle at the centre of the hyperbola
x 2 y2
– =1, then its eccentricity is
a 2 b2
A
5 1 § b2 ·
¨¨ ae, ¸¸
(a) (b) 3 © a ¹

2 X
C S (ae, 0)
(0, 0)
5 5 3 § b2 ·
¨¨ ae,– ¸¸
(c) (d) B © a ¹

2 2
Solution
mAC×mBC= –1
b2 – b2
u = –1
a.ae a.ae
b4=a4e2

37
b4
a 4 =e
2

(e2–1)2=e2
e4–3e2+1=0
3r 5 3 5 3 5
e2= Ÿ e2 = Ÿ e=
2 2 2

3 5
[if e = <1 but ecentricity of hyperbola >1 so neglecting this value of e]
2
And. d

PRACTICE QUESTIONS
4
1. The equation of hyperbola whose foci are (8,3), (0,3) and eccentricity is is
3
(x – 4) 2 ( y – 3) 2 (x – 4) 2 ( y – 3) 2
a. – 1 b. – 1
9 7 7 9

(x – 3) 2 ( y – 4 ) 2
c. – 1 d. none of these
9 7
2. If S and Sc be the foci, C the centre and P be a point on a rectangular hyperbola then SP× Sc P
is equal to
a. 2.SP b. (SP)2 c. (CP)2 d. 2.CP

3. If e and ec be the eccentricity of a hyperbola and its conjugate, then


1 1 1 1
a. – 2 1 b. e2+ ec 2=1 c. e2– ec 2=1 d.  2 1
e 2
ec e 2
ec
x 2 y2
4. The foci of a hyperbola coincide with the foci of the ellipse  1 , The equation of hyperbola
25 9
if its eccentricity is 2 is
a. 3x 2–y2=12 b. 4x 2–y2=12 c. x2–3y2=12 d. x 2–4y2=12

5. The equation ( x – 4 ) 2  ( y – 2 ) 2 + ( x  4) 2  ( y – 2) 2 =8 represents


a. an ellipse b. a parabola
c. a pair of coincident line segment d. hyperbola
x2 y2
6. For hyperbola – =1, which of the following remains constant with change in D.
cos 2 D sin 2 D
a. abscissae of vertices b. abscissae of foci
c. eccentricity d. directrix

38
7. Two rods are rotating about two fixed points in opposite directions. If they start from their
position of co-incidence and one rotates at the rate double that of the other, then locus of point
of intersections of two rods is
a. a parabola b. a circle c. an ellipse d. a hyperbola
x2 y2
8. The equations – =1 , r > 1 represents
1– r 1 r
a. an ellipse b. a circle c. a hyperbola d. None of these

9. The equation 2x2+3y2–8x–18y+35=k represents


a. no locus if k > 0 b. an ellipse if k < 0
c. a point if k = 0 d. a hyperbola if k > 0

10. A hyperbola having the transverse axis of length 2sinT is confocal with the ellipse 3x2+4y2=12.
Then its equation is
a. x2cosec2T– y2sec2T=1 b. x 2sec2T– y2cosec2T=1
c. x sin T– y cos T=1
2 2 2 2
d. x2cos2T– y2sin2T=1

Answers

1. a 2. c 3. d 4. a 5. c
6. b 7. d 8. d 9. c 10. a

39
HYPERBOLA - III
Equation of Hyperbola
Equation of a Hyperbola referred to two perpendicular lines Y
Let equation of hyperbola be
P
x 2 y2 N (x, y)
– =1
a 2 b2 X
O
From diagram PM=y and PN=x
PN 2 PM 2
– 2 =1
a2 b
ie. if perpendicular distance of a point P(x,y) from two mutually perpendicular lines say l 1=a1x+b1y+c1=0
and l2=a2x+b2y+c2=0 then

§a xb yc · §a xb yc ·


¨ 1 1 1¸ ¨ 2 2 2 ¸
¨ ¸
a 1  b1 ¹ ©
2 2 ¨ a 2  b 2 ¸¹ =1
2 2
© –
a2 b2
then the locus of point P denotes a hyperbola
* centre of the hyperbola , we get after solving l1=0 and l2=0
* Transverse axis : l2=0
* Conjugate axis : l1=0
a 1x  b1 y  c1
* Foci : The foci of the hyperbola is the point of intersection of the lines r ae and
a 12  b12
l2=0
a1 x  b1 y  c1 a
* Directrix : r
a12  b12 e
* Length of transverse axis = 2a
* Length of conjugate axis = 2b
2b 2
* Length of latus Rectum =
a
Example 1 :
Find the eccentricity and centre of the hyperbola
3x – 4 y – 12 2 – 4x  3y – 12 2
=1
100 225
Solution :
l1=3x–4y–12, l2=4x+3y–12
a=10, b=15
225 13
e = 1
100 2

40
3x–4y–12=0
4x+3y–12=0
84 12 § 84 12 ·
x= , y=– Centre ¨ ,– ¸
25 25 © 25 25 ¹
Example 2:
Find the eccentricity of the conic 4(2y–x–3)2–9(2x+y–1)2=80
Solution :
2 2
4 ( 2y – x – 3 )
– 9 ( 2x  y – 1 ) =
80
80 80 80
2 2
( 2y – x – 3 )( 2x  y – 1 )
20

8 0 / 9 =1
2 2 80
a = 20, b = 9

1 b
2
1 80 1 4 13
e= = = 9
=
a2 9x 20 3
2
Example 3:
Find the coordinates of the centre, foci and vertices, length of axes and latus rectum, equation
of axes and directries, and eccentricity of the conic 9x 2–16y2–18x+32y–151=0
Solution :
9x 2–16y2–18x+32y–151=0
9(x2–2x+1–1) –16(y2–2y+1–1)–151=0
9(x–1)2–16(y–1)2=144
x – 1 2 – y – 1 2
=1
16 9
Let x–1=X, y–1=Y, a=4, b=3
Centre : X=0, Y=0 Ÿ x=1, y=1 i.e. (1, 1)
9 5
Eccentricity : e= 1 =
16 4
Foci : X= r ae, Y=0 Ÿ x=1 r 5, y=1 i.e. (6, 1) and (–4, 1)
Vertices : X= r a, Y=0 Ÿ x=1 r 4, y=1 i.e. (5, 1) and (–3, 1)
Length of transverse axis = 2a = 8
Length of conjugate axis = 2b = 6
2b 2 9
Length of latus rectums = =
a 2
Equation of transverse axis : Y=0 Ÿ y–1=0
Equation of conjugate axis : X=0 Ÿ x–1=0
a 16
Equation of directries X = r Ÿ x–1= r i.e. 5x–21=0 and 5x+11=0
1=0
e 5

41
Example 4:
The equation of the transverse and conjugate axes of a hyperbola are respectively 3x+4y–7=0,
4x–3y+8=0 and their respective lengths are 4 and 6. The equation of the hyperbola is
a. 17x2+312xy+108y2–634x–312y–715=0
b. 108x2+312xy+17y2–312x–634y–715=0
c. 108x2–312xy+17y2–312x–634y–715=0
d. none of these
Solution :
The equations of hyperbola is
2 2
§ 3x  4 y – 7 · § 4 x – 3 y  8 ·
¨ ¸ ¨ ¸
¨ 2 ¸ ¨ 2 ¸
© 3 2
 4 ¹ –© 4 2
 3 ¹
§4·
2
§6·
2
=1
¨ ¸ ¨ ¸
©2¹ ©2¹

3x  4 y – 7 2 – 4 x – 3y  8 2
=1
100 225
9(9x2+16y2+49+24xy–42x–56y)–4(16x 2+9y2+64–24xy+64x–48y)=900
17x2+312xy+108y2–634x–312y–715=0
Ans (a)
Line and Hyperbola

x 2 y2
Let equation of line be y=mx+c and equation of hyperbola be 2 – 2 1
a b
x 2 (mx  c) 2
– 1
a2 b2
x 2 m 2 x 2 c 2 2mc
– 2 – 2 – 2 x –1=0
a2 b b b
§ 1 m 2 · 2mc § c2  b 2 ·
x 2 ¨¨ 2 – 2 ¸¸ – 2 x – ¨¨ 2
¸¸ 0
©a b ¹ b © b ¹

§ b2 ·
x 2 ¨¨ 2 – m 2 ¸¸ – 2mcx – (c 2  b 2 ) 0
©a ¹

42
§ b2 – a 2m 2 · § 2 2 b2 c2 - m2 c 2 + b4 - m2 b 2 · § 2 2
– 2 2·
¨¨m c + 2 ¸¸ = 4b2 ¨ c + b 2 a m ¸
? D=4m c +4.¨¨
2 2
a2
¸¸ (c2+b2) = 4 a a2
¨ a ¸
© ¹ © ¹ © ¹

i. D < 0 i.e. c2–a2m2+b2 < 0 line do not intersect hyperbola.


ii. D = 0 i.e. c2–a2m2+b2 = 0 line touches the hyperbola.
iii. D > 0 i.e. c2–a2m2+b2 > 0 line intersect hyperbola at two points.
Hence y = mx r a 2 m 2 – b 2 is a tangent to hyperbola. Y
P(h, k)
Let this tangent passes through a point (h, k) then k = mh r a 2 m 2 – b 2 T

(k–mh)2= a2m2–b2
X
m2(h2–a2)–2mkh + k2+b2=0 O
Hence maximum two tangents can be drawn through a point P.
2kh
Now m1 + m2 =
h – a2
2

k 2  b2
m1 . m2 =
h2 – a2
If T is the angle between the two tangents, then
§ m –m ·
tanT ¨¨ 1  m m ¸¸
1 2

© 1 2 ¹
2
§ 2kh · § k 2  b2 ·
¨ 2 2 ¸
– 4¨¨ 2 ¸
©h –a ¹ © h – a 2 ¸¹
( m1  m2 ) – 4 m1m 2
2
2
Ÿ tan2T = § k 2  b2 ·
(1  m1m 2 ) 2 ¨¨1  2 ¸
2 ¸
© h –a ¹

4k 2 h 2 – 4(h 2 – a 2 )(k 2  b 2 )
Ÿtan T2
(h 2  k 2 – a 2  b 2 ) 2

4(a 2 b 2  a 2 k 2 – h 2 b 2 )
Ÿtan T2
(h 2  k 2 – a 2  b 2 ) 2
If T=90° then h2+k2–a2+b2=0
i.e. h2+k2=a2–b2
Locus of (h,k) is x 2+y2=a2–b2
Hence, Locus of point of intersection of two perpendicular tangents is known as Director Circle. Its
equation is x2+y2=a2–b2
If a=b, director circle is a point circle.
If a < b, no real director circle is possible.
(x – h)2 (y – h)2
For equation of hyperbola – equation of tangent in slope from is y–k=m(x–
a2 b2
h) r a 2 m 2 – b 2 .

43
PRACTICE QUESTIONS
x 2 y2 x 2 y2
1. If the foci of the ellipse  1 and the hyperbola – 1 coincide then b2 =
232 b 2 144 81
a. 3 b. 5 c. 7 d. 9
x 2 y2
2. If PQ is a double ordinate of the hyperbola 2 – 2 1 such that OPQ is an equilateral triangle,
a b
O being the centre of the hyperbola, the range of eccentricity is
§ 2 · § 2 · § 4· §4 ·
a. ¨ 0, ¸ b. ¨ ,f¸ c. ¨ 0, ¸ d. ¨ ,f¸
© 3¹ © 3 ¹ © 3¹ ©3 ¹
3. An ellipse and hyperbola are confocal and the conjugate axis of the hyperbola is equal to the
minor axis of the ellipse. If e1 and e2 are the eccentricities of the ellipse and hyperbola then
1 1 1 1
a. e12+e22=2 b. e1+e2=2 c.  2 d.  2
e1 e 2 e12 e 22
3x  4 y – 7 2 4 x – 3y  8 2
4. The centre of a hyperbola – =1 is
100 225
§ – 11 52 · § 11 – 52 ·
a. ¨ , ¸ b. ¨ , ¸ c. (0, 0) d. (10, 15)
© 25 25 ¹ © 25 25 ¹
5. The equations of the transverse and conjugate axes of a hyperbola are x+2y–3=0 and
2
2x–y+4=0 respectively and their respective lengths are 2 and , equation of hyperbola is
3
a. 2(2x–y+4)2–3(x+2y–3)2=1 b. 2(x+2y–3)2–3(2x–y+4)2=1
c. 2(2x–y+4)2–3(x+2y–3)2=5 d. 2(x+2y–3)2–3(2x–y+4)2=5
6. For all real values of m the straight line y=mx+ 9m 2 – 4 is a tangent to the hyperbola
a. 4x 2–9y2=36 b. 9x 2–4y2=36 c. x2–36y2=9 d. 36x 2–y2=36
7. The equation of tangents to the curve 4x –9y =1 which is parallel to 4y=5x+7 is
2 2

a. 4y=5x–30 b. 4y=5x+24
c. 24y–30x= 161 d. 30x–24y– 161 =0
8. If the line 5x+12y=9 touches the hyperbola x –9y =9 then point of contact is
2 2

§ –4· § 4·
a. (5, 4) b. ¨ 5, ¸ c. ¨ 5, ¸ d. (5, –4)
© 3 ¹ © 3¹
x 2 y2
9. If y=mx+ 51 is a tangent to the hyperbola – =1, then m =
100 49
a. 1 b. 17 c. –1 d. 2
x 2 y2
10. The locus of the point of intersection of perpendicular tangents to – =1 is
25 16
a. x2+y2=25 b. x2+y2=16 c. x2+y2=41 d. x2+y2=9
Answers
1. c 2. b 3. d 4. a 5. c 6.a 7. c 8. b 9. c 10. d

44
HYPERBOLA - IV
Equation of Tangent
Equation of Tangent
x 2 y2 Y
i. Point Form : – =1
a 2 b2
Differentiate w.r.t. x P(x , y )
1 1

2 x 2 y dy O
X

a 2 a 2 dx =0

dy b2 x
dx b2 y

b 2 x1
Slope of tangent = 2
a y1

b 2 x1
Equation of tangent, y–y1= 2 (x–x1)
a y1

xx1 yy1 x12 y12


– 2 –
a2 b a 2 b2
xx1 yy1
– 2 =1 (But P(x1,y1) lies on hyperbola)
a2 b
xx1 yy1
or T = – 2 –1
a2 b
Equation of tangent is T=0
(x – h )2 ( y – k )2
Equation of tangent at point P(x 1 ,y 1 ) to the hyperbola – =1 is
a2 b2
( x – h )( x1 – h ) ( y – k )( y1 – k )
– =1
a2 b2
(ii) Parametric Form :
Parametric equation of hyperbola is x=asecT, y=btanT
Equation of tangent is
x y
secT– tanT=1
a b
(iii) Slope Form :
y = mx r a 2 m 2 – b 2

45
§ x 2 y2 ·
Hyperbola ¨¨ 2 – 2 1¸¸ Point of contact.
©a b ¹
xx1 yy1
Point form : – 2 =1 (x1,y1)
a2 b
x y
Parametric Form : secT– tanT=1 (asecT,btanT)
a b
§ a 2m b2 ·
Slope form: y = mx r a 2 m 2 – b 2 ¨r , r ¸
¨ ¸
© a 2m2 – b2 a 2m2 – b2 ¹

Example 1 :
The equation of a tangent to the hyperbola 16x 2–25y2–96x+100y–356=0, which makes an
S
angle with the transverse axis, is
4
a. y=x+2 b. y=x–5 c. y=x+3 d. x=y+2
Solution :
The equation of the hyperbola is
16(x 2–6x) –25(y2–4y)= 356
x – 3 2 y – 2 2
– =1
25 16
S
The equation of tangent of slope m = tan =1 to this hyperbola are
4
y–2=1(x–3) r 25u 1 – 16
y–2=x–3 r 3
Ÿ y=x+2 or y=x–4
Ans. (a)

Example 2 :
x 2 y2
The point of intersection of two tangents to the hyperbola 2 – 2 =1, the product of whose
a b
slopes is c , lies on the curve
2

a. y2–b2=c2(x 2+a2) b. y2+a2=c2(x 2–b2)


c. y2+b2=c2(x 2–a2) d. y2–a2=c2(x 2+b2)
Solution :
x 2 y2
Let P(h,k) be the point of intersection of two tangents to the hyperbola – =1
a 2 b2
The equation of tangent to hyperbola is

46
y=mx ± a 2 m 2 – b 2
If it passes through (h, k) then
k=mh ± a 2m2 – b2
Ÿ (k–mh)2=a2m2–b2
Ÿ m2(h2–a2)–2mkh+k2+b2=0
k 2  b2
Let m1 and m2 be the slopes of the tangents passing throgh P. Then m 1.m2=
h2 – a2
k 2  b2
Ÿ c 2=
h2 – a2
Hence locus of P(h, k) is y2+b2=(x 2–a2)c2
Ans. (c)

Example 3 :
x 2 y2
If the tangents drawn from a point on the hyperbola x –y =a –b to the ellipse 2 + 2 =1
2 2 2 2
a b
make angles D and E with the transverse axis of the hyperbola, then
a. tanD–tanE=1 b. tanD+tanE=1 c. tanD.tanE=1 d. tanD.tanE=–1
Solution :
Let P(h,k) be the point on the hyperbola x 2–y2=a2–b2, then h2–k2=a2–b2 .........(i)
x 2 y2
The equation of tangent to the ellipse is 2 + 2 =1 is
a b
y=mx ± a 2 m 2  b 2
If it passes through (h, k) then
k=mh ± a 2m2  b2
(k–mh)2=a2m2+b2
m2(h2–a2)–2mkh+k2–b2=0
Let m1 and m2 be the roots of this equation, then
k 2 – b2
m1.m2= =1 (from equation (i))
h2 – a2
tanD.tanE=1
Ans (c)

Example 4 :
If the line 2x+ 6 y=2 touches the hyperbola x 2–2y2=4, then the point of contact is

§1 1 ·
a. – 2, 6 b. – 5,2 6 c. ¨ ,
©2 6¹
¸ d. 4,– 6

47
Solution :

x2 y2
Equation of hyperbola is 2 –
2 2 2 =1

2 2
Equation of tangent is y = – x
3 3

2 2
m= – , a=2, b = 2 , c= 3 and c =a m –b
2 2 2 2
3

§ a 2m b2 ·
¨
Point of contact is ¨
©
r
a 2m 2 – b2
, r
a 2m 2 – b2
¸
¹

¸ i.e. B 4,r 6
Ans. (d)

Example 5 :
x 2 y2
Number of real tangents can be drawn from the point (5, 0) to the hyperbola – =1 is
16 9
a. 2 b. 1 c. 0 d. 4
Solution :
25 0
S 1= – –1 > 0
16 9
? Point (5, 0) lies inside the hyperbola. Hence no tangents can be drawn.
Ans. (c)

PRACTICE QUESTIONS
x 2 y2
1. P is a point on the hyperbola 2 – 2 =1, N is the foot of the perpendicuilar from P on the
a b
transverse axis. The tangent to the hyperbola at P meets the transverse axis at T. If O is the
centre of the hyperbola, then OT×ON is
a. b2 b. a2 c. ab d. none of these
x 2 y2
2. Tangents drawn from the point (c,d) to the hyperbola – =1make angles D and E with
a 2 b2
the x-axis. If tanD tanE=1, then c2–d2=
a. a2–b 2 b. a2.b 2 c. a2+b 2 d. none of these
x 2 y2 y2 x 2
3. The common tangents to the two hyperbolas – =1 and – =1 is
a 2 b2 a 2 b2
a. y=x+ a 2 – b 2 b. y=x+ a 2  b 2 c. y=2x+ a 2 – b 2 d. none of these

48
4. The equation of the tangent to the curve 4x 2–9y2=1 which is parallel to 4y=5x+7 is
2 15
a. 30x+24y=720 b. 7y=6x–15 c. y=3 x+ d. none of these
7 7
6. The locus of a point P(h, k) moving under the condition that the line y=hx+k is a tangent to the
x 2 y2
hyperbola – =1 is
a 2 b2
a. Parabola b. circle c. ellipse d. hyperbola

Answers
1. b 2. c 3. a 4. d
5. c 6. d

49
Hyperbola - V
Normals
Equation of Pair of Tangents
x 2 y2
Let equation of hyperbola be – =1 and a point P(x 1,y1) then the combined equation of
a 2 b2
tangents PA and PB is SS 1 = T2 where Y

x 2 y2 A
S= 2 – 2 –1 (x1, y1)
a b P
X
2 2 O
x1 y B
S1= 2
– 12 –1
a b
xx1 yy1
T= – 2 –1 Yc
a2 b
Example 1 :
x 2 y2
A pair of tangents drawn from the point (4, 3) to the hyperbola – =1 then angle between
16 9
the tangents is
§3· S §4· §1·
a. tan–1 ¨© 4 ¸¹ b. 2 c. tan –1 ¨ ¸
©3¹ d. tan –1 ¨ ¸
©2¹
Solution :
x 2 y2 16 9 4x 3y
S= – –1, S1 = – –1=–1, T= – –1
16 9 16 9 16 9
Equation of pair of tangents is SS1=T2
2
§ x 2 y2 · § x y ·
– 1¨¨ – – 1¸¸ ¨ – – 1¸
© 16 9 ¹ ©4 3 ¹

x 2 y2 x 2 y2 xy x 2 y
–  +1=  +1– – 
16 9 16 9 6 2 3
3x2–4xy–12x+16y=0
3x–4y=0, x–4=0
3
m1= m2= f
4
S 3 3 4
T= –tan–1 =cot–1 =tan–1
2 4 4 3
Ans (c)

50
Equation of Normal to the Hyperbla :
i. Point Form:
x 2 y2 Y
Tangent
– =1
a 2 b2
Normal
2 P(x , y )
b x1
1 1

Slope to tangent = + 2 Xc X
a y1 O

a 2 y1
Slope to normal = – 2
b x1
Yc

a 2 y1
Equation of normal y–y1= – 2 (x–x1)
b x1

a 2x b2 y
+ =a2+b 2
x1 y1
ii. Parametric Form: P(asecT, btanT)
ax by
+ =a2+b 2
sec T tan T
or axcosT+bycotT= a2 + b2
iii. Slope Form:
m(a 2  b 2 )
y = mx r
a 2 – m2b2

§ a2 mb 2 ·
¨
Point of contanct is ¨ r 2 ,B ¸
¸
© a – m2b2 a 2 – m2b2 ¹
Exercise 1
x 2 y2
A normal to the hyperbola 2 – 2 =1 meets the axes in M and N and lines MP and NP are
a b
drawn perpendiculars to the axes meeting at P. The locus of P is
a. a2x2–b2y2=(a2+b2)2 b. a2x2–b2y2=a2+b2
c. b2x2–a2y2=a2+b2 d. a2x2–b2y2=a2–b2
Solution :
Y
Equation of normal is axcosT+bycotT=a2+b2
P
N
§a b · 2 2
§ a b ·
2 2

Normat meets the axes at M ¨¨ ,0 ¸¸ and N ¨¨ 0, ¸¸


© a cos T ¹ © b cot T ¹ Xc X
O M
a b2 2
Now equation of PM is x =
a cos T
Yc

51
ax
Ÿ secT=
a  b2
2

a 2  b2
and equation of PN is y =
b cot T
by
Ÿ tanT=
a  b2 2

sec2T–tan2T=1
a2x2 b2 y 2

a 2
 b2 a
2 2
 b2 2 =1

? a2x2–b2y2=(a2+b2)2
Ans. (a)
Exercise 2
x 2 y2
The line lx+my–n=0 will be normal to the hyperbola 2 – 2 =1 if
a b

a. l a +b m =
2 2
(a 2  b 2 )
2 2
b.
a 2 b2

a 2
 b2
2

n l2 m 2 n2

c.
a 2 b2

a 2
 b2
2

d.
a 2 b2

a 2  b2
l2 m 2 n2 l2 m2 n2
Solution :
Equation of normal is axcosT+bycotT=a2+b2
This equation compare with lx+my=n
a cos T b cot T a 2  b2
i.e.
l m n
an
Ÿ secT=
l( a  b 2 )
2

bn
Ÿ tanT=
m(a 2  b 2 )
? sec2T–tan2T=1
a 2 b2 (a 2  b 2 ) 2

l2 m 2 n2
Ans. (c)
Exercise 3
x 2 y2
The line xcos D +ysin D = p touches the hyperbola 2 – 2 =1 if
a b
a. a cos D+b sin D=p
2 2 2 2
b. a cos2D–b2sin2D=p2
2

52
c. a2cos2D+b2sin2D=p 2 d. a2cos2D–b2sin2D=p
Solution :
x 2 y2
The line y=mx+c touches the hyperbola 2 – 2 =1, then c2=a2m2–b2
a b
Given equation is y = – cotD+pcosecD y = – x cot D + p cosec
? p2cosec2D=a2(–cotD)2–b2
p2 = a2cos2D–b2sin2D
Ans. (b)
Example 4 :
The normal at P to a hyperbola of eccentricity e, intersects its transverse and conjugate axes at
L and M respectiely. The locus of the mid point of LM is a hyperbola then eccentricity of the
hyperbola is
e 1 e
a. b. 2 c. e d. none of these
e –1 e –1
Solution :
x 2 y2
The equation of the normal at P (asecT,btanT) of the hyperbola 2 – 2 =1 is
a b
axcosT+bycotT=a +b2 2

§ a 2  b2 · § a 2  b2 ·
This intersect the transverse and conjugate axes at L ¨¨ sec T,0 ¸¸ and M ¨¨ 0, tan T ¸¸
© a ¹ © b ¹
respectively.
Let N(h,k) be the mid point of LM, then
a 2  b2 a 2  b2
h= secT and k = tanT
2a 2b
2ah 2kb
Ÿ secT= 2 and tanT=
a b 2
a  b2
2

? sec2T–tan2T=1

Ÿ 4a2h2–4b2 k2=(a2+b2 )2
Thus locus of (h,k) is 4a2x2–4b2y2=(a2+b2)2
Let e1 be the eccentricity of this hyperbola. Then
a 2 a 2  b2 a 2e2 e2
e1 =1+ 2 =
2
= 2 2 =
b b2 a (e – 1) e 2 – 1
e
Ÿ e 1= 2
e –1
Ans. (b)

53
Some Important Results
1. Normal other than transverse axis never passes through the focus.
x 2 y2
2. Locus of the feet of the perpendicular drawn from focus of the hyperbola 2 – 2 =1 upon any
a b
tangent is its auxiliary circle i.e. x2+y2=a2.
Y

T P
Xc X
O S

Tc

Yc

3. The product of the feet of these perpendiculars is b 2.


4. The portion of the tangent between the point of contact and the directrix subtends a right angle
at the corresponding focus.
YZ

P
Xc X
O S
T

Yc Zc

5. The tangent and normal at any point of a hyperbola bisect the angle between the focal radii.
This spells the reflection property of the hyperbola as an incoming light ray aimed towards one
focus is reflected from the outer surface of the hyperbola towards the other focus.
Y

P
Xc X
Sc O S

Yc

54
6. If an ellipse and a hyperbola have the same foci, they cut at right anlges at any of their common
points.
Y

90°

Xc X
S2 O S1

Y’

x 2 y2 x2 y2
7. The ellipse + =1 and the hyperbola – =1 (a>k>b>0) are confocal and
a 2 b2 a 2 – k 2 k 2 – b2
therefore orthogonal.
8. The foci of the hyperbola and the points P and Q in which any tangent meets the tangents at the
vertices are concyclic with PQ as diameter of the circle.
Y

Xc X
S2 O S1

PRACTICE QUESTIONS
x 2 y2
1. Normal is drawn at one of the extremities of the latus rectum of hyperbola 2 – 2 =1, which
a b
meets the axis at points A and B. Then area of triangle OAB (O being the origin) is
1 2 5
a. a 5 e2 b. a 2 e5 c. ae d. none of these
2
x 2 y2
2. If the normal at P(T) on the hyperbola – =1meets the conjugate axis at G then AG× Ac G
a 2 2a 2
is (Where A and Ac are the vertices of the hyperbola)
a. a2e2sec2T–a2 b. a4e2sec2T–a4 c. a2e4sec2T–a2 d. none of these
x 2 y2
3. Normals are drawn to the hyperbola – =1 at points T1 and T2 meeting the conjugate axis
a 2 b2
S
at G1 and G2 respectively. If T1+T2= then CG1 × CG2 is (where C is centre of hyperbola and
2
e is eccentricity of hyperbola)

55
a 2e 4 a 2e 2 a 4e 4
a. b. c. d. none of these
e2 – 1 e2 – 1 e2 – 1
S
4. Let P(asecT, btanT) and Q(asecI, btanI), where T+I= , be two points on the hyperbola
2
x 2 y2
– =1. If (h, k) is the point of intersection of the normals at P and Q, then k is equal to
a 2 b2

a 2  b2 § a 2  b2 · a 2  b2 § a 2  b2 ·
a. b. – ¨¨ ¸¸ c. d. – ¨¨ ¸¸
a © a ¹ b © b ¹
5. If the normal at the point P to the rectangular hyperbola x 2–y2=a2 meet the axes in N and Nc
and C(0, 0) is the centre of the hyperbola then
a. PN=P Nc =PC b. PC=PNzP Nc c. PN=P Nc zPC d. none of these

Answers
1. c 2. c 3. a 4. d
5. a

56
HYPERBOLA - VI
Asymptotes

Equation of Chord of Contact


x 2 y2
Let equation of hyperbola be 2 – 2 =1,
a b
Y
Equation of Chord of contact is
P(x , y )
xx1 yy1 1 1

– 2 –1=0
a2 b
X
or T=0 Q
R

xx1 yy1
where T = – 2 –1
a2 b
Example 1 :
x 2 y2
If tangents of the parabola y =4ax intersect the hyperbola 2 – 2 =1 at P and Q, then locus of
2
a b
point of intersection of tangents at P and Q is
a. a3y2+b4x2=0 b. a3y2+b4x=0 c. a2y2+b2x2=0 d. none of these
Solution :
Let P1(h, k) be the point of intersections of tangents at P and Q. Therefore, the equation of
chord of contact PQ is
xh yk
– =1
a 2 b2
xb 2 h b 2
y= – which touches the parabola y2=4ax
a 2k k

b2 a
?–
k § b2h ·
¨¨ 2 ¸¸
©a k¹

b2 a 3k

k b2h
b4
? Locus of (h, k) is y2=– x
a3
a3y2+b4x=0
Ans. (b)

57
Equation of the chord of the hyperbola whose mid point is given :
A is the mid point of PQ, then equation of chord is
xx1 yy1 x12 y12 Y
– 2 = 2 – 2
a2 b a b
P
ie. T=S1 where
A (x1, y1)
xx1 yy1 X
T = 2 – 2 –1
a b Q
2 2
x
1 y 1
S1 = 2
– –12
a b

Example 2:
The locus of the middle points of the chord of hyperbola 3x2–2y2+4x–6y=0 parallel to y=2x is
a. 3x+4y=4 b. 4x+3y=12 c. 3x–4y=4 d. none of these
Solution :
Let mid point of the chord be (h, k), equation of the chord is T=S 1 i.e.
3xh–2yk+2(x+h)–3(y+k)=3h2–2k2+4h–6k
Ÿ x(3h+2) –y(2k+3) –3h 2+2k2–2h+3k=0
3h  2
Its slope is =2 (slope of y=2x)
2k  3
3h–4k=4
Locus of (h, k) is 3x–4y=4
Ans. (c)
Asymptotes of Hyperbola
An asymptotes of any hyperbola is a straight line which touches in it two points at infinity.
OR
If the length of the perpendicular let fall from a point on a hyperbola
Y
to a straight line tends to zero as the point on the hyperbola moves
to infinity along the hyperbola, then the straight line is called b b
y – x y x
asymptote of the hyperbola. a a

x 2 y2
The equation of two asymptotes of the hyparbola – =1 X
a 2 b2 O

b x y
are y= r x or r =0
a a b
x 2 y2
Pair of asymptotes : – =0
a 2 b2
x 2 y2
1. If b=a, then – =1 reduces to x 2–y2=a2. The asymptotes of rectangular hyperbola
a 2 b2
x2–y2=a2 are y= r x which are at right angles.
2. A hyperbola and its conjugate hyperbola have the same asymptotes.

58
x 2 y2 §b·
– –1 ¨ ¸
3. The angle between the asymptotes of 2 2 =1 is 2 tan © a ¹
a b
4. The asymptotes pass through the centre of the hyperbola.
5. The bisectors of the angle between the asymptotes are the coordinate axes.
x 2 y2
6. Let H = 2 – 2 –1=0
a b
x 2 y2
A = 2 – 2 =0
a b
x 2 y2
and C = – 1 0
a 2 b2
be the equation of the hyperbola, asymptotes and the conjugate hyperbola respectively, then
clearly
C+H=2A
Example 3:
The asymptotes of the hyperbola xy–3y–2x=0 are
a. x–3=0 b. x+y=0 c. y–2=0 d. x–y=0
Solution :
Since equation of a hyperbola and its asymptotes differ in constant terms only. Pair of asymptotes
is given by
xy–3y–2x+c=0
It represents a pair of straight line

1
0 –1
2
1 –3
0
? 2 2 =0
–3
–1 c
2

1§ c 3· § 3·
– ¨ – ¸ – ¨ – ¸ =0
2©2 2¹ © 4¹
c 3 3
–   =0
4 4 4
c=6
? xy–3y–2x+6=0
(x–3) (y–2)=0
Asymptotes are x–3=0 and y–2 0
Ans : a, c

59
Example 4:
The equation of the hyperbola which has 3x–4y+7=0 and 4x+3y+1=0 as its asymptotes and
which passes through the origin is
a. x2–y2=12xy b. 12x2–7xy–12y2+31x+17y=0
c. 12x2–12y2=7xy d. 12x2+7xy–12y2+25x–19y=0
Solution :
Combined equation of the asymptotes is
(3x–4y+7) (4x+3y+1)=0
12x2–7xy–12y2+31x+17y+7=0
Since equation of hyperbola and combined equation of its asymptotes differ by a constant,
therefore equation of hyperola may be
12x2–7xy–12y2+31x+17y+c=0
But it passes through the origin. So c=0
Hence equation of hyperbola is
12x2–7xy–12y2+31x+17y=0
Ans. (b)
Example 5:
The product of the lengths of perpendiculars drawn from any point on the hyperbola x 2–2y2=2
to its asymptotes is
3 2
a. b. 1 c. 2 d.
2 3
Solution :
Given hyperbola is
x 2 y2
– =1
2 1
Let P ( 2 secT, tanT) a point on the hyperbola.
x x
Equation of asymptotes is –y 0 and y 0
2 2
sec T – tan T . sec T  tan T
Product of the lengths of perpendiculars = 1 1
1 1
2 2

sec 2 T – tan 2 T
= 3
2
2
=
3
Ans (d)

60
Exercise 6
The angle between the asymptotes of the hyperbola
x 2 y2
– =1
16 9
§3· § 24 · §4· §4·
a. tan–1 ¨ ¸ b. tan–1 ¨ ¸ c. 2tan–1 ¨ ¸ d. 2tan–1 ¨ ¸
©4¹ © 7 ¹ ©3¹ ©5¹
Solution :
x y x y
Equations of asymptotes are – =0 and + =0
4 3 4 3
3
slope of first asymptote is tanT=
4
§3·
T= tan–1 ¨ ¸
©4¹
§3·
Angle between the asymptotes is 2T=2tan–1 ¨ ¸
©4¹

§ 3 ·
¨ 2u ¸
¨ 4 ¸
=tan–1 ¨ 9 ¸
¨1– ¸
© 16 ¹

§ 24 ·
=tan–1 ¨ ¸
© 7 ¹
Ans. (b)
PRACTICE QUESTIONS
x 2 y2
1. If angle between asymptotes of hyperbola – =1 is 120° and product of perpendiculars
a 2 b2
drawn from foci upon its any tangent is 9, then locus of point of intersection of perpendicular
tangents of the hyperbola can be
a. x2+y2=6 b. x2+y2=9 c. x2+y2=3 d. x2+y2=18
2. For a hyperbola whose centre is at (1,2) and asymptotes are parallel to lines 2x+3y=0 and
x+2y=1, then equation of hyperbola passing through (2, 4) is
a. (2x+3y–5) (x+2y–8)=40 b. (2x+3y–8) (x+2y–5)=40
c. (2x+3y–8) (x+2y–5)=30 d. none of these
x 2 y2 x 2 y2
3. Asymptotes of the hyperbola 2 – 2 =1and 2 – 2 =1are perpendicular to each other then,
a1 b1 a 2 b2

a1 b1
a. =
a 2 b2 b. a1a2=b 1b 2 c. a1–a2=b 1–b 2 d. a1a2+b1b2=0

61
4. If S=0 be the equation of the hyperbola x2+4xy+3y2–4x+2y+1=0, then the value of k for which
S+k=0 represents its asymptotes is
a. –22 b. 18 c. –16 d. 20
5. A hyperbola passes through (2, 3) and has asymptotes 3x–4y+5=0 and 12x+5y–40=0, then
the equation of its transverse axis is
a. 77x–21y–265=0 b. 21x–77y–265=0
c. 21x–77y–265=0 d. 21x+77y–265=0
6. The combined equation of the asymptotes of the hyperbola 2x 2+5xy+2y2+4x+5y=0 is
a. 2x2+5xy+2y2+7=0 b. 2x2+5xy+2y2+4x+5y+7=0
c. 2x2+5xy+2y2+4x+5y+2=0 d. None of these
7. The asymptotes of the hyperbola xy+hx+ky=0 are
a. x+h=0 and y+k=0 b. x+h=0 and y–k=0
c. x–k=0 and y–h=0 b. x+k=0 and y+h=0
8. If foci of hyperbola lie on y=x and one of the asymptote is y=2x, then equation of the hyperbola,
given that it passes through (5, 4) is
a. 2x2–2y+5xy+5=0 b. 2x 2+2y2–5xy+10=0
c. x2–y2–xy+5=0 d. None of these
Linked comprehension type (for problems 9 - 11)
In hyperbola portion of tangent intercept between asymptotes is bisected at the point of contact.
Consider a hyperbola whose centre is at origin. A line x+y=2 touches this hyperbola at P(1, 1)
and intersects the asymptotes at A and B such that AB=6 2 units.
9. Equation of asymptotes are
a. 2x2+2y2–5xy=0 b. 2x2+5xy+2y2=0
c. 3x2+6xy+4y2=0 d. None of these
§ 7·
10. Equation of tangent to the hyperbola at ¨ – 1, ¸ is
© 2¹
a. 3x+2y=4 b. 3x+4y=11 c. 5x+2y=2 d. none of these
11. Angle subtended by AB at centre of the hyperbola is
§4· §2· §3·
–1 ¨ ¸ –1 ¨ ¸
a. tan © 3 ¹ b. tan © 3 ¹ c. tan © 4 ¸¹
–1 ¨
d. none of these

Answers
1. d 2. b 3. d 4. a
5. d 6. c 7. c 8- b
9. b 10.. a 11. c

62
HYPERBOLA - VII
Rectangular Hyperbola
Rectangular Hyperbola:
A hyperbola whose asymptotes include a right angle is said to be rectangular hyperbola.
OR
If the lengths of transverse and conjugate axes of any hyperbola be equal it is called rectangular
or equilateral hyperbola.
According to first definition
Y
§b· S
2tan–1 ¨ ¸ =
©a¹ 2 xy=c2
y=–x y=x
§ a a ·
b S ¨ ,
© 2 2¹
¸

tan–1 = A x2–y2=a2
a 4
b S X
= tan O (a, 0)
a 4
b=a
x2 y2
then – = 1 becomes x 2–y2 = a2
a2 b2
According to second definition
x2 y2
When a = b, – 2 = 1 becomes x 2–y2 = a2
a2 b

b2
Eccentricity e = 1  = 2
a2
Then asymptotes of x 2–y2 = a2 are x+y = 0 and x–y = 0 . Each of these two asymptotes is
inclined at an angle of 45° with the transverse axis. So, if we rotate the coordinate axes through
S
an angle of keeping the origin fixed, then the axes coincide with the asymptotes of the
4
hyperbola.
Now equation of asymptotes of new hyperbola is x y = 0
Then equation of hyperbola is xy = k (constant)
§ a a ·
The hyperbola passes through the point ¨ , ¸
© 2 2¹

a2
? k=
2
a2 a2
Then equation of hyperbola is xy = or xy = c where c =
2 2
2 2
If the asymptotes of a rectangular hyperbola are x = a, y = b, then its equation is (x–a)(y–b) = c2
xy = c2

63
1 Asymptotes : x = 0, y = 0
2 Transverse axis : y = x
Conjugate axis : y = – x
3 Vertices A(c,c), Ac (–c,–c)
4
Foci : S c 2 , c 2 , Sc – c 2 ,– c 2
5 Length of transverse axis = A Ac = 2 2c
6 Equation of auxiliary circle x2+y2 = 2c2
7 Equation of director circle x 2+y2 = 0
8 x2–y2 = a2 and xy = c2 intersect at right angles
Properties of Rectangular Hyperbola
1 Eccentricity of rectangular hyperbola is 2.
c
2 Since x = ct, y = satisfies xy = c2
t
§ c· c
? (x,y) = ¨ ct , t ¸ (t z 0) is called a ‘t’ point on the rectangular hyperbola. The x = ct, y =
© ¹ t
represents its parametric equation with parameter ‘t’
§ c· § c ·
3 Equation of chord joining P ¨¨ ct1 , ¸¸ and Q ¨¨ ct 2 , ¸¸ is
© t1 ¹ © t2 ¹
x+y t1 t2 – c(t1+t2) = 0
1
Slope of chord = – t t
1 2

4 Equation of tangent at (x1,y1) is xy1+yx1 = 2c2


x
5 Equation of tangent at t is +yt = 2c
t
1
Slope of tangent = –
t2
6 Equation of normal at (x1,y1) is xx1–yy1 = x12–y12
Equation of normal at t is xt3–yt–ct4+c = 0
Slope of normal = t2
7 Point of intersection of tangents at t1 and t2 is
§ 2ct1t 2 2c ·
¨¨ , ¸¸
© t 1  t 2 t1  t 2 ¹
8 Point of intersection of normal at t1 and t2 is
§ ct 1 t 2 ( t 12  t 1 t 2  t 2 2 ) – c ct 13 t 2 3  c( t 12  t 1 t 2  t 2 2 ) ·
¨ , ¸
¨ t .t ( t  t ) t .t ( t  t ) ¸
© 1 2 1 2 1 2 1 2 ¹

64
Example 1
If the normal at the point t1 to the rectangular hyperbola xy = c2 meets it again at the point t2 then
(a) t1t2 = – 1 (b) t13t2 = – 1 (c) t1t23 = – 1 (d) t12t22 = – 1
Solution
§ c·
Equation of normal at ¨¨ ct1 , t ¸¸ to the hyperbola xy = c2 is
© 1¹

xt1 –yt1–ct1 +c = 0
3 4

§ c·
But this passes through ¨¨ ct 2 , t ¸¸ then
© 2 ¹

c
ct2t13– t t1–ct14+c = 0
2

t1
t2t13– t –t14+1 = 0
2

t t –t –t t +t2 = 0
2 3
2 1
4
1 1 2
t t (t –t )+1(t2–t1) = 0
3
1 2 2 1
(t –t ) (t t +1) = 0
2 1
3
1 2
Ÿt t = – 1
3
1 2
(' t1 z t2)
Ans (b)
Example 2
A triangle has its vertices on a rectangular hyperbola xy = c2
The orthocenter of the triangle lies on
(a) x2+y2 = c2 (b) x2–y2 = c2 (c) xy = c2 (d) None of these
Solution
§ c· § c· § c·
Let A ¨¨ ct1 , t ¸¸ B ¨¨ ct 2 , t ¸¸ C ¨¨ ct 3 , t ¸¸ be the vertices of atriangle lies on xy = c2
© 1¹ © 2 ¹ © 3 ¹

§c c· Y
¨t t ¸ 1 C
Now slope of BC = © 3 2 ¹ = – t t
ct3  ct2 2 3
E
D
B
Hence slope of AD = t2t3 X
Equation of AD is A
c
y– t = t2t3 (x–ct1)
1

t1y–c = xt1t2t3–ct12t2t3______________(1)
Similarly
1
slope of AC = – t t
1 3

65
slope of BE = t1t3
Equation of BE is t2y–c = xt1t2t3–ct22t1t3____________(2)
c
Solving (1) and (2) we get y = –ct1t2t3 & x = – t t t
1 2 3

Which satisfies xy = c2
Therefore orthocentre lies on xy = c2
Ans (c)
Example 3
If PN is the perpendicular from a point on a rectangular hyperbola xy = c2 to its asymptotes then
locus of the midpoint of PN is
c2 c2 Y
(a) xy = (b) xy = (c) xy = 2c2 (d) None of these
2 4
Solution P
Let P(x1, y1) be a point on xy = c2 y1 Q
X
Le Q(h,k) be the Midpoint of PN then x1 = h and k = N
2
(x ,1y )1 lies on xy = c2 Ÿ h(2k) = c2 Locus of

(h,k) is 2xy = c2
Ans (a)
Example 4
PQ and RS are two perpendicular chords of the rectangular hyperbola xy = c2. If O is the centre of
the rectangular hyperbola, then product of the slopes of OP, OQ, OR and OS is
(a) –1 (b) –2 (c) 1 (d) 2
Solution
§ c·
Let coordinates of P, Q, R, S be ¨¨ ct i , ¸¸ , i = 1, 2, 3, 4
© ti ¹

Now PQ A RS Ÿ m PQ u m RS = –1

c c c c
– –
t 2 t1 t4 t3
Ÿ u =–1
ct 2 – ct1 ct 4 – ct 3

1 –1
Ÿ– t t u t t = – 1
1 2 3 4

Ÿ t1t2t3t4 = – 1
c
1
Now slope of OP = t1 = 2
ct1 t1

66
1
Similarly slope of OQ = 2
t2
1
Similarly slope of OR = t 2
3

1
Similarly slope of OS = 2
t4
1 1
Product of their slopes = t 2 .t 2 .t 2 .t 2 = =1
1 2 3 4 (–1) 2
Ans (c)
Example 5
The angle between the rectangular hyperbolas
(y–mx) (my+x) = a2 and (m2–1) (y2–x2) + 4mxy = b2 is
S S S §b·
(a) (b) (c) (d) tan–1 ¨ ¸
2 3 4 ©a¹
Solution
(y–mx) (my+x) = a2
§ dy · § dy ·
¨ – m ¸ (my+x) + ¨ m 1¸ (y–mx) = 0
© dx ¹ © dx ¹
dy dy
(my+x) + m(y–mx) = m(my+x) – (y–mx)
dx dx
dy m 2 y  mx – y  mx m 2 y  2mx – y
= = = m1________(1)
dx my  x  my – m 2 x – m 2 x  x  2my
For another hyperbola
(m2–1) (y2–x2) + 4mxy = b2
§ dy · § dy ·
(m2–1) ¨ 2 y – 2 x ¸ + 4m ¨ y  x ¸ = 0
© dx ¹ © dx ¹
dy dy
(m2–1) y – x (m2–1)+2my+2mx =0
dx dx
dy
(m2y–y+2mx) = m2x–x–2my
dx

dy m 2 x – x – 2my
= 2 = m2 _________(2)
dx m y  2mx – y
Now m1×m2 = – 1

67
S
? angle =
2
Ans (a)
Example 6
The family of the curves which intersect the family of rectangular hyperbola xy = c2 orthogonally is
(a) family of circle b) family of parabola
(c) family of ellipse (d) family of hyperbola
Solution
xy = c2
Differentiate w.r.t. x
dy
y+x =0
dx
dy dx
Replace by – , we get
dx dy
dx
y–x =0
dy
y dy – xdx = 0
y2–x2 = k (where k is paramenter)
family of hyperbola.
Ans (d)
PRACTICE QUESTIONS
1 The coordinates of the foci of the rectangular hyperbola xy = c2 is
§ c c · § c c·
(a) ( r c, r c) (b) ¨r
© 2
,r

¸ (c)
r c 2 ,r c 2 (d) ¨ r ,r ¸
© 2 2¹
2 The equation of directories of the rectangular hyperbola xy = c2 is
c
(a) x+y = r c (b) x+y = r c c (c) x+y = r (d) xr y=0
c
3 If the line ax+by+c = 0 is a normal to the hyperbola xy = 1, then
(a) a>0, b>0 (b) a<0, b<0 (c) a>o, b<0 (d) a<o, b>0
4 Consider the set of hyperbolas xy = k, k  R. Let e1, be the eccentricity when k = 4 and e2 be the
eccentricity when k= 9, then e1–e2 is equal to
3
(a) 0 (b) 1 (c) (d) 2
2
5 If chords of the hyperbola x2–y2 = a2 touch the parabola y2 = 4ax then the locus of the middle points
of these chords in the crane
(a) y2 = x3 (b) y2(x–a) = x2 (c) y3(x–a)=x2 (d) y2(x–a) = x3
6 If the tangent and normal to a rectangular hyperbola xy = 4 cut off intercepts a1 and a2 on one axis
and b1 and b2 on the other, then a1a2+b1b2 is
(a) –4 (b) 0 (c) 1 (d) 4

68
7 The points of intersection of the cranes whose parametric equations are x = t2+1, y = 2t and x = 25,
2
y= is given by
5
(a) (1,2) (b) (2,2) (c) (1,–3) (d) (–2,4)
8 Number of maximum tangents from any point to the hyperbola xy = c2 is
(a) 1 (b) 2 (c) 3 (d) 4
9 The length of the latus rectum of the hyperbola xy–3x–3y+7 = 0 is
(a) 2 (b) 4 (c) 2 2 (d) None of these
10 If x = 9 is the chord of contact of the hyperbola x –y = 9, then the equation of the corresponding
2 2

pair of tangent is
(a) 9x2–8y2–18x+9 = 0 (b) 9x2–8y2–18x–9 = 0
(c) 9x2–8y2+18x+9 = 0 (d) 9x2–8y2+18x–9 = 0
11 The equation of the chord joining two points (x 1,y1) and (x2,y2) on the rectangular hyperbola xy =
c2 is
x y x y
(a) x1 – x 2 + y1 – y 2 = 1 (b) y1  y 2 + x1  x 2 = 1
x y x y
(c) x1  x 2 + y1  y 2 = 1 (d) y1 – y 2 + x1 – x 2 = 1
Answers :
1. c 2. b 3. c, d 4. a 5. d
6. b 7. b 8. b 9. b 10. a 11. c

69
HYPERBOLA - VIII
Practice Problems
Practice Problems
1 Equation of conjugate axis of hyperbola xy–3y–4x+7 = 0 is
(a) x+y = 7 (b) x+y = 3 (c) x–y = 7 (d) None of these
Solution:
xy–3y–4x+7 = 0 Y
xy–3y–4x+12 = 5 x=3
(x–3) (y–4) = 5
Equation of asymptotes are x–3 = 0 and y-4 = 0
Since the hyperbola is rectangular hyperbola, axes y=4
(3, 4)
are bisectors of asymptotes
X
Hence their slopes are r 1
? Equation of conjugate axis is
y–4 = – 1(x–3)
x+y = 7
Ans (A)
2 If S1 and S2 are the foci of the hyperbola whose transverse axis length is 4 and conjugate axis
length is 6, S3 and S4 are the foci of the conjugate hyperbola, then the area of the quadrilateral
S1S3S2S4 is
(a) 156 (b) 36 (c) 26 (d) None of these
Solution :
S1S3S2S4 forms a square.
S3
1
So required area = 4×area of ' S1 O S3 = 4× ae × be1
2
S2
= 2 abee1 = 2. 2. 3. e e1
S1
o = 12ee1
9 13 9 13
S4 Now e = 1 = & e1 = 1 =
4 2 4 2

13 13
Hence area = 12× × = 26 sq.units
2 2
Ans (c)
3 The ellipse 4x2+9y2 = 36 and the hyperbola a2x2–y2 = 4 intersect at right angles then the equation
of the circle through the points of intersection of two conic is
(a) x2+y2 = 25 (b) 5(x2+y2 )+3x+4y = 0
(c) 5(x2+y2)–3x–4y = 0 (d) (x2+y2) = 5
Solution:
Since ellipse and hyperbola intersect orthogonally, they are confocal.
4 5
e = 1– =
9 3

70

foci of ellipse r 5 ,0
(ae)2 = a2+b2
4
5= + 4Ÿ a = 2 Y
a2
Let point of intersection in the first quadrant be P(x1, y1).
P
P lies on both the curves.
4x12+9y12 = 36 and 4x12–y12 = 4 Xc
O
X

Adding these two, we get 8x12+8y12 = 40


x12+y12 = 5
Equation of circle is x +y = 5
2 2

x2 y2
4 If e is the eccentricity of the hyperbola – = 1 and 2 T is angle between the asymptotes
a2 b2
then cos T =
1 1– e 1 e
(a) (b) (c) (d) None of these
e e e
Solution:

b2
e = 1
a2

§b· b
we know 2 T = 2tan–1 ¨ a ¸ Ÿ tan T =
© ¹ a
1
e = 1  tan T 2 = sec T Ÿ cos T = .
e
Ans (a)
5 From a point p (1,2) pair of tangents are drawn to a hyperbola in which one tangent to each
arm of hyperbola. Equation of asymptotes of hyperbola are 3 x–y+5 = 0 and 3 x+y–1 = 0
then eccentricity of hyperbola is
2
(a) 3 (b) (c) 2 (d) None of these
3
Solution:
Equation of asymptotes are Y

3 x–y+5 = 0
P(1,2)
– 3 x–y+1 = 0
? a1a2+b1b2 = –3+1<0 X
? origin lies in acute angle and P(1,2) lies in obtuse angle.
? e = sec T where 2 T is the angle between asymptotes.
S S
2T = Ÿ T=
3 6

71
S 2
e = sec =
6 3
Ans (b)
e ec
6 If a variable line has its intercepts on the coordinate axes e, ec where , are the eccentricities
2 2
of a hyperbola and its conjugate hyperbola, then the line always touches the circle x2+y2 = r2,
where r =
(a) 4 (b) 3
(c) 2 (d) Can not be decided
Solution
4 4 e 2 (e c) 2
Now + =1 Ÿ 4=
e2 (e c) 2 e 2  (e c) 2
Line passing through the points (e,0) and (0, ec ) is ec x+ey = e ec
It is a tangent to the circle x2+y2 = r2

ee c
? =r
e 2  (e c) 2
2=r
Ans (c)
x2 y2
7 If angle between asymptotes of hyperbola – = 1 is 120° and product of perpendiculars
a2 b2
drawn from foci upon its any tangent is 9, then locus of point of intersection of perpendicular
tangents of the hyperbola can be
(a) x2+y2 = 18 (b) x2+y2 = 6 (c) x2+y2 = 9 (d) x2+y2 = 3
Solution
b b 1
2tan–1 = 60° Ÿ =
a a 3
b2 = 9 ? a2 = 27
Required locus is director circle i.e. x2+y2 = 27 – 9
x2+y2 = 18
b
If = tan 60° = 3
a
a2 = 3
Then equation of director circle is x2+y2 = 3 – 9 = – 6 which is not possible.
Ans (a)
8 The equation of the transverse axis of the hyperbola
(x–3)2+(y+1)2 = (4x+3y)2 is
(a) 3x–4y = 0 (b) 4x+3y = 0 (c) 3x–4y = 13 (d) 4x+3y = 9

72
Solution
(x–3)2+(y+1)2 = (4x+3y)2
2
§ 4 x  3y ·
(x–3)2+(y+1)2= 25 ¨ ¸
© 5 ¹
PS = 5 PM
Directrix is 4x+3y = 0 and focus is (3,–1)
3
Equation of transverse axis is y+1 = (x–3)
4
3x–4y = 13.
Ans (c).
PRACTICE QUESTIONS
1 The equation of common tangents to the parabola y2 = 8x and hyperbola 3x2–y2 = 3 is
(a) x r 2y–1 = 0 (b) x r 2y+1 = 0 (c) 2x r y+1 = 0 (d) 2x r y–1 = 0
x 9
2 A tangent to the hyperbola y = passing through the origin is
x 5
(a) x–2y =0 (b) 5x–y = 0 (c) 5x+y = 0 (d) x+225y =0
3 The equation of the common tangent to the curves y2 = 8x and xy = – 1 is
(a) y = x+2 (b) y = 2x+1 (c) 2y = x+8 (d) 3y = 9x+2
x2 y2
4 Let PQ be a double ordinate of the hyperbola 2 – 2 = 1. If O be the centre of the
a b
hyperbola and OPQ is an equilateral triangle, then eccentricity e is
2
(a) > 3 (b) >2 (c) > (d) None of these
3
5 The difference between the length 2a of the transverse axis of a hyperbola of eccentricity e
and the length of its latus rectum is
(a) a(2e2–1) (b) 2a(e2–1) (c) 2a|3–e2| (d) 2a|2–e2|
x2 y2 y2 x2
6 The slopes of common tangents to the hyperbolas – = 1 and – = 1 are
9 16 9 16
(a) r2 (b) r 2 (c) r1 (d) None of these

y2 x2 b
7 The two conics 2 – 2 = 1 and y2 = – x intersect ifff
b a a
1 1
(a) 0<b d (b) 0<a d (c) b2 < a2 (d) b2 > a2
2 2
x2 y2
8 The point on the hyperbola – = 1 which is nearest to the line 3x+2y+1 = 0 is
24 18
(a) (–6,3) (b) (3,–6) (c) (–6,–3) (d) (6,3)

73
9 If (asec T , btan T ) and (asec I , btan I ) be the coordinates of the ends of a focal chord of the
x2 y2 T I
hyperbola 2 – 2 = 1, then tan tan =
a b 2 2
1 e 1– e e –1
(a) (b) (c) (d) None of these
1– e 1 e e 1
10 If the latus rectum of a hyperbola through one focus subtends 60° angle at the other focus, then its
eccentricity e is
(a) 2 (b) 3 (c) 5 (d) 6

Answers
1. c 2. b 3. a 4. c
5. d 6. c 7. a 8. d
9. b 10. b

74
COMPLEX NUMBER - I
Algebra of Complex Number

Our motive for the introduction of complex numbers is to make every algebraic equation solvable.
Let us consider the equation z2+4 = 0. This equation has no solution in the set of real numbers.
There is no real no. x whose square is –4. In order to remedy this situation, a new kind of
numbers were introduced and were given the name complex nos. Eular was the first to introduce
the symbol i for – 1 with the property i2 = –1.
i is also called the symbol as the imaginary unit.
i is called an imaginary number.

Powers of i
(i) i0 1 i1 i i2 –1 i3 i 2 .i –i
i4 1 i5 i 4 .i i i6 i 4 .i 2 –1 i7 i 4 .i 3 –i
8 4 4 9 8
i i .i 1 i i .i i i10 –1 i11 –i
-------------------------------------------------------------------------------------------------
-------------------------------------------------------------------------------------------------
i4p 1 i 4 p 1 i i 4p2 –1 i 4p3 –i
1 i 1 1
i –4 1 i –3 i i–2 –1 i –1 –i
i3 i4 i2 i

Complex numbers
An expression of the form x+iy, where x & y are real numbers and i is a symbol, is called a
complex number and usually denoted by z.

z = (x) + i (y)

Real part of z or Re (z) Imaginary part z or Im (z)


Note: If Re z = 0, then z is Note: If Imz = 0, then z is
called purely imaginary called purely real.
* Every real number is a complex number.
* 0 is both purely real and purely imaginary number.
* A complex is an imaginary no. if and only if its imaginary part is non-zero. Here real part
may or may not be zero. 7+11i is an imaginary number but not purely imaginary.

Addition of complex number


Let z1 = a1+ib1 & z2 = a2 + ib2
be two complex number
Then their sum z1 +z2 is defined as the complex number (a1+a2) + i(b1+b2)

75
Multiplication of Complex Number
Let z1 = a1+ ib1 & z2 = a2 + ib2 be two complex number. Then the multiplication of z1 with z2 is
denoted by z1 z2 and is defined as (a 1a2–b1b2) + i(a1a2+ b1b2 ).

Division of Complex Number


The division of a complex number z1 by a non-zero complex z2 is defined as the multiplicaiton of
z1
z1 by the multiplication inverse of z2 and is denoted by z
2

z1 a1  ib1 a1  ib1 a 2 – ib 2
z2 a 2  ib 2 a 2  ib 2 a 2 – ib 2
a1a 2  b1b 2 i a 2a1 – b1b2
= + a 22  b22
a b
2
2
2
2

Conjugate of Z
Let z = a + ib be a complex number. Then the conjugate of z is denoted by z and is equal to a–ib
thus z = a + ib
Ÿ z = a – ib
e.g. if z = 3 +4i then z = 3 – 4i

Modulus of Z
The modulus of a complex number x + iy is denoted by x  iy and is denoted as x  iy =

x 2  y 2 = non-negative square root of x2 + y2 e.g. z = 3 – 4i, then z = 3 2  –4 2 5.

Equality of complex number


Two complex nos z1 = x1 + iy1 & z2 = x2 + iy2 are said to be equal if and only if x1 = x2 & y1 = y2
i.e. Z1 = Z2 Ÿ Re(z1) = Re (z2) & Im (z1) = Im (z2).

Example: 1 Evaluate
5 3
 ^ n
 n 1
 n 2
 n 3


1
G.E = i–(4x13+1) + [in (1+i+i2+i3) +4] 2 .i

ª 1 1
º
^
« i  i  1  i   1   i  4
n
` 2
.i »
¬ ¼
ª 1 1
º
« i  ^i (0 )  4 ` .i »
n 2

¬ ¼
ª 1 1
º
« i  ( 4 ) .i »
2

¬ ¼
ª 1
Ÿ ¬i  2i¼ º
ª1 º i
Ÿ «  2i» Ÿ 2  2i Ÿ i  2i i
¬i ¼ i

76
Example: 2 Show that the polynomial x 4 p  x 4q  1  x 4r  2  x 4 s  3 is divisible by
x3+x 2+x+1 ,where p,q,r,s  N.
Solution: Let f(x) = x 4 p  x 4q  1  x 4r  2  x 4 s  3
Now x3+x2+x+1 = (x2+1)(x+1) =(x+i)(x-i)(x+1)
4 p 4q  1  i 4 r  2  i 4 s  3
f(i) = i  i = 1+ i+i2+i3 = 1+ i -1- i =0
4p
( i)  (i)  (i)  (i)
( 4 q 1) (4r 2) ( 4 s  3)

f(-i) = = +1+(– i) (+1) + (–i) 2 + (–i) 3 = 1-.i –1+i=0


f(–1) = 0
4p 4 q  1  x 4r  2  x 4 s  3
Thus by division theorem x3 +x2 +x +1 is factor of x  x .

1
Exercise: 3 Express in the form x+iy
1 – cos lj  2i sin lj
1
1
1 – cos lj  2i sin lj
= lj lj
2sin 2  4sin cos
2 2
lj
2
i
§ lj lj· lj lj
¨ sin  2 i cos ¸  2 i cos
sin
© 2 2¹ 2 2
= ljª lj ljºª lj ljº ljª 2lj ljº
2 sin « sin  2 i cos » « sin  2 i cos » 2 sin « sin  4 cos 2 »
2¬ 2 2¼¬ 2 2¼ 2¬ 2 2¼

§ lj lj·
¨ sin  2 i cos ¸
© 2 2¹
= ljª
sin « 2 sin 2 lj ljº
 8 cos 2 »
2¬ 2 2¼

§ lj lj·
¨ sin  2 i cos ¸
© 2 2¹
= lj
sin >1  cos lj  4  4 cos lj @
2

§ lj lj·
¨ sin  2 i cos ¸
© 2 2¹
= lj
sin 5  3 cos lj
2

77
lj
= 1
2 cot
2
i

5  3 cos lj 5  3 cos lj

Exercise: 4 Find the multiplicative inverse of complex no 3+2i


Let z = 3+2i

3  2i 3  2i 3 2
 i
3  2i 3  2i 94 13 13

PRACTICE QUESTIONS

1. If a < 0, b > 0, then a . b is equal to


a. – ab b. – a .bi c. ab d. none of these

2. The value of the sum , where i – 1 ,is

a. i b. i –1 c. –i d. 0
n
§1 i ·
3. The smallest positive integral value of n of which ¨ ¸ is purely imaginary with positive
©1 i ¹
imaginary part, is
a. 1 b. 3 c. 5 d. none of these
4. If n is an odd integer, i – 1 , then (1+i)6n +(1–i)6n is equal to
a. 0 b. 2 c. –2 d. none of these
3  2i sin ș
5. If is a real number and 0 < ș < 2ʌ , then ș =
1 – 2i sin ș
ʌ ʌ ʌ
a. ʌ b. c. d.
2 2 6
1  iz
6. If b+ic=(1+a)z and a2 + b2 + c2 =1, then
1  iz
a  ib a  ib a  ib a  ib
a. b. c. d.
1 c 1 c 1 c 1 c
1 x y
7. If x  iy 3 = a + ib then  =
a b
a. 0 b. 1 c. –1 d. none of these

78
8. If (a + ib)5 = Į  iȕ , then (b+ai)5 is equal to
a. ȕ  iĮ b. Į – iȕ c. ȕ – iĮ d. – Į – iȕ
9. The set of values of a  R for which x + i(a–1)x+5 = 0 will have a pair of conjugate complex
2

roots is
a. R b. {1}
c. {a : . a2 – 2a  21 ! 0} d. none of these
1 – ix
10. The relation between the real numbers a and b, which satisfy the equation =a – ib, for some
1  ix
real value of x, is
a–b
a. (a – b) (a + b) = 1 b.
ab
c. a2+b2 = 1 d. none of these

Answers :
1.b 2.b 3.b 4.a 5.a 6.d 7.d 8.a 9.b 10.c

79
COMPLEX NUMBER - II
Square Root & Polar Form
Square root of a complex number
1. If the square root of a+ib is to evaluated let a  ib x  iy , x , yR
2. Square both sides and equate real and imaginary part which will give value of (x 2–y2) and 2xy.
3. Find x 2+y2 by
4. From x 2 – y2 & x2 + y2, we get the value of x.
5. Put x in xy, we obtained corresponding value of y.
6. Now a  ib x  iy

Direct Formula
The square root of z = a + ib are
ª z a z –a º
r« i »
«¬ 2 2 »¼ for b > 0

ª z a z –aº
& r« –i » for b 0
¬« 2 2 ¼»
Note:
i. a b ab is ture only when at least-one of a & b is non-negative.
ii. The square root of Ȧ are r Ȧ 2
iii. The square root of Z  are r Ȧ

iv. The square root of i are

v. The square root of – i are


Example: Find the square root of –7–24i
Solution: Here a =-7 & b = -24 0

z – 7 2  – 24 2 25
Now by using formula,
ª z a z –a º ª 25 – 7 25  7 º
– 7 – 24i = r « – i » r« –i » r 3 – 4i
«¬ 2 2 » ¬ 2 2 ¼
¼

80
Geometrical Representation of a Point is:
Coordinate Geometry Vector Complex Number

P(a,b) P (z=x+iy)
P a î  b ˆj r̂ r z x 2  y2
b r̂
y
bĵ T x
a
x Complex plane, Gaussian plane
Cartesian Plane Cartesian Plane or Argand plane

Modulus
The modulus of a complex number x+iy is denoted by x  iy = x 2  y 2 = non negative square
root of x 2+y2
e.g. z = 3 – 4i, then z = 3 2  – 4 2 5

Argument or Amplitude of a complex number

z = x+iy P(x,y)
r x y
2 2

y
T
x

x y
cosș & sin ș
x 2  y2 x 2  y2

x x
cos ș & sin ș
r r
x = rcos ș & x = rsin ș
The argument of a complex number z = x+iy is the value of ș which satisfies the two equations
x y
cos ș and sin ș
x y2 2
x 2  y2
Argument of z is denoted by argument z or amplitude z.
There will be infinite number of values of ș satisfying the above equations and all these values
will be the argument of z but usually we take only that value of ș to which 0 d lj  2S
Example: z = –1 –i here x = –1, y = –1
x 1 3ʌ 5 ʌ
cos ș – ș ,
x y2 2
2 4 4

81
y 1 5ʌ 7 ʌ
sin ș – ș ,
x y2 2
2 4 4

? since ș satisfies both the equation
4
5ʌ 5ʌ
? argument z = and general value of argument z = 2nʌ  , where n = 0, r 1,r2 .....
4 4

Another way of finding argument of a complex number


Working Rule
y ʌ
i. Take tan ș and from this find the value of ș lying between 0 and
x 2
ii. Then find in which quadrant the point z lies.
iii. Argument of z will be ș , ʌ – ș, ʌ  ș or 2ʌ  ș according as the point z lies in the 1st, 2nd,
3rd or 4th quadrants.
Exercise: Let z = –1 –i here x = –1, y = –1
y –1 ʌ
tan ș 1 tan
x –1 4
ʌ ʌ
Ÿ ș (between 0 and )
4 2
Since the point z = –1 –i { (–1,–1) lies in 3rd quadrant
ʌ 5ʌ
? argument = ʌ  ș = ș
4 4
Principal vaule of the argument: There are infinite values of ș satisfying the equation
x y
cos ș and sin ș
x y
2 2
x  y2
2

But there will be a unique value of ș such that – ʌ  ș d ʌ . The value of argument ș satisfying the
inequality – ʌ  ș d ʌ is called principal value of argument.
For above example
5ʌ – 3ʌ
Principal value of argument = – 2ʌ
4 4
Note: If argument ! ʌ , subtract 2ʌ from it to get the principal value of argument and if ar-
gument d – ʌ , add 2ʌ to it, to get the principal value of argument.

Polar form of a complex Number


z = x+iy
= r cos ș  i sin ș is called the polar form of complex Number..

82
For above example
r z 2

ª 5ʌ 5ʌ º
Polar form of z is 2 «cos  i sin »
¬ 4 4¼

ª § – 3ʌ · § – 3ʌ · º
or 2 «cos¨ ¸  i sin ¨ ¸
¬ © 4 ¹ © 4 ¹ »¼
Complex Number

Cartesian Representation Polar Repersentation


z = a+ib z = r cos ș  i sin ș
= r cos ș  i sin ș
= reiT (using euler’s formula)
Algebraic operations
Cartesian form Polar form

z = re iT ,w= se ˳
i
Complex No. z = a + ib, w= c + id

Addition z +w= (a + c) + i (b + d) z +w= r cos ș  s cos ˳ +


i r sin ș  s sin ˳

Subtraction z –w= (a – c) + i (b – d) z – w= r cos ș – s cos ˳ +


i r sin ș – s sin ˳

Multiplicaiton zw= (ac – bd) + i zw = rsei(T+I)


(ad + bc)

z a  id c – id z r i T – M
Division e
Z c  id c – id w s
ac  bd bc – ad
= i 2
c d
2 2
c  d2

Conjugate of a Complex Number


The conjugate of a complex number z =x + iy is denoted by z = x  iy and is defined as
iT  iT
z = xiy and if z = r e (Eular’s form), then z = r e

83
Equality of Complex Number
Two complex numbers z1 =x1 + iy1 and z2 = x2 +iy2 are said to be equal if and only if x 1 = x 2 & y1 =
+y2
i.e. z1 = z2 , then Re(z1) =Re(z2) & Im(z1) = Im(z2)

Properties of Conjugate
1. z –z z –z
2. z z
ʌ
3. >arg z – arg z @ = arg (–z) – arg – z = 2
2
4. zz z
5. z1 r z 2 z1 r z 2
6. z z
n n

7. z1.z 2 z1.z 2

§ z1 · z1
8. ¨¨ ¸¸
© z2 ¹ z2
Example-Write the following in Polar form
(1)z =1 + 3 i
(2) z =  1  3i
(3) z =  1  3i
(4) z = 1  3i
Solution : z = 1 + 3 i { (1, 3 )

r = z = (1)
2
 3 =2
2

y 3 S
Then tan D = x 3 tan
1 3
(1) As z lies in 1st quadrant
S
so, argz = T = D =
3
Therefore polar form of z is
S S
2(cos + i sin )
3 3

84
2S
(2) z lies in 2nd quadrant so argz = T = S  D =
3
Therefore polar form of z is
2S 2S § 2S 2S ·
2(cos + i sin ) Ÿ 2 ¨ cos 3  i sin 3 ¸
3 3 © ¹
4S
(3) z lies in 3rd quadrant so argz = T = S  D =
3
Therefore polar form of z is
4S 4S
2(cos + i sin )
3 3
S
(4) lies in 4th quadrant so argz = T =  D =
3
Therefore polar form of z is
S S
2(cos( ) + i sin( ))
3 3

PRACTICE QUESTIONS
1. Im(z) is equal to
1
z  z i z  z 1
z  z i
a. b. c. d. none of these
2 2i 2
2. If z1 = 9y2–4–10ix, z2 = 8y2–20i where z1 z 2 then z = x+iy is equal to
a. –2+2i b. – 2 r 2i c. – 2i d. none of these
3. If z is a complex number satisfying the relation z  1 z  2 1  i , then z is
1 1 1 1
a. 1  4i b. 3  4i c. 1 – 4i d. 3 – 4i
2 2 2 2
4. For a complex number z, the minimum value of z  2 – z is
a. 1 b. 2 c. 3 d. none of these
1 z
5. If z 1 , then is equal to
1 z
a. z b. z c. z+ z d. none of these
6. If z1 – 1  1 , z 2 – 2  2 , z 3 – 3  3 , then z1  z 2  z 3
a. is less than 6 b. is more than 3
c. is less than 12 d. is between 6 and 12
7. If z1,z2 are two non-zero complex numbers such that z1  z 2 z1  z 2 , then complex arg ( z 1 is
z2
equal to
ʌ
a. ʌ b. –ʌ c. 0 d.
2

85
3 i
8. If z = , then the fundamental argument of z is
3 –i
–ʌ ʌ ʌ
a. b. c. d. none of these
3 3 6
9. If z = x+iy satisfies amp(z–1) = amp (z+3i) then the value of (x–1): y is equal to
a. 2:1 b. 1:3 c. –1:3 d. none of these
10. If (1+x)n = a0+a1x+a2x 2+.....+anx n, then (a0– a2+......)2 + (a1– a3+......)2 is equal to
1 – 2n
a. 3 n
b. 2 n
c. d. none of these
1  2n
§ 1 1 · 1 2
11. If x = 2+5i and 2 ¨  ¸ + 5! , then the value of x 3–5x2 +33x–19 is equal to
© 1!9! 3!7! ¹ 5! b!
a. a b. b c. a–b d. none of these
z –1 ʌ
12. If x + iy such that z  1 z – 1 and complex = then
z 1 4
a. x = 2  1, y = 0 b. x = 0, y = 2  1
c. x = 0, y = 2 –1 d. x = 2 –1, y = 0

x 2 y 2 1 § y x · 31 §x y i ·
13. If the square root of  2  ¨¨  ¸¸  is r ¨  – ¸ , then m is
y 2
x 2i © x y ¹ 16 © y x m¹
a. 2 b. 3 c. 4 d. none of these
334 365
ª 1 3º ª 1 3º
14.If i = – 1 , then 4+5 « –  i » + 3« –  i » is equal to
¬ 2 2 ¼ ¬ 2 3 ¼

a. 1– i 3 b. –1– i 3 c. i 3 d. –i 3
1 1 1
15. If z1, z2 and z3 are complex numbers such that z1 z2 z3 =   1,
z1 z 2 z 3

then z1  z 2  z 3 is
a. equal to 1 b. less than 1 c. greater than 3 d. equal to 3
16. The complex number sinx + icosx and cosx – isinx are conjugate to each other for
§ 1 · ʌ or x n ʌ ʌ
a. x nʌ b. x=0 c. x ¨ n  ʌ¸ d. x
© 2 ¹ 4 4
Answers :
1.c 2.b 3.c 4.b 5.a 6.c 7.c 8.b 9.b 10.b
11.b 12.b 13.c 14.c 15.a 16.d

86
COMPLEX NUMBER - III
Arguments and Modulus

Props of Arg
x arg( z ) = – arg z
z = x+iy
= r(cos T + isin T ) ___ polar form
= rei T ____Euler’s form
arg z = T
z = r (cos T – isin T )
= re– i T
= r (cos(– T )+isin(– T )
arg z = – T = – arg z.
x arg(z1z2) = arg z1+argz2
z1= r1 (cos T 1+isin T 1) arg z1 = T 1
z2= r2 (cos T 2+isin T 2) arg z2 = T 2
z1z2 = r1r2 e 1 2
i (T T )

= r1r2[cos[( T 1+ T 2)+isin( T 1+ T 2)]


arg(z1z2) = T 1 + T 2
= arg z1+arg z2
In general , we write arg(z1z2) = arg z1+argz2+2k S , wherek is -1 or 0 or 1
§ z1 ·
x arg ¨¨ ¸¸ = arg (z1) – arg z2 + 2k S where k = 0, –1 ,1
© z2 ¹

§ z1 · § r1 ·
¨¨ ¸¸ = ¨¨ ¸¸ i ( T1 – T2 )
e
© z 2 ¹ © r2 ¹

§ r1 ·
= ¨¨ r ¸¸ [cos( T 1– T 2)+isin( T 1– T 2)]
© 2¹

§ z1 ·
arg ¨¨ ¸¸ = T 1– T 2
© z2 ¹
= arg z1 – arg z2
x arg(zn) = n arg z
z = r (cos T + isin T )
z = r e iT
zn = rn e inT

87
n
Z = r [cos(n T )+isinn T ]
n

arg zn = n T = n arg z

Q.1 If arg z1 = 170° and arg z2 = 70°, then final the principal argument of z1 z2
arg (z1z2) = arg(z1)+arg z2 = 240°
Thus z1 z2 lies in 3rd quadrant. Hence its principal argument is –120°.
Q.2 If z1 and z2 are conjugate to each other, then find arg (–z1z2).
' z1 & z2 are conjugate to each other i.e z2 = z1
? arg (–z1z2) = arg (–z1 z1 ) = arg (–|z1|2)
= arg (negative real no.)
= S.
§ z1 · § z2 ·
Q.4 If z1, z2& z3, z4 are two pairs of conjugate complex nos, then find the value of arg ¨¨ z ¸¸ + arg ¨¨ ¸¸ .
© 4¹ © z3 ¹
z2 = z1 & z4 = z3 therefore
z1 z2 = |z1|2 & z3 z4 = |z3|2
§ 2
· § 2
·
§ z1 · § z2 · § z1z 2 · ¨ z1 ¸ ¨
arg ¨
z1 ¸
¸
Now arg ¨¨ z ¸¸ + arg ¨¨ z ¸¸ = arg ¨ z z ¸ = arg ¨¨
¨ ¸ 2¸
¸ ¨ ¸ =arg (positive real no.)=0
© 4¹ © 3¹ © 3 4¹ © z3 ¹ © z3 ¹
Properties of Modulus

1 |z1z2| = |z1| |z2|


Explanation:Let z1 =x1 + iy1 &z2 = x2 +iy2
Now|z1z2|=|(x1+iy1)(x2+iy2)|=|(x1x2-y1y2)+ i(x1y2+x2y1)|= x1 x 2  y1 y 2  x1 x 2  y1 y 2
2 2

§ 2 2
·§ 2 2
· § 2 2
· § 2 2
·
= ¨
© x1  y1 ¸¨
¹© x2  y2 ¸
¹
¨
© x1  y1 ¸
¹
¨
© x2  y2 ¸
¹ z z 1 2

as |z1z2.................zn| = |z1||z2|.......................|zn|
so, |zn| = |z.z..............to n factors| = |z||z|.......................to n factors=|z|n.

z1 z1
2 = z (try yourself)
z2 2

3 |z1+z2|2 = |z1|2 + |z2|2 + 2 re (z z 2 )


4 |z1–z2|2 =|z1|2 + |z2|2 – 2 re (z z 2 )

88
2
Explanation of (3) &(4):' z z z
z r z z r z z r z z r z z z  z z2 r z z2 r z z
2

? z1r z 2 1 2 1 2 1 2 1 2 1 1 2 1 2 1

Ÿ |z1+z2|2 = |z1|2 + |z2|2 r 2 re (z1 z 2 ) ( z1 z 2 zz zz


1 2 1 2
)

5 |z1+z2|2 + |z1 –z2|2 = 2 (|z1|2 + |z2|2)


Explanation : Adding (3) & (4), we get the result.

6 |z1+z2| d |z1| + |z2| (Triangle Inequality)


Explanation : z  z r cosT  i sinT  r cosT 2  i sin T 2
1 2 1 1 1 2 1 1

= r1 cosT 1  r 2 cosT 2  i r1 sin T 1  r 2 sin T 2 =

= r1 cosT 1 r 2 cosT 2  r1 sinT 1 r 2 sinT 2


2 2

= r1 cos T 1  sin T 1  r 2 cos T 2  sin T 2


2 2 2 2
2 2

1
 2 r r (cosT cosT
1 2 1 2
 sin T 1 sin T 2)

= r1  r 2 2 2
 2 r1 r 2 cos(T 1  T 2)

 2 r1 r 2 (' cos(T 1  T 2) d 1)
2 2
d r1  r 2
? |z1+z2| d r 1 r 2
2

or ? |z1+z2| d r1  r 2
Thus |z1+z2| d |z1| + |z2|
7 |z1–z2| t ||z1| – |z2||

Explanation: z  z r cosT  i sinT  r 1 2 1 1 1 2


cosT 2  i sinT 2
1 1

= r cosT  r cosT  i r sin T  r sin T =


1 1 2 2 1 1 2 2

= r1 cosT 1 r 2 cosT 2  r1 sinT 1 r 2 sinT 2 2 2

r1 cos T  sin T  r 2 cos T  sin T 2  2 r r (cosT cosT


2 2
 sin T 1 sin T 2 )
2 2 2 2
= 1 1 2 1
1 2 1 2

 2 r1 r 2 cos(T 1  T 2)
2 2
= r1  r 2
2 2
t r1  r 2  2 r1 r 2 (' cos(T 1  T 2) d 1)

? |z1z 2| t r1 r 2
2

or ? |z1- z 2| t r r1 2 z 1  z 2

Thus |z1-z 2| t |z1|  | z 2 |

89
COMPLEX NUMBER - IV
Practice Problems
E–D
1 If D , E are two different complex numbers such that | D | = 1 or | E | = 1 then the expression
1 – DE
equals
1
(a) (b) 1 (c) 2 (d) None of these
2
Solution
E–D E–D
= ( D D = | D |2 = 1)
1 – DE DD – DE

E–D |E – D|
= =
D ( D – E) | D (D – E) |

| (–1)(D – E) | | –1 || Į – ȕ | 1
= = |Į| Į–ȕ = =1
| D || D – E |  |D|

§ z –1·
2 If z = x+iy such that |z+1| = |z–1| and amp ¨ ¸ = S /4, then
© z 1 ¹
(a) x= 2 +1, y = 0 (b) x = 0, y = 2 +1
(c) x = 0, y = 2 –1 (d) x= 2 –1, y = 0
let z = x+iy
' |z+1| = |z–1| Ÿ |(x+1)+iy| = |(x–1)+iy|
Ÿ ( x  1) 2  y 2 = ( x – 1) 2  y 2
Ÿ (x+1)2+y2 = (x–1)2+y2 Ÿ x2+2x+1= x2–2x+1
Ÿ 4x = 0 Ÿ x = 0
§ z –1· § iy – 1·
amp ¨ ¸ = S /4, Ÿ amp ¨ ¸ = /4 ....................(1)
© z 1 ¹ © iy 1¹ S

iy – 1 iy – 1 1 – y 2 – 2iy (1– y2 ) 2iy


' iy  1 × iy – 1 = – ( y 2  1) = (y2 1) +
1  y2

2y – ( y 2  1) – 2y
× = tan S /4 Ÿ =1
y 1
2
(1 – y )2
1 – y2
Ÿ 1–y2 = – 2y
Ÿ y2– 2y–1 = 0
2r 44 2r2 2
y= = = 1r 2
2 2

90
1– 2 can be neglected as it is negative but tan T lies in Ist quadrant.
3 For any complex number z, the maximum value of |z|–|z–1| is
1 3
(a) (b) 1 (c) (d) 2
2 2
' |z1–z2| t ||z1|–|z2|| t |z1| – |z2|
|z|–|z–1| d |z–(z–1)| = 1
? Maximum value of |z| – |z–1| = 1
z1 – 2 z 2
4 If z1 & z2 are two complex numbers such that 2 – z .z is uni moduler, while z 2 is not uni moduler, then
1 2

the value of |z1| is


(a) 1 (b) 2 (c) 3 (d) 4
Solution
z1 – 2 z 2 | z1 – 2 z 2 |
2 – z1 z 2 = 1 Ÿ | 2 – z1 z 2 |
=1

Ÿ | z1 – 2 z 2 | 2 = | 2 – z1 z 2 | 2
Ÿ ( z1 – 2z 2 ) ( z1 – 2z 2 ) = (2–z1 z 2) (2 – z1 z 2 )
Ÿ (z1–2z2) ( z1 –2 z 2 ) = (2 – z1z 2 ) (2 – z1z 2 )
Ÿ z1 z1 –2z1 z 2 –2z2 z1 +4 z2 z 2
= 4–2z1 z 2 – z1 z2 +z1 z1 z 2 z 2
Ÿ |z1|2+4|z2|2 = 4+|z1|2 |z2|2
Ÿ |z1|2(1–|z2|2)–4(1–|z2|2) = 0
Ÿ (|z1|2–4) (1–|z2|2) = 0
|z2| = 1, |z1| = 2 since |z 2| = 1 1(not unimodular) so, |z1 | = 2
5 If |z–3+2i| d 4, then the sum of least and greatest values of |z| is
(a) 2 11 (b) 3 11 (c) 2 13 (d) 3 13
|z–3+2i| d 4 ..........(1)
Ÿ |z–3+2i| t ||z|–|3–2i||
Ÿ |z–3+2i| t ||z|– 13 |............(2)
From (1) & (2)
Ÿ 4 t |z–3+2i| t ||z|– 13 |
Ÿ ||z|– 13 | d 4
Ÿ –4 d |z|– 13 d 4
Ÿ 13 –4 d |z| d 4+ 13
? greatest value of |z| is 13 +4
&least value of |z| is 13 –4
? their sum = 13 +4+ 13 –4

91
= 2 13
6 If z1, z2, z3 are three distinct complex numbers and a, b, c are three positive real numbers such that
a b c a2 b2 c2
| z 2 – z 3 | = | z 3 – z1 | = | z1 – z 2 | , then z 2 – z 3 + z 3 – z1 + z1 – z 2 is

a 2 ( z 2 2  z 3 2 )  b 2 ( z1 2  z 3 2 )  c ( z 1 2  z 2 2 )
(a)
z12  z 2 2  z 3 2
(b) 0
a 2 ( b 2 – c 2 ) z1 2  b 2 ( c 2 – a 2 ) z 2 2  c 2 ( a 2 – b 2 ) z 3 2
(c)
z1 z 2  z 2 z 3  z 3 z1
(d) None of these
a b c
Let | z 2 – z3 | = | z 3 – z1 | = | z1 – z 2 | = k (ray)
Ÿ a2 = k2 |z2–z3|2 etc
a2
Ÿ 2 z z
z 2 – z 3 = k ( 2 – 3 )
b2
= k2 ( z 3 – z1 )
z 3 – z1

c2
= k2 ( z1 – z 2 )
z1 – z 2

a2 b2 c2

? z – z + z – z z – z = k2 | z 2 – z 3 + z 3 – z1 + z1 – z 2 ) = 0
2 3 3 1 1 2

Cube root of unity


z3 = 1
z3 – 1 = 0
(z–1) (z2+z+1) = 0
z = 1, z2+z+1 = 0
– 1 r 3i
z=
2

– 1  3i – 1 – 3i
z = 1, z = , z=
2 2
p p
Ȧ Ȧ2
1+ Ȧ + Ȧ 2 = 0
Ȧ3 = 1

92
w19 = w3×6+1 = (w3)2. Ȧ = Ȧ

De Mover’s Theorem
(cos T +isin T )n =cosn T +isinn T .

nth roots of unity


1 = cos0+isin0
11/n = (cos0+isin0)1/n
ª 2rS  0 2rS  0 º
= «cos  i sin
¬ n n »¼
2rS 2rS
= cos + isin
n n
r = 0, 1/2 .............(n–1)
i 2 rS
=e n where r = 0, 1/2 ............(n–1)
i 2 rS i4S i6S i 2( n –1) S
= 1, e n ,e n ,e n ........... e n

i 2 rS
= 1, D , D 2................. D n–1 where D = e n
(1) 1+ D + D 2+.........+ D n–1 = 0
(2) 1. D . D 2........... D n–1 = (–1)n–1
Important Relations
1 x2+x+1 = (x– Ȧ ) (x– Ȧ 2)
2 x2–x+1 = (x+ Ȧ ) (x– Ȧ 2)
3 x2+2y+y2 = (x–y Ȧ ) (x–y Ȧ 2)
4 x2–xy+y2 = (x+y Ȧ ) (x+y Ȧ 2)
5 x2+y2 = (x+iy) (x–iy)
6 x3+y3 = (x+y) (x+y Ȧ ) (x+y Ȧ 2)
7 x3–y3 = (x–y) (x–y Ȧ ) (x–y Ȧ 2)
8 x2+y2+z2–xy–yz–zx = (x+y Ȧ +z Ȧ 2) (x+y Ȧ 2+z Ȧ )
9 x3+y3+z3–3xyz = (x+y+z) (x+y Ȧ +z Ȧ 2) (x+yw2+z Ȧ )
§ 1 3 9 27 ·
¨    ...........Į ¸
© 2 8 32 128 ¹
1 If w is a cube root of unity, then Ȧ + Ȧ is
(a) +1 (b) i2 (c) 0 (d) None of these
1
1 3 9 27 1 ª 3 9 27 º 1 1 4
    ...........D = «1  4  16  64  ...........D » = 2 1 – 3 = 2 × 1 = 2
2 8 32 128 2 ¬ ¼ 4
? w+w2 = –1 = i2
2 D , E real J are the roots of x3–3x2+3x+7 = 0
§ Į – 1 ȕ – 1 Ȗ – 1·
(w is the cube root of unity), then ¨   ¸ is
© ȕ – 1 Ȗ – 1 Į – 1¹

93
(a) 3/ Ȧ (b) Ȧ 2 (c) 2Ȧ2 (d) 3Ȧ2
3 we have
x3–3x2+3x+7 = 0
Ÿ (x–1)3+8 = 0 Ÿ (x–1)3 = (–2)3
3
ªx –1º x –1
Ÿ «¬ – 2 »¼ = 1 Ÿ = 1, Ȧ , Ȧ 2
–2
Ÿ x = –2+1, x–1 = –2 Ȧ , x–1, = –2 Ȧ 2
x = –1, x= 1–2 Ȧ , x = 1–2 Ȧ 2
D = –1, E = 1–2 Ȧ , J = 1–2 Ȧ 2
D –1 = –2, E –1 = –2 Ȧ , J –1 = –2 Ȧ 2
2 – 2Ȧ  2Ȧ 2
? G.E = + +
 2Ȧ – 2Ȧ 2 2
= Ȧ +Ȧ +Ȧ
2 2 2

= 3Ȧ2
3 The common roots of eqin
z3+2z2+2z+1 = 0 & z1985+z100+1 = 0 are
(a) 1, Ȧ (b) 1, Ȧ 2 (c) Ȧ,Ȧ2 (d) None of these

z3+2z2+2z+1 = 0..................(1)
Ÿ (z+1) (z2+z+1) = 0
Ÿ z = –1, Ȧ , Ȧ 2 are the roots of (1)
z = –1, z1985+z100+1 = 1 z 0
z = Ȧ , ( Ȧ )1985+ Ȧ 100+1 = Ȧ 2+ Ȧ +1 = 0
z = Ȧ 2, Ȧ 3970+ Ȧ 200+1 = 0
? The common roots are w,w2
? (c) is correct.
xD  yE  zJ
4 If D , E , J are the cube roots of p, p<0 then for arang x,y & z is
xE  yJ  zD

x 3  y3  z3
(a) w 2
(b) –1 (c) (d) None of these
D 3  E3  J 3
Let x3 = p
Ÿ x = (1.p)1/3
Ÿ x = p1/3. 11/3
x = p1/3, p1/3 Ȧ , p1/3 Ȧ 2
x.p1/ 3  yp1/ 3Ȧ  zp1/ 3Ȧ 2
? Given expression =
x.p1/ 3Ȧ  y.p1/ 3Ȧ 2  z.p 1 / 3

x  yȦ  zȦ2 Ȧ2 ( x  Ȧy  Ȧ 2 z)
= = (multiply and divide by Ȧ 2)
xȦ  yȦ2  z ( x  Ȧy  zȦ 2 )
= Ȧ2

94
5 The value of the expression
§ 1· § 1· § 1· § 1· § 1· § 1· § 1· § 1·
¨1  ¸ ¨1  2 ¸ + ¨ 2  ¸ ¨ 2  2 ¸ + ¨ 3  ¸ ¨ 3  2 ¸ +.........+ ¨ n  ¸ ¨ n  2 ¸
© Ȧ¹ © Ȧ ¹ © Ȧ¹ © Ȧ ¹ © Ȧ¹ © Ȧ ¹ © Ȧ¹ © Ȧ ¹

§ 1· § 1·
Tk = ¨© k  ¸¹ ¨© k  2 ¸¹ = (k+ Ȧ 2) (k+ Ȧ ) = k2–k+1
Ȧ Ȧ
n n
2
? Sn = ¦ Tk = ¦ (k – k  1)
k 1 k 1
2
= ¦ k – ¦ k  ¦1
n (n  1)(2n  1) n (n  1)
= – +n
6 2
ª 2n 2  3n  1 n  1 º
=n « –  1»
«¬ 6 2 »¼

ª 2n 2  3n  1 – 3n – 3  6 º
=n« 6
»
«¬ »¼

§ 2n 2  4 ·
= ¨¨ 6 ¸¸
n
© ¹

n ( n 2  2)
= Ans
3
6 If w and w2 are the cube roots of unity and
1 1 1 1 1 1 1
+ + = 2Ȧ2 & 2 + 2 + 2 = 2 Ȧ then the value of +
aȦ bȦ cȦ aȦ bȦ cȦ a 1
1 1
+ is
b 1 c 1
(a) 1 (b) –1 (c) Ȧ (d) 2
Note that Ȧ & Ȧ are roots of
2

1 1 1 2
+ + = .................(1)
ax bx cx x
3x 2  2(a  b  c) x  b c  ca  ab 2
=
(a  x )(b  x )(c  x ) x
Ÿ x –(bc+ca+ab)x–2abc = 0
3

The roots of the equation are Ȧ & Ȧ 2


If D in the third root of this equation then D + Ȧ + Ȧ 2 = 0 Ÿ D = 1
Put D = 1 on equation (1)
1 1 1
+ + = 2.
a 1 b 1 c 1

95
PRACTICE QUESTIONS
1.
5

5
The value of 1  Ȧ  Ȧ 2  1  Ȧ  Ȧ 2 ,where Z and Ȧ 2 are the complex cube roots of unity,,
is
a. 0 b. 32 Z c. -32 d. 32
2. For any two complex numbers z 1 and z 2 and any real numbers a and b,
2 2
az1  bz 2  bz 1  az 2 isequal to

a. a 2

 b 2 z1  z 2
b. a 2  b 2 z 1  z 2
2 2


c. a 2  b 2 z 1 2  z 2 2
d. none of these

3. For any two complex numbersz1 and z2 , the relation z1  z 2 z1  z 2 holds,if


ʌ
a. arg(z1)=arg(z2) b. arg(z1)+arg (z2)=
2
c. z1z2 d. |z1|=|z2|
4. If Z and Ȧ 2 are the two imaginary cube roots of unity, then the equation whose roots are
aȦ 317 and aȦ 382 is
a. x2 +ax -a2 = 0 b. x2 +a2x +a2 = 0 c. x2 +ax +a2 = 0 d. x 2 -a2x +a= 0
5. If 1,a1,a2,..............a n-1 are roots of unity, then the value of (1-a 1)(1-a2)........(1-an-1) is
a. 0 b. 1 c. n d. n2
6. If Ȧ z 1 is a cube root of unity, then the value of
1  Ȧ 2  2 Ȧ100 Ȧ2 1
1 1  w  2Ȧ
100 200
Ȧ
Ȧ Ȧ2 2  Ȧ100  Ȧ 200
a. 0 b. 1 c. Z d. Ȧ 2
7. If the area of the triangle formed by the points z,z +iz and iz is 50sq. unit , then |z| is equal to
a. 5 b. 8 c. 10 d. none of these
8. Let Z be an imaginary roots of x = 1 .Then, (5 - Z )(5 - Ȧ ).............( 5  Ȧ ) is
n 2 n 1

5n  1 5n  1
a. 1 b. c. 4 1
n d.
4 4
9. If Ȧ(z 1) is a cube root of unity and 1  Ȧ 7 A  BȦ
a. 0,1 b. 1,1 c.1,0 d.-1,1
10. If x Z  Z  2 then the value of x + 3x +2x - 11x -6 is
2 4 3 2

a. 1 b.-1 c.2 d. none of these


z 2n  1
11. If z = cosș  isinș ,then 2n is equal to (n is an integer)
z 1
a. i cotn ș b. i tan n ș c.tann ș d. cotn ș
12. The cube root of unity

96
a. lie on the circle |z|=1 b. are collinear
c.form an equilateral triangle d. none of these
13. The complex number z satisfying the equation |z| - 4 = |z - i| - |z +5 i| = 0, is
a. 3 i b. 2 3 c. - 2 3  2i d. 0

§ z - z1 · ʌ
14. If z1 = 8 + 4 i, z 2 = 6 + 4 i and arg¨¨ z  z ¸¸
4 , then z satisfies
© 2 ¹

a. |z -7 -4 i| =1 b. |z - 7 -5 i|= 2 c. z - 4i 8 d. z - 7i 18
15. If z =cos ș + i sin ș , then
1 1
a. zn  2cosnș b. zn  2 n cosnș
zn zn
1 1
c zn  2 n isinnș d. zn  (2i) n sinnș
zn zn

ANSWERS

1.d 2.b 3.a 4.c


5.c 6.a 7.c 8.d
9.b 10.a 11.b 12c
13.c 14.b 15.a

97
COMPLEX NUMBER - V
Loci in Complex plane

1. Distance formula
Distance between A & B
i.e. AB = z1 – z 2
Imz
B(z2)

A(z1)

Rez
2.Section formula
If P divides AB in the ratio
m:n (internally), then P is
§ mz 2  nz1 ·
¨ ¸
© mn ¹
B(z2)
n
z
m P

A(z1)

mz 2  nz1
In case of external division ,P is .
mn
If P is the mid point of AB then
§ z1  z 2 ·
P is ¨© ¸
2 ¹
Straight line
OP OA  AP
i.e.
t is some suitable real number

Now writing corresponding complex number, we get

98
z = z1 + t (z2–z1) ......(1)
Since Value of t is different for different position of P.
? t is called a parameter & the equation (1) is called the parametric equation of straight
line passing through the fixed points A (z1) & B (z2).
Cartesian equation of (1) Put z = x+iy, z1=x1+iy1 & z2 = x2+iy2 in (1)
If we get x+iy = x1+iy1+t((x2–x1)+(y2–y1))
x = x 1+t(x 2–x 1) & y = y1+t(y2–y1)
x – x1 y – y1
or t
x 2 – x1 y 2 – y1
z – z1
From (1) t
z 2 – z1
' t is Purely real number.
so, -----------------(2)

(z z1 ,then z is purely real)

Ÿ
z – z1 z – z1
Ÿ z 2 – z1 z 2 – z1
Ÿ zz 2 – zz1 – z1z 2  z1z1 zz 2 – z1z 2 – z1z  z1z1
1 1 1 z z 1
Ÿ z z1 0 Ÿ z1
z2 z1 1 0 ........(3)
z z1 z2 z2 z2 1
z –z
Slope of this line is 1 2 .
z1 – z 2
Since, z z1 – z 2 – z z1 – z 2  z1.z 2 – z1.z 2 0
Ÿ i z1 – z 2 z – i z1 – z 2 z  i z1.z 2 – z1.z 2 0
Ÿ az  az  b 0 .......(4)
where b = i z1z 2 – z1.z 2

= i z1 z 2 – z1. z 2
= i.2i Im z1z 2
= –2Im z1z 2 = a real number & a z 0 as z1 z z 2

z1 z1 1
* Points z1,z2,z3 are collinear if and only if z 2 z2 1 0
z3 z3 1

99
* Equation (2) & (3) represent the non parametric form of equation of straight line
passing through the point A(z1) & B(z2)

* Equation (4) is called general equation with slope = –
Equation of Perpendicular Bisector

We consider two fixed points A(z1) & B(z2) and variable point P(z) moving such that AP = BP
i.e. z – z1 z – z2 ......(5)

z –z2

From Geometry, it is obvious that P lies on the perpendicular bisector of the segment AB.
? Equaiton (5) is the equation of perpendicular bisector of the segment joining A(z1) & B(z2).

Another form of perpendicular bisector


z

2 2
z – z1 z – z 2 Ÿ z – z1 z – z2

* If A(z1) and B(z2) are two fixed points and P is a variable moving on the line segment AB then
AP+BP = AB
i.e. z – z1  z – z 2 z1 – z 2 .......(6)
? above equation represents the line segment AB
z
z

z z

z z

i.e. z – z1 – z – z 2 z1 – z1 .......(7) i.e. z – z1 – z – z 2 – z 1 – z 1 .......(8)


From equation (7) & (8), it is clear that p is lying on the line passing through A & B but not lying
on the segment AB.

100
If z1 and z2 are two unequal complex numbers represented by points P and Q,
z1 – z 2
then is called the complex slope of the line joining z1 &z2.
z1 – z 2
z1 – z 2
Thus w =
z1 – z 2

z1 – z 2 z1 – z 2
then = = 1
z1 – z 2 z1 – z 2
* Two lines having complex slopes w1 and w2 are parallel if and only if w1 = w2
z1 – z 2
w1 = (complex slope of AB)
z1 – z 2
z3 – z 4
w2 = (complex slope of CD)
z3 – z 4
AB||CD Ÿ angle b/w AB and CD = 0 or ʌ
§z –z ·
Ÿ argument ¨ 2 1 ¸ 0 or ʌ z
© z 4 – z3 ¹

Ÿ is real z
z

§ z 2 – z1 · z 2 – z1 §z –z · z1 – z 2
Ÿ ¨ ¸ Ÿ¨ 1 2¸ z
© z 4 – z3 ¹ z 4 – z3 © z3 – z4 ¹ z3 – z 4

z1 – z 2 z3 – z 4
Ÿ Ÿ w1 w2
z1 – z 2 z3 – z 4
* The two lines having complex slope w1 & w2 are perpendicular if and only if w1 = –w2.
* The equation of a line parallel to az  az  b 0 is
az  az  Ȝ 0 , where
* The equation of a line perpendicular to the line az  az  b 0 is
az – az  Ȝ 0 , where
* The length of perpendicular from a point P(z0) to the line az  az  b 0 is
az 0  az 0  b
.
2a

101
CIRCLE

Consider a fixed point c(z 0) and a variable


point P(z) which is moving, keeping its distance
from the point C a constant R
? At any position, P(z) satisfies the
equation z – z 0 R ......(1)
z
Equation(1) represents the points on a
circle whose centre is c(z 0) & radius is R.

Cartesian form
2
z – z0 R2 .
Let z = x+iy &z 0 = x 0 + iy0
x – x 0  i y – y 0 2 R2
(x–x0)2 + (y–y0)2 = R2
General equation of a circle is
zz  az  az  b 0 where b is real. The cen-
tre of this circle is –a and radius
2
is aa – b a –b
Explanation: Let z0 be the centre of the circle
and P(z) be any point on the circle.
2
Then z – z 0 r or z – z 0 r2 z

Ÿ
(z–z0) z – z 0 = r2 Ÿ (z–z0) z – z 0 = r2
2
Ÿ zz – z 0 z – z 0 z  z 0 – r 2 0
Ÿ zz – az  az  b 0 ......(1)
2
Where a = –z0, b = z 0 – r 2 0 real number clearly centre of the circle (1) is z 0 i.e. –a &
radius=

* Equation of the circle whose one diameter is the line


segment joining z1 & z2 is
z – z1 z – z1
 0
z – z 2 z – z2
ʌ
' ‘APB
2

102
§ z – z· ʌ z –z
argument ¨ 2 ¸ Ÿ 2 is purely imaginary
© z1 – z ¹ 2 z1 – z

§ · §z –z·
¨ z2 – z ¸
or ¨¨ ________ ¸¸ – ¨¨ 2 ¸¸
© z1 – z ¹ © z1 – z ¹

z2 – z z 2 – z z1 – z z2 – z
Ÿ – Ÿ –
z1 – z z1 – z z2 – z z1 – z

z – z2 z – z2 z – z1 z – z1
Ÿ – or  0
z – z1 z – z1 z – z 2 z – z 2

Ÿ (z–z1) z – z 2 +(z–z2) z – z1 = 0

* z – z1 k z – z 2 Ÿ AP KBA Ÿ Locus of P(z) is a circle k z 1


* z – z1  z – z 2 k z z1 – z 2 Ÿ PA+PB = k be locus of z is an ellipse where A & B are foci of
the ellipse.
* z – z1 – z – z 2 k z z1 – z 2 Ÿ PA–PB = k be locus of z is an ellipse where A & B are foci at
A & B.
2 2 2
* z – z1  z – z 2 z1 – z 2 , locus of z is a circle.

* argument (fixed), locus of z is segment of a circle.

* argument , locus of z is a circle with z1 and z2 as the vertices of diameter..

* argument , locus of z is a straight line passing through z1 & z2.

103
PRACTICE QUESTIONS
1  iz
1. If Z = x + iy and Z = ,then Z 1 implies that, in the complex plane
z i
a. z lies on the imaginary axis b. z lies on the real axis
c. z lies on the unit circle d. none of these
2. The points z1,z2,z3,z4 in the complex plane are the vertices of a parallelogram taken in order if
and only if
a. z1 +z4 = z2 +z3 b. z1 +z3 =z2 +z4 c. z1 +z2 = z3 +z4 d. none of these
3. For all complex numbers z 1,z2 satisfying z1 =12 and z 2  3  4 i 5 , the minimum value of

z1  z 2 is
a. 0 b. 2 c. 7 d. 17
4. If z 1 , then the point representing the complex number  1 3z will lie on
a. a circle b. a straight line c. a parabola d. a hyperbola
5. If z = O  3  i 5  O2 ,then the locus of z is
a. ellipse b. semicircle c. parabola d. straight line
6. If z is complex number then the locus of z satisfying the condition 2 z  1 z  1 is
a. perpendicular bisector of line segment joining 1/2 and 1 b. circle
c. parabola d. none of the above curves
§ 2z  1·
7. If Im ¨ ¸ 2 then the locus of the point representing z in the complex plane is
© iz  1 ¹
a. circle b. a straight line c. a parabola d. none of these
8. If z1 z 2 z 3 z 4 then the points representing z1,z2,z3,z4 are
a. concyclic b. vertices of a square
c. vertices of a rhombus d. none of these
9. The equation z z  ( 4  3i ) z  ( 4  3i ) z  5 0 represents a circle whose radius is
5
a. 5 b. 2 5 c. d. none of these
2
10. The equation z  i  z  i k, k ! 0 , can represent an ellipse if k2 is
a. <1 b. <2 c. >4 d. none of these
11. The equation z  i  z  i k represents hyperbola if
a. 2k 2 b. k!2 c. 0k2 d. none of these
12. Let z 1  t  i t 2  t  2 ,where t is a parameter. The locus of z in the Argand plane is
a. a hyperbola b. an ellipse
c. a straight line d. None of these

104
2 2
13. Let z1 and z2 be two non-real complex cube roots of unity and z  z1  z  z 2 O be the
equation of a circle with z1,z2 as ends of a diameter, then the value of O is

a. 4 b. 3 c. 2 d. 2
2 2
14. If the equation z  z1  z  z 2 k represents the equation of a circle, where z 1 = 2 + 3i,
z 2 = 4 + 3i are the extremities of a diameter, then the value of k is
a. 1/4 b. 4 z c. 2 d. none of these
15. Let Į and ȕ be two fixed non-zero complex numbers and z a variable complex number. If the
lines Į z  Įz  1andȕ z  ȕz  1 0 are mutually perpendicular, then
a. Įȕ  Įȕ 0 b. Įȕ  Įȕ 0 c. Įȕ  D ȕ 0 d. Įȕ  ĮE 0

ANSWERS

1.b 2.b 3.b 4.a


5.b 6.b 7.b 8.a
9.a 10.c 11.a 12.a
13.b 14b 15d

105
COMPLEX NOMBER -VI
Problem Solving and nth Roots of Unity
1 Let x,y R & i = – 1 , then z = x+iy is called a complex number
Re(z) = x, Im(z) = y
If x = 0, z is purely imaginary
If y = 0, z is purely real
2 For two complex numbers z1 = x1+iy1 z2 = x2+iy2
(i) z1 = z2 if and only if x1 = x2 and y1=y2
Note that no order relation is possible among the set of complex numbers. i.e. it is wrong to
say 1+i<4+3i. But |1+i| < |4+3i|
(ii) z1 r z2 = (x1 r x2)+i(y1 r y2)
(iii) z1z2 = (x1x2–y1y2)+i(x1y2+x2y1)
z1 ( x1x 2  y1 y 2 )  i( x 2 y1 – x1 y 2 )
(iv) = 2 2 ; z2 z 0
z2 x 2  y2
3 Conjugate of z
Conjugate of z = x+iy is defined as z = x–iy (i.e. replace i by –i)
Properties
(i) z = z
(ii) z = z œ z is puraly real
(iii)z = – z œ z is purely imaginary
(iv) z+ z = 2Re(z) = 2Re( z )
(v) z– z = 2iIm(z)
(vi) z1 r z 2 = z1 r z 2
(vii)
z 2 z 2 = z1 z 2 (also z n = ( z )n)

§ z1 · z1
(viii) ¨¨ ¸¸ =
© z2 ¹ z2
4 Geometric representation
Coordinate Complex number Vector
representation representation representation
Point P :P(x,y) Point P : z = (x+iy) Point P : Position vector
of P OP = x î +y ˆj
5 Modulus and amplitude
|z| = r = x 2  y 2 , the modulus of z (distance of z from the origin)

§ y·
T = tan–1 ¨ x ¸ = arg z, the inclination of OP with positive direction of x–axis where P is (x,y).
© ¹
Here – S < arg z d S
z = x+iy ; x,y R (algebraic form)

106
= r(cos T +isin T ) or r cis T (polar/trigonometric form)
= r e iT (Eulers form)
Properties of modulus
(i) –|z| d Re(z),Im(z) d |z|
(ii) |z|=| z |+|–z|+|– z |
(iii) z z =|z|2
(iv) |z1z2|=|z1||z2|
Also |zn|=|z|n
z1 | z1 |
(iv) = |z |
z2 2

(vi) | z1 | – | z 2 | d |z1 r z2| d |z1|+|z2| (Triangle inequality)

i.e.|z1|+|z2| is the maximum and | z1 | – | z 2 | is the minimum value of |z1 r z2|

­| z1 |2  | z 2 |2 r2 Re(z1 z 2 )
°°
(vii) |z1 r z2|2 = ®| z1 |  | z 2 | r2 | z1 || z 2 | cos(T1 r T 2 )where
2 2

°
°¯T1 arg( z1 )&T 2 argz 2
(viii) |z1+z2|2+|z1–z2|2=2(|z1|2+|z2|2) (parallelogram law)
Also |az1–bz2|2+|bz1+az2|2=(a2+b2)(|z1|2+|z2|2); a,b  R
1 a  a2  4
Note : If z  = a, the greatest and least value of |z| are respectively and
z 2
– a  a2  4
.
2
Properties of argument
(i) arg(z1z2)=arg(z1)+arg(z2)+2k S , where k= –1 or 0 or 1
Also arg(zn) = narg(z)+2k S
§ z1 ·
(ii) arg ¨¨ z ¸¸ = arg z1–arg z2 + 2k S , where k = –1 or 0 or 1
© 2¹

§z·
(iii)arg ¨ ¸ = 2arg(z)+2k S , where k = –1 or 0 or 1
©z¹
(iv) argument of zero is not defined
arg(z) = 0 œ z is real and positive
arg(z) = S œ z is real and negative
S
arg(z) = œ z is purely imaginary, Imz>0
2

107
–S
arg(z) = œ z is purely imaginary, Imz<0
2
(v) |z1+z2| = |z1| + |z2|
z1
argz1 – argz2 = 0, z >0
2

0, z1, z2 are collinear and z1, z2 lie on the same side of 0.


(vi) |z1–z2| = |z1| + |z2|
z1
argz1–argz2 = S , z <0
2

0, z1, z2 are collinear and 0 lies between z1 & z2.


(vii) |z1–z2| = |z1+z2|
S
argz1–argz2 = r
2
z1
z 2 and z1 z2 are purely imaginary..
6 De Moivres theorem
(i) For any rational number n, then
(cos T + i sin T )n = cosn T +i sinn T
i.e. , ( e iT )n = e inT
(ii) (cos T 1+ i sin T 1) (cos T 2+ i sin T 2)..................(cos T n + i sin T n )
= cos( T 1+ T 2+......+ T n) + i sin( T 1+ T 2+...............+ T n)
(iii) If z = r(cos T + i sin T ) and n Z+, then
§ § 2rS  T · § 2rS  T · ·
z1/n = r1/n ¨¨ cos¨ ¸  isin ¨ ¸ ¸¸ where r = 0, 1, 2, ...........(n–1)
© © n ¹ © n ¹¹
(nth roots of z)
7 The nth roots of unity
z = n 1 = 11/n = e i 2 rS / n , r = 0, 1, 2, ............,(n–1)
Let z = D r where D = e i 2 S / n
2S 2S
The nth roots of unity are ( D °=) 1, D , D 2,......... D n–1 where D = ei2 S /n = cos + i sin
n n
Properties
(i) nth roots of unity are solutions of the equation z = 11/n
i.e. zn = 1
z n–1 = (z–1)(z– D ) (z– D 2)..........(z– D n–1)
nth roots of –1 are the solutions of zn +1 = 0
(ii) nth roots of unity lie on a unit circle |z| = 1 and divide the circumference into n equal parts and
are the vertices of a regular polygon of n sides inscribed in the circle |z| = 1

108
(ii) Product of nth roots of unity = (–1)n–1
(iii) Sum of nth roots of unity is always zero.
(v) nth roots of unity form a G.P with common ratio e i 2 S / n
(vi) Sum of pth power of nth roots of unity
­0; if pisnotamultipleof n
= 1+ D p+( D 2)p+( D 3)p +..............(+ D n–1)p = ®
¯n; if pisamultipleof n
8 Cube roots of unity
2rS 2rS
11/3 = cos + i sin ; r = 0, 1, 2
3 3
–1 i 3 –1– i 3
= 1, ,
2 2
If one of the non real complex roots be Z , then the other non real complex root will be Z 2.
–1 i 3 –1– i 3
i.e. if Z = , then Z 2 =
2 2
? the 3 cube roots of unity are 1, Z & Z 2.
Properties
(i) z3–1 = (z–1) (z– Z ) (z– Z 2)
(ii) Z & Z 2 are roots of z2+z+1 = 0 i.e., z2+z+1 = (z– Z ) (z– Z 2)
(iii) arg Z = 2 S /3 & arg Z 2 = 4 S /3
(iv) cube roots of unity lie on the unit circle |z| = 1 and divide its circumference into three equal
parts
(v) If A(1), B( Z ) & C( Z 2), then ' ABC is an equilateral triangle.
(vi) Z 3 = 1 ; 1+ Z + Z 2 = 0 ; Z 3n = 1 ; Z 3n+1 = Z ; Z 3n+2 = Z 2
1 1
(vii) Z = = Z 2 ; Z2 = 2 = Z
Z Z
(viii) x x2+y2 = (x+iy) (x–iy)
x x3+y3 = (x+y) (x+ Z y) ( x+ Z 2y)
x x3–y3 = (x–y) (x– Z y) ( x– Z 2y)
x x2+xy+y2 = (x– Z y) ( x– Z 2y)
x x2–xy+y2 = (x+ Z y) ( x+ Z 2y)
x x2+y2+z2–xy–y z–zx = (x+y Z +z Z 2) (x+y Z 2+z Z )
x x3+y3+z3–3xy z = (x+y+z) (x+y Z +z Z 2) (x+y Z 2+z Z )
Note ? x If a+b+c = 0 = a2+b2+c2 , then a : b : c = 1: Z : Z 2
realpart
x Any complex number for which imaginary part = 1 : 3 or 3 : 1, can be expressed

in terms of Z , Z & i
2

9 Square root of a complex number


Let z = a+ib

109
­ 1 1 ½
a  ib = r ® (| z |  Re( z))  i (| z | – Re( z)) ¾
¯ 2 2 ¿

­ 1 1 ½
i.e. a  ib = r ® ( a  b  a )  i ( a  b – a ) ¾
2 2 2 2

¯ 2 2 ¿
To find the square root of a–ib, replace i by – i in the above result.
10 Logarithm of a complex number
Let z = a+ib = r e iT
|z| = r = a 2  b2
log z = log |z| + iarg z
11 Expansions
(i) cosn T = nC0 cosn T –nC2cosn–2 T sin2 T +nC4cosn–4 T sin4 T +............
(ii) sinn T = nC1cosn–1 T sin T –nC3cosn–3sin3 T +nC5cosn–5sin5 T ........ (using De-Moivres theo-
rem)
1
(iii) cosn T = {nC0cosn T +nC1cos(n–2) T +nC2cos(n–4) +....................}
2 n –1
(–1) n / 2
(iv) sin T =
n
{cosn T –nC1cos(n–2) T +nC2cos(n–4) T +...........}
2 n –1
n –1

sin T = (–1n)–1 {sinn T –nC1sin(n–2) T +nC2sin(n–4) T +...........}


2
n

2
Solved Examples
a  bZ  cZ 2 a  bZ  cZ2
1 If Z is a complex cube root of unity, then the value of + is equal to
c  aZ  bZ 2 b  cZ  aZ2
(a) –1 (b) 2 (c) 2 Z (d) None of these
Solution :
1 (a  bZ 2  cZ3 ) 1 (aZ 2  bZ3  cZ 4 )
+
Z aZ  aZ 2  c Z2 b  cZ  aZ 2
1 1
= + Ÿ Z 2+ Z = – 1
Z Z2
Ans (a)
2 2 2
§ 1· § 2 1 · § 27 1 ·
2 If x +x+1 = 0, then the value of ¨ x  ¸ + ¨ x  2 ¸ + ..............+ ¨ x  27 ¸ is
2
© x¹ © x ¹ © x ¹
(a) 27 (b) 72 (c) 54 (d) None of these
Solution:

110
Roots of x2+x+1 = 0 are Z and Z 2
Put x= Z
2 2 2 2
§ 1· § 2 1 · § 3 1 · § 27 1 ·
? ¨ Z  ¸ + ¨ Z  2 ¸ + ¨ Z  3 ¸ ...........+ ¨ Z  27 ¸
© Z¹ © Z ¹ © Z ¹ © Z ¹
= (–1) + (–1) + (2) + .................+(2)
2 2 2 2

= 18×1 + 9×4 = 54
Ans (c)
3 If D , E , J are roots of x3–3x2+3x+7 = 0 and Z is a complex cube roots of unity, then
D –1 E –1 J –1
+ + is equal to
E –1 J –1 D –1
(a) Z (b) 2 Z (c) 2 Z2 (d) 3 Z2
Solution:
(x–1)3 = –8
x–1 = z
x –1 = – 2, –2 Z , –2 Z 2
x= – 1, 1–2 Z , 1–2 Z 2
Let D = –1 E = 1–2 Z , J = 1 –2 Z 2
Ÿ D – 1 = – 2 ; E –1 = – 2 Z , J =1 –2 Z
2

2
D –1 E –1 J –1 –2 – 2Z – 2Z
? + + = + +
E –1 J – 1 D – 1 – 2Z – 2Z 2 –2
1 1
= + + Z2 = 3 Z2
Z Z
Ans (d)
4 If (x–1)4 –16 = 0, then the sum of non real complex roots of the equation is
(a) 2 (b) 0 (c) 4 (d) None of these
Solution:
(x–1)4 = 16
1
x–1 = (16) 4
x–1 = r 2, r 2i
x = 1 r 2, 1 r 2i
Sum of non-real roots is (1+2i) + (1–2i) = 2
Ans (a)
5 If z is a non-real root of 7 – 1 , then z86 +z175+ z289 is equal to
(a) 0 (b) –1 (c) 3 (d) 1
Solution :
z = 7 – 1 Ÿ z7 = –1
? z86 + z175 + z289 = (z7)12 z2 + (z7)25+(z7)41.z2

111
= z2–1–z2 = –1
Ans (b)
PRACTICE QUESTIONS
6
§ 2Sk 2Sk ·
1 The value of ¦ ¨© sin 7
– i cos
7 ¹
¸ is
k 1
(a) –1 (b) 0 (c) –i (d) i
2 For positive integers n1, n2 the value of expression
(1  i) n1 + (1  i 3 ) n1 + (1  i 5 ) n 2 + (1  i 7 ) n 2 is a real number if and only if
(a) n1 = n2+1 (b) n1 = n2–1 (c) n1 = n2 (d) n1>0, n2>0
3 The minimum value of |a+b Z +c Z | where a, b & c are all not equal integers and Z ( z 1) is a cube
2

root of unity, is
1
(a) 3 (b) (c) 1 (d) 0
2
4 If x is a complex root of the equation
1 x x 1– x 1 1
x 1 x 1 1– x 1 1
+ = 0, then x 2005 + is
2005
x x 1 1 1 1– x x
(a) 1 (b) –1 (c) i (d) Z
5 If z = i log e (2– 3 ), then cosz =
(a) 0 (b) 1 (c) 2 (d) None of these
1 1 1
6 If Z is a complex cube roots of unity and a, b, c are such that + + = 2 Z2
aZ bZ cZ
1 1 1 1 1 1
and 2 + 2 + 2 = 2 Z , then + + =
aZ bZ cZ a 1 b 1 c 1
(a) 1 (b) –1 (c) 2 (d) –2
7 If Z ( z 1) be a cube root of unity and (1+ Z 2)m = (1+ Z 4)m, then the least positive integral value of
m is
(a) 2 (b) 5 (c) 1 (d) 3
§ § x  iy · ·
8 cot ¨¨ – i log e ¨¨ ¸¸ ¸¸
© © x – iy ¹ ¹

x 2 – y2 2xy 2xy y2 – x 2
(a) (b) (c) (d)
2 xy x 2 – y2 x 2  y2 2 xy
2S 4S 8S
9 sin + sin +sin =
7 7 7
7 –1 1
(a) (b) (c) (d) None of these
2 2 8

112
n –1
Di
10 If D 0, D 1, D 2,.............., D n–1 be the , n roots of the unity then the value of
th ¦ (3 – D ) is equal
i 0 i

to
3 n –1 n 1 n2
(a) n (b) (c) (d)
3 –1 3n – 1 3n – 1 3n – 1
Answers
1 .d 2. d 3. c 4. a 5. c
6. c 7. d 8. a 9. a 10. a

113
COMPLEX NUMBERS - VII
Rotation Theorem
GEOMETRY OF COMPLEX NUMBERS
1. Distance between two complex numbers z1 & z2 is z1 – z 2
2. Section formula
Two points P & Q have affixes z1 & z2 respectively in the argand plane, then the affix of a point
R dividing PQ is the ratio m:n
mz 2  nz1
i. internally is
mn
mz 2 – nz1
ii. externally is
m–n
z z
iii. If R is the mid point PQ, then affix of R is 1 2
2
3. Special points of a triangle
Segment/Line Figure point of concurrency Formula
Perpendicular A Circumcentre z1 sin2A z2 sin2B z3 sin2C
z3
sin2A sin2Bsin2C

¦ z z – z
2
bisectors (Equidistant from 1 2 3

¦ z z – z
S
1 2 3
B C
vertices)
Angle A

az1  bz 2  cz 3
bisectors Incentre (equidistant z3=
abc
from the sides) A to the sides) a z 2 – z3 , b z1 – z 3 ,
B C
c z1 – z 2

Medians A
Centroid ( distance z1  z 2  z 3
Gz
3
from vertex to G is
G 2
of total median)
3
B C

A
z1 tanA  z2 tanB  z3 tanC
Altitudes Orthocentre (can be ZH
tanA  tanB tanC
H inside, outside or on
A the right angle) ¦zz z – z z  z – z
1 2 3 2 3 1
B C
¦z z – z 1 2 3

114
1
4. Area of the triangle with vertices z1, z2 & z3 is z1 z 2 – z 3
4
5. Equilateral triangle
In equilateral ǻABC , where the vertices are given by z1, z2, z3 and the circumcentre is z0,
i. z12  z 22  z 32 3z 02
ii. z12  z 22  z 32 z 1z 2  z 2 z 3  z 3 z1
1 1 1
iii.   0
z1 – z 2 z 2 – z 3 z 3 – z1
6. If z1,z2,z3,z4 are vertices of a parallelogram if and only if z 1+z3 = z2+z4.
7. If z1,z2,z3,z4 are vertices of a square in the same order, then
i. z 1+ z 3 = z 2+ z 4
ii. z1 – z 2 z 2 – z3 z3 – z4 z 4 – z1

iii. z1 – z 3 z2 – z4

z1 – z 3
iv. is purely imaginary
z2 – z4
8. Equation of the straight line joining z1 & z2 is
§ z – z1 · § z – z1 · z – z1 z – z1
i. arg ¨ ¸ ʌor0 i.e. ¨ ¸ must be real i.e. or
© z 2 – z1 ¹ © z 2 – z1 ¹ z 2 – z1 z 2 – z1

z z 1
z1 z1 1 0 (non-parametric form)
z2 z2 1
ii. z = Ȝ z1+(1– Ȝ )z2 (parametric form)
iii. General equation of a line is az  az  b 0 where b  R.
z 3 – z1
Note: Condition for 3 points z 1,z2,z3 to be collinear is that arg 0orʌ
z3 – z 2

Two points z1 & z2 lie on the same side or opposite side of the line az  a z  b 0 according as
az1  a z1  b and az 2  a z 2  b have same sign or opposite sign.
z1 – z 2
9. Slope of the line segment joining z1 & z2 is .Two lines with slopes Ȧ1 & Ȧ2 are
z1 – z 2
a. perpendicular if Ȧ 1 + Ȧ 2 = 0
b. parallel if Ȧ1 = Ȧ2

115
a z1  az1  b
10. Length of the perpendicular from z1 to a z  az  b 0 is
2a
11. Equation of the perpendicular bisector of the line segment joining z1 & z2 is
z z1 – z 2 + z z1 – z 2 = z1 – z 2
2 2

12. Equation of a circle


i. z – z0 r or z z – z 0 z – z 0 z  z 0 z 0 – r 2 0 where z0 the centre & r is the radius
2 2 2
ii. z – z1  z – z 2 z1 – z 2 (circle described on the line joining z1 & z2 as diameter)

iii. General equation of a circle is zz  a z  az  b 0 where b  R . Centre is –a and radius


is aa – b
z1 – z 3 z 2 – z 4
* Four points z1,z2,z3,z4 are concyclic if and only if z – z z – z is purely real.
1 4 2 3

13. Loci in complex plane


i. z – z0 = r
circle with centre z 0 & radius r.
l z – z 0 < r : interior of this circle

l z – z 0 > r : exterior of this circle

l z – z1 = z – z 2
perpendicular bisector of the segment joining z1 & z2
­isanellipseif k ! z1 – z 2
iii. z – z1 + z – z 2 = k °
®
°̄isastraightlineif k z1 – z 2

­isahyperbolaif k  z1 – z 2
iv. z – z1 – z – z 2 = k °
®
°̄isastraightlineif k z1 – z 2

§ z – z1 ·
v. argument ¨ ¸ a , a fixed angle is a circle.
© z – z2 ¹
14. Complex number as a rotation arrow in the argand plane. Y
Q (zeiD )
Let z = reiș
Ÿ OP = r & ‘XOP ș
ze iĮ is a complex number whose modulus is r and P(z)

argument ‘XOP ș  Į . To obtain ze iĮ , rotate OP through an D


T X
angle Į in anticlockwise direction. O

116
Note: Let z1 & z2 be two complex represented by P & Q such Y
Q (z2 )

that ‘POQ

Į . z1e is a vector of magnitude
z1 i Į
OP z1 along OQ . e is a unit vector along OQ . P(z1 )
z1
D
z X

? A vector of magnitude z 2 OQ units is given by z 2 1 eiĮ O

z1

z2
i.e. z2 z1eiĮ
z1
Note: general formula for rotation
If AB is rotated to AC but AB z AC, then
C (z3 )

B (z2 )

r3

r2

I
T

A(z 1)

z 3 – z1 z 2 – z1 i˳
e
r3 r2
l Multiplication of z with i rotates the vector z through a right angle in anticlockwise direction
' i
1eiʌ / 2 D

l Multiplication of z by – 1 rotates the vector z through Q (z2)

an angle of 1800 in anticlockwise direction.


l Let AB & CD intersect at z 0. Let P(z1) & Q (z 2) be z0
T
A P(z1) B
two points on AB & CD. Then the angle ș is given by
§ z2 – z0 ·
ș = arg ¨ ¸ = arg (z 2– z0) – arg (z 1– z0)
© z1 – z 0 ¹
C

Solved Examples
1. If z1,z2,z3 are the vertices of an equilateral triangle having its circumcentre at the origin such that
z1 = 1+i, find z2 and z3.
A(z1)

2ʌ / 3
2ʌ / 3

2ʌ / 3

B(z2) C(z3)

117
Solution: Clearly, OB and OC are obtained by rotating OA through 2ʌ / 3 and 4 ʌ / 3 respectively..
? OB = OA e i 4 ʌ / 3 and, OC = OA e i 4 ʌ / 3
Ÿ z2 = z1 e i 2 ʌ / 3 and, z 3 = z1 e i 4 ʌ / 3
Ÿ z2 =(1+i) cos 2 ʌ / 3  i sin 2 ʌ / 3 and, z 3 1  i cos 4ʌ / 3  i sin 4ʌ / 3
Ÿ
z2 = (1+i) – 1 / 2  i 3 / 2 and z 3 1  i – 1 / 2 – i 3/2
§ 3  1· § 3 – 1· §1 – 3 · §1 3·
Ÿ z2= – ¨ ¸  i¨ ¸ and z3 = – ¨ ¸ – i¨ ¸
© 2 ¹ © 2 ¹ © 2 ¹ © 2 ¹
Example: 2 Let z1 and z2 are roots of the equation z2+pz+q = 0, where the coefficients p and q may
be complex numbers. Let A and B represent z1 and z2 in the complex plane. If ‘AOB Į z 0 and
OA = OB, where O is origin, prove that p 2 = 4qcos2 Į / 0 .
Solution: Since z1 and z2 are roots of the equation z 2+pz+q = 0
? z1+z2 = –p and z 1z2 = q
Since OA = OB. So, OB is obtained by rotating OA in

anticlockwise sense through angle Į .


? OB = OA e iĮ
Ÿ z2 z1e iĮ
z2
Ÿ cosD  i sinD
z1
z2
Ÿ  1 1  cos Į  i sin Į
z1

118
z2  z1 D§ D D· D iD /2
Ÿ 2 cos ¨ cos  i sin ¸ 2 cos e
z1 2© 2 2¹ 2
z 2  z1 Į
Ÿ 2 cos eiĮ / 2
z1 2
2
§ z 2  z1 · Į iĮ
Ÿ ¨ ¸ 4 cos 2 e
© z1 ¹ 2
2
§ z 2  z1 · Į § z2 ·
Ÿ ¨ ¸ 4 cos 2 ¨ ¸
© z1 ¹ 2 © z1 ¹
Į
Ÿ (z2+z1)2 = 4z1z2cos2
2
Į
Ÿ (–p)2 = 4qcos2
2
Į
Ÿ p2 = 4qcos2
2
Example: 3 If the points A,B,C represent the complex numbers z 1,z2,z3 respectively and the angles
§ ʌ – Į· § Į·
of the ǻABC at B and C are both ¨© ¸ . Prove that (z 3 – z2)2 = 4(z3–z1) (z1–z2)sin2 ¨ ¸ .
2 ¹ © 2¹
Solution: We have,
ʌ–Į
‘B ‘C
2
§ ʌ – Į ʌ – Į·
? A ʌ–¨  ¸ Į
© 2 2 ¹
Since AC is obtained by rotating AB through angle Į .

119
? AC = AB e iĮ
Ÿ (z3 – z1) = (z2 – z1) e iĮ
z 3 – z1
Ÿ cos Į  i sin Į
z 2 – z1
z 3 – z1
Ÿ – 1 –1  cos Į  i sin Į
z 2 – z1
z3 – z 2 Į Į Į
Ÿ –2 sin 2  2i sin cos
z 2 – z1 2 2 2

z3 – z2 Į§ Į Į·
Ÿ 2i sin ¨ cos  i sin ¸
z 2 – z1 2© 2 2¹
z3 – z 2 Į
Ÿ 2i sin eiĮ / 2
z 2 – z1 2
2
§ z3 – z 2 ·
Ÿ ¨ ¸
© z 2 – z1 ¹
4i 2 sin 2
2

Į iĮ / 2
e
2

2
§ z3 – z 2 · Į iĮ
Ÿ ¨ ¸ –4 sin 2 e
© z 2 – z1 ¹ 2
2
§ z3 – z 2 · Į § z 3 – z1 ·
Ÿ ¨ ¸ –4 sin 2 ¨ ¸
© z 2 – z1 ¹ 2 © z 2 – z1 ¹

Į
Ÿ z 3 – z 2 2 4 z 3 – z1 z1 – z 2 sin 2
2
Example: 4 Show that the area of the triangle on the Argand plane formed by the complex
1 2
numbers z, iz and z + iz is z
2
Solution: We have, iz = zeiʌ / 2
This implies that iz is the vector obtained by rotating vector

120
z in anticlockwise direction through 900. Therefore, OA A AB. So,
1
Area of ǻOAB OAxOB
2
1 1 2
= z iz z .
2 2
Example: 5 Let z1,z2,z3 be the affixes of the vertices A,B and C respectively of a ǻABC . Prove
that the triangle is equilateral if z12  z 22  z 32 z1z 2  z 2 z 3  z 3 z1 .
Solution: First, let z1,z2,z3 be the affixes of the vertices A,B,C of an equilateral triangle ABC.
Then, we have to prove that z12  z 22  z 32 z 1z 2  z 2 z 3  z 3 z1 .
Since ǻABC is an equilateral triangle.
Therefore,
AB = BC = AC and ‘A ‘B ‘C ʌ / 3.
Clearly, AC can be obtained by rotating AB in anticlockwise sense through 600.
? z3 – z1 = (z2–z1) e iʌ / 3 ......(1)
ʌ
Also BC can be obtained by rotating BA by anticlockwise
3
? z2 – z3 = (z1–z3) e i ʌ / 3 ......(2)
From (1) and (2)
§ z 3 – z1 · § z 2 – z3 ·
¨ ¸ ¨ ¸
© z 2 – z1 ¹ © z1 – z 3 ¹

Solving we get z12  z 22  z 32 z 1z 2  z 2 z 3  z 3 z1 .

121
PRACTICE QUESTIONS
z – 5i
1. The complex number z = x+ iy which satisfy the equation =1 lie on
z  5i
a. the x-axis b. the straight line y = 5
c. a circle passing through the origin d. none of these
2. Let z & w be two complex numbers such that z d 1, w d 1 and z  iw z – iw = 2 then z =
a. 1 or i b. i or –i c. 1 or –1 d. i or –1
3. Let z1 & z2 be the nth roots of unity which subtend a right angle at the origin, then n must be of
the form (where k  z )
a. 4k+1 b. 4k+2 c. 4k+3 d. 4k
§z –z · 1– i 3
4. The complex number z 1,z2 & z3 satisfying ¨ 1 3 ¸ are the vertices of a triangle
© z2 – z3 ¹ 2
which is
a. of area zero b. right angled isosceles
c. equilateral d. obtuse-angled isosceles
5. For all complex numbers z 1,z2 satisfying z1 12 and z 2 – 3 – 4i =5, the minimum value of
z1 – z 2 is
a. 0 b. 2 c. 7 d. 17
6. The shaded region where P = (–1,0), Q=(–1+ 2 , 2 ), R = (–1+ 2 ,– 2 ), S = (1,0) is
represented by
Y

P
O S X

(PQ=PS=PR)

ʌ ʌ
a. z  1 ! 2, arg z  1  b. z  1  2, arg z  1 
4 2
ʌ ʌ
c. z  1 ! 2, arg z  1 ! d. z – 1  2, arg z  1 !
4 2
7. A man walks a distance of 3 units from the origin towards the North-East (N45 0 E) direction.
Form there, he walks a distance of 4 units towards the North-West (N45 0 W) direction to

122
reach a point P. Then the position of P is the Argand plane is
a. 3e iʌ / 4  4i b. 3 – 4i eiʌ / 4 c. 4  3i eiʌ / 4 d. 3  4i eiʌ / 4
8. A particle P starts from the point z 0 = 1+2i. It moves first horizontally away from origin by 5
units and then vertically away from origin by 3 units to reach a point z 1. From z1 the particle
ʌ
moves 2 units in the direction of the vector î  ˆj and then it moves through an angle in
2
anticlockwise direction on a circle with centre at origin to reach a point z2. The point z2 is given
by
a. 6+7i b. –7+6i c. 7+6i d. –6+7i
9.* Match the following
Column I Column II
a. The set of points z satisfying p. an ellipse with eccentricity 4/5
z–iz z  i z is contained in
or equal to q. the set of points z satisfying Im(z) = 0
b. The set of points z satisfying r. the set of points z satisfying Im z d 1
z4  z–4 0 is contained in
or equal to
c. If w 2 , then the set of points s. the set of points satisfying Re z d 2
1
z w– is contained in or equal to t. the set of points z satisfying z d 3
w
d. If w 1 , then the set of points
1
z w is contained in or equal to
w
10. If a,b,c and u,v,w are complex numbers representing the vertices of two triangles such that
c = (1–r) a +rb, w = (1– r) u + rv, where r is a complex number, then the two triangles
a. have the same area b. are similar
c. are congruent d. none of these
11. The locus of the centre of a circle which touches the circles z – z1 = a and z – z 2 = b externally
is
a. an ellipse b. a hyperbola c. circle d. none of these
12.* If one of the vertices of the square circumscribing the circle z – 1 2 is 2  i 3 , then which of
the following can be a vertex of it?
a. 1 – 3i b. – 3i c. 1 3i d. none of these

13. Read the passage and answer the following questions:


A(z1), B(z2), C(z3) are the vertices a triangle inscribed in the circle z 2 . Internal angle bisector
of the angle A meets the circum circle again at D(z4).

123
i. Complex number representing point D is
z1z 2 z 2z3 z1z 2
a. z4 b. z4 c. z4 d. none of these
z3 z1 z3
ii. argument (z4 / (z2–z3) is
ʌ ʌ ʌ 2ʌ
a. b. c. ` d.
4 3 2 3
iii. For fixed positions of B(z2) and C(z3) all the bisectors(internal) of ‘A will pass through a
fixed point which is
a. H.M. of z 2 and z3 b. A.M. of z 2 and z3
c. G.M. of z 2 and z3 d. none of these
Note:* Questions with more than one option is correct.

ANSWERS

1. a 2. c 3. d 4. c 5. b 6. a 7. d

8. d 9. a o q,r; b o p; c o p,s,t; d o q,s,r,t


,t 10. b 11. b

12. a,b,c 13. (i) d (ii) c (iii) c

124
COMPLEX NUMBERS AND QUADRATIC EQUATIONS - I
Quadratic Equations
Quadratic equations
The general form of a quadratic equation over real numbers is ax 2+bx+c = 0 where a, b,c  R & a z
0. The solution of the quadrati equation ax 2+bx+c = 0 is given by

– b r b 2 – 4ac
x= . The expression b2–4ac is called the discriminant of the quadratic equation
2a
and is denoted by D.
Nature of roots : For the quadratic equation ax 2+bx+c = 0, where a, b, c  R and a z 0, then

D>O D=O D<0


Roots are real and Roots are real & Roots are imaginary{If p+iq is
unequal (distinct) equal (coincident) one of the roots then the other
must be the conjugate p–iq
where p,q  R & i = –1 }
For the quadratic equation ax 2+bx+c = 0 where a, b, c  Q and a z 0, then :

D>0 and is a perfect square. D>0 and not a perfect square.


Roots are rational and unequal (distinct). Roots are irrational{If p+ q is one root,
then the other root must be the conjugate
p – q }ie, Irrational roots occurr in
pairs if a, b, c  Q.
Note1 : If D is a root of ƒ(x) = 0, then the polynomial ƒ(x) is exactly divisible by x– D or (x– D ) is
a factor of ƒ(x) and vice versa.
Note 2 : ax2+bx+c = 0 cannot have three different roots. If it has, then the equation becomes an
identity in x. ie, a = b = c = 0.
Relation between roots and coefficients
If D 1, D 2,......... D n are roots of the equation ƒ(x)=anxn+an–1xn–1+an–2xn–2+....... ..a2x2+a1x+a0 = 0,
then ƒ(x) = an(x– D 1)(x– D 2)(x– D 3)..............(x– D n)
? anxn+an–1xn–1+an–2xn–2+.............+a2x2+a1x+a0 = an (x– D 1) (x– D 2)............(x– D n)
Comparing the coefficients of like powers of x an both sides, we get,
– a n –1 – coeft.of x n –1
S1 = D 1+ D 2+.............+ D n = ¦ Di = an =
coeft of x n

+................ ¦ i j
DD a n–2 coeft.of x n – 2
S 2 = D 1 D 2+ D 1 D 3 = (–1) a = (–1)
22
iz j n coeft of x n
a n –3
D +................ ¦
DDD coeft.of x n –3
S3 = D 1 D 2 D 3+ D 2 D 3 i j k = (–1) a = (–1)
3 3
4 i z jz k n coeft of x n
......................................
.....................................

125
a0 constant term
Sn = D 1 D 2 D 3...................... D n = (–1)n a = (–1)n coeft of xn
n

Here Sk denotes the sum of the products of the roots taken ‘k’ at a time.
Particular cases :-
Quadratic equation : If D & E are the roots of the quadratic equation ax2+bx+c = 0, then
–b c
S1 = D + E = , & S2 = D E =
a a
Cubic equation : If D , E , J are the roots of the cubic equation ax3+bx2+cx+d = 0, then
–b
S1 = D + E + J =
a
c c
S2 = D E + E D + J D = (–1)2 =
a a
d –d
S3 = D E J = (–1)3 =
a a
Biquadratic equation : If D , E , J , G are roots of the biquadratic equation ax4+bx3+cx2+dx+e = 0,
then cons tant term
–b coeft of x n
S1 = D + E + J + G =
a
c
S2 = D E + E J + D G + E J + E G + J G = (–1)2
a
c
Or S2=( D + E ) ( J + G )+ D E + J G =
a
d
S3 = D E J + E J G + J G D + D E G = (–1)3
a
–d
Or S3 = D E ( J + G )+ J G ( D + E ) =
a
e e
and S4 = D E J G = (–1)4 =
a a
Formation of a polynomial equation from given roots
If D 1, D 2, D 3, ................, D n are the roots of an nth degree equation, then the equation is
xn–S1xn–1+S2xn–2–S3xn–3+..............+(–1)n Sn = 0 where Sk denotes the sum of the products of roots
taken k at a time.
Particular cases
Quadratic equation : If D , E are the roots of a quadratic equation, then the equation is
x2–S1x+S2 = 0 ie, x2–( D + E )x+ D E = 0.

126
Cubic equation : If D , E , J are the roots of a cubic equation. Then the equation is ,
x3–S1x2+S2x–S3 = 0 ie, x3–( D + E + J ) x2+( D E + E J + J D ) x– D E J = 0
Biquadratic equation : If D , E , J , G are the roots of a biquadratic equation, then the equation is
x4–S1x3+S2x2–S3x+S4 = 0
i e , x 4– ( D + E + J + G ) x 3+ ( D E + E J + J G + D G + E G + D J ) x 2–
( D J E + D E G + E J G + D J G )x+ D E J G = 0.
Quadratic Expression : An expression of the form ax2+bx+c, where a, b, c R & a z 0 is called a
quadratic expression in x. So in general quadratic expression is represented as: ƒ(x)=ax2 +bx+c or y =
ax2+bx+c.
Graph of a quadratic Expression
Let y = ax2+bx+c where a z 0.

§ 2 b c· § 2 bx b 2 c b 2 ·
Then y = a ¨ x  x  ¸ Ÿ y = a ¨x   2  – 2¸
© a a¹ ¨ a 4a a 4a ¸¹
©
2 2
b 2 – 4ac § b · D § b ·
Ÿy + =a ¨ x  ¸ Ÿ y+ = a¨x  ¸
4a © 2a ¹ 4a © 2a ¹
D b
Let y+ = Y & x+ =X
4a 2a
Y
? Y = aX2 or X2 =
a
§ – b – D·
Clearly it is the equation of a parabola having its vertex at ¨ , ¸.
© 2a 4a ¹
If a>0, then the parabola open upwards.
If a<0, then the parabola open downwards.
Sign of quadratic Expression
(1) The parabola will intersect the x–axis in two distinct points if D>0.
(i) a>0 (ii) a<0

D E D E
X X

Let ƒ(x) = 0 have 2 real roots Let ƒ(x) = 0 have 2 real roots
D & E ( D < E ).Then ƒ(x)>0 D & E ( D < E )Then ƒ(x)<0
 x  (– f , D ) ‰ ( E f ) and ƒ(x)<0  x  (– f , D ) ‰ ( E f )
 x ( D , E ) & ƒ(x)>0 for all x  ( D , E )

127
(2) The parabola will touch the x–axis at one point if D = 0
(i) a>0 (ii) a<0
X

X
ƒ(x) t 0  x R ƒ(x) d 0  x R
(3) The parabola will not intersect x–axis if D<0.
(i) a>0 (ii) a<0

ƒ(x)>0  x R ƒ(x)<0  x R
NOTE : Condition that a quadratic function ƒ(x,y) = ax +2hxy+by2+2gx+2ƒy+c may be resolved
2

into two linear factions is that


a h g
h b f
abc+2ƒgh–aƒ2–bg2–ch2 = =0
g f c
NOTE :
–b –D
(i) For a>0, ƒ(x) = ax2+bx+c has least value at x= . This least value is given by
2a 4a
–b –D
(ii) For a<0, ƒ(x) = ax2+bx+c has greatest value at x= . This greatest value is given by
2a 4a
Solved Examples
1 If D , E are roots of ax2+bx+c = 0 ; D +h and E +h are roots of px2+qx+r = 0 and D1, D2 are their
discriminants, then D1: D2 =
a2 b2 c2
(a) (b) (c) (d) None of these
p2 q2 r2
Solution : D – E = ( D +h)–( E +h)
Ÿ ( D – E )2 = (( D +h) – (E +h))2
Ÿ ( D + E )2 –4 D E = (( D +h)–
+ (E +h))2 – 4( D +h)( E +h)
2 2
§ –b· c §–q· 4r
¨ ¸ –4 = ¨¨ ¸¸ –
© a ¹ a © p ¹ p

128
b 2 – 4ac q 2 – 4pr Ÿ D1 D2
= 2 =
a2 p2 a p2

D1 a2
ŸD =
2 p2
Ans : (a)
2 If a Z and the equation (x–a)(x–10)+1 = 0 has integral roots, then the values of a are
(a) 10, 8 (b) 12, 10 (c) 12, 8 (d) None of these
Solution : (x–a)(x–10) = – 1
Ÿ x–a = 1 & x–10 = –1 OR x–a = –1 & x–10=1
9–a = 1 x= 10 – 1 11 – a = – 1 x = 11
a=8 x=9 a =12
Ans : (c)
3 If D , E are roots of the equation (x–a)(x–b)+c = 0 (c z 0), then then roots of the equation (x–c–
D )(x–c– E ) = c are
(a) a and b+c (b) a+c and b (c) a+c and b+c (d) None of these
Solution: x2–(a+b)x+ab+c = 0
Ÿ D + E = a+b and D E = ab+c
Now (x–c– D )(x–c– E ) = c
Ÿ (x–c)2 –( D + E )(x–c)+ D E –c = 0
(x–c)2–(a+b)(x–c) +ab = 0
(x–c)2– a(x–c) –b(x–c) +ab = 0
Ÿ (x–c–a)(x–c–b) = 0
? x = c+a and b+c
Ans (c)
4 Let ' 2 be the discriminant and D , E be the roots of the equation ax2+bx+c = 0. Then 2a D + ' and
2a E – ' can be roots of the equation
2
(a) x2+2bx+b2 = 0 (b) x2–2bx+b = 0
(c) x2+2bx–3b2–16ac = 0 (d) x2+2bx–3b2+16ac = 0

– b r '2
Solution : D , E =
2a
–b' –b–' –b–' –b'
D= and E = Or D= and E =
2a 2a 2a 2a
2a D = –b+ ' 2a E = –b– ' Ÿ 2a D + ' = – b & 2a E – ' = – b
2a D + ' = –b+2 ' 2a E – ' = –b–2 ' ? S = –2b and P = b2
? S = – 2b and P = +b2–4 ' 2
= +b2–4(b2–4ac)
D
S = (2a + ' )+(2a E - ') = - 2b = –3b2+16ac Heve S means sum of the roots
P = (2a D + ' ). (2a E- ') = 16ac -3b
2
P means product of the roots

129
? quadatic equation is ? quadratic equation is
x +2bx–3b +16ac = 0
2 2
x2+2bx+b2 = 0
Ans : (a) and (d)
5 The polynomial equation (ax2+bx+c)(ax2–dx–c) = 0 , ac z 0 has
(a) four real roots (b) atleast two real roots
(c) atmost two real roots (d) No real roots
Solution : ac z 0
Ÿ ac>0 or ac<0
Now D1=b2–4ac & D2 = d2+4ac
When ac> 0 Ÿ D2>0 but D1 may positive or negative
When ac<0 Ÿ D1>0 but D2 may be positive or negative
In either case the polynomial has atleast two real roots
Ans (b)
D 2  2D  1 E 2  2E  1
6 If D , E are roots of x –p(x+1) –q = 0 , then the value of 2
2
+ is
D  2D  q E2  2E  q
(a) 1 (b) 2 (c) 3 (d) None of these
Solution : x2–px–p–q = 0
Ÿ D +E = p and D E = – p–q
Now ( D +1)( E +1) = ( D + E )+ D E +1
= p–q–p+1 = 1–q
D  2D  1
2
E  2E  1
2

Now 2 + 2
D  2D  q E  2E  q
(D  1) 2 (E  1) 2
= +
(D  1) 2  q – 1 (E  1) 2  q – 1
(D  1) 2 (E  1) 2
= +
(D  1) 2 – (D  1)(E  1) (E  1) 2 – (D  1)(E  1)
1 1 D+ 1 E +1
+ = +
= E +1 (D+1 ) (D+1 ) – (E +1) E + 1– (D+1 )

1 D ( +1 ) 1 – E +1

D 1 E D 1 – E –1
= + =
D –E E–D D –E
D –E
Ÿ D –E = 1
Ans (a)
7 Let D , E , J be the roots of the equation x3+4x+1 = 0, then ( D + E )–1+( E + J )–1+( J + D )–1 equals
(a) 2 (b) 3 (c) 4 (d) 5
Solution : D + E + J = 0, D E + E J + J D = 4, D E J = –1
1 1 1 1 1 1 DE  EJ  JD § 4 ·
? DE+E J + JD = – J – –E = – = –¨ ¸ = 4
D DEJ © – 1¹
Ans : (c)
130
PRACTICE QUESTIONS
1 The minimum value of ƒ(x) = x2+2bx+2c2 is greatest than the maximum value of
g(x)= –x2–2cx+b2, then (x being a real )
|b| |c|
(a) |c|> (b) >|b|
3 2
(c) –1<c< 2 b (d) Non real value of b &c exist
2 If P(x) is a polynomial of degree less than or equal to 2 and S is the set of all such polynomials so that
2
P(1) = 1, P(0) = 0 and P1(x)>0  x  [0,1], then S = Here, P(x) = b x + ax + c
(a) I (b) {(1–a)x2+ax, 0<a<2}
(c) {(1–a)x2+ax,a>0} (d) {(1–a)x2+ax,0<a<1}
3 In the quadratic equation ax2+bx+c = 0 if ' = b2–4ac and D + E , D 2+ E 2 and D 3+ E 3 are in G.P, .P,
where D , E are the roots of the equation, then
(a) ' z 0 (b) b ' = 0 (c) c ' = 0 (d) ' = 0
4 If a, b, c are the sides of a triangle ABC such that x –2(a+b+c)x+3 O (ab+bc+ca) = 0 has real roots,
2

then
4 5 § 4 5· §1 5·
(a) O< 3 (b) O> 3 (c) O ¨ 3 , 3¸ (d) O ¨3, 3¸
© ¹ © ¹
5 Let D & E be the roots of x2–6x–2 = 0, with D > E . If an = D n – E n for n t 1, then the value of
a10 – 2a 8
2a 9 is

(a) 1 (b) 2 (c) 3 (d) 4


6 If x –10ax–11b = 0 have roots c & d. x –10cx–11d = 0 have roots a & b, then a+b+c+d is
2 2

(a) 1210 (b) 1120 (c) 1200 (d) None of these


1 1
7 If tn denotes the nth term of an A.P. and tp = and tq = , then which of the following is necessarily
q p
a root of the equation (p+2q–3r)x +(q+2r–3p)x+(r+2p–3q) = 0 is
2

(a) tp (b) tq (c) tpq (r) tp+q


8 The curve y = ( O +1) x +2 intersect the curve y = O x+3 in exactly one point, if O equals
2

(a) {–2, 2} (b) {1} (c) {–2} (d) {2}


9 Read the passage and answer the following questions.
Consider the equation x4+(1–2k) x2+k2–1 = 0 where k is real. If x2 is imaginary, or x2<0, the equation
has no real roots. If x2>0, the equation has real roots.
(i)* The equation has no real roots if k
§ 5· §5 ·
(a) (– f –1) (b) (–1,1) (c) ¨1, ¸ (d) ¨ ,f¸
© 4¹ ©4 ¹
(ii) The equation has only two real roots if k
(a) (– f –1) (b) (0, 1) (c) (1, 2) (d) (–1, 1)

131
(iii) The equation has four real roots if k
§ 5·
(a) (– f ,0) (b) (–1, 1) (c) ¨1, ¸ (d) (1, f )
© 4¹
10 If D , E are the roots of the equation ax 2 +bx+c = 0, then the value of

1 cos(E – D) cos D
cos(D – E) 1 cos E
is
cos D cos E 1

(a) sin( D + E ) (b) sin D sin E (c) 1+cos( D + E ) (d) None of these
11* If (1+k) tan2x–4tan x–1+k = 0 has real roots, then
S
(a) k2 d 5 (b) tan(x1+x2)=2 (c) for k=2, x1=
(d) for k =1, x1 = 0
4
12 If p,q {1,2,3,4}, the number of equations of the form px2+qx+1 = 0 having real roots is
(a) 15 (b) 9 (c) 7 (d) 8
S p Q
13 In ' PQR ‘ R = . If tan & tan are the roots of the equation ax2+bx+c = 0 (a z 0) then
2 2 2
(a) a+b=c (b) b+c = 0 (c) a+c = b (d) b = c

Note : * Questions with more than one option is correct

Answers
1. b 2. b 3. c 4. a 5. c
6. a 7. c 8. c 9 (i) a, d (ii) d (iii) c
10. d 11. a, b, c, d 12. c 13. a

132
COMPLEX NUMBERS AND QUADRATIC EQUATIONS - II
Quadratic Equations(Location o f Roots)
Let ƒ(x) = ax 2+bx+c, a,b,c  R , a z 0 and Į, ȕ Į  ȕ be the roots of ƒ(x)=0. Let k be any
real number

Cases Figure Condition


Į  k and i. a > 0 i. D t 0 (roots may be equal)
D E
ȕ  k Both the k
X
ii. a.ƒ(k) > 0
roots are less than k iii. 2k > Į  ȕ i.e 2k > sumof
D Ek
X –b
ii. a < 0 roots or k >
2a

Į ! k and ȕ ! k i. a > 0 k
D E
X i. D t 0 (roots may be equal)
Both the roots are ii. a.ƒ(k) > 0
greater than k iii. 2k < Į  ȕ i.e 2k < sumof
kD E
X
–b
ii. a < 0 roots or k <
2a

D k E
X

Įkȕ i. a > 0 i. D > 0


k lies between (distinct roots)
the roots D E
X ii. a ƒ(k) < 0
k
ii. a < 0

Wavy Curve Method


Let ƒ(x) = x – a 1 k 1 x – a 2 k 2 …… x – a n k n – (1)
Where k i  Ni & ai  R such that a1 < a2 < ……< an. Mark a1,a2…… an on real axis check the
sign of ƒ(x) in each interval. The solution of ƒ(x) > 0 is the union of all intervals in which we have put
plus sign and the solution of ƒ(x) < 0 is the union of all intervals in which we have put the minus sign.

Exponential Equations
If we have an equation of the form
ax = b where a > 0, then
x if b d 0 ; x = log a b if b > 0, a z 1 ;
x  if a = 1, b z 1 ; x  R , if a = 1, b = 1

133
Lagrange’s Identity
If a1, a2, a3, b1, b2, b3  R then
a 2
1
 a 22  a 32 b12  b 22  b 32 – a 1 b1  a 2 b 2  a 3 b 3 2

= a 1 b 2 – a 2 b1 2 + a 2 b 3 – a 3 b 2 2 + a 3 b1 – a 1 b 3 2
a c e
Note: If , then each ratio is equal to
b d f
a  c  e  ....
i.
b  d  f  ....

§ pa n  qc n  re n  .... ·
1/ n

ii. ¨ n ¸ where p, q, r, n  R
© pb  qd n  rf n  ....¹
n
ac ace...
iii. n
bd bdf ...
SOLVED EXAMPLES
1. The values of m for which both roots of the equation x2–mx+1=0 are less than unity is
a. – f,–2 b. (–f,–2] c. – 2, f d. none of these
Solution:
Dt0 a.f(1) > 0 Įȕ  2
(–m)2 –4.1.1 t 0 1.(1–m+1) > 0 m<2
(m –2) (m+2) t 0 Ÿm<2 Ÿ m  – f,2 .......(3)
Ÿ m  (–f,–2] ‰ [ 2, f ) .... (1) m  – f,2 ....(2) Form (1), (2) and (3) we have
m  (– f,–2]

Ans: b

2. The values of m m  R , for which both roots of the equation x2–6mx+9m2–2m+2 = 0 exceed
3 is
a. (–f,1] b. – f,1 c. [1, f) d. none of these
Solution:
Dt0
(–6m)2 –4.1.(9m2 –2m+2) t 0 a.f(3) > 0 Įȕ ! 6
8m–8 t 0 1.(3 –18m+9m –2m+2) > 0 6m > 6
2 2

Ÿ m t 1 ........(1) 9m2 – 20m + 11 > 0 m > 1 ...(3)


9m – 9m – 11m + 11 > 0
2

(9m – 11) (m–1) > 0


11
Ÿ < ......(2)
9

134
§11 ·
From (1), (2) and (3) m  ¨© , f ¸¹
9
Ans: d

3. The values of p for which 6 lies between the roots of the equation x 2+2(p–3)x+9 = 0 is
§ 3· § 3º
a. ¨ – f,– ¸
© 4¹
b. ¨ – f,– »
© 4¼
c. – f,1@ d. none of these

Solution: D ! 0 a.ƒ(6) < 0


(–2(p–3))2 –4.1.9 > 0 1.(36+12(p–3) +9)) < 0
p2–6p > 0 Ÿ 12p+9 < 0
p (p – 6) > 0
–3
p > 0, p> 6 .....(1) P< .....(2)
4
§ – 3·
From (1) and (2) p  ¨© – f, ¸¹
4
Ans: a

4. If a, b, c  R , and the equation ax 2+bx+ c = 0 has no real roots, then


a. (a+b+c) > 0 b. a(a+b+c) > 0
c. b (a+b+c) > 0 d. c(a+b+c) < 0
Solution:
a>0 a<0
ƒ(0) > 0 Ÿ c > 0 ƒ(0) < 0 Ÿ c < 0
ƒ(1) > 0 Ÿ a+b+c > 0 ƒ(1) < 0 Ÿ a+b+c < 0
aƒ(1) > 0 & cƒ(1) > 0 ? aƒ(1) > 0 & c.ƒ(1) > 0
a(a+b+c) > 0 and c (a+b+c) > 0 a(a+b+c) > 0 and c (a+b+c) > 0
Ans: b

PRACTICE QUESTIONS
1. If the roots of equation x –2ax+a2+a–3 are less than 3, then
2

a. a<2 b. a>4 c. 3<a<4 d. –2 < a < 3


2. Read the following passage and answer the questions:-
ƒ(x) = ax 2+bx+c = a x – Į x – ȕ , where Į  ȕ are the roots of ƒ(x) = 0. If ǻ b 2 – 4ac is
negative, then its sign is same as that of a, the coefficient of x2. If ƒ(x) = a x – Į x – ȕ , where
Į  ȕ , a is positive, then for any number p which lies between Į & ȕ; ƒ(p) is negative and for
any number q or r which do not lie between D & E , ƒ(q) or ƒ(r) both will be positive. Also if a2
< x 2 < b2, then a < x < b or –b < x < –a.
i. If x 2 – 2 4Ȝ – 1 x + 15Ȝ2 – 2Ȝ – 7 > 0  x  R , then Ȝ 
a. (0,2) b. (1,3)
c. (2,4) d. none of these

135
ii. Let ƒ(x) be a quadratic expression which is positive for all real x.If g(x) = ƒ(x)
+ ƒ c( x )  ƒ cc( x ), then for any real x,
a. g(x) > 0 b. g(x) t 0 c. g(x) d 0 d. g(x) < 0
x2 – x – 2
iii. The inequality > 2 holds only if
2 x – x2 – 2

–2 2
a. –1 x  only b. only for  x 1
3 3
2 2
c. –1 < x < 1 d. –1 < x < or  x  1
3 3

iv* for real x, the function


x – a x – b will assume all real values, provided
x–c
a. a<b<c b. a>b>c
c. a>c >b d. a<c<b
3. Values of ‘a’ for which the roots of the equation (a+1)x 2–3ax+4a = 0 a z –1 greater than
unity is
ª – 16 º § – 16 ·
a. a « ,–1» b. a ¨ ,–1¸
¬ 7 ¼ © 7 ¹

§ – 16 º
c. a ¨ ,–1» d. none of these
© 7 ¼
4*. If x  R satisfies (log10(100x))2 + (log10(10x))2 + log10x d 14 , then the solution set contains
the interval
a. 1,10@ b. >10 –9 / 2
@
,1 c. 0, f d. – 1, f
5*. If a, b are the real roots of x 2+px+1 = 0 and c, d are the real roots of x 2+qx+1 = 0, then
(a – c) (b – c) (a + d) (b + d) is divisible by
a. a–b–c–d b. a+b+c–d
c. a+b+c+d d. a+b–c–d

6. Match the following:-


Column I Column II
§ 5 – 1º
a. The value of x for which loge(x – 3) < 1 is (p) ¨ 0, »
© 2 ¼
b. The value of x for which log1/2x t log1/3x is (q) (0, 1)
c. If log0.3(x – 1) < log0.09(x– 1), then x lies in the interval (r) (2, 8)
d. If log cos x sin x t 2 and x >0,3ʌ@ then sinx lies in the interval (s) (3, 3 + e)

§ 5  1º
(t) ¨ 0, »
© 2 ¼
136
7. Read the paragraph and answer the questions that follow:


Let a  b + a – b = A, where Ȝ  N, A  R and a – b = 1 ? a  b a – b
Qx Q x –2Ȝ
2
1
i.e Ÿ a r b a  b or a – b
r1 r1

If 4  5 + 4 – 5 = 62, then
> @x > @ x
i.
a. x >– 3,–2 ‰ >1,2 b. x >– 3,2 ‰ >– 2,1
c. x >– 2,–1 ‰ >2,3 d. x >– 2,3 ‰ >– 1,2

Solution of 2  3
x 2 – 2 x 1 x 2 – 2 x –1 4
ii.  2– 3 are
2– 3
a. 1 r 3 ,1 b. 1 r 2 ,1
c. 1 r 3 ,2 d. 1 r 2 ,2
iii. The number of real solutions of the equation 15  4 14  15 – 4 14
t
t
30 are where
t = x 2–2 x
a. 0 b. 2
c. 4 d. 6
3x 2  9 x  17
8. The maximum value of ƒ(x) = is 5k+1, Then k is
3x 2  9 x  7
a. 41 b. 40 c. 8 d. none of these
x 2 – yz y 2 – zx z 2 – xy
9. If , then (x+y+z) (a+b+c) is
a b c
x yz
a. ax+by+cz b. a+b+c c. d. none of these
3
log 3 x 2 – 2 log x 9
10 The value of x satisfying the equation x – 1 x – 1 7 is
a. 3 b. 9 c. 27 d. 81
If x – 9 x  20 ! x – 9 x  20 then which is true?
2 2
11.
a. x d 4 or x t 5 b. 4dxd5
c. 4<x<5 d. none of these
12. If x2+px+1 is a factor of ax 3+bx+c, then
a. a2+c2 = –ab b. a2– c2 = –ab
c. a2– c2 = ab d. none of these
Ȝ ȝ
13. If Ȝ z ȝ and Ȝ2 5Ȝ – 3, ȝ 2 5ȝ – 3 , then the equation whose roots one ȝ and is
Ȝ
a. x2–5x–3 = 0 b. 3x +19x+3 = 0
2

c. 3x 2–19x+3 = 0 ‘ d. x 2+5x–3 = 0

137
14. If the equation (cos p – 1)x 2+ x (cos p) + sin p = 0, in the variable x, has real roots then ‘p’
can take any value in the interval.
a. 0,2ʌ b. – ʌ,0
§ – ʌ ʌ·
c. ¨ , ¸
© 2 2¹ d. 0, ʌ
15. If (cosD +isinD) is a root of the equation ax 2+bx+c = 0, a, b, c  R , then
a. a cos 2Į  b sin Į  c 0 b. a cos 2Į  b cos Į  c 0
c. a sin 2Į  b sin Į  c 0 d. none of these

Note:* Questions with more than one option is correct.

ANSWERS

1. a 2. (i) c (ii) a (iii) d (iv) c,d 3. a 4. a,b 5. c,d

6. a o s; b o q; c o r; d o p 7. (i) c (ii) b (iii) c 8. c 9. a 10. d

11. c 12. c 13. c 14. d 15. b

138
COMPLEX NUMBERS AND QUADRATIC EQUATIONS - III
Quadratic Equations(Location o f Roots)
Let ƒ(x) = ax 2+bx+c, a, b, c  R , a z 0 and Į, ȕ Į  ȕ be the roots of ƒ(x) = 0. Let k 1, k2 be two
real numbers such that k1 < k2

Cases Figure Conditions

Exactly one root k1


D k2 E
X i. D > 0 (distinct roots)
lies is the interval (k1,k2) i. a > 0 ii. ƒ(k1) ƒ(k2) < 0

k1 D E
X
k2
ii. a < 0

E
Both the roots lie i. a > 0 k1
D
k2
X i. D t 0
between k1 & k2 (roots may be equal)
k1 < Į d ȕ  k 2 ii. a < 0 k1 D E k2
ii. a ƒ(k1) > 0 & a ƒ(k2) > 0
X

–b
iii. k1 <  k2
2a
k1 & k2 lie between i. a > 0 D k1 k2 E
X
i. D > 0 (distinct roots)
the roots Į  k1  k 2  ȕ ii. a ƒ(k1) < 0 & a ƒ(k2) < 0
D E
ii. a < 0 k1 k2
X

Logarithmic Equations
If we have an equation of the form as log a ƒ(x) = b where a > 0, a z 1 can be written as ƒ(x) = a b
when ƒ(x) > 0.

Logarithmic Inequalities

For a > 1 For 0 < a < 1

0 < x < y and 0 < x < y and


log a x  log a y are log a x ! log a y are
equivalent equivalent

log a x  p log a x  p
Ÿ0 < x < aP Ÿx > aP

log a x ! p log a x ! p
Ÿ x > aP Ÿ 0 < x < aP

139
Descartes Rule of signs
The maximum number of positive real roots of a polynomial equation ƒ(x) = 0 is the number of
changes of signs from positive to negative and negative to positive.
The maximum number of negative real roots of a polynomial equation ƒ(x) = 0 is the number of
changed signs from positive to negative and negative to positive in ƒ(– x) = 0
Solved examples
1. The values of m for which exactly one root of x2 – 2mx + m2 –1= 0 lies in the interval (–2, 4) is
a. (–3, –1) ‰ (3, 5) b. (–3, –1) c. (3, 5) d. none
Solution: D > 0 ƒ(–2)ƒ(4) < 0
(–2m)2 –4.1.(m2–1) > 0 (4+4m+m2–1) (16 –8m+m2–1) < 0
4>0 (m2 +4m+3) (m2 –8m+3)
Ÿ m  R ......(1) (m +1) (m + 3) (m – 3) (m – 5) < 0
Ÿm  (–3, –1) ‰ (3, 5) ......(2)
From (1) and (2) , m  (–3, –1) ‰ (3, 5)

Ans: a
2. The values of a for which both the roots of the equation 4x 2 –2x + a = 0 lie in the interval
(–1, 1) is.
§ 1º § 1º
a. – 2, f b. ¨ – f, »
© 4¼ c. ¨© – 2, 4 »
¼
d. none of these
Solution:
Dt0 a.ƒ(–1) > 0 a. ƒ(1) > 0
(–2)2 – 4.4.a t 0 4.(4+2+a) > 0 4.(4 –2+a) > 0
4a –1 d 0 a+6 > 0 a > –2
1
ad – (1) a> –6 – (2) a  (–2, f ) – (3)
4
§ 1º
From (1),(2) and (3),a  ¨ – 2, »
© 4¼
Ans: c
3. The all possible values of a for which one root of the equation (a –5)x 2 –2ax+a – 4 = 0 is
smaller than 1 and the other greater than 2 is
a. [5, 24) b. (5, 24] c. (5, 24) d. none of these
Solution: D t 0 (a –5) ƒ(1) < 0 (a –5) ƒ(2) < 0
(–2a)2– 4(a – 5) (a – 4) t 0 (a –5) (a–5 –2a+ a –4) < 0 (a –5) (4(a –5) –4a + a –4) < 0
Ÿ 9a –20 t 0 Ÿ (a –5) 9 > 0 Ÿ(a–5)(–9) < 0 Ÿ(a – 5) (a – 24) < 0
20
a t – (1) a>5 – (2) Ÿ5 < a < 24 –(3)
9
From (1), (2), and (3) a  (5,24)
Ans: c
4. If a, b, c  R and the equation x2 + (a+b) x + c = 0 has no real roots, then

140
a. c (a+b+c) > 0 b. c + (a+b+c) c > 0
c. c – (a+b–c) c > 0 d. c (a+b–c) > 0
Solution:
ƒ(0) > 0 Ÿc > 0 ƒ(0) < 0 Ÿc > 0
ƒ(1) > 0 Ÿ1+a+b+c > 0 ƒ(1) < 0 Ÿ1+a+b+c < 0
ƒ(–1) > 0 Ÿ– (a+b) + c > 0 ƒ(–1) < 0
? ƒ(0). ƒ(1) > 0 and ƒ(0). ƒ(–1) > 0 Ÿ1– (a+b) + c < 0
gives b and c ? ƒ(0) ƒ(1) > 0 and ƒ(0). ƒ(–1) > 0
gives (b) and (c)
Ans b and c

PRACTICE QUESTIONS
1. The values of a for which 2x 2–2(2a+1)x + a(a+1) = 0 may have one root less than a and other
root greater than a are given by
a. 1>a>0 b. –1< a < 0 c. a t0 d. a > 0 & a < –1
2. The value of a for which the equation (1–a )x +2ax–1= 0 has roots belonging to (0, 1) is
2 2

1 5 1 5
a. a! b. a>2 c.  a  2 d. a> 2
2 2
3. If a, b, c, x, y, z,  R be such that (a+b+c) 2=3(ab+bc+ca–x 2–y2–z2), then
a. a=b=c=0=x=y=z b. x = y = z = 0, a = b = c
c. a = b = c = 0; x = y = z d. x=y=z=a=b=c
4. Number of positive integers n for which n2+96 is a perfect square is
a. 8 b. 12 c. 4 d. infinite
5. The curve y = Ȝ  1 x 2  2 intersects the curve y = Ȝx  3 is exactly one point, if Ȝ equals
a. {–2, 2} b. {1} c. {–2} d. {2}
6. A quadratic equation whose product of roots x 1 & x 2 is equal to 4 and satisfying the
x1 x2
relation x – 1  x – 1 2 is
1 2

a. x2–2x+4 = 0 b. x 2+2x+4 = 0 c. x2+4x+4 = 0 d. x2–4x+4 = 0


7. If a,b,c,d  R , then the equation (x +ax–3b) (x –cx+3b) (x –dx+2b) = 0 has
2 2 2

a. 6 real roots b. at least 2 real roots


c. 4 real roots d. 3 real roots
8. Suppose P,Q,R are defined as P=a 2b+ab2–a2c–ac2, Q=b2c+bc2–a2b–ab2 & R=a2c+ac2–b2c–
bc2, where a>b>c and the equation Px 2+Qx+R=0 has equal roots, then a,b,c are in
a. A.P b. G.P c. H.P d. AGP
9. If a (p+q)2+2abpq+c=0 & a(p+r) 2+2abpr+c=0 a z 0 then
c
qr = p 
2
a. qr=p2 b. c. qr= – p2 d. none of these
a

141
10. x2–xy+y2–4x–4y+16=0 represents
a. point b. a circle
c. a pair of straight line d. none of these
k 1 k  2
11. If the roots of the equation ax 2+bx+c=0 are of the form & , then (a+b+c)2 is equal
k k 1
to
a. 2b2– ac b. ¦a 2
c. b2–4ac d. b2–2ac
12. Read the passage and answer the following questions:-
aƒ( ȝ ) < 0 is the necessary and sufficient condition for a particular real number ȝ to the between
the roots of a quadratic equation ƒ(x)=0, where ƒ(x)= ax2+bx+c. Again if ƒ ȝ1 ƒ ȝ 2 <0, then

exactly one of the roots will lie between ȝ1 & ȝ 2 .


1. If b ! a  c , then
a. One root of ƒ(x)=0 is positive, the other is negative.
b. Exactly one of the roots of ƒ(x)=0 lies in (–1, 1).
c. 1 lies between the roots of ƒ(x)=0.
d. Both the roots of ƒ(x)=0 are less than 1
2. If a(a+b+c) < 0 < (a+b+c) c, then
a. one root is less than 0, the other is greater than 1.
b. Exactly one of the roots lies in (0, 1)
c. Both the roots lie in (0, 1)
d. At least one of the roots lies in (0, 1)
3. If (a+b+c) c < 0 < a(a+b+c), then
a. one root is less than 0, the other is greater than 1
b. one root lies in – f, o and the other in (0, 1)
c. both roots lie in (0, 1)
d. one root lies in (0, 1) and other in 1, f
13. Match the following:-
Column I Column II
(Number of positive integers for which) (p) 0
a. One root is positive and the other
is negative for the equation
(m–2)x2–(8–2m)x – (8–3m) = 0
b. Exactly one root of the equation (q) infinite
x2–m(2x–8)–15=0 lies in the interval
(0, 1) (r) 1
c. The equation x2+2(m+1)x+9m–5=0 has
both roots negative
d. The equation x2+2(m–1)x+m+5=0 has (s) 2
both roots lying on either sides of 1

142
1
14. If Į, ȕ are the roots of 375x2–25x–2=0 & Sn = Į n  ȕ n , then the value of is.....
§ n
·
3¨ lim ¦ Sr ¸
© n of r 1 ¹

1 1 1
15. If x,y,z are distinct positive number such that x  y z , then xyz = .......
y z x

ANSWERS

1. d 2. b 3. a.b 4. c 5. c 6. a 7. b 8. c 9. b 10. a 11. c

12. (i) b (ii) a (iii) b 13. a orb or c oqd op  

143
COMPLEX NUMBERS AND QUADRATIC EQUATIONS - IV
Quadratic Equations
Problem solving skills.
If one root of ax2+bx+c=0 is n times the other, then (n+1)2 ac = nb2
If one root of ax2+bx+c=0 is square of the other, then (a2c)1/3+(ac2)1/3+b=0
If D , E are roots of ax2+bx+c=0 then
(i) – D ,– E are roots of ax2–bx+c=0
1 1
(ii) , are roots of cx2+bx+a=0 ; ac z 0
D E
(iii) k D , k E are roots of ax2+kbx+k2c=0
(iv) D 2, E 2 are roots of a2x2–(b2–2ac)x+c2=0
If the sum of the coefficient of ƒ(x) = 0 is 0, then 1 is always a root of ƒ(x) = 0. Also x–1 is a facter
of ƒ(x).
In particular, for ax2+bx+c=0 if
c c
a+b+c=0, then 1 is always a root and the other root = (' product of roots = ).
a a
ƒ(x)=(x–a1) +(x–a2) +.....................+(x–an) , where ai R  i.
2 2 2

a 1  a 2  .......  a n
ƒ(x) assumes its least value when x=
n
While solving an equation, if you have to square, then additional roots will occur as the degree of
the equation will change. In such cases, you have to check whether the roots satisfy the original
equation or not.
Solved Examples
1 If D , E are roots of the equation x2–2x+3=0
Then the equation whose roots are
D 3–3 D 2+5 D –2 and E 3– E 2+ E +5 is
(a) x2+3x+2=0 (b) x2–3x–2=0 (c) x2–3x+2=0 (d) None
Solutions :
D 2–2 D +3=0 and E 2–2 E +3=o
? D 3=2 D 2–3 D and E 3=2 E 2–3 E
? P= D 3–3 D 2+5 D –2 = 2 D 2–3 D –3 D 2+5 D –2 = – D 2+2 D –2
=3–2 = 1
Similarly, we can show that Q = E 3– E 2+ E +5 = 2
? Sum = 1+2 = 3 and product = 1×2=2
Hence x2–3x+2=0
Ans (c)
1
2 If D , E are roots of the equatio x2+px– =0,  p  R–{0}, then the minimum value of D 4+ E 4
2p 2

144
is
(a) 2 2 (b) 2 – 2 (c) 2 (d) 2 2
Solutions :
D 4+ E 4 = ( D 2+ E 2)2 –2 D 2 E 2 = (( D + E )2–2 D E )2 – 2( D E )2
2
§ 2 1 · 1 1
= ¨¨ p  2 ¸¸ – = p 4
+ +2
© p ¹ 2p 4 2p 4

2
§ 2 1 ·
= ¨¨ p – ¸
2 ¸ + 2+ 2
© 2 p ¹
? Min value is 2+ 2 .
Ans (d)
1 1
3 Let p(x) be a polynomial of least possible degree with rational coefficients, having 3 + 3 as
7 49
one of its roots, then the product of all roots of p(x) = 0 is
(a) 56 (b) 63 (c) 7 (d) 49
Solutions :
1 1
Let x = +
73 49 3
3 3 11
§ 1· § 1· § 1 1·
Cubing x = ¨ 7 3 ¸ + ¨ 49 3 ¸ +3. 7 . 49 3
3 3 ¨ 7 3  49 3 ¸
¨ ¸ ¨ ¸ ¨ ¸
© ¹ © ¹ © ¹

§ 1 2·
¨ 7 3  49 3 ¸
Ÿ x3 = 7+49+3.7. ¨ ¸
© ¹
Ÿ x3 = 56+21x
Ÿ x3+0 x2–21x–56 = 0
? Product of roots is 56
Ans : (a)
4 If D , E , J , G are roots of x4+4x3–6x2+7x–9=0, then the value of (1+ D 2)(1+ E 2)(1+ J 2)(1+ G 2) is
(a) 9 (b) 11 (c) 13 (d) 5
Solution :
x4+4x3–6x2+7x–9 = (x– D )(x– E )(x– J )(x– G )
Put x=i, i4+4i3–6i2+7i–9=(i– D )(i– E )(i– J )(i– G )
Ÿ –2+3i = (i– D )(i– E )(i– J )(i– G )...............................(1)
Put x = – i
– 2–3i = (– i – D )(– i – E )(– i – J )(– i – G ).....................(2)
Multiply (1) & (2)

145
4–9i2=( D 2–i2)( E 2–i2)( J 2–i2)( G 2–i2)
Ÿ 13 = (1+ D 2)(1+ E 2)(1+ J 2)(1+ G 2)
Ans : (c)
5 If D , E , J , are roots of 8x3+1001x+2008 =0, then the value of ( D + E )3+( E + J )3+( J + D )3 is
(a) 251 (b) 751 (c) 735 (d) 753
Solution :
D +E + J = 0
? ( D + E )3+( E + D )3+( J + D )3 = (– J )3+(– D )3+(– E )3
§ – 2008 ·
= – 3D E J = – 3¨ ¸ = 753
© 8 ¹
Ans : (d)
6 Total number of integral values of ‘n’ so that the equation x2+2x–n = 0 (n N) and n [5, 100] has
integral roots is
(a) 2 (b) 4 (c) 6 (d) 8 and n  [5,100]
Solution :
x2+2x–n = 0
Ÿ x2+2x+1 = n+1
Ÿ (x+1)2=n+1
x+1 = r n  1 Ÿ n+1 should be perfect square
n  [5,100]
? n+1  [6,101]
Perfect squares in the given interval are
9, 16, 25, 36, 49, 64, 81, 100
8 values ?
Ans : (d)
7 If the equation p(q–r)x2+q(r–p)x+r(p–q) = 0
2
has equal roots, then is equal to
q
1 1 1 1
(a)  (b) p+r (c)  r (d) p
p r p r
Solution :
r (p – q)
Clearly x = 1 is one root and the other root is . roots are equal, we have
p (q – r ) '

r (p – q) § r (p – q ) ·
¨¨' Pr oductof roots ¸
p(q – r ) ¸¹
=1
p (q – r ) ©
Ÿ rp–rq = pq–rq
2rp=pq+rq

146
2 1 2
Ÿ = +
q p r
Ans : (a)
PRACTICE QUESTIONS
1 The largest interval for which x12–x9+x4–x+1>0 is
(a) –4<x d 0 (b) 0<x<1 (c) –100<x<100 (d) – f <x< f
2 Read the following passage and answer the questions.
If a continuous function ƒ defined on the real line R, assumes positive and negative values in R, then
the equation ƒ(x) = 0 has a root in R, for example, if it is known that a continous function ƒ on R is
positive at some point and its minimum value is negative , then the equations ƒ(x) = 0 has a root in R.
Consider ƒ(x) = kex–x,  x  R where k R is a constant.
(i) The line y=x meets y=kex for k d 0 at
(a) no point (b) one point
(c) two point (d) more than two points
(ii) The value of k for which ke –x=0 has only one root is
x

1
(a) (b) e (c) log e 2 (d) 1
e
(iii) For k>0, the set of all values of k for which kex–x=0 has two distinct roots is
§ 1· §1 · §1 ·
(a) ¨ 0, ¸ (b) ¨ ,1¸ (c) ¨ ,f¸ (d) (0, 1)
© e¹ ©e ¹ ©e ¹

26 – 15 3
3 If = a2, then a is
5 2 – (38  5 3 )

1 1
(a) (b) (c) 3 (d) None of these
3 3
4* Solution of 2 log x a + log ax a + 3 log b a = 0 , where a>0, b=a2x is
(a) a–1/2 (b) a–4/3 (c) a1/2 (d) None of these
5 Solution of the system of equations
3x – y x  3y
x+ 2 2 = 3, y–
= 0 is _________ or ________________
x y x 2  y2
6 The number of ordered 4–tuple (x, y, z, w) where x, y, z, w  [1, 10], which satisfies the inequality
cos 2 y sin 2 z cos 2 w
t 120 is
2
2sin x 3 4 5
(a) 0 (b) 144 (c) 81 (d) infinite.
7 The number of solutions of the following inequality
1 1 1 1
2
sin x 2 . sin 2 x 3 . sin 2 x 4 . .......... sin 2 x n d n! where
2 3 4 n
xi (0,2 S ) for i = 1,2,3.............n is
(a) 1 (b) 2n–1 (c) nn (d) infinite

147
8 The number of solutions of |[x]–2x| = 4 is
(a) infinite (b) 4 (c) 3 (d) 2
9 How many roots does the equation 3 |2–|x|| = 1 possess?
|x|

(a) 1 (b) 2 (c) 3 (d) 4


10 Let S be the set of values of ‘a’ for which 2 lie between the roots of the quadratic equation x2+(a+2)x–
(a+3)=0, then S is gives by
(a) (– f ,–5) (b) (5, f ) (c) (– f ,–5] (d) [5, f )
11 Match the following
For what values of m, the equation 2x2–2(m+1)x+m(m+1) = 0 has (m R)
ColumnI Columan II
(a) both roots are smaller than 2 (p) {0,3}
(b) both roots are grater than 2 (q) (0,3)
(c) both roots lie in the interval (2, 3) (r) (– f ,0) ‰ (3, f )
(d) exactly one root lie in the interval (2,3) (s) I
­ 81  6625 81 – 6625 ½
(e) one root is smaller than 1, the other root (t) ® , ¾
is greater than 1 ¯ 32 32 ¿

(f) both 2 & 3 lie between the roots (v) (– f ,–1) ‰ [3, f )
12* The real roots of the equation

x  2 x  2 x  ........  2 x  2 3x = x is
(nradicalsigns)
(a) 0 (b) 3 (c) 1 (d) None of these
13 Solution of the equation 1+3 = 2 is ___________
x/2 x

14 The number of real solutions of the system of equations

2z 2 2x 2 2y2
x= , y= , z= is
1  z2 1 x2 1  y2
(a) 1 (b) 2 (c) 3 (d) 4
15 If a, b, c > 0, a = bc and a+b+c = abc, then the least value of a +a +7 must be equal to
2 4 2

(a) 19 (b) 20 (c) 21 (d) 18

'Note : Questions with * have more than one correct option'


Answers
1. d 2. (i) b (ii) a (iii) a 3. d 4. a,b
5. (2,1),(+1,–1) 6. c 7. b 8. b 9. d 10. a
11. a oq ; b op ; c or ; d ov ; e os ; f ot 12. a,b 13. 2 14.a 15.

148
SEQUENCES AND SERIES - I
Arithmetic Progression
Sequence
A sequence is a function of natural numbers with codomain as the set of real numbers. It is said to
be finite or infinite according it has finite or infinite number of terms. Sequence a1, a2,........ an is
usually denoted by {an} or <an>
Series
By adding or subtracting the terms of a sequence we get a series.
Arithmetic Progression (A.P.)
A sequence is called an arithmetic progression, if the difference of two consecutive term
is the same always.
i.e. an–an–1 = d, (constant), nN.
Here d is called the common difference (If d= 0 sequence is a constant sequence. if d>0 the
sequence is increasing; if d<0,the sequence is decreasing)
nth term of an A.P = a+(n–1)d
nth term from end of an A.P = an+(n–1) (–d)
where an is the last term and d, the common difference of the A.P.
Sum of n terms of an A.P
The sum Sn of n terms of an A.P is given by
­n
°° 2 >2 a  ( n – 1) d @
Sn = ® or
° n (a 1  a n )
°¯ 2
Where a is the first term, an last term, d common difference
Note that a sequence is an A.P if and only if its nth term is a linear expression in n, and in that
case its common difference is the coefficient of n.
Also sum to n terms is of the form An2+Bn where A & B are constants, and the common
difference of the A.P is 2A.
Note :
(i) d = an– an–1
= (Sn–Sn–1)–(Sn–1–Sn–2)
= Sn–2Sn–1+Sn–2
(ii) Sn–3Sn–1+3Sn–2–Sn–3
= (Sn–Sn–1) –2 (Sn–1–Sn–2)+(Sn–2–Sn–3)
= an–2an–1+an–2
= 0 as an–2,an–1, an are in A.P
Selection of terms in an A.P
In case of an odd number of terms the middle term is a and common difference d while in
case of even number of terms, middle terms are a–d, a+d and common difference is 2d.
No. of terms Terms Common difference.
3 a–d, a, a+d d
4 a–3d, a–d, a+d, a+3d 2d

149
5 a–2d, a–d, a, a+d, a+2d d
Insertion of Arithmeitc Means
ac
If a, b, c are in A.P, then b= is called the single arithmetic mean of a & c. Let a & b be two
2
given numbers and A1, A2,........... An are n A.M’s between them. Then a, A1,A2,...An, b are in A.P.
b–a
Common difference of this sequence d= .
n 1
A1= a+d, A2= a+2d etc. we can find all the arithmetic means.
Properties of A.P.
1 If a1, a2, a3, ........ are in A.P; then a1 r k, a2 r k,a3 r k,..................... are also in A.P..
a1
2 If a1, a2, a3,................. are in A.P, then a 1 O , a2 O ,a3 O ,.................... and ,
O
a 2 a3
, .............. are also in A.P ( O z 0)
O O
3 If a1, a2,.......... an are in A.P, then an,an–1,...............a2,a1 is also an A.P with common
difference (–d)
4 If a1, a2, a3, ..................and b 1, b 2, b 3, .......................... are two A.P.s then
a1 r b1,a2 r b2,a3 r b3,..... are also in A.P..
5 If a1, a2, a3,.............. and b1, b2, b3,................are two A.P.s then a1b1, a2b2, a3b3,...........and
a1 a 2 a 3
, , ,................. are NOT in A.P..
b1 b 2 b 3
6 If 3 numbers are in A.P we may take them as a–d, a, a+d. If 4 numbers are in A.P, we
can take them as a–3d, a–d, a+d, a+3d.
7 In an arithmetic progression, sum of the terms equidistant form the beginning and
end is a constant and equal to sum of first and last term.
Iie for {an},a1+an = a2+an–1=a3+an–2=......
a r –k  a rk
Also ar = , 0 d k d n–r..
2
8 Sum of n arithmetic means between two given numbers a & b is n times the single
A.M between them .
§a b·
ie. A1+A2+...............+An = n ¨ ¸
© 2 ¹

­n (middleterm);if nisodd.
°
9 Also Sn = a1+a2+......+an= ® n (sum of two middle terms); if n is even
°̄ 2        

150
Solved Examples
1 If x, y, z are real numbers satisfying the equation 25(9x2+y2)+9z2–15(5xy+yz+3zx) = 0, then
x,y,z are in
(a) A.P (b) G .P (c) H.P (d) None of these
Solution:
We have (15x)2+(5y)2+(3z)2–(15x)(5y)–(5y)(3z)–(3z)15x) = 0
or (15x–5y)2+(5y–3z)2+(3z–15x)2 = 0
1
' a2+b2+c2–ab–bc–ca = {(a–b)2+(b–c)2+(c–a)2}
2
Ÿ (15x–5y)= 0,(5y–3z)= 0,(3z–15x) = 0
x y z
Ÿ 15x = 5y = 3z Ÿ O
1 3 5
?x = O, y = 3O, z = 5O
So x, y, z are in A.P
Ans (a)
2 The number of common terms of the two sequences 2,5,8,11..................299 and
3,5,7,9,11.....201.
(a) 17 (b) 33 (c) 50 (d) 147
Solution
Sequence of common terms is
5,11,17..........whose nth term is
an = 5+(n–1)6 = 6n–1
an d 201 Ÿ 6n–1 d 201
2
Ÿ n d 33 ? n = 33
3
Ans (b)
1 1 1 5 1
3 The value of x+y+z = 15. If a, x, y, z, b are in A.P, while the value of + y + is . If ,
x z 3 a
1 1 1 1
, , , are in A.P, then
x y z b
(a) a = 1, b = 9 (b) a=9,b=1 (c) can not find (d) None of these
Solution:
5
a+x+y+z+b = (a+b)
2
3
Ÿ x+y+z = (a+b)
2
3
Ÿ 15 = (a+b)
2
Ÿ a+b = 10

151
1 1 1 1 1 5 § 1  1·
Also + + y + + = ¨ ¸
a x z b 2 ©a b¹

1 1 1 3 § 1  1·
+ y + = ¨ ¸
x z 2 ©a b¹

5 3 §a b·
= ¨ ab ¸
3 2 © ¹
5 3 10
Ÿ = . Ÿ ab = 9
3 2 ab
Solving we get a = 1, b=9 or a= 9, b = 1
Ans = a,b
4 Let a1, a2, a3,............a11, be real numbers satisfying a1 = 15, 27–2a2>0 and ak = 2ak–1–ak–2 for
2 2
a1  a 2  .......  a11
2
a 1  a 2  .......  a 11
k = 3,4,..............11. If = 90, then the value of is
11 11
equal to
Solution
a1 = 15, ak=2ak–1–ak–2 Ÿ a1, a2........a111 are in A.P.
2 2 2
a1  a 2  .......  a11 (15) 2  (15  d) 2  .........  (15  10d) 2
? = = 90
11 11
2 2 2
–9 2
­ (15u 11)  d (1+----+10 )  3od (1+2+10)
Ÿ 7d +30d+27 = 0 Ÿ d = –3,
2
7 °° 11 11 11
®Ÿ 225+350 d 2+150 d =90
27 –9 °
given a2<
2
? d = –3 & d z
7 °¯ 35d 2+ 150 d +135 = 0

a 1  a 2  .......  a 11 11 (30 – 3 u 10)


Ÿ = =0
11 2 11
Ans 0
5 Suppose A, B, C are defined as A = a2b+ab2–a2c–ac2, B = b2c+bc2–a2b–ab2 and C = a2c+c2a–
1 1 1
cb2–c2b, where a>b>c>0 and the equation Ax2+Bx+C = 0 has equal roots then , , are in
a b c
(a) A.P (b) G.P (c) H.P (d) None of these
Solution :
' A+B+C = 0, x = 1 is a root of Ax2+Bx+C = 0
The other root = 1 (' roots are equal)
C
? 1×1 = Ÿ C =A
A
a2c+c2a–cb2–c2b = a2b+ab2–a2c–ac2
c(a–b)(a+b+c) = a(b–c)(a+b+c)
Ÿ ac–cb = ab–ac (' a+b+c z 0)
Ÿ 2ac = ab+bc

152
2 1 1
Ÿ = +
b c a
1 1 1
Ÿ , , are in A.P.
a b c
Ans: (a)
6 If a, b, c, d, are distinct integers in an increasing A.P such that d = a2+b2+c2, then a+b+c+d =
(a) –1 (b) 0 (c) 1 (d) 2
Solution :
a, b, c, d  Z
Let b = a+ O , c= a+2 O , d = a+3 O , O  Z
a+3 O = a2+(a+ O )2+(a+2 O )2
Ÿ 5 O 2+3(2a–1) O +3a2–a = 0 .................(1)
O is real
Dt 0
9(2a–1)2–4.5.(3a2–a) t 0 Ÿ 24a2+16a–9 d 0
–1 70 –1 70
– d ad +
3 12 3 12
' a  I a = –1,0
If a = –1, O = 1 and if a = 0 O  Z
4
a+b+c+d = (a+d) = 2(2a+3 O ) = 2(–2+3) = 2
2
Ans (d)
7 Consider the sequence in the form of groups (1), (2,2), (3,3,3), (4,4,4,4), (5,5,5,5,5),............,
the 2000th term of the sequence is not divisible by
(a) 3 (b) 9 (c) 7 (d) None of these
Solution
Let us write the terms in the groups as follows:
(1), (2,2), (3,3,3).........consisting of 1,2, 3, 4, ............terms
Let 2000th term fall in nth group. Then
1+2+3+...........+(n–1)<2000 d 1+2+....................+n
n(n – 1) n(n  1)
<200 d
2 2
Ÿ n(n–1)<400 d n(n+1)
n(n–1)<400 and n(n+1) t 400
n2–n–400<0 and n2+n–400 t 0
Ÿ n = 63
i.e. 2000th term falls is 63 rd group.
Also 2000th term is 63
Ans (d)
8 If x18 = y21 =z28, then 3logyx,3logzy, 7logxz are in
(a) A.P (b) G.P (c) H.P (d) None of these

153
Solution
Let x18 = y21 =z28= O
18logx = 21log y = 28 logz = log O
21 28 18
logyx = , logzy = , logxz =
18 21 28
7 9
Ÿ 3logyx = , 3logzy = 4, 7logxz =
2 2
Ÿ 3, 3logyx, 3logzy, 7logxz are in A.P
Ans (a)

PRACTICE QUESTIONS
1* If 1, logyx, logzy, –15logxz are in A.P, then
1
(a) z3 = x (b) x = y–1 (c) y= (d) None of these
z3
a
2 If 51+x+51–x, , 25x+25–x are three consecutive terms of an A.P, then a is
2
(a) d 12 (b) t 12 (c) = 12 (d) None of these
3 If sin D , sin2 D , 1, sin4 D and sin5 D are in A.P, where – S < D < S , then D lies in the interval
§ – S S· § – S S· § – S S·
(a) ¨ , ¸ (b) ¨ , ¸ (c) ¨ , ¸ (d) None of these
© 2 2¹ © 3 3¹ © 6 6¹
4 If the roots of x3–12x2+39x–28 = 0 are in A.P, then their common difference will be
(a) r 1 (b) r 2 (c) r 3 (d) r 4
5 If the sides of a right triangle are in A.P, then the sum of the sines of two acute angles is
7 4
(a) (b)
5 3

5 –1 5 1
(c) + (d) None of these
2 2
6 Read the passage and answer the following questions. Two consecutive numbers from 1, 2,
105
3, .........,n are removed. The arithmetic mean of the remaining numbers is .
4
(i) The value of n lies in
(a) [45,55] (b) [52,60] (c) [41,49] (d) None of these
(ii) The removed numbers
(a) lie between 10 and 20 (b) are greater than 10
(c) are less than 15 (d) none of these
(iii) Sum of all numbers
(a) exceeds 1600 (b) is less than 1500
(c) lies between 1300 and 1500 (d) none of these
7 Concentric circles of radii 1, 2, 3, .....100 cm are drawn. The interior of the smallest circle is

154
coloured red and the angular regions are coloured alternately green and red, so that no two
adjacent regions are of same color. The total area of the given regions in sq.cm is equal to
(a) 1000 S (b) 5050 S (c) 4950 S (d) 515 S
8* If three identical fair unbiased dice are thrown together such that the numbers a, b and c, where
a,b, c  {1,2,3,4,5,6} appear on each of them respectively. If r represents all possible distinct
cases, D represent the number of ways in which a+b+c = 9 and E represents the number of
ways of obtaining a+b+c = 8, then match the following.
Column I Column II
(a) If D represents the common difference of an A.P
such that the arithmetic mean of the squares of these
quantities exceeds the square of A.M by 9, then the
number of terms of A.P are (p) 5C2

(b)
max( D,E )
¦
r min( D,E )
max(D,E)

Cr – 5 (q) 2

(c) If 6th term in the expansion of ( D + E )n is the greatest


term, then n is (r) 1
ª§ J – E ·§ D ·º
(d) «¨¨ ¸¸¨¨ ¸¸» is equal to where [.] denotes the
¬ © J – D ¹© E ¹¼
greatest integer function (s) 10
9 A person is to count 4500 currency notes. Let an denotes the number of notes he counts in the
nth minute. If a1 = a2 = ............a10 = 150 and a10,a11...........are in A.P with common difference –
2, then the time taken by him to count all notes is
(a) 24 min (b) 34min (c) 125min (d) 135min
10 If a1, a2...............a n are in A.P with common difference d z 0 then (sind) (seca 1 seca2+
seca2seca3+.........secan–1secan) is equal to
(a) cotan–cota1 (b) cota1–cotan (c) tanan–tana1 (d) tana1–tanan
Nnote:* Question with more than one option is correct.

Answers
1. a,b,c 2. b 3. d 4. c 5. a 6. (i) a (ii) c (iii) b 7. b 8. a o q ; b o q ; c o p,s ; d o r
9. b 10. c

155
SEQUENCES AND SERIES - II
Geometric Progression

A sequence of non-zero numbers is called a geometric progression if ratio of a term and the
term preceding to it is always a constant. This constant is called the common ratio of the G.P.
a n 1
i.e. a1, a2, .........., an is in G.P. if a r = const., nHN .
n

nth term of a G.P. = an = a rn–1.


n 1
§1·
nth term from end of a G.P = an. ¨ ¸ where an is the last term and r the common ratio of the
©r¹
G.P.
Sign of a +ve +ve –ve –ve
Range of r r>1 0<r<1 r>1 0<r<1
G.P. is increasing decreasing decreasing increasing
If r = 1, the sequence will be a constant sequence. If r is negative the terms of G.P. are alternately
positive and negative and so the G.P is neither increasing nor decreasing.
Sum of n terms of a G.P
Let the first term of G.P be a common ratio r and last term a n, then
i) Sum to n terms

­ a(r n  1) §1 rn ·
° or a ¨ ¸ ; when r z 1
® r 1 © 1 r ¹
Sn =
°na ; when r 1
¯

a  anr a r a
Also Sn = or n ; when r z 1
1 r r 1
Sum of an Infinite G.P
Sum of an infinite G.P. with first term a and common ratio r (–1 < r < 1; r z 0 or 0<|r|<1) is given
a
by S =
1 r
If r>1, then the sum of an infinite G.P tends to infinity.
Selection of terms in a G.P

156
In case of an odd number of terms the middle term is a and common ratio is r while in case of
a
even number of terms, middle terms are , ar and common ratio is r2.
r
No. of terms Terms Common ratio

a
3 , a, ar r
r

a a
4 , , ar, ar3 r2
r3 r

a a
5 , , a, ar, ar2 r
r2 r
Note that it is convenient to take the terms as a, ar, ar 2 ......... if the product of numbers is not
given.
Insertion of geometric means
Note : If a & b are two numbers of opposite signs then geometric mean between them does not exist.
Solved Examples

3
1. Suppose a, b, c are in A.P and a2, b2, c2 are in G.P. If a<b<c and a+b+c = , then the value of
2
a is
1 1 1 1 1 1
a. b. c. 1  d. 
2 2 2 3 2 3 2 2
Solution : Let the numbers be A – d, A , A + d
1
Then A =
2
1 1 1
? Numbers are – d, , +d
2 2 2
a2, b2, c2 are in G.P.
2
§ § 1 ·2 · 2 2
¨ ¨ ¸ ¸ = §¨ 1  d ·¸ §¨ 1  d ·¸
¨© 2 ¹ ¸ ©2 ¹ ©2 ¹
© ¹
2
1 §1 2· 1 1
Ÿ = ¨  d ¸ Ÿ  d2 = r
16 © 4 ¹ 4 4

1 1
Ÿ d2 = Ÿ d= +
2 2

157
1 1
Since a < b < c, a = –
2 2
Ans. d
2 3 n
3 § 3· § 3· n –1 § 3 ·
2. If an = – ¨ ¸  ¨ ¸  .........  (–1) ¨ ¸ and bn = 1–an, then the minimum natural number
4 © 4¹ © 4¹ © 4¹
no such that bn > an & n > no is
a. 4 b. 5 c. 6 d. 12

3 §¨ §  3 · ·
n

1 ¨ ¸ ¸
4 ¨© © 4 ¹ ¸
¹
Solution : an = 3
1
4

3 § § 3· ·
n

i.e. an = 7 ¨¨1  ¨ 4 ¸ ¸
¸
© © ¹ ¹
bn > an Ÿ 1 – an > an Ÿ 2an < 1

6 §¨ §  3 · ·
n n n
1 ¨ ¸ ¸ <1 § 3· 7 § 3· 1
Ÿ 7 ¨© © 4 ¹ ¸ Ÿ 1– ¨ ¸ < – ¨ ¸ <
¹ © 4 ¹ 6 © 4 ¹ 6

Ÿ (–3)n+1 < 22n–1


For n to be even, the inequality always holds. For n to be odd, it holds for n > 7.
? Least natural number for which it holds is 6.
Ans. c
3. If a be the arithmetic mean of b and c and G 1, G2 be the two geometric means between them,
then G13 + G23 =
a. G1G2a b. 2G1G2a c. 3G1G2a d. none of these
Solution : b, a, c are in A.P
2a = b+c
Also b, G1, G2, c are in G.P
G 12 = b G 2 Ÿ G13 = bG1G2
G 22 = G 1c Ÿ G23 = cG1G2
G13 + G23 = G1G2 (b+c) = 2G1G2a
Ans. b

158
f
1 S2 f
1
4. It is known that ¦ (2r  1) 2 8
, then ¦
r 1 r2
is
r 1

S2 S2 S2
a. b. c. d. none of these
24 3 6
Solution:

1 1 1
Let    ..........f = x
12 2 2 32
f
1 §1 1 1 · §1 1 1 ·
¦
r 1 r 2 = ¨ 2
©1
 2  2  .......... ¸ + ¨ 2  2  2  ..........f ¸
3 5 ¹ ©2 4 6 ¹

S2 1 §1 1 1 ·
x= + ¨ 2  2  2  ..........f ¸
8 4 © 1 2 3 ¹

S2 1 3x S2 S2
x= + x Ÿ = Ÿ x=
8 4 4 8 6
Ans. c
5. If ai H R, i = 1, 2, 3, .......n and all a i’s are distinct such that
§ n 1 2 · 2 § n –1 · n

¨¦ i ¸a x  2¨ ¦ i i 1 ¸
a a x  ¦ a i2 d 0 , then a1, a2, ......... are in
©i1 ¹ ©i1 ¹ i 2

a. G.P b. A.P c. H.P d. none of these


n 1 n 1

Solution : ¦
i 1
(aix + ai+1) < 0
2
Ÿ ¦i 1
(aix + ai+1)2 = 0

Ÿ aix + ai+1 = 0  i = 1, 2, ......... n–1

a i 1
Ÿ a i = – x  i = 1, 2, ...... n –1

? a1, a2, ......... an are in G.P.

159
Ans . a
6. The 1025th term is the sequence
1, 22, 4444, 88888888, ........... is
a. 29 b. 210 c. 211 d. none of these
Solution : Number of digits in each term are in G.P.
Let 1025th term =2n
then
1+2+4+8+........+2 n–1 < 1025 d1+2+4+8+.....+2 n

(2 n – 1) (2 n1 – 1)
Ÿ 1 < 1025 d1.
2 –1 2 –1
Ÿ 2n–1 < 1025 d 2n+1–1
Ÿ 2n+1 t 1026 > 1024 = 2 10
Ÿ n+1 > 10 Ÿn > 9
? n=10
Ans. b
7. If a1/x=b1/y=c1/z and a, b, c are in geometrical progression, then x,y, z are in
a. A.P. b. G.P. c. H.P. d. None of these
Solution : a1/x=b 1/y=c1/z =k
Ÿ a=kx, b=ky, c=kz
a, b, c are in G.P. Ÿb2 = ac
Ÿk2y=kx+z
Ÿ2y=x+z
Ans. a
8. If the arithmetic mean of two numbers be A and geometric mean be G, then the numbers will be

a. A r (A2–G2) b. A r A2 – G2

A r ( A  G )(A – G )
c. r (A  G )(A – G ) d. 2

160
Solution : Let the number be a and b

A = a  b G2=ab
2
a and b are the roots of x 2–2Ax+G2=0

2A r 4A2 – 4G2
x=
2
x= A r (A2–G2)
Ans. a
PRACTICE QUESTIONS
1. The sum of an infinite geometric series is 2 and the sum of the geometric series made from the
cubes of this infinite series is 24. Then the series is
3 3 3 3 3 3
a. 3 –  ....... b. 3   ......
2 4 8 2 4 8
3 3 3
c. 3–  – ...... d. none of these
2 4 8
2. Read the passage and answer the following questions.
Let A1, A2,.....Am be arithmetic means between –2 and 1027 and G 1, G2, .... Gn be Geometric
means between 1 and 1024. Product of geometric means is 245 and sum of arithmetic means is
1025×171.
i. The value of n is
a. 7 b. 9 c. 11 d. none of these
ii. The value of m is
a. 340 b. 342 c. 344 d. 346
iii. The value of G1+G2+G3+....+Gn is
a. 1022 b. 2044 c. 512 d. none of these
iv. The common difference of the progression A1, A3, A5.....An is
a. 6 b. 3 c. 2 d. 1
v. The numbers 2A171, G 52  1 , 2A172 are in
a. AP b. GP c. HP d. AGP
3. The difference between two numbers is 48 and the difference between their arithmetic mean
and their geometric mean is 18. Then, the greater of two numbers is
a. 96 b. 60 c. 54 d. 49
§1 1 1 ·
4. If ax=by=cz=dw, the value of x ¨¨   ¸¸ is
©y z w¹
a. loga(abc) b. loga(bcd) c. logb(cda) d. logc(dab)
5. If three positive numbers x, y, z are in A.P. and also tan –1x, tan–1y, tan–1z are in A.P., then
a. x=y=z b. xz y=z c. x = yz z d. none of these
6. If a1, a2, a3 are three consecutive terms of a G.P. with common ratio k. Then the values of k for
which the inequality a3 > 4a2 – 3a1, is satisfied is (if a1 > 0)

161
a. (1, 3) b. – f,1 ‰ 3, f c. R d. none of these
7. The three successive terms of a G.P. will form the side of a triangle if the common ratio r lies in

§ 5 –1 5 1· § –1 5 ·
¨ ¸ ¨ ¸
a. ¨ 2 , 2 ¸ b. ¨ 2 ,f¸
© ¹ © ¹

§ 5 –1·
c. ¨ – f, ¸ d. none of these
¨ 2 ¸
© ¹
S
8. If for 0 < x < , exp (sin2x+sin4x+sin6x+........ f ) loge2) satisfies the quadratic equation
2
sin x – cos x
x 2–9x+8=0, is
sin x  cos x
a. 2– 3 b. 2+ 3 c. 3 –2 d. none of these
9. Match the following
Column I Column II
a. If a, b, c are non zero real numbers such that
3(a2+b2+c2+1) = 2 (a+b+c+ab+bc+ca) then a, b, c are in (p) AP
b. If the square of difference of three numbers be in AP, then
their differences are in (q) GP
c. If a–b, ax–by, ax 2–by2(a, b z 0) are in G.P. then
ax – by
x, y, are in (r) HP
a–b
(s) equal
10. If 1+p+p +......+p =(1+p) (1+p )(1+p )(1+p ) (1+p ) then the value of n (nHN) is
2 n 2 4 8 16

a. 32 b. 16 c. 31 d. 15
11. If sinT, 2 (sinT+1), 6sinT+6 are in G.P., then the fifth term is
a. 81 b. 82 2 c. 162 d. none of these
12. If x H{1, 2, 3, ....9} and fn(x)=xxx......x(n digits) then f n (3)+fn(2) is equal to
2

a. 2f2n(1) b. fn2(1) c. f2n(1) d. 2f2n(4)


13. The number of divisors of 1029, 1547 and 122 are in
a. AP b. GP c. HP d. none of these
14. Let x1, x2,.....,xn be a sequence of integers such that
(i) –1d x i d 2 for i=1, 2,....n
(ii) xi+x2+....+xn=19
(iii) x12+x 22+....+xn2=99
Let m and M be the minimum and Maximum possible values of x13+x23+....xn3 respectively, then
M
the value of is _____
m
15. Let 16, 4, 1, .... be a geometric sequence. Define Pn as the product of the first n terms. Then the

162
f

¦ n Pn
value of n 1 is _____.
4
1
a. 64 b. c. 32 d. none of these
64
Note:* Questions with more than one option is correct

ANSWERS

1. c 2. (i) b (ii) b (iii) a (iv) a (v) a 3. d 4. b

5. a 6. b 7. a 8. a

9. a o p,q,s; b o r; c o p,q,s 10. c 11. c 12. c

13 a 14. 7 15. c

163
SEQUENCES AND SERIES - III
Harmonic Progression
A sequence a1,a2......an of non-zero numbers is called a harmonic sequence if the sequence
1 1 1
, ,........ ,....... is an A.P..
a1 a 2 an
1 1 1
Eg: The sequence , , ...... is an H.P. because the sequence 3,5,7...... is an A.P..
3 5 7
nth term of a H.P.
The nth term of a H.P. is the reciprocal of nth term of the corresponding A.P. and the common
1 1
difference of the corresponding A.P is d. i.e. d – .
a 2 a1
i. nth term of the H.P is given by
1 1 a 1a 2
an = 1
§ 1 1· a 2  n – 1 a1 – a 2
 n – 1 d 1
 n – 1 ¨ – ¸
a1 a1 © a 2 a1 ¹
ii. nth term of the H.P. from end
1 1
a1n
1 1 § 1 1·
 n – 1 – d – n – 1 ¨ – ¸
an an © a 2 a1 ¹

a 1a 2 a n
= a1a 2 – n – 1 a n a1 – a 2
Note: No term of H.P. can be zero and there is no general formula for finding out the sum
of n terms of a H.P.

Harmonic Mean
If a,b,c are in H.P., then b is called the H.M. between a & c.
Now a,b,c are in H.P.
1 1 1
Ÿ , , are is A.P..
a b c
2 1 1
Ÿ 
b a c
2ac
Ÿ b=
ac
2ac
i.e. H.M. between a & c is
ac

164
2ab
Note: The single H.M. between a & b is .
ab
The single H.M. (H) of n positive numbers a 1,a2,.....an is given by
1 1 1 1
   ....... 
1 a1 a 2 a 3 an 1 § 1 1 1 1·
= ¨    .......  ¸
H n n © a1 a 2 a 3 an ¹
Insertion of Harmonic Means
Let a and b be two given numbers and H1,H2.....Hn be the H.M.’s between them. Then a,
H1, H2.......Hn , b will be in H.P. Let d the common differece of the corresponding A.P.
b n  2 th term of H.P..
1
b
1
 n  2 – 1 d
a
1 1

1 1 b a
 n  1 d Ÿ d
b a n 1
a–b
d
n  1 ab
1 1 1 a–b
? d 
H1 a a n  1 ab
1 1 1 2 a – b
 2d 
H2 a a n  1 ab
.
.
.
1 1 1 n a – b
 nd 
Hn a a n  1 ab
2 2
E.g: Insert 4 H.M’s between and
3 13
1 1 13 3
– –
d b a 2 2
n 1 4 1
d=1
1 3 5 2
? 1 or H1
H1 2 2 5

165
1 3 7 2
2 or H 2
H2 2 2 7
1 3 9 2
3 or H 3
H3 2 2 9
1 3 11 2
4 or H 4
H4 2 2 11
Note: The sum of reciprocals of n H.M’s between two numbers is n times the reciprocal of
single H.M. between them.
i.e. if a,b are the numbers and H1,H2.....Hn be the H.M’s between them,
1 1 1 § a  b·
then   ........  n¨ ¸
H1 H 2 Hn © 2ab ¹

§ 1 ·
= n¨ ¸
© H.M.of aandb ¹
Solved Examples
1. If a1,a2,a3......a10 are in A.P. and h 1,h2,h3,.....h10 are in H.P. If a1=h1 = 2 and a10 = h10 = 3,
then a4 h7 is
a. 6 b. 7 c. 18 d. none of these
Solution:
1
a10 = 3 Ÿ 3 = 2+9d Ÿ d
9
h1,h2.....h10 are in H.P.
1 1
 9d 1 Ÿ 1 1
 9d 1 Ÿ d1
–1
h10 h1 3 2 54
1 1 7
a4 = a1+3d = 2+3x = 2 
9 3 3
1 1 18
and h  6d Ÿ h 7
7 h1 7
7 18
? a4h7 = x 6
3 7
Ans: a
2. If a1,a2,a3.....an are in H.P., then a1a2 + a2a3+.........+an–1an will be equal
a. a1an b. na1an c. (n–1)a1an d. none of these
Solution:
1 1 1
, ,........... are in A.P..
a1 a 2 an
1 1 1 1 1 1
Ÿ – – ....... – d
a 2 a1 a3 a2 a n a n –1

166
a1 – a 2 a2 – a3 a n –1 – a n
Ÿ ........ d
a1a 2 a 2a 3 a n a n –1
Ÿ a1 – a2 = da1a2
Ÿ a2 – a3 = da2a3
Ÿ an–1–an = danan–1
Adding, we get a1– an = d(a1a2+a2a3+.......+anan–1)

·
a1 – a n
§
i.e. a1a2+a2a3+......+anan–1= 1 1
d =  (n-1)d
an a1

=
a1 – a n a a n – 1
a1 – a n 1 n ¨ Ÿ 1 – 1 = (n-1)d
an a1 ¸
=
Ans: c
a1an(n–1) ©
a1 – a n
d
= (n-1)a1an ¹
3. If A1,A2 ; G1,G2 and H1,H2 be two A.M.’s , G.M.’s and H.M.’s between two numbers a & b
G1G 2 H1  H 2
repectively, then x
H1 H 2 A1  A 2
a. 1 b. 0 c. 2 d. 3
Solution:
a, A1, A2, b are in A.P Ÿ A1+A2 = a+b
a, G1, G2, b are in G.P Ÿ G1G2 = ab
1 1 1 1
a, H1, H2, b are in H.P Ÿ , , , are is A.P
a H1 H 2 b

1 1 1 1
Ÿ  
H1 H 2 a b

G1G 2 § 1 1 · ab § 1 1 ·
Given expression is A  A ¨ H  H ¸ = ¨  ¸ 1
1 2 © 1 2¹ a  b © a b¹
Ans: a
6
4. If nine A.M.’s and nine H.M.’s are inserted between 2&3, then A  ....(where A is any of
H
the A.M.’s and H the corresponding H.M.)
a. 5 b. 3 c. 15 d. none of these
Solution:
2,A1,A2.......A9,3 are in A.P.
b–a 1
d=
n 1 10
i
Ai = 2 + id = 2+ ; i = 1,2,......9
10

167
1 1 1 1 1
, , ,.......... , are in A.P..
2 H1 H 2 H9 3

a–b –1
D= ; i = 1,2,......9
(n  1)ab 60

1 1 1 § – 1·
 iD  i¨ ¸ ; i 1,2,.....,9
Hi 2 2 © 60 ¹

6 i
3–
Hi 10 ; i 1,2,..........9

6
? Ai + = 5 ; i = 1,2,3.........9
Hi

Ans: a
H1  2 H 20  3
5. If H1,H2.....H20 be 20 harmonic means between 2&3, then 
H1 – 2 H 20 – 3
a. 20 b. 21 c. 40 d. 38
Solution:
H1  2 H 20  3

H1 – 2 H 20 – 3

1 1 1 1
 
2 H1 3 H 20
= +
1 1 1 1
– –
2 H1 3 H 20

1 1 1 1
 d  –d
= 2 2 + 3 3
1 1 1 1
– –d – d
2 2 3 3

2
–d
1 d 3 –1 2
=  – 1 –1
–d d d 3d
1§2 ·
= ¨ – 1¸ – 2
d©3 ¹

168
1 § 1 1 2 ·
= u 42d – 2 ¨'  22 – 1 d Ÿ – 1 42d¸
d © 3 2 3 ¹
Ans: 40

PRACTICE QUESTIONS
1. If x,y,z are in H.P., then the value of expression log(x+z) + log(x–2y+z) will be
a. log(x–z) b. 2log(x–z) c. 3log(x–z) d. 4log(x–z)
2. If a,b,c,d are positive real numbers such that a+b+c+d = 2, then M = (a+b) (c+d) satisfies the
equation
a. 0 < Md1 b. 1dMd2 c. 2dMd3 d. 3dMd4
a1 a2 an
3. If a1,a2,a3.......,an are in H.P., then a  a .....  a , a  a .....  a ,............, a  a .....  a
2 3 n 1 3 n 1 2 n –1

are in.
a. A.P. b. G.P. c. H.P. d. none of these
4. If a,a1,a2.....a2n–1, b are in A.P. ; a,b1,b2.....b2n–1,b are in G.P. ; a,c1,c2.....c2n–1,b are in H.P.
where a,b, are positive, then the equation a nx2 – bnx+cn = 0 has its roots
a. real and unequal b. real and equal
c. imaginary d. none of these
x x x
5. If b–c, 2b–x, b–a are in H.P, then a – , b – and c – are in
2 2 2
a. A.P b. G.P c. H.P d. none of these
2 12
6. The first two terms of a H.P are and respectively. Then the largest term is
5 13
a. 2nd term b. 6 term
th
c. 4th term d. none of these
p r
7. If a,b,c are in A.P, p,q,r are in H.P and ap, bq, cr are in G.P. then  is equal to
r p

a c a c b q b q
a. – b.  c.  d. –
c a c a q b q b

§ 1 1 1· § 1 1 1·
8.* If a,b,c are in H.P, then the value of ¨©  – ¸¹ ¨©  – ¸¹ is
b c a c a b

2 1 1§ 3 2 1 ·
a. – 2 b. ¨  – ¸
bc b 4 © c 2 ca c 2 ¹
3 2
c. 2
– d. none of these
b ab
H1  a H n  b
9. If H1,H2......Hn be n harmonic means between a and b, then H – a  H – b is equal to
1 n

a. 0 b. n c. 2n d. 1

169
10. If a,b,c be in G.P. and a+x,b+x,c+x in H.P., then the value of x is (a,b,c are district numbers)
a. c. b. b c. a d. none of these
11.* The harmonic mean of two numbers is 4, their A.M.A and G.M.G satisfy the relation
2A +G 2=27. The numbers are
a. 6 and 3 b. 3 and 6 c. can’not find d. none of these
12. If n be a root of the equation
x2(1–ab) –x(a2+b2) – (1+ab) = 0, then H 1–Hn =
b–a
a. ab (a–b) b. ab(b–a) c. d. none of these
ab
x z
13. If x,y,z are in A.P, ax,by,cz in G.P. and a,b,c in H.P., then 
z x
a c 2ac
a. 2 b.  c. d. none of these
c a ac
§ 1 1 ·§ 1 1 ·
14. If a,b,c,d are in H.P., then the value of ¨© 2 – 2 ¸¹ ¨© 2 – 2 ¸¹ is
a d b c
a. 1 b. 2 c. 3 d. 4

Note:*Questions with more than one option is correct

ANSWERS

1.b 2.a 3. c 4. c 5. b

6. a 7. b 8. a,c 9. c 10. b

11. a,b 12. a 13. b 14. c

170
SEQUENCES AND SERIES - IV
Inequalities based on A.M., G.M. and H.M.

Properties of A.M, G.M & H.M


Let A, G, H be the arithmetic, geometric and harmonic means of two positive numbers a & b.
ab 2ab
Then, A = ,G= ab , H =
2 ab

a  b 2ab
i) A. H = . = ab = G2
2 ab
i.e. G2 = A.H
G is the geometric mean between A & H.

Again A – G =
ab
– ab =
a b
2

>0
2 2
Ÿ A>G
Also G2 = A.H

G A
>1
H G

G
Ÿ > 1 or G > H
H
Combining, A > G > H
Note : If the numbers are equal, then A = G = H. Thus, A > G > H, equality holds when the
numbers are equal.
ii) The equation with a and b as its roots is x 2 – 2Ax + G2 = 0.

a A  A2  G2
or if A & G be the A.M and G.M between two positive numbers a & b then
b A  A2  G2
iii) If A, G, H be the A.M, G.M, and H.M between three given numbers, a, b and c , then the
equation having a,b,c as its roots is
3G 3
x – 3Ax +
3 2
x – G3 = 0
H

abc
Proof : A = Ÿ a + b + c = 3A
3
G = (abc)1/3 Ÿ G3 = abc

171
1 1 1
  ab  bc  ca 1
1 a b c Ÿ
H 3 3abc H

3abc 3G 3
or ab + bc + ca =
H H
Equation having a, b, c as roots is
x 3 – (a + b + c)x + (ab + bc + ca)x 2 – abc = 0

3G 3
x – 3Ax +
3
x – G3 = 0
H
Example : For distinct positive numbers x, y, z, prove that (x +y)(y + z)(z + x) > 8xyz
Solution : We have A > G

For positive numbers x and y, x + y > 2 xy

For positive numbers y and z, y + z > 2 yz

For positive numbers z and x, z + x > 2 zx

Multiplying, (x +y)(y + z)(z + x) > 8 x 2 y 2 z 2

i.e., (x +y)(y + z)(z + x) > 8xyz

Solved Examples
1. If a, b, c are positive then prove that ((1+a)(1+b)(1+c)) 7 > 77a4b4c4.
Solution : (1+a)(1+b)(1+c) = 1 + a + b + c + ab + bc + ca + abc
> a + b + c + ab + bc + ca + abc
> 7(a . b . c . ab . bc . ca . abc) 1/7 ( 'A > G )
i.e. (1+a)(1+b)(1+c) > 1 + 7(a4b4c4)1/7 > 7(a4b4c4)1/7
((1+a)(1+b)(1+c)) 7 > 77a4b4c4
2. Maximum value of xyz for positive values of x, y, z if yz + zx + xy = 12 is
a. 64 b. 43/2 c. 8 d. none of these
Solution : Apply A > G for yz, zx & xy

yz  zx  xy
> (x2y2z2)1/3
3

172
12
> (xyz)2/3
3
(xyz) < 43/2
xyz < 8
Ans: c
3. Maximum value of x2y3 where x & y lie in 1st quadrant in the line 3x + 4y = 5.
5 3 5 3
a. b. c. d.
16 8 8 16
Solution : x2y3 = x . x . y . y . y
3x 3x 4y 4y 4y
3x + 4y = + + + +
2 2 3 3 3
A>G

3x 3x 4 y 4 y 4 y
    1/ 5
2 2 3 3 3 t § 3x . 3x . 4 y . 4 y . 4 y ·
¨ ¸
5 © 2 2 3 3 3¹

1
5 § 16 x 2 y 3 · 5
t¨ ¸
5 © 3 ¹

3
Ÿ x 2y3 <
16
Ans. d
4. If a2 + b2 + c2 = 1 = x 2 + y2 + z2, then maximum value of ax + by + cz is (a, b, c, x, y, z are
positive real numbers)
a. 4 b. 3 c. 2 d. 1
Solution : A>G

a2  x2
! a 2x 2 Ÿ a2 + x 2 > 2ax
2
Similarly b2 + y2 > 2by
c2 + z2 > 2cz
adding, (a2+b2+c2) + (x 2+y2+z2) > 2(ax+by+cz)
1 1
Ÿ ax+by+cz <
2
Ans. d

173
a b c
5. If a, b, c are positive then the minimum value of   is
bc ca ab

2 3
a. b. c. 1 d. none of these
3 2

1 1 1
Solution : Apply A > H for , ,
bc ca a b

1 1 1
 
bc ca a b t 3
3 abbcca

1 1 1 9
Ÿ   t
b  c c  a a  b 2(a  b  c)

abc abc abc 9


Ÿ   t
bc ca ab 2

a b c 9
Ÿ   3t
bc ca ab 2

a b c 3
Ÿ   t
bc ca ab 2
Ans. b

§ 1 1 1·
6. If x, y, z are three positive numbers, then (x+y+z) ¨¨   ¸¸ > ______
©x y z¹

1
a. 3 b. 9 c. d. none of these
3

xyz
Solution : > (xyz)1/3 (' A > G)
3

1 1 1
  1/ 3

Also x y z § 1 1 1· (' A > G)


! ¨¨ . . ¸¸
3 ©x y z¹

§1 1 1·
( x  y  z)¨¨   ¸¸
Multiplying, ©x y z¹ > 1
9

174
Ans. b (or apply A > H for x, y, z to get the result)

§ 2n ·
7. Prove that : n
C1 . ( C2) ( C3) ....... ( Cn) < ¨¨
n 2 n 3 ¸¸ n n
© n 1 ¹
Solution : Let S = nC1 + 2nC2 + 3nC3 + ....... n nCn
n n

= ¦
r 1
r..nCr = ¦
r 1
n. n–1
Cr–1 = n 2n–1

Now A > G

n

C1  n C 2  n C 2  n C3  n C 3  n C3  ...... n C n  n C n  ...... n C n
1  2  3  ..........  n

t
C C ........ C
n
1
n
2
2 n
n
n
1
1 2 ..... n

t C C ........ C
n 1 2
n .2 n n 2 n n n ( n 1)

Ÿ n (n  1) 1 2 n

n ( n 1) n 1
C2
§ 2n · 2 § 2n ·
Ÿ ( C1 . ( C2) ( C3) ....... ( Cn) ) d ¨¨
n n 2 n 3 n n ¸¸ = ¨¨ ¸¸
© n 1¹ © n 1¹

8. If a, b, c, d are in H.P. then


a. a+d > b+c b. ad > bc c. ad = bc d. none of these
Solution : a, b, c are in H.P Ÿ b is the H.M of a & c.

ac
A.M of a & c =
2

ac
we have A > H Ÿ > b Ÿ a+c > 2b
2
Similarly b + d > 2c
Adding, a+b+c+d > 2b+2c Ÿ a+d > b+c
Also a, b, c are in H.P. Ÿ b is the H.M of a & c

G.M of a & c = ac

G > H Ÿ ac > b

175
Similarly bd > c

Multiplying, abcd > bc Ÿ ad > bc


Ans. a, b

PRACTICE QUESTIONS
1. If a+b+c = 1, then find k such that

k § 1 ·§ 1 ·§ 1 ·
! ¨  1¸¨  1¸¨  1¸ > k
27abc © a ¹© b ¹© c ¹
a. 8 b. 7 c. 3 d. none of these
2. A rod of fixed length k slides along the coordinate axes. If it meets the axes at A(a,0) and
2 2
§ 1· § 1·
B(0,b), then the minimum value of ¨ a  ¸  ¨ b  ¸ is
© a¹ © b¹

4 4
c. k  4  k2  4 
2
a. 0 b. 8 d.
k2 k2
3. If positive numbers a, b, c be in H.P, the equation x 2 – kx + 2b101 – a101 – c101 = 0 (k H R) has
a. both roots imaginary b. one root is positive and other is negative
c. both roots positive d. both roots negative
2n
§ n 1·
n
4. If n H N, n ¨ ¸ >k where k is
© 2 ¹
a. (2n!)3 b. 2(n!)3 c. (n!)3 d. none of these

yz xz xy
5. If x, y, z H R+, then is   is always
yz xz xy

1 1 1 1
a. < (x+y+z) b. > xyz c. < (x+y+z) d. > xyz
2 3 3 2
f f f
1
6. ¦¦¦ 3 3 3
i 0 j 0 k 0
i j k is

(i z j z k)

1 81
a. b. c. 1 d. none of these
27 208

176
( x  1)( x  2)
7. Minimum value of :  x > 3 is
x 3

a. 3 2  2 b. 3  2 2 c. 3  2 3 d. 3 2  2
8. The least value of 6tan2T+ 54cot2T + 18 is
i) 54 when A.M. > G.M is applied for 6tan 2T 54cot2T , 18
ii) 54 when A.M > G.M is applied for 6tan2T, 54cot2T and 18 is added further.
iii) 78 when tan2T = cot2T.
a. (iii) is correct b. (i) is correct (ii) is flase
c. (i) and (ii) are correct d. none of these
9. If A, G, H are A.M, G.M, H.M between the same two numbers, such that A–G=15 and
A–H=27, then the numbers are
a. 100, 50 b. 120, 30 c. 90, 60 d. none of these
10. If a, b, c H R, the square root of a 2+b2+c2–ab–bc–ac is greater than or equal to

3 3
a. max {|b–c|, |c–a|, |a–b|} b. max {|b–c|, |c–a|, |a–b|}
2 2

3
c. max {|b–c|, |c–a|, |a–b|} d. max {|b–c|, |c–a|, |a–b|}
4
11. If x1, x2, x3, x4 are four positive real numbers such that

1 1 1 1
x1 + x = 4, x 2 + x = 1, x 3 + x = 4, x 4 + x = 1 then
2 3 4 1

a. x1 = x 3 and x 2 = x 4 b. x 2 = x 4 but x 1 z x 3
c. x1 x 2 = 1, x 3x 4 = –1 d. x 3x 4 = 1, x, x 2 z 1

1 1 1
12. If a, b, c > 0 and a (1–b) > , b (1–c) > , c (1–a) > , then
4 4 4
a. never possible b. always true c. cannot be discussed d. none of these

1 1 1
13. If a, b, c are the sides of a triangle, then , , are also the sides of the triangle is
bc ca ab
a. sometimes true b. always true c. cannot be discussed d. never true
14. Given n4 < 10n for a fixed positive integer n > 2, then
a. (n+1)4 < 10n+1 b. (n+1)4 > 10n+1 c. nothing can be said d. none of these

177
Answers
1. a 2. d 3. b 4. c 5. a 6. b
7. b 8. c 9. b 10. a 11. a 12. a
13. b 14. a

178
SEQUENCES AND SERIES - V
Arithmetico Geometric Series & Special Sequences

Arithmetico-geometric Series
A series is said to be an arithmetico geometric series if its each term is formed by multiplying
the corresponding terms of an A.P and a G.P.
E.g. 1+2x+3x2+4x3+...........
Here 1,2,3,4.....are in A.P and 1,x,x 1,x 3.......are in G.P

Sum to n terms
Let Sn = a+(a+d)r + (a+2d)r 2+......(a+(n–1)d)r n–1........(1)
Multiply by r on both the sides
rSn ar  a  d r 2  ........ a  n – 1 d r n .......(2)

(1) – (2) Ÿ Sn 1 – r a  dr  dr 2  .....dr n –1 – a  n – 1 d r n
(n–1)terms

= a  dr
1 – r – a  n – 1 d r
n –1
n

1– r
a



dr 1 – r n –1 a  n – 1 d r n
Sn =
1– r 1 – r 2 1– r
Sum to infinity
If r  1 and n o f , then nlim rn 0
of

a dr
? S= 
1 – r 1 – r 2
Note: If we take the first term of a G.P to be b, then

Sn = 


ab dbr 1 – r n –1 a  n – 1 d br n
1– r 1 – r 2 1– r
ab dbr
If r  1 , then sum to infinity, S 
1 – r 1 – r 2
Use of Natural numbers
1 Let Sr = 1r + 2r + 3r +.......................+nr, then
n (n  1)
(i) S1 = 1 + 2 + 3 +..................+n =
2
n ( n  1)(2n  1)
(ii) S2 = 12 + 22 + 33 +..................+n2 =
6

179
n 2 (n  1) 2
(iii) S3 = 13 + 23 + 33 +..................+n3 = = S12
4
n (n  1)(2n  1)(3n 2  3n – 1) S2
(iv) S4 = 14 + 24 + 34 +..................+n4 = = (6S1–1)
30 5
n 2 (n  1) 2 (2n 2  2n – 1) 1
(iv) S5 = 15 + 25 + 35 +..................+n5 = = S12(4S1–1)
12 3
2 1 + 3 + 5 + ....................to n terms = n2
n (4n 2 – 1)
3 1 + 3 + 5 + .................to n terms =
2 2 2
3
4 1 + 3 + 5 + .................to n terms = n (2n2 –1)
3 3 3 2

1 – (–1) n
5 1 – 1 + 1 –..................to n terms =
2
1 – (–1) n (2n  1)
6 1 – 2 + 3 –.....................to n terms =
4
(–1) n –1 n (n  1)
7 1 – 2 + 3 –...................to n terms =
2 2 2
= (–1)n–1S1
2
(–1) n –1 (4n 3  6n 2 – 1) – 1
8 1 – 2 + 3 –.................to n terms =
3 3 3
8

Application
If nth term of a sequence is given by
Tn = an3 + bn2 + cn + d, where a, b, c, d  R, then
Sn = ¦ Tn = T1 + T2 +..............+Tn

= a ¦ n 3 + b ¦ n 2 + c ¦ n + d ¦1
Note: If Tn is expressible as product of m consecutive numbers beginning with n,
i.e. Tn = n(n+1) (n+2)....(n+m –1) then
§ n  m·
Sn = n(n+1) (n+2)....(n+m –1) ¨© ¸
m 1¹
n n  1
Eg. If Tn =n, then Sn (Here m = 1)
2
n n  1 n  2
E.g. Tn = n(n+1), then S n = (Here m = 2)
3
Method of differences
If the differences of successive terms of a series are in A.P. or G.P., we can find Tn as follows
(a) Denote nth term and the sum up to n terms by Tn &Sn respectively
(b) Rewrite the given series with each term shifted by one place to the right

180
(c) Subtracting the above two forms of the series, find Tn.
(d) Apply Sn = ¦T n .
Note : Instead of determining the nth item of a series by the method of difference, we can use the
following steps to obtain the same
(i) If the differences T2 –T1, T3 – T2,................etc are in A.P. Then take the nth term as
Tn = an2 + bn + c, a, b, c  R
Determine a, b, c by putting n = 1, 2, 3 and equating them with the values of corresponding
terms of the given series.
(ii) If the differences T2 –T1, T3 –T2, ...........etc are in G.P , with common ratio r, then take Tn=
arn–1 + bn + c, a, b, c  R
Determine a, b, c by putting n = 1, 2, 3 and equating them with the values of corresponding
terms of the given series.
(iii) If the differences of the differences computed in step (i) are in A.P, then take T n = an3 +
bn2 + cn +d
Determine a, b, c by putting n = 1, 2, 3, 4 and equating them with the values of corresponding
terms of the given series.
(iv) If the differences of differences computed in step (i) are in G.P with common ratio r, then take
Tn = arn–1 + bn2 + cn + d
Determine a, b, c by putting n = 1, 2, 3, 4 and equating them with the values of corresponding
terms of the given series.
Summation by " ¦ " (sigma) operator
n

i. ¦T r T1  T2  ......  Tn
r 1

ii. ¦1 1  1  1  ......  ntimes n


r 1

n n

iii. ¦ kTr k ¦ Tr ; k is a constant


r 1 r 1

¦ T r T = ¦ T r ¦ T
n n n
1 1
iv. r r r r
r 1 r 1 r 1

n n
§ n ·§ n ·
v. ¦ ¦ Ti Tj ¨ ¦ Ti ¸ ¨ ¦ Tj ¸
© i 1 ¹© j 1 ¹
j 1 i 1

(Note that i & j are independent here)

vi Now consider ¦ ¦ f i u f j Here three types of terms occur, for which i < j, i > j and
0 d i  jd n

i = j. Also note that the sum of terms when i < j equal to the sum of the terms when i > j
if f(i) and f(j) are symmetrical. In such case,

181
n n

¦ ¦ f i f j ¦ ¦ f (i)f ( j)  ¦ ¦ f (i)f ( j) + ¦ ¦ f (i)f ( j)


i 0 j 0 0d j i d n 0 d i  jd n i j

= 2 ¦ ¦ f (i)f ( j)  ¦ ¦ f (i)f ( j)
0d i  jd n i j

n n

Ÿ ¦ f (i)f ( j) ¦ ¦ f (i)f ( j) – ¦¦ f (i)f ( j)


0 d i  jd n i 0 j 0 i j

2
When f(i) and f(j) are not symmetrical, the sum can be obtained by listing all the terms.
4 3

Example:1 ¦ ¦ ij
i 1 j 1
¦ i 1  2  3 ¦ i  2i  3i
(1+2+3) + (2+4+6) + (3+6+9) + (4+8+12) = 6+12+18+24 = 60
4 3 4 3
4 x 5 3x 4
Also, ¦ ¦ ij ¦ i¦ j 2
x
2
60 (Since i & j are independent)
i 1 j 1 i 1 j 1

Note that 2 ¦ a i a j a1  a 2  .....  a n – a 21  a 22  .....  a 2 n


i j 1

Example: 2 ¦ ¦1 0 d i d j n

n n

¦ ¦1 – ¦ ¦1
i 1 j 1 i j
=
2

§ n ·§ n · n
¨ ¦ 1¸ ¨ ¦ 1¸ – ¦ 1
= © i 1 ¹© j 1 ¹ j 1
2
n.n – n n n – 1 n ij 1 2 3 ...... n
= C2
2 2
1 1.1 1.2 1.3 ...... 1.n
n n
Example:3Consider ¦ ¦ ij
i 1 j 1 2 2.1 2.2 2.3 ...... 2.n
There are 3 types of terms in this summation,
i. Those terms when i < j (upper triangle) 3 3.1 3.2 3.3 ...... 3.n
ii. Those terms when i > j (lower triangle) ..
iii. Those terms when i = j (diagonal) ...
.
It is shown in the diagram
n n.1 n.2 n.3 ...... n.n

182
n n

¦ ¦ ij = sum of terms in upper triangle + sum of terms in lower triangle + sum of terms in
i 1 j 1

diagonal.
n n

¦ ¦ ij = 2 ¦ ¦ ij  ¦ ¦ ij
i 1 j 1 0 d i d jd n i j
(' sum of terms in upper + lower tringles are same)

n n n n n

¦ ¦ ij ¦ ¦ ij – ¦ ¦ ij ¦ ¦ i – ¦ i 2

i 1 j 1 i j i 1 j 1 i 1
0 d i d jd n
2 2
n n  1 n n  1 n n  1 2n  1
. –
= 2 2 6
2
n n - 1 ( 3 n + 2 )
2

=
24
Solved Examples
1. Find sum of the series to n terms
Solution:
1+3x+5x2+7x3+.........
Let Sn = 1+3x+5x 2+7x 3+.........+(2n–3)x n–2 + (2n–1)x n–1........(1)
xSn = x+3x 2+5x 3+............... +(2n–3)x n–1+(2n–1)x n ........(2)
(1) – (2) given
Sn (1–x) = 1+ (2x+2x 2+2x 3+.........+2x n–1) – (2n–1)x n
n–1 terms

§ 1 – x n –1 ·
1  2 x¨ ¸ – 2n – 1 x
n
=
© 1– x ¹

1
 2x –¨

1 – x n –1 § 2n – 1· n
¸x

? Sn =
1– x 1 – x 2 © 1 – x ¹
1 1 1
2. 3 x9 x 27 x........tof
4 8 16

1
a. 3 b. 9 c. d. none of these
3
Solution:
1 1 1 1 2 3
3 4 x9 8 x 27 16 x........tof 3 4 x3 8 x 316 x........tof
1 2 3
   ..........tof
= 3 4 8 16
= 3S..............(1)
1 2 3
Where S =    ..........tof................( 2)
4 8 16

183
1 1 2
S =   ..........tof.......................(3)
2 8 16
§ 1· 1 1 1
(2) – (3) Ÿ S ¨©1 – ¸¹ =   ..........tof
2 4 8 16

1
4 1
=
1 2
1–
2
? S=1
Substituting in (1)
1 1 1
3 4 x9 8 x 27 16........tof = 31 3
Ans: a
f f
m2n
3. ¦ ¦ 3m n.3m  m.3n
m 1n 1

16 9
a. b. c. 1 d. none of these
9 16
f f
m2n
Solution: Let S = ¦¦ m
m 1 n 1 3 n.3  m.3
m n

f f
1
= ¦¦ §3
m 1n 1 · § 3m 3n ·
m

¨ ¸¨  ¸
© m ¹© m n ¹

f f
1 § 3m 3n ·
S= ¦ ¦ a a
m 1n 1  an
............(i) ¨ wherea m
© m
& an

¸
m m

Intercharging m & n
f f
1
S= ¦ ¦ a a
m 1n 1  an
...............(ii)
n m

Adding (i) & (ii)


f f
1
2S = ¦ ¦
m 1n 1 a ma n

f f
mn
= ¦¦ 3 mn
m 1n 1

184
§ f m · § f n · Where f
m f
n 1 2 3
= ¨ ¦ m ¸ ¨ ¦ n ¸ = s1 s1 =
©m 13 ¹ © n 1 3 ¹
¦ 3m ¦3
m 1
n =
3
+ 2 + 3 + ........
3 3
f ) =S
m 1

1 2 3
§ 3· § 3· S=
1
+ 2 + 3 + ........ f _ (i)
= ¨ ¸ .¨ ¸ 3 3 3
© 4¹ © 4¹
1 1 1 2 3
9 S = 2 + 3 + 4 + ........ f _ (ii)
2S = 3 3 3 3
16 Subtratiy equation (ii) From (i)
9 2 1 1 1 1
S = S = + 2 + 3 + ........
32 3 3 3 3
2 1
S=
3
1 3
Ans: d S=
4
4. Sum to n terms of the series 1 2–22+32–42+..........is
Solution: Clearly nth term is negative or positive according n is even or odd.
Case I when n is even.
In this case the series is
(12–22)+(32–42)+..................+((n–1) 2–n2)
– n ( n  1)
= –{1+2+3+......+(n–1)+n} =
2
Case II when n is odd.
In this case the series is
(12–22)+(32–42)+..................+{(n–2) 2 – (n–1)2}+ n2
– n ( n – 1)
=  n2
2
– n  n  2n 2 n 2  n n (n  1)
=
2 2 2
5. Find the sum of all possible products of first n natural numbers taken two by two
1 ­° n n ½°
Solution: ¦ ¦ x i x j = ®¦ ¦ ij – ¦ ¦ ij¾
1d i d jd n 2 °̄ i 1 j 1 i j °¿

1 ­°§ n n  1 · n n  1 2n  1 ½°
2

= ®¨ ¸ – ¾
2 °̄© 2 ¹ 6 °¿

1
= n n  1 n – 1 3n  2
24
1 2 3
6. Find sum to n terms of    ..........
1  1  1 1  2  2 1  32  34
2 4 2 4

n n
1 n2  n 1 – n2 – n 1
Tn
1 n  n
2 4

n  n 1 n2 – n 1
2

2 n2  n 1 n2 – n 1

185
[ 1 – 3  3 – 7
1§ 1 1 · 1 1 1 1 1 1 1
 .....
Tn ¨ 2 – 2 ¸ – 2

2 © n – n  1 n  n  1¹ Sn = 2 2 
n - n 1 n +n 1
Putting n = 1,2,3......n and adding
1­ 1 ½ n2  n
Sn ¦ Tn ®1 – 2 ¾
2 ¯ n  n  1¿
2 n2  n 1
3 5 7 9
7.     .........tof is
4 36 144 400
a. 2 b. 1 c. 3 d. none of these
3 5 7 9
Solution:     .........tof
4 36 144 400
3 5 7 9
=     ..........tof
1x 2 2x3 3x 4 4x5 2
2 2 2

22 – 12 32 – 2 2 4 2 – 32
=    ..........tof
12 x 2 2 2 2 x 32 32 x 4 2
§1 1· § 1 1· § 1 1·
= ¨ 2 – 2 ¸  ¨ 2 – 2 ¸  ¨ 2 – 2 ¸  ..........tof
©1 2 ¹ © 2 3 ¹ © 3 4 ¹
1
= 1
12
Ans: b

8. Find the nth term of the following series


i. 3+7+13+21+.......
Solution:
1st consecutive differences 4,6,8,........are in A.P.
? Tn = an2+bn+c
Putting n = 1,2,3,
a+b+c = 3, 4a+2b+c = 7, 9a+3b+c = 13
Ÿ a = 1, b = 1, c = 1
? Tn = n2+n+1
ii. 5+7+13+31+85+..........
Solution:
1st consecutive differences 2,6,18,54,....... are in G.P. with common ratio 3
? Tn = a.3n–1+bn+c
Putting n = 1,2,3 we get
a+b+c = 5, 3a+2b+c =7, 9a+3b+c = 13
Ÿ a = 1, b = 0, c = 4
Tn = 3n–1+4

186
Vn method
1 1 1
1. If Sn   ......... 
a1a 2 ......a r a 2 a 3 ......a r 1 a n a n 1a n ......a n  r –1
1 1
Tn ,Vn (leave 1st term in denominator of Tn)
a n a n 1......a n  r –1 a n 1......a n  r –1

1 1
Vn–Vn–1 –
a n 1a n  2 ......a n  r –1 a n a n 1......a n  r – 2
1
= a n – a n  r –1
a n a n 1......a n  r –1
Vn–Vn–1= Tn (1–r)d
–1 –1
Tn = (Vn – Vn –1 ) Ÿ Sn (Vn – V0 )
d(r – 1) d (r – 1)
1 1 1 n
E.g   .......... 
1.2 1.3 n (n  1) n 1

1 1 1 1§ 1 1 ·
  .................  ¨ – ¸
1.2.3 2.3.4 n n  1 n  2 2 © 1.2 n  1 n  2 ¹

1 1 1 1§ 1 1 ·
  .................  ¨ – ¸
1.2.3.4 2.3.4.5 n n  1 n  2 n  3 3 © 1.2.3 n  1 n  2 n  3 ¹
2. If Sn = a1a2.....ar+a2a3......ar+1+.......+anan+1.....an+r–1

Tn = anan+1.......an+r–1 Vn = anan+1.......an+r–1an–r
Vn–Vn–1 = anan+1.....an+r – an–1an......an+r–1
= anan+1.....an+r–1 (an+r – an–1)
= Tn (r+1)d
1 1
Tn = (Vn – Vn –1 ) Ÿ Sn (Vn – V0 )
d(r  1) d (r  1)
1.2+2.3+.....+n(n+1) = n(n+1) n  2
3
1.2.3.4+2.3.4.5+........+ n(n+1)(n+2)(n+3) = n(n+1)(n+2)(n+3) n  4
5
Alternative method
1 1 1
1. S   .............. 
1.2.3 2.3.4 n (n  1)(n  2)

187
1 ­ 3 –1 4 – 2 ( n  2) – n ½
= ®   .............  ¾
2 ¯1.2.3 2.3.4 n (n  1)(n  2) ¿

1 ­§ 1 1 · § 1 1 · § 1 1 ·½
= ®¨ – ¸ ¨ – ¸  ......  ¨ – ¸¾
2 ¯© 1.2 2.3¹ © 2.3 3.4 ¹ © n (n  1) (n  1)(n  2) ¹ ¿

1­ 1 1 ½
= ® – ¾
2 ¯1.2 (n  1)(n  2) ¿

(n )(n  1)(n  2)(n  3)


2. 1.2.3+2.3.4+......+n(n+1)(n+2) =
(3  1)
t n ( n  3)
=
31
Tn (n  m)
i.e. Sum of the product of m consecutive natural Sn
m 1
PRACTICE QUESTIONS
1. If R  0, ʌ denote the set of values which satisfies the equation

2 1 cosx  cos2x  cos3 x  ......


4 , then R =
­– ʌ½ ­ ʌ 2ʌ ½ ­ – ʌ 2ʌ ½ ­ ʌ – 2ʌ ½
a. ® ¾ b. ® , ¾ c. ® , ¾ d. ® , ¾
¯ 3 ¿ ¯3 3 ¿ ¯ 3 3¿ ¯3 3 ¿
n i j

2. Find the value of the expression ¦ ¦ ¦ 1


i 1 j 1 k 1

20
n
3. If in a series tn = , Then ¦ t n is equal to
n  1 ! n 1

20!–1 21!–1 1
a. b. c. d. none of these
20! 21! 2(n – 1)!
§1 1 1 ·
4. The value of 0.2 log 5¨
©
  ......f ¸
4 8 16 ¹ is
1
a. 1 b. 2 c. d. 4
2
5. lim (1+3–1) (1+3–2) (1+3–4)(1+3–8)........ 1  3 –2n is equal to
nof
1 3
a. 1 b. c. d. none of these
2 2

188
6. lim 1.2.3  2.3.4  3.4.5  ..........uptonterms is equal to
nof
n 1.2  2.3  3.4  .........uptonterms
3 1 1 5
a. b. c. d.
4 4 2 4
1 2 1 2 1
7.* Sum to infinite terms of the series tan –1  tan –1  tan –1  tan –1  tan –1  ......
2 9 8 25 18
1 1
a. tan –1 3 b. cot –1 c. tan –1 d. cot –1 3
3 3
f (x )
8. If f(x) = a0+a1x+a2x 2+......+anxn +.......and =b +b x+b2x 2+.......+b nx n+......If a0 = 1 and
1– x 0 1
b1 = 3 and b10 = k11–1, and a0, a1,a2,......are in G.P then k is
1
a. 2 b. 3 c. d. none of these
2
1 1
9. The value of n for which 704+ (704)+ (704)+......up to n terms =
2 4
1 1
1984 – (1984)+ (1984).......upto n terms is
2 4
a. 5 b. 3 c. 4 d. 10
10. If 12+22+32+......+2003 2 =(2003) (4007) (334) and (1) (2003) +2(2002) +(3) (2001).......+
(2003) (1) = (2003) (334) (x), then x equals
a. 2005 b. 2004 c. 2003 d. 2001
11. Read the passage and answer the following questions
Let T1,T2 .....Tn be the terms of a sequence and let (T 2–T1) = T21, (T3–T2) = T21 ,......
(Tn–Tn–1) = T1n–1
Case I: If T11, T21...... T1n–1 are in A.P., then T n is quardratic in ‘n’. If T 11–T21,
T21–T31,....are in A.P., then Tn is cubic is n.
Case II: If T11, T21...... Tn1–1 are not in A.P., but in G.P. , then T n = arn+b where r is the
common ratio of the G.P. T11, T21,T31.....and a,b R. Again if T11, T21...... Tn1–1 are not in
G.P., but T21–T11,T31–T21,..... are in G.P., then T n is of the form ar n–1+bn+c and r is
the C.R. of the G.P. T 21–T11,T31–T21, ...... and a,b,c R.
i. The sum of 20 terms of the series 3+7+14+24+37+...... is
a. 4010 b. 3860 c. 4240 d. none of these
ii. The 100th term of the series 3+8+22+72+226+1036+.....is divisible by 2 n,then maxi
mum value of n is
a. 4 b. 2 c. 3 d. 5
Tn
iii. For the series 2+12+36+80+150+252+....., the value of lim is (where Tn is the nth
n of n3
term)

189
1
a. 2 b. c. 1 d. none of these
2
12.* Match the following
Column I Column II
5 2 13
a. If the sum of the series    ...... p. 28
7 3 21
up to n terms is 5, then n =
b. A term of the sequence 1,3,6,......is q. 10
c. Sum of the series 1+2+3+......upto
7 terms is r. 36
d. If ¦n 3
1296 , then ¦ n is equal to s. 21

Note:* QuestionS with more than one option is correct

ANSWERS

n ( n  1)( n  2)
1. b 2. 3. b 4. d
6
5. c 6. a 7. a,b 8. a

9. a 10. a 11. (i) c (ii) c (iii) c

12. a o q,s; b o p,q,r,s; c o p; d o p

190
SEQUENCES AND SERIES - VI
Sequences and Series - Problem Solving
Sequence
A sequence is a function of natural numbers with codomain as the set of real numbers. It is said to
be finite or infinite according it has finite or infinite number of terms. Sequence a1, a2,........ an is
usually denoted by {an} or <an>
Series
By adding or subtracting the terms of a sequence we get a series.
Arithmetic Progression (A.P.)
It is a sequence in which the difference between two consecutive terms is the same.
For a sequence {an} which is in A.P, nth term an=a+(n–1)d= A (last term) which is always a linear
expression in n)
d=an–an–1 (If d= 0, then sequence is a constant sequence. if d>0 the sequence is increasing; if
d<0,the sequence is decreasing)
nth term from the end an1 = A +(n–1)(–d)
= A –(n–1)d

­n
°° 2 2a  (n – 1)d
Sum to n terms = ® or
° n
¯° (a  A)
2
1
(Sn is a quadratic expression in n; common difference = coefficient of n2)
2
Also an = Sn–Sn–1
Arithmetic mean
ac
If a, b, c are in A.P, then b= is called the single arithmetic mean of a & c. Let a & b be two
2
given numbers and A1, A2,........... An are n A.M’s between them. Then a, A1,A2,...An, b are in A.P.
b–a
Common difference of this sequence d= .
n 1
A1= a+d, A2= a+2d etc. we can find all the arithmetic means.
Properties of A.P.
1 If a1, a2, a3, ........ are in A.P; then a1 r k, a2 r k,a3 r k,..................... are also in A.P..
a1 a 2 a 3
2 If a1, a2, a3,................. are in A.P, then a1 O , a2 O ,a3 O ,.................... and , , ..............
O O O
are also in A.P ( O z 0)
3 If a1, a2,.......... an are in A.P, then an,an–1,...............a2,a1 is also an A.P with common difference (–
d)
4 If a1, a2, a3, ..................and b1, b2, b3, .......................... are two A.P.s then a1 r b1,a2 r b2,a3 r b3,.....
are also in A.P.

191
5 If a1, a2, a3,.............. and b1, b2, b3,................are two A.P.s then a1b1, a2b2, a3b3,...........and
a1 a 2 a 3
, , ,................. are NOT in A.P..
b1 b 2 b 3
6 If 3 numbers are in A.P we may take them as a–d, a, a+d. If 4 numbers are in A.P, we can take
them as a–3d, a–d, a+d, a+3d.
7 In an arithmetic progression, sum of the terms equidistant form the beginning and end is a constant
and equal to sum of first and last term.
ie for {an},
a1+an = a2+an–1=a3+an–2=......
a r –k  a rk
Also ar = , 0 d k d n–r..
2
8 Sum of n arithmetic means between two given numbers a & b is n times the single A.M between
them .
§a b·
ie. A1+A2+...............+An = n ¨ ¸
© 2 ¹

­n (middleterm);if nisodd.
°
9 Also Sn = a1+a2+......+an= ® n (sum of two middle terms); if n is even
°̄ 2        

Geometric Progression (G.P.)


It is a sequence in which the ratio of any two consecutive terms is the same. For a sequence {an}
which is in G.P. nth term an = arn–1 (last term)
an
Common ratio r = a (r z 0 . If r>1, the sequence is an increasing sequence, if 0<r<1 then the
n –1
sequence is decreasing )
n –1
§1·
n term from the end a = an ¨ ¸
th 1
(an1 = nth term from end)
n
©r¹
Note : No term of G.P. can be zero
­ a (r n – 1)
° ,rz1
Sum to n terms Sn = ® r – 1
°na , r 1
¯  
a
If |r|<1, the sum of the infinite G.P is given by S f =
1– r
Geometric mean
If a, b, c are in G.P, then b2 = ac or b = ac is called the single geometric mean of a & c. Let a &
b be two given numbers and G1, G2, .....Gn are n G.M.s between them. Then a, G1, G2,.........Gn,

192
1
§ b · n 1
b are in G.P. Common ratio of this sequence r = ¨ ¸
©a¹
G1, = ar, G2 = ar2 etc. we can find all the geometric means.
Properties of G.P.
a1 a 2 a 3
1 If a1, a2, a3................... are in G.P., then a1k, a2k, a3k,.............. and , , , ...............are
k k k
also in G.P (k z 0).
1 1 1
2 If a1, a2, a3,................ are in G.P., then a , a , a ,....................and a1n, a2n, a3n,..........are also
1 2 3

in G.P.
3 If a1, a2, a3,......... an are in G.P with common ratio r, then an, an–1.............a2, a1 is also in G.P. With
1
common ratio .
r
4 If a1, a2, a3........ and b1, b2, b3,.......... are two G.P.s then a1 r b1, a2 r b2, a3 r b3,........ are NOT in
G.P.
5 If a1, a2, a3, .............. and b1, b2, b3,................are two G.P.s then a1b1, a2b2, a3b3,...........and
a1 a 2 a 3
b1 , b 2 , b 3 ,.............. are also in G.P.
.P.

a
6 If 3 numbers are in G.P., we may take them as , a, ar. If 4 numbers are in G.P., we can take them
r
a a
as , , a r, a r3.
r3 r
7 In a geometric progression, product of the terms equidistant from the beginning and end is a
constant and equal to product of first and last term.
ie For {an}
a1an = a2 an–1 = a3 an–2 =.......
Also ar =
a r – k a r  k , 0 d k d n–r..
8 Product of n geometric means between two given numbers a & b is nth power of the single G.M.
between them.
ie G1G2G3..........Gn = ab n
9 If a1, a2, a3,............... are in G.P. (ai>0  i), then loga1,loga2,loga3,...... are in A.P. Its converse is
also true.
Harmonic Progression (H.P.)
A sequence is said to be in H.P if the reciprocals of its terms are in A.P.
1 1 1
ie. if a1,a2,a3,....... an are in H.P., then a , a ,...... a are in A.P..
1 2 n

193
For a sequence {an} which is in H.P.,
1 a1a 2
nth term an = =
1 § 1 1 · a 2  (n – 1)(a1 – a 2 )
 (n – 1)¨¨ – ¸¸
a1 © a 2 a1 ¹

1 a 1a 2 a n
nth term from end an1 = =
1 § 1 1 · a 1a 2 – a n (n – 1)(a 1 – a 2 )
– (n – 1)¨¨ – ¸¸
an © a 2 a1 ¹
Note : No term of H.P. can be zero. There is no general formula for finding out the sum of n terms
of H.P.
Harmonic mean
2ac
If a,b,c are in H.P; then b = is called the single H.M. between a & c. Let a & b be two given
ac
numbers and H1, H2,..............,Hn are n H.M.s between them. then a, H1, H2,...... Hn, b are in H.P.
The common difference d of the corresponding A.P is
a–b
d=
(n  1)ab
1 1 1 1
= d, =  2d etc. we can find all the harmonic means.
H1 a H2 a
Note: The sum of reciprocals of n Harmonic means between two given numbers is n times the
reciprocal of single H.M. between them.
§1 1·
1 1 1 ¨  ¸
ie + +....... = n ©a b¹
H1 H 2 Hn
2
Note : If a, b, c are three successive terms of a sequence. Then

­a
° a Ÿ a ,b,careinA.P.
°
a–b °a
= ® Ÿ a ,b,careinG.P.
b–c °b
°a
° c Ÿ a ,b,careinH.P.
¯
Relation between A.M., G.M., and H.M.
For positive numbers a1, a2, a3, .................an
a1  a 2  .......  a n
A.M. = A =
n

194
1
G.M. = G = a a .......a n
1 2 n

n
H.M = H = 1 1 1 ,
  ...... 
a1 a 2 an

A t G t H and G2 = AH.
(equality holds if a1 = a2 =...............an)
2 2
a 1  a 2  ......  a n
2
a1  a 2  ........  a n
Note : Also t
n n
(Root mean square inequality)
Note : The quadratic equation having a, b as its roots is x2–2Ax+G2 = 0 and a : b = A+ A 2 – G 2

:A– A 2 – G 2 where A,G are respectively the A.M. and G.M. of a & b
Note : Formation of progressions
Two consecutive terms determine the required progression. If two numbers a & b are given, then
(i) a, b, 2b–a is A.P.
b2
(ii) a, b, is G.P..
a
ab
(ii) a, b, is H.P..
2a – b
Solved Examples.
1 If the p , q and r terms of an A.P are in GP, then the common ratio of the G.P is
th th th

pq r–q p–r


(a) (b) (c) (d) None of these
rq q–p p–q
Solution : Tp, Tq, Tr are in G.P
Tq
Tr Tq Tr
ŸT – 1 –1
p
= Tq Ÿ Tp = Tq

Tq – Tp Tr – Tq Tq Tr – Tq
Ÿ Tp = Tq ? T = T –T
p q p

Tq
(A  (r – 1)D) – (A  (q – 1)D) r–q
Ÿ T = (A  (q – 1)D) – (A  (p – 1)D) = q – p
p

Ans : (b)
2 If 4a2+9b2+16c2 = 2(3ab+6bc+4ca), where a, b, c are non–zero real numbers then a, b, c are in
(a) A.P. (b) G.P. (c) H.P. (d) None of these
Solution : Multiply by 2 on both sides

195
4a2+4a2+9b2+9b2+16c2+16c2–12ab–24bc–16ca = 0
Ÿ (2a–3b)2+(3b–4c)2+(4c–2a)2=0
Ÿ 2a=3b=4c= O
O O O
Ÿa ,b ,c
2 3 4
1 1 1
2,3,4 are in AP Ÿ , , are in H.P..
2 3 4
O O O
Ÿ , , are in HP gives
2 3 4
a, b, c are in HP
Ans (c)
3 If a, a1, a2, a3, ..........,a2n, b are in AP and a, g1, g2, g3, .....................g2n, b are in G..P. and h is the
single harmonic mean of a & b, then
a1  a 2n a 2  a 2 n –1 a n  a n 1
g 1g 2 n + g 2g 2n –1 +...................+ g n g n 1 is equal to
2n n
(a) (b) 2nh (c) nh (d)
h h
Solution :
a1+a2n = a2+a2n–1 =......................= an+an+1 = a+b and
g1g2n = g2.g2n–1=.....................=gn.gn+1 = ab
2ab
Also h =
ab
ab ab ab
? Given expression = + +............. (n times)
ab ab ab
ab n .2 2n
=n = =
ab h h
Ans : (a)
S
4 If 0<x< , then the minimum value of
2
(sinx+cosx+cosec2x)3 is
27 27
(a) 27 (b) (c) (d) None
2 4
Solution : Apply A.M t GM
1
sinx  cosx  cosec2x
Ÿ t sinx.cosx.cosec2x 3

3
1
sinx  cosx  cosec2x § sin x cos x · 3
t ¨ ¸
3 © 2 sin x cos x ¹

196
Cubing both sides
sin x  cos x  cos ec2x 3 1
t 2
27
27
Minimum of sinx  cosx  cosec2x 3 =
2
Ans : (b)
5 Sum of certain odd consecutive positive integers is 572 –132, then the integers are
(a) 25, 27, 29,..........111 (b) 27, 29, ..............113
(c) 29, 31, 33,...........115 (d) None of these
Solution :
(2m+1)+(2m+3)+..............n terms = 572–132
n
{2.(2m+1)+(n–1)2} = 572–132
2
Ÿ n(2m+n)=572–132
n2+2mn+m2–m2=572–132
(n+m)2–m2=572–132
Ÿ n+m =57 and m = 13, Solve to get n = 44
Hence, the series is
27, 29, 31,.......................,113
Ans : (b)
xy yz
6 If x, y, z are three positive numbers in A.P, then the minimum value of + is
2y – x 2y – z
1
(a) 2 (b) 4 (c) (d) None of these
4
Solution :
zx
put y= in the given expression
2
zx zx
x z
= 2 + 2
zx –x zx–z
3x  z 3z  x
= 
2z 2x
3x 1 3z 1
=   
2z 2 2 x 2
x z
3§x z · 2  z x x z
= ¨  ¸ Now ' AM t GM Ÿ z x t . Ÿ  t2
2© z x¹ 2 2 x z z x

197
3
t 2 u 2+1 = 4
Ans : (b)
7 If n arithmetic means are inserted between 50 and 200, and n harmonic means are inserted
between the same two numbers, then a2.hn–1 is equal to
(a) 500 (b) 5000 (c) 10,000 (d) None of these
Solution :
50, a1, a2,..................an, 200 are in AP ............................................... (1)
Also, 50, h1, h2,................,hn 200 are in H.P
1 1 1 1 1
Ÿ , h , h ,..................... h , are in AP
50 1 2 n 200

1 1 1 1 1
Ÿ , h , h ,..................... h , are in AP
200 n n –1 1 50
Multiply by 200×50 = 10,000
10,000 10,000 10,000 10,000
Ÿ 50, h , h , ......................... h , h , 200 are in AP................. (2)
n n –1 2 1
Now (1) and (2) are identical.
10,000
Ÿ a2 = h n –1 gives a2.hn–1 = 10,000
Ans : (c)
PRACTICE QUESTIONS
1 If a1, a2, .......... an are positive real numbers whose product is a fixed number c, then the minimum
value of a1+a2+.........+an–1+2an is
(a) n(2c)1/n (b) (n+1)c1/n (c) 2nc1/n (d) (n+1)(2c)1/n
3
2 If a, b, c are in A.P. and a2, b2, c2 are in G.P. If a<b<c and a+b+c = , then the value of a is
2
1 1 1 1 1 1
(a) (b) (c) – (d) –
2 2 2 3 2 3 2 2
3 Let ƒ(x) = ax +bx+c, a z 0 and ' =b2 – 4ac. If D + E , D 2+ E 2 & D 3+ E 3 are in G.P,
2
.P, then
(a) ' z 0 (b) b ' =0 (c) c'= 0 (d) bc z 0
bc bc b–c
4 If = =3 , then a, b, c, d are in
ad ad a –d
(a) A.P (b) G.P (c) H.P (d) A.G.P.
5* The 4th term of the A.G.P. 6, 8, 8, ............. is
32 64
(a) 0 (b) 12 (c) (d)
3 9
6 If x= 111......1(20digits), y=333.........3(10digits) and

198
x – y2
z=222...........2(10digits) then =
z
1
(a) 1 (b) 72 (c) (d) 3
2
7 Read the passage and answer the questions that follow.
An odd integer is the difference of two squares of integers.
The cube of an integer is difference of two squares.
The cube of an odd integer can be expressed as difference of two squares in two different
ways.
The difference of the cubes of two consecutive integers is difference of two squares.
(i) If 103= a2–b2, then a–b =
(a) 5 (b) 0 (c) 10 (d) 15
(ii) If 9 =a –b = c –d , a+b+c+d =
3 2 2 2 2

(a) 720 (b) 750 (c) 800 (d) 810


(iii) 15 –14 =a –b , ab =
3 3 2 2

(a) 90000 (b) 95940 (c) 99550 (d) 99540


8 Match the following :-
a n 1  b n 1
For the given number a and b, is
a n  bn
Column I Column II
(a) A.M. (p) for n=1
(b) G.M (q) for n=1/2
(c) H.M. (r) for n=0
(s) for n= –1/2
(t) for n= –1
9 The sum of the products of the ten numbers r 1, r 2, r 3, r 4, r 5 taking two at a time is
(a) 165 (b) –55 (c) 55 (d) None of these
10 Let a1 = 0 and a1, a2, a3,...............an be real numbers such that |ai| = |ai–1+1| for all i, then the A.M.
of the numbers a1, a2, a3................ an has the value A where
–1 –1 –1
(a) A< (b) A<–1 (c) At (d) A=
2 2 2
Note : Questions with more than one option is correct.

Answers
1. a 2. d 3. c 4. c 5. c,d 6. a 7. (i) c (ii) d (iii) d
8. a o r, b o s, c o t 9. b 10. c

199
SEQUENCES AND SERIES - VII
Sequences and Series - Problem Solving
Some important Logarithmic and Exponential formulae
1 If ax = y, then x = log ay
2 log aa = 1 & log a1 = 0
3 a log a n n
4 logamn = logam+logan
§m·
5 loga ¨ ¸ = logam – logan
©n¹
6 logamn = n logam
log c a
7 logba = log b
c

m
8 log a n a m =
n
1
9 logba = log b
a

x2 x3 x4
10 loge (1+x) = x – + – +............
2 3 4
x2 x3 x4
11 loge (1 – x) = – x – – – ............
2 3 4
§ x2 x4 x6 ·
12 loge(1+x) (1–x) = loge(1–x ) = –2 ¨¨ 
2   ............¸¸
© 2 4 6 ¹

§1 x · § x3 x5 ·
13 loge ¨ ¸ ¨
= 2¨ x    ................... ¸¸
©1 – x ¹ © 3 5 ¹
1 1 1
14 loge 2 = 1 – + – +..................
2 3 4
x x2 x3
15 e = 1 + + + + ...............
x
1! 2! 3!

§ x2 x4 ·
16 e +e = 2 ¨¨1 
x –x   ............¸¸
© 2! 4! ¹

§ x x3 x5 ·
17 e –e = 2 ¨¨ 
x –x  .......... .. ¸¸
© 1! 3! 5! ¹

200
1 1 1
18 e = 1+ + + +...............
1! 2! 3!
e is an irrational number & it lies between 2 & 3. e # 2.7183
y2 y3
19 ay = e y log e a = 1+y(logea) + (logea)2+ (logea)3+..................
2! 3!

20 log2 log2 ............... 2 = n where n is the number of square roots

21 log b
a a = b b
log a

Vn - Method
(i) To find the sum of series of the form
1 1 1
a 1a 2 ............a r + a 2 a 3 .......... ..a r 1 + ....................+ a n a n 1............a n  r –1 where a1, a2,

§ 1 ·
¨¨ Here Tn ¸
© a n a n 1............a n  r –1 ¸¹

1
........are in A.P. Let Vn = a a ............a (avoiding first term for Vn ie an in Tn)
n 1 n  2 n  r –1

–1 1
Tn = (Vn–Vn–1) = (V –V )
d (r – 1) d(r – 1) n–1 n
put n = 1, 2, 3............. n and add to get Sn.
1
Sn = T1+T2+........Tn = (V0–Vn)
d(r – 1)

1 § 1 1 ·
= ¨¨ – ¸
d(r – 1) © a1a 2 ......a r –1 a n 1a n  2 ......a n  r –1 ¸¹

1 1 1 1 § 1 1 ·
¨¨ – ¸
(n  1)(n  2) ¸¹
Eg: (a) + + ............... + =
1.2.3 2 .3 .4 n ( n  1)(n  2) 1.(3 – 1) © 1.2
1 1
= –
4 2(n  1)(n  2)
1 1 1
(b) + + ..........
1.2.3.4 2 .3 .4 .5 n (n  1)(n  2)(n  3)

1 § 1 1 ·
¨¨ – ¸
(n  1)(n  2)(n  3) ¸¹
=
1.(4 – 1)© 1.2.3
(ii) Summation of series of the form a1a2......ar+a2a3........ar+1+.....................+anan+1.........an+r–1 where
a1, a2.......are in A.P

201
Here Tn = anan+1.........an+r–1
Let Vn = anan+1..............an–r+1an–r (Take one term extra in Tn for Vn)
1
Tn = (V –V ).
(r  1)d n n–1
Put n = 1, 2, 3 ........and add to get Sn
1 1
Sn = T1+T2+.............+Tn = (Vn–V0) = (a a ..........an+ r – a0a1a2...........ar) where
d(r  1) d(r  1) n n+1
a0=a1 – d
1 n ( n  1)(n  2)
Eg (a) 1.2+2.3 +....................+ n (n+1) = ((n+1)(n+2) – 0.1.2) =
1.(2  1) 3
(b) 1.2.3.4 + 2.3.4.5 +...........................+ n(n+1)(n+2)(n+3)
1
= (n(n+1)(n+2)(n+3)(n+4) – 0.1.2.3.4)
1.(4  1)
1
= n(n+1)(n+2)(n+3)(n+4)
5
SOLVED EXAMPLES
1 If the sides of a triangle are in A.P and the greatest angle of the triangle is double the smallest, then
the ratio of sides of the triangle is
(a) 3 : 4 : 5 (b) 4 : 5 : 6 (c) 5 : 6 :7 (d) None of these
Solution :
Applying sine rule, we have
A
a–d a a d
= =
sin T sin(S – 3T) sin 2T S –T

a–d
a–d a ad a+d

Ÿ = =
sin T 3 sin T – 4 sin T
3
2 sin T cos T
a –d a ad ad B
2T T
C
Ÿ = = gives cos T =
1 3 – 4 sin 2 T 2 cos T 2(a – d )
a
Also, 3–4sin2 T =
a–d
2
a §a d· a
Ÿ 3–4(1–cos T ) =2
gives –1+ ¨ ¸ =
a–d ©a –d¹ a–d
4ad
Ÿ = a Ÿ 4ad = a2–ad gives a = 5d
a–d
? sides a–d : a : a+d
5d–d : 5d : 5d+d = 4 : 5: 6
Ans : (b)

202
2 The sum of
2 3
3 1 4 §1· 5 §1·
. + ¨ ¸ + .¨ ¸ +............. n terms is
1 .2 2 2 . 3 ©2¹ 3 .4 © 2 ¹
1 1 1
(a) 1– (b) 1– (c) 1+ (d) None of these
n  1 2 n n.2 n –1 n  1 2 n
Solution :
n2 1 §2 1 · 1
Tn = . n =¨ – ¸. n
n.(n  1) 2 © n n 1¹ 2
1 1
Ÿ Tn = n.2 n –1 – (n  1).2n
Putting n = 1, 2, 3, ..............,n
1 1
? T1 = 0

1 .2 2 .21
1 1
T2 = –
2 .2 3 .2 2
1

1 1
T3 = 2

3 .2 4 .3 2
.
.
.
1 1
Tn = –
n.2 n –1
n  1 2 n
Adding , T1+T2+T3+.........+Tn = Sn
1
Sn = 1 –
n  1 2 n
Ans : (a)
3 Coefficient of x49 in the expansion of (x–1)(x–3)(x–5).........................(x–99) is
(a) –992 (b) 1 (c) –2500 (d) None of these
Solution :
(x–1)(x–3)(x–5) ................ (x–99)
= x50–S1x49+S2x48.................
? Coefficient of x49 is –S1 50
(1+99) = 2
2
Ans : (c)
4 The coefficients of x15 in the product
(1–x)(1–2x)(1–22x)..............(1–215x) is
(a) 2105–2 121 (b) 2121–2 105 (c) 2120–2 104 (d) None of these

203
Solution :
(1–x)(1–2x)(1–22x)..............(1–215x)
= (–1)16(x–1)(2x–1)(22x–1).................. (215x–1)
§ 1· § 1· § 1 ·
= 21.22.23...................215(x–1) ¨ x – ¸ ¨ x – 2 ¸ ........... ¨ x – 15 ¸
© 2¹ © 2 ¹ © 2 ¹

§ 1· § 1· § 1 ·
= 2120.(x–1) ¨ x – ¸ ¨ x – 2 ¸ ........... ¨ x – 15 ¸
© 2¹ © 2 ¹ © 2 ¹

­ 1 1 1 ½
? coeff of x15 is –2120 ®1  2  2 2  . . .  215 ¾
¯ ¿

­ 1 ½
®1 – 16 ¾ § 1 ·
= –2120.1 ¯ 2 ¿ = –2121 ¨1 – 16 ¸ = 2105–2121
1 © 2 ¹
1–
2
Ans : (a)
5 The sum to 2n terms of the series
3 7 15 31 63 127
+ + + + + +........................ is
4 4 16 16 64 64
2 § 1 · 10 § 1 · 13 1
(a) 3n– ¨1 – n ¸ (b) 3n– ¨1 – n ¸ (c) 3n– (d) None of these
3 © 4 ¹ 21 © 4 ¹ 21 4 n
Solution :
Given expression
§ 1· § 1· § 1· § 1·
= ¨1 – ¸ + ¨ 2 – ¸ + ¨1 – ¸ + ¨ 2 – ¸ +...........................2n terms
© 4¹ © 4 ¹ © 16 ¹ © 16 ¹

§1 1 ·
= 3n –2 ¨   ..........nterms ¸
© 4 16 ¹

§ 1 ·
¨1 – n ¸
1 © 4 ¹
= 3n – 2 1
4 1–
4

2§ 1 ·
= 3n – ¨1 – n ¸
3© 4 ¹
Ans : (a)

204
6 The sum to n terms of the series
1 1 1
1 + 1 + 1
+ ......... f
1– 1  3 – 1  3  5 –
4 4 4
2n 4n 2
(a) (b) (c) (d) None
2n  1 2n  1 2n  1
Solution :
1 1
Tn = =
1  3  5  ............  nterms n 2 – 1
4

§ 1· § 1·
1 ¨n  ¸ – ¨n – ¸
© 2¹ © 2¹
§ 1 ·§ 1 ·
= ¨ n  ¸¨ n – ¸ = § 1 ·§ 1·
© 2 ¹© 2¹ ¨ n – ¸¨ n  ¸
© 2 ¹© 2¹

­ 1 1 ½
Tn = 2® – ¾
¯ 2n – 1 2n 1¿

§1 1 ·
? T1 = 2¨ 1 – 3 ¸
© ¹
§1 1·
T2 = 2¨ – ¸
©3 5¹
.
.
.
§ 1 1 ·
Tn = 2¨ – ¸
© 2n – 1 2n 1 ¹
§ 1 · 4n
Adding, Sn = T1 +T2+.............+Tn=2 ¨1 – ¸=
© 2n  1 ¹ 2n  1
Ans : (b)
7 The sum of the series
1 2 3
+ + + ...................... f is
1.3 1.3.5 1.3.5.7
1 3
(a) 1 (b) (c) (d) None
2 2
Solution :
n
Tn =
1.3.5.............. 2n – 1 2n  1

205
1 ­ 2n  1 – 1 ½
= ® ¾
2 ¯1.3.5.............(2n – 1)(2n  1) ¿

1 ­ 1 1 ½
= ® – ¾
2 ¯1.3.5......... 2n – 1 1.3.5.......... 2n 1 ¿

1 §1 1 ·
? T1 = ¨ – ¸
2 © 1 1 .3 ¹

1 § 1 1 ·
T2 = ¨ – ¸
2 © 1 .3 1 . 3 .5 ¹
.
.
.
1§ 1 1 ·
Tn ¨¨ – ¸
2 © 1.3.5.....( 2n – 1) 1.3.5.....(2n  1) ¸¹

1§ 1 ·
Sn ¨¨1 – ¸
2 © 1.3.5.....(2n – 1)(2n  1) ¸¹
1 1
? Sf = (1–0) =
2 2
Ans : (b)
PRACTICE QUESTIONS
1* Let S1, S2,................be squares such that for each n t 1 the length of a side of Sn equals the
length of a diagonal of Sn+1. If the length of a side of S1 is 10 cm, then for which of the following
values of n is the area of Sn less than 1 sq. cm.
(a) 7 (b) 8 (c) 9 (d) 10
2 If a, b, c are is A.P and a , b , c are in H.P, then b =
2 2 2 2

ca – ca
(a) (b) 2ca (c) (d) –2ca.
2 2
3 Let the HM & GM of two positive numbers a & b be in the ratio 4:5 then a : b is
(a) 1:2 (b) 2 : 3 (c) 3:4 (d) 1 : 4
y
4 If cos(x–y), cos x, cos(x+y) are in H.P., then the value of cos x sec is
2
1
(a) r1 (b) r (c) r 2 (d) r 3
2
5 If x & y are positive real numbers and m, n are positive integers, then the minimum value of
x m yn
is
(1  x 2 m )(1  y 2 n )

206
1 1
(a) 2 (b) (c) (d) 1
4 2
6* There are two numbers a & b whose product is 192 and the quotient of A.M. by H.M. of their
169
greatest common divisor and least common multiple is . The smaller of a & b is
48
(a) 2 (b) 4 (c) 6 (d) 12
7 Consider the sequence 1, 2, 2, 3, 3, 3, ...............where n occurs n times. The number that occurs
as 2007th term is
(a) 61 (b) 62 (c) 63 (d) 64
8 Read the following paragraph and answer the questions.
Let A1, G1, H1 denote the A.M., G.M., H.M of two distinct positive numbers.
For n t 2, let An–1 and Hn–1 have A.M., G.M., H.M as An, Gn, Hn respectively.
(i) Which of the following statements is correct ?.
(a) G1>G2>G3 >....................
(b) G1<G2<G3< ....................
(c) G1=G2=G3 =....................
(d) G1<G3<G5.................and G2>G4>G6>......
(ii) Which are of the following statement is correct ?.
(a) A1>A2>A3>...................
(b) A1<A2<A3<...................
(c) A1>A3>A5>................... and A2<A4<A6<.........................
(d) A1<A3<A5<................... and A2>A4>A6>...................
(iii) Which are of the following statement is correct ?.
(a) H1>H2>H3>....................
(b) H1<H2<H3<.................
(c) H1>H3>H5>................... and H2<H4<H6<..................
(d) H1<H3<H5<................... and H2>H4>H6>.................
xyz
9 If x,y, z>0 and x+y+z = 1, the is necessarily
(1 – x )(1 – y)(1 – z)
1 1
(a) t8 (b) d (c)  (d) None of these
8 8
10* Match the following :
Column I Column II
(a) 3 numbers a,b, c between 2 & 18 such that (p) G–L = 4
ªG º
(i) a+b+c = 25 (q) «¬ L »¼ = 4
(ii) 2, a, b are consecutive terms of an A.P. (r) G–L = 7
ªG º
(iii) b, c, 18 are consecutive terms of a G.P. (s) «L» = 3
« »

207
ªG º ªG º
If G = max {a, b, c} & L = min {a, b, c}then (t) «¬ L »¼ + «« L »» = 3
(b) 3 numbers a, b, c are in G..P. Such that
(i) a+b+c = 70
(ii) 4a, 5b, 4c are is A.P.
If G = Max{a, b, c} and L = Min{a, b, c}, then
11 The coefficient of x203 is the expansion of (x–1)(x2–2)(x3–3) ....................(x20–20) is
(a) –35 (b) 21 (c) 13 (d) 25
12 Read the following passage and answer the questions : -
–D
Let ABCD be a unit square and 0< D <1. Each side of the square D D C

is divided in the ratio D : 1– D , as shown in the figure. These D


–D
points are connected to obtain another square. The sides of new
square are divided in the ratio D : 1– D and points are joined to
obtain another square. The process is continued indefinitely.
Let an denote the length of side and An the area of the nth square –D D

f A D B
8 –D
(i) The value of D for which ¦ An = is
n 1 3
(a) 1/3, 2/3 (b) 1/4, 3/4 (c) 1/5, 4/5 (d) 1/2
(2) The value of D for which side of nth square equals the diagonals of (n+1)th square is
(a) 1/3 (b) 1/4 (c) 1/2 (d) 1/ 2
f
(iii) If a = 1/4 and Pn denotes the perimeter of the nth square then ¦ Pn equals
n 1

(a) 8/3 (b) 32/3 (c) 16/3 (d)


8
3

4  10
Answers
1. b, c, d 2. c 3. d 4. c 5. b 6. b,d
7. c 8. (i) c (ii) a (iii) b 9. b
10. a o r, s, b o q 11. c 12. (i) b (ii) c (iii) d

208
SEQUENCES AND SERIES - VIII
Special Series - Problem Solving
Arithmetico - Geometric Series (A.G.S.)
If a1, a2, ............ an is an A.P. and b1, b2, ...........bn is a G.P, then the sequence a1b1, a1b2, .......anbn
is said to be an A.G.S. The sequence is of the form ab, (a+d) br, (a+2d) br2,.......... ..............
ab dbr(1 – r n –1 ) (a  (n – 1)d)br n
Sum to n terms = Sn = + –
1– r (1 – r ) 2 1– r
If –1 < r < 1, sum to infinite numbers is given by
ab dbr
Sf = +
1– r (1 – r ) 2
Important results
1 Let Sr = 1r + 2r + 3r +.......................+nr, then
n ( n  1)
(i) S1 = 1 + 2 + 3 +..................+n =
2
n ( n  1)( 2n  1)
(ii) S2 = 12 + 22 + 33 +..................+n2 =
6
n 2 (n  1) 2
(iii) S3 = 13 + 23 + 33 +..................+n3 = = S12
4
n (n  1)(2n  1)(3n 2  3n – 1) S2
(iv) S4 = 1 + 2 + 3 +..................+n =
4 4 4 4
= (6S1–1)
30 5
n 2 (n  1) 2 (2n 2  2n – 1) 1
(iv) S5 = 1 + 2 + 3 +..................+n =
5 5 5 5
= S12(4S1–1)
12 3
2 1 + 3 + 5 + ....................to n terms = n 2

n (4n 2 – 1)
3 12 + 32 + 52 + .................to n terms =
3
4 1 + 3 + 5 + .................to n terms = n (2n2 –1)
3 3 3 2

1 – (–1) n
5 1 – 1 + 1 –..................to n terms =
2
1 – (–1) n (2n  1)
6 1 – 2 + 3 –.....................to n terms =
4
(–1) n –1 n (n  1)
7 1 – 2 + 3 –...................to n terms =
2 2 2
= (–1)n–1S1
2
(–1) n –1 (4n 3  6n 2 – 1) – 1
8 1 – 2 + 3 –.................to n terms =
3 3 3
8
Note 1 : (x+1)(x+2)(x+3)..................(x+n) = xn + A1 xn–1 + A2xn–2 + A3xn–3+...................

209
n ( n  1)
Then A1 =
2
( n – 1) n ( n  1)(3n  2)
A2 =
24
(n – 1)(n – 2)n 2 (n  1) 2
A3 =
48
Note 2 : To obtain the sum ¦
i j
ai aj we use the identity

2¦ ai aj = (a1+a2+.........+an) –(a1 +a2 +...+an )


2 2 2 2

i j

More methods of summation of series


If nth term of a sequence is given by
Tn = an3 + bn2 + cn + d, where a, b, c, d  R, then
Sn = ¦ Tn = T1 + T2 +..............+Tn

= a ¦ n 3 + b ¦ n 2 + c ¦ n + d ¦1
I Method of differences
If the differences of successive terms of a series are in A.P. or G.P., we can find Tn as follows
(a) Denote nth term and the sum up to n terms by Tn &Sn respectively
(b) Rewrite the given series with each term shifted by one place to the right
(c) Subtracting the above two forms of the series, find Tn.
(d) Apply Sn = ¦T n .
Note : Instead of determining the nth item of a series by the method of difference, we can use the
following steps to obtain the same
(i) If the differences T2 –T1, T3 – T2,................etc are in A.P. Then take the nth term as
Tn = an2 + bn + c, a, b, c  R
Determine a, b, c by putting n = 1, 2, 3 and equating them with the values of corresponding
terms of the given series.
(ii) If the differences T2 –T1, T3 –T2, ...........etc are in G.P , with common ratio r, then take
Tn= arn–1 + bn + c, a, b, c  R
Determine a, b, c by putting n = 1, 2, 3 and equating them with the values of corresponding
terms of the given series.
(iIi) If the differences of the differences computed in step (i) are in A.P, then take Tn = an3
+ bn2 + cn +d
Determine a, b, c by putting n = 1, 2, 3, 4 and equating them with the values of corresponding
terms of the given series.
(iv) If the differences of differences computed in step (i) are in G.P with common ratio r, then
take
Tn = arn–1 + bn2 + cn + d
Determine a, b, c by putting n = 1, 2, 3, 4 and equating them with the values of corresponding
terms of the given series.

210
II Sum of series whose nth term is
1
Tn =
[a  (n – 1)d][(a  nd )]
Resolve Tn into partial fractions, (or express the Nr of Tn in terms of factors of Dr and
simplify), then find T1, T2, ............ Tn and add to get Sn.
III Sum of series in special form
(a) Let the series consists of terms whose nth term
1
Tn =
a (a  d)(a  2d)..............(a  (n – 1)d )
To find sum of such a series ( factors of Dr are in A.P.) as shown above, remove the least
factor and multiply the denominator by the number of factors left out (here n–1), and also
by the common difference (here d) change the sign and add a constant C.
1
Thus, Sn = +C
(n – 1)d(a  d)(a  2d )..................(a  (n – 1)d )
Find S1 and hence value of C. This gives the required sum.
(b) Let the series consists of terms whose nth term Tn = a(a+d) (a+2d)..........(a+(n–1)d).
To find sum of such a series, as shown above, add one more factor and divide by the total
number of factors (here (n+1)) and also by the common difference (here d). Also add a
constant C.
a (a  d )(a  2d )........(a  (n – 1)d )(a  nd )
Thus Sn = +C
(n  1)d
Find S1 and hence value of C. This gives the required sum.
Note :
2 2
§ n 1· § n –1·
(i) for odd n, n= ¨ ¸ –¨ ¸
© 2 ¹ © 2 ¹

§ n n  1 ·
2 3
§ n (n – 1) ·
(ii) For any n, n =¨
3 ¸ –¨ ¸
© 2 ¹ © 2 ¹
3 2
§ n3 1· § n3 –1· § n (n  1) ·
2
§ n (n – 1) ·
2

(iii) For odd n, n = ¨¨


3 ¸¸ – ¨¨ ¸¸ = ¨ ¸ –¨ ¸
© 2 ¹ © 2 ¹ © 2 ¹ © 2 ¹
SOLVED EXAMPLES
1 1 1 S4 1 1 1
1 It is given that 4 + 4 + 4 + ............. f = then, 4 + 4 + 4 + ............. f is equal to
1 2 3 90 1 3 5
S4 S4 89 S 4
(a) (b) (c) (d) None of these
96 45 90
1 1 1
Solution : Let 4 + 4 + + ............. f = S
1 3 54

211
1 1 1 S4
Now + + + ............. f =
14 24 34 90
§1 1 1 · § 1 1 1 · S4
¨ 4  4  4  ......... ¸ + ¨ 4  4  4  .........¸ =
©1 3 5 ¹ ©2 4 6 ¹ 90
1 §1 1 1 · S4
¨  
Ÿ S+ 4 14 2 4 34  .........f ¸=
2 © ¹ 90
1 S4 S4
Ÿ S+ . =
16 90 90
S 4 §1 – 1 · S 4 15 S4
Ÿ S= ¨ ¸ = × =
90 © 16 ¹ 90 16 96
Ans : (a)
n
1  2  2 2  ..........  r terms
2 If Sn = ¦ , then
r 1 2r
Sn is equal to
(a) 2n –(n+1) (b) 1 –2–n (c) n–1 + 2–n (d) 2n –1
Solution :
1  2  2 2  ...................rterms.
Tr =
2n
1(2 r – 1)
= = 1 – 2–r
(2 – 1)2 r
1
Tr = 1 –
2r
­1 1 1 ½
Sn = n – ®  2  3  ................nterms¾
¯2 2 2 ¿

1§ 1 ·
Sn = n – 2 ¨©1 – 2n ¸¹ = n – (1– 2–n)
1
1–
2
= n –1 + 2 –n
Ans : (c)
3 The sum to n terms of the series
n (n  1) 2
12 + 2.22 + 32 + 2.42 + 52 +.................... = when n is even. When n is odd, the sum is
2
n 2 (n  1) n (n 2 – 1)
(a) (b) (c) n(n+1)2(2n+1) (d) None of these
2 2
212
Solution : Let n = 2k
? 12 + 2.22 + 32 + 2.42 + 52 +................+(2k–1)2 + 2(2k)2
2k (2k  1) 2
=
2
Let n = 2k + 1 (odd)
? 12 + 2.22 + 32 + 2.42 +................+(2k–1)2 + 2(2k)2 + (2k+1)2
2k (2k  1) 2
= + (2k + 1)2
2
(2k  1) 2 (2k  2) n 2 (n  1)
= =
2 2
Ans : (a)
4 If the sum to n terms of an A.P is cn(n–1); c z 0, then the sum of squares of these terms is
2c 2
(a) c2 n2 (n+1)2 (b) n(n–1)(2n–1)
3
2c 2
(c) n(n+1)(2n+1) (d) None of these
3
Solution : Tn = Sn –Sn–1 = cn(n–1) –c(n–1)(n–2)
= c(n–1){n–n+2}
= 2c(n–1)
? Tn2 = 4c2(n–1)2
? Sn = 4c2{02+12+22+..................+(n–1)2}
2

n ( n – 1)( 2n – 1)
= 4c2
6
2c 2
= n(n–1)(2n–1)
3
Ans (b)
5 Let tr = 2r/2 + 2–r/2
10

The ¦ t r is equal to
2

r 1

2 21 – 1 2 21 – 1 2 21 – 1
(a)  20 (b)  19 (c) – 19 (d) None of these
210 210 2 20
Solution : tr2 = 2r + 2–r + 2
§1 1 1 ·
? S102 = (21+22+.........+210) + ¨ 2  2 2  ........  210 ¸ + 20
© ¹
1§ 1 ·
2(210 – 1) ¨1 – 10 ¸
= + 2 © 2 ¹ + 20
2 –1
1
1–
2

213
1
= 211 –2 +1 – + 20
210
1 2 21 – 1
= 2 – 10 + 19 =
11
+ 19
2 210
Ans : (b)
6 Sum to n terms
1.(3n–1)+2. (3n–2) + 3.(3n–3) + .............n terms is
n ( 2n  1)(5n – 1) n ( 2n  1)(5n  1)
(a) (b)
3 3
n ( n  1)(7 n – 1)
(c) (d) None of these.
6
Solution :
Tr = r(3n–r)
= 3nr–r2
n n n
Sn = ¦ Tr = 3n ¦ r – ¦r2
r 1 r 1 r 1

n.(n  1) n ( n  1)(2n  1)
= 3n. –
2 6
n ( n  1) ­ 2n  1 ½
= ®3n – ¾
2 ¯ 3 ¿

n ( n  1) ­ 9n – 2n – 1½ n ( n  1)(7 n – 1)
= ® ¾=
2 ¯ 3 ¿ 6
Ans : (c)
n n m
7 ¦r2 – ¦ ¦ r is equal to
r 1 m 1 r 1

1 §¨ n 2 n ·¸ 1 §¨ n 2 n ·¸
(a) ¦r  ¦r¸
2 ¨© r 1 (b) ¦r – ¦r¸
2 ¨© r 1
r 1 ¹ r 1 ¹
(c) 0 (d) None of these
n n
m(m  1)
Solution : ¦ r – ¦
2

r 1 m 1 2
n n
r2  r
= ¦r – ¦
2

r 1 r 1 2

214
n § 2 r2 · n
r
= ¦ ¨¨ r – ¸¸ –

¦
r 1 © r 1 2
n n
1 1§ n 2 n ·
= ¦ r 2 – 12 ¦ = 2 ¨ ¦ r –¦ r ¸
r
2 r 1 r 1 ©r 1 r 1 ¹

PRACTICE QUESTIONS
1*. For a positive integer n let
1 1 1 1
a(n) = 1 + + + +................+ n then
2 3 4 (2 ) – 1
(a) a(100) d 100 (b) a(100)>100 (c) a(200) d 100 (d) a(200)>100
2 11 –10 +9 –8 +7 –6 +5 –4 +3 –2 +1 =
3 3 3 3 3 3 3 3 3 3 3

(a) 756 (b) 724 (c) 648 (d) 812


n
d
3 If a1, a2, ..........an+1 are in A.P with common difference d, then ¦ tan–1 1  a a
r 1 r r 1

nd (n  1)d
(a) tan–1 1  a a (b) tan–1 1  a a
1 n 1 1 n 1

(n – 1)d ( n – 1)d
(c) tan–1 1 – a a (d) tan–1 1 – a a
1 n 1 1 n 1
4 The sum to 50 terms of
3 5 7
2 + 2 2 + 2 + ............. is
1 1 2 1  2 2  32
50 100 150 200
(a) (b) (c) (d)
17 17 17 17
5 The sum of the first 10 common terms of the series 17, 21, 25, .................... and 16, 21, 25, ... is
(a) 1100 (b) 1010 (c) 1110 (d) 1200
6 Match the following :
Column I Column II
(a) 1 –2 +3 –................. to 21 terms
2 2 2
(p) 680
(b) 1 –2 +3 –4 + ........... to 15 terms
3 3 3 3
(q) 2556
(c) 12+32+52 + ................ to 8 terms (r) 1856
(d) 13+33+53 + ............... to 6 terms (s) 231
f
7 The sum of the series ¦ cos ec –1 4r 4  1 is
r 1
(a) S (b) S /2 (c) S /4 (d) None of these
n n
8 Let ¦r 4
= ƒ(n), then ¦ (2r–1)4 is equal to
r 1 r 1

215
§ n –1·
(a) ƒ(2n) – 16ƒ(n),  n N (b) ƒ(n)–16ƒ ¨ ¸ when n is odd
© 2 ¹
§n·
(c) ƒ(n)–16ƒ ¨ ¸ when n is odd (d) None of these
©2¹
9 Match the following : -
Column I Column II
(a) If ¦ n = 210, then ¦ n 2 is divisible by
the greatest prime number which is greater than (p) 16
(b) Between 4 & 2916 is inserted odd number (2n+1)
G.M’S. Then the (n+1)th G.M. is divisible by greatest (q) 10
odd integer which is less than
(c) In a certain progression, three consecutive terms
are 40, 30, 24, 20. Then the integral part of the (r) 34
next term of the progression is more then
4 7 10 a
(d) 1+  2  3 +............. to f = , where (s) 30
5 5 5 b
HCF(a,b) = 1, then a–b is less then
1 1 1 1 ª1º
10 If S= 2 + 2 + 2 + 2 +............... f then the value of « » is ..............
3 1 4 2 5 3 6 4 ¬S¼
§ 1 1 1 · § 1 1 1 ·
11 The value of the ratio ¨1  2  2  2  ............ ¸ ¨1 – 2  2 – 2  ................. ¸ is .......
© 2 3 4 ¹ © 2 3 4 ¹
n
lim r
12 nof ¦ 1.3.5.7.9...................(2r  1)
is equal to
r 1

1 3 1
(a) (b) (c) (d) None of these
3 2 2
n 2
13 If (12–t1) + (22–t2) + ............. + (n2–tn) = (n –1), then tn is equal to
3
(a) n2 (b) 2n (c) n2–2n (d) None of these
14 If (1+3+5+.........+p) + (1+3+5+.............+q) = (1+3+5+.................+ r) where each set of
parantheses contains the sum of consecutive odd integers as shown, the smallest possible value of
p+q+r (where p>6) is..........................
(a) 12 (b) 21 (c) 45 (d) 54
Answers
1. a,d 2. a 3. a 4. b 5.c 6. a o s, b o r, c o p,d o q
7. c 8. a 9. a o p,q,r,s; b o r,s; c o p,q; d o r,s ,s 10. 2 11.2
12.c 13.d 14. b

216
PERMUTATION AND COMBINATIONS -I
Properties of nPr and nCr
Fundametal principle of counting
(i) Addition principle : If an operation can be performed in ‘m’ different ways and another operation
which is independent of the first operation, can be performed in ‘n’ different ways then either of
them can be performed in (m+n) ways.
(ii) Multiplication Principle. If an operation can be performed is ‘m’ different ways; following with
a second operation can be performed in ‘n’ different ways, then the two operations in succession
can be performed in ‘mn’ ways.
Factorial
The continued product of first n natural numbers is n!.
n! = 1 . 2 . 3 . ............. n
value of 0! is 1
Exponent of prime p is n!
Let n be a positive integer and p, a prime number. Then the exponent of p is n! is given by

where s in the largest positive integer such that

ps < n < ps+1


Permutation
Number of permutations of n distinct things taking r(1 d r d n) at a time is denoted by nPr.
n n!
Pr =
(n – r )!
x Number of ways of filling r places using n things if repetition is allowed = nr
Circular Permutation
Number of circular permutations of n things = (n–1)!
n
Pr
Number of circular permutations of n different things taken r at a time =
r
Number of circular permutations of n different things when clockwise and anticlockwise circular
( n – 1)!
permutations are considered as same is
2
Note : When position are marked, circular arrangement is assumed to be linear.
Combination
Number of combinations of n distinct things taking r at a time is denoted by nCr.
n!
n
Cr =
r!(n – r )!
Note: nPr = r! (nCr)

217
Properties of nPr and nCr
i) Pn = n! ; nCo = nCn = 1
n

ii) Pr =
n n–1
Pr–1+ r n–1
Pr–1
n

iii) ¦r
r 1
. rPr = n+1Pn+1 –1

iv) Cr = nCs Ÿ either r=s or r+s = n


n

n
r ; if n is even
2
v) Cr is greatest ,
n
n r1
r ; if n is odd.
2

n
vi) Cr = nCn–r =
n n–1
Cr–1
r
n
Cr n  r 1
vii) n
C r –1 r

viii) Cr–1 + nCr =


n n+1
Cr

Solved Examples

1. How many 5-digit numbers divisible by 3 can be formed using digits 0, 2, 4, 6, 8, 9, if repetition not
allowed?
Soluton:
Sum of digits =0+2+4+6+8+9=29
so 5-digit numbrs can be formed using the digits 0, 4, 6, 8, 9 or 0, 2, 4, 5, 9 as sum of digits is
divisible by 3.
? total number of numbers formed is
2(4.4.!)=192
Ans. 192
2. How many 5-digit numbers divisible by 4 can be formed using digits 0, 2, 4, 7, 8, 9, if repetition not
allowed?
Soluton:
Number is divisible by 4, if the last two digits of the number are a multiple of 4, so we can have 04,
08, 20, 24, 28, 40, 48, 72, 80, 84, 92 at the last two places. Hence total number of numbers can
be 5.(4P3)+6.3.3P2 as the first place cannot have zero.
? the answer is 228.
Ans. 228

218
3. How many 5-digit numbers divisible by 6 can be formed using digits 0, 2, 4, 5, 6, 8, if repetition not
allowed.
Soluton:
Sum of digits is
0+2+4+5+6+8=25
? the numbers can be formed with the digits 0, 2, 5, 6, 8 as their sum is a multiple of 3 provided
unit’s place has an even number. So it can be done in 4!+3.3.3!=78 ways.
4. The total number of odd natural numbers that can be formed with the digits 1, 3, 1, 5,4, 1, 4 and
are greater than 2 million are
(a) 120 (b) 160 (c) 180 (d) none
Soluton:
Odd digits Even digits
1, 3, 5 4
1 4
1
Number of arrangements can be

5!
5
3!2!
3
5!
1
2!2!
5!
1
2!
4
5!
3
3!
5!
5
3!
5!
1
2!2!
5
5!
3
3!2!
Toal no. = 10+30+60+20+20+30+10=180
Ans. c
5. The total number of 4 digit numbers greater than 4000, whose sum of digits is odd is
a. 2800 b. 3000 c. 3600 d. none of these
Soluton:
Thousands place can be filled in 6 ways. Hundred’s and Ten’s place can be filled in 10 ways each.
First 3 places given the sum either odd or even. In either case last place can be filled in 5 ways.
? Number of 4 digit numbers is
= 6×10×10×5=3000

219
Ans. b
6. Mohan writes a letter to five of his friends and addresses them. The number of ways in which the
letters can be placed in the envelopes so that that three of them are in the wrong envelopes is
a. 44 b. 119 c. 21 d. 20
Soluton:
§ 1 1 1·
C2.3! ¨1 –  – ¸ =20 (dearrangement of n pairs)
5
© 1! 2! 3! ¹
Ans. d

7. In how many ways the squares of the figure given below be filled up with letters of the word
‘ROHINI’, so that each row contains atleast one letter.

Soluton:

C
8
6
6!
2!

–2 u
=26×360=9360 ways
as six squares can be slected in such a way that no row is empty is (8C6–2)=26 ways.
Ans. 9360
PRACTICE QUESTIONS
1. Number of polynomials of the form x3+ax2+bx+c that are divisible by x2+1, where a, b, c H {1,
2, 3, .....9, 10}
a. 30 b. 1000 c. 10 d. none of these
2. The number of ways in which we can select four numbers from 1 to 30 so as to exclude every
selection of four consecutive number is
a. 27378 b. 27405 c. 27399 d. none of these
3. The number of ordered pairs of integers (x, y) satisfying the equation x 2+6x+y2 = 4 is
a. 2 b. 8 c. 6 d. none of these
4. How many six digit numbers are there in which sum of the digits is divisible by 5
a. 180000 b. 540000 c. 5x105 d. none of these
5. Ten IIT and 2 DCE students sit in a row. The number of ways in which exactly 3 IIT students sit
between 2 DCE students is
a. 10
C3 u 2! u 3! u 8! b. 10! 2! 3! u 8! c. 5! u 2! u 9! u 8! d. none of these
6. Let there are n>3 circles. The value of n for which the number of radical centres is equal to the
number of radical axis is (assume that all radical axis and radical centres exist and are different)
a. 7 b. 6 c. 5 d. none of these
7. Total number of integers ‘n’ such that 2 < n < 2000 and HCF of n and 36 is one, is equal to
a. 666 b. 667 c. 665 d. none of these

220
8. Match the following :-
Column I Column II
a) The number of five digit numbers having p) 77
the product of digits 20 is
b) A man took 5 space plays out of an engine q) 31
to clean them. The number ofways in which
he can place atleast two plays in the engine
from where they came out is
c) The number of integers between 1 and 1000 r) 50
inclusive in which atleast two consecutive
digits are equal is

1
d) Value of 15 ¦¦ i j s) 181
1di d jd9

9. The number of ordered triplets (a, b, c) such that LCM (a, b) = 1000, LCM (b, c) 2000 and
LCM (c, a) = 2000 is _______
10. Read the paragraph and answer the questions that follow :-
Number of ways of distributing n different things into r different groups is rn when blank groups
are taken into account and is rn –rC1 (r–1)n + rC2(r–2)2 ........ + (–1)r–1 rCr–1 when blank groups
are permitted.
i) 4 candidates are competing for two managerial posts. In how many ways can the candidates be
selected?
a. 42 b. 4C2 c. 24 d. none of these
ii) 8 different balls can be distributed among 3 children so that every child receives at least one
ball is
a. 38 b. 8C3 c. 83 d. none of these
iii) 5 letters can be posted into 3 letter boxes in
a. 35 ways b. 53 ways c. 5C3 ways d. none of these

Answers
1. c 2. 3. b 4. a 5. a
6. c 7. a 8. a-r, b-q, c-s, d-p 9. 70
10. (i)-b, (ii)-d, (iii)-a

221
PERMUTATION AND COMBINATIONS -II
Simple Applications on nPr and nCr

Important Results
1. Sum of digits in the unit place of all numbers formed by a1, a2.........an taken all at a time is given by
(n–1)! (a1+a2+......an) if repetition of digits is not allowed.
2. Sum of all the numbers which can be formed using the digits a1, a2.........an (repetition not allowed)

=
n

(n–1)! (a1+ a2+........+an) 10 – 1
9
=
(n–1)! (sum of digits) 11n.......
times

1
3. Number of integral solutions of linear equations and unequations (Multinomial Theorem)
i. Total number of non negative integral solutions of x 1+x2+......xr = n is n + r–1Cr–1
Total number of positive integral solutions of x 1+x2+......xr = n is n–1Cr–1
ii. In order to solve inequations of the form x1+x2+......+xr d n, we introduce artificial (dummy)
variable xr+1 such that x1+x2+.......+xr+xr+1 = n where xr+1 t 0.
Number of solutions of this equation are same as the number of solutions of inequation
x1+x2+.....xr d n.
iii. Number of solutions of D  2E  3J ..... qT n is


­°coefficient of x n in 1 – x –1 1 – x 2 –1 1 – x 3 –1....... 1 – x q –1 if zeroisincluded
®
–1

°̄coefficient of x n inx1 2...... q 1 – x –1 1 – x 2 ....... 1 – x q if zeroisnotincluded
–1

4. Number and sum of divisors


Let N = ap bq cr where a, b, c are primes & p, q, r Z .
i. Number of divisors of N = (p+1) (q+1) (r+1)
Sum of divisors of N = (1+a+a 2+....ap) (1+b+b2+....bq) (1+c+c2+....cr)
ii. Number of ways in which N can be resolved as a product of two factors is

­1 ½
°° 2 a  1 b  1 c  1 if Nisnotperfectsquare °°
® ¾
°­® 1 a  1 b  1 c  1  1½¾if Nisaperfectsquare°
°¯¯ 2 ¿ °¿
5. Number of ways in which a composite number N can be resolved into two factors which are
relatively prime (or co prime) to each other is 2 n–1 where n is the number of different prime
factors is N.
6. Divisibility
Condition for divisibility of a number
A number abcde will be divisible
1. by 4 if 2d+e is divisible by 4

222
2. by 8 if 4c+2d+e is divisible by 8
3. by 3 if a+b+c+d+e is divisible by 3
4. by 9 if a+b+c+d+e is divisible by 9
5. by 5 if e = 0 or 5
a
 c 
e – bN d
6. by 11 if Sum of digits sumof digit
is divisible by 11
1
 
atoddplaces atevenplaces

7. by 6 if e = even and a+b+c+d+e is divisible by 3


8. by 18 if e = even and a+b+c+d+e is divisible by 9

Solved Examples
1. Ten different letters are printed round a circle. The number of different ways in which we can select
three letters so that no two of them are consecutive is
a. 26 b. 50 c. 56 d. 72
Soluton:
Number of selections is
10
C3–10 – 10.6C2= 50 ways as total ways of selection is 10C3 (number of selections is
n
C3–n – n.n–4C2)
Number of ways when three are consecutive is 10
Number of ways when two are consecutive is 10.6C2. Subtract these two cases from the total
number of ways.
Ans. b

2. The numberof triangles whose vertices are the vertices of an octagon but none of whose sides
happen to come from the sides of octagon is
a. 24 b. 52 c. 48 d. 16
Soluton:
Proceding in a similar way as in solved example 1, we have number of selection as
8
C2 – 8 –8.4C2 =16 (number of selections is nC3–n – n.n–4C2)
Ans. d
3. The maximum number of points in which 4 circles and4 straight lines intersect is
a. 26 b. 50 c. 56 d. 72
Soluton:
Maximum number of points of line - line intersection = 4C2=6
Maximum number of points circle - circle intersction is 4P2=12
Maximum number of points of line circle intersection is (2×4)×4=32
? total number of points of intersection is 6+12+32=50.
Ans. b
4. In a plane two families of lines are given by y = x+r and (y = –x+p) where r {0, 1, 2, 3, 4}and
p  {0, 1, 2, ......, 9}. The number of squares of diagonals of length 3 units formed by these
lines is
a. 36 b. 24 c. 20 d. none of these
Soluton:
It means we have to select two lines from each family in such a way that there is a gap of 2
lines between the selected lines. First pair can be selected in two ways and second pair can
be selected in seven ways. Hence, number of squares selected is 7×2 = 14

223
Ans: d
5. The total number of words that can be made using letters of the word CALCULATE so that
each word starts and ends with a consonant is
5.7! 3.7!
a. b. c. 2.7! d. none of these
2 2
Solution:
Consonants Vowels
CLT AUE
CL A
Arrangements can be as follows.
7!
C C
2!2!
7!
C L
2!
7!
L C
2!
7!
C T
2!2!
7!
T C
2!2!
7!
C T
2!2!
7!
L T
2!2!
7!
T T
2!2!
7! § 1 1 1 1 1 1·
? Total number of arrangements can be ¨ 11     ¸
2! © 2 2 2 2 2 2¹
5.7!
=
2
6. The number of 5 digit numbers of different digints in which middle digits is the largest is
9

a. ¦ n
P4 b. 33(3!) c. 30(3!) d. none of these
n 4

Solution: Fix the middle digit. Number of arrangements is


(4P4 – 3P3) + (5P4 – 4P3) +........+ ( 9P4 – 8P3)
Ans: d

224
PRACTICE QUESTIONS
1. Last digit of (1!+2!+.....+2005!) 500 is
a. 9 b. 2 c. 7 d. 1
2. Number of integral solutions of x+y+z = 0 with x t –5, y t –5, z t –5 is
a. 134 b. 136 c. 138 d. 140
3. Let x1,x2.....,xk be divisors of positive integer n (excluding 1 & n). If x 1+ x2+.....+xk = 75, then
k
1
¦x
i 1
is equal to
i

75 75 75
a. 2 b. c. d. none of these
n n k
4. The total number of ways in which n2 number of identical balls can be put in n numbered boxes
(1,2,3,....,n) such that ith box contains at least i number of balls is
n2 n2 n–2
a. C n –1 b. n 2 –1
C n –1 c. d. none of these
2
C n –1
5. Total number of positive integral solutions of 15 < x1+x2+x3 d 20 is
a. 685 b. 785 c. 1125 d. none of these
6. If n is selected from the set {1,2,3....10} and the number 2 +3 +5 is formed. Total number of
n n n

ways of selecting n so that the formed number is divisible by 4 is equal to


a. 50 b. 49 c. 48 d. none of these
7. A is a set containing n different elements. A subset P of A is chosen. The set A is reconstructed
by replacing the elements of P. A subset Q of A is again chosen. The number of ways of choos-
ing P and Q so that P ˆ Q contains exactly two elements is
a. n
C 3×2 n b. n
C 2×3 n–2 c. 3n–2 d. none of these
8. The number of three digit numbers of the form xyz such that x < y and z d y is
a. 276 b. 285 c. 240 d. 244
9. Number of ordered triplets (x,y,z) such that x,y,z are primes and x y +1 = z is
a. 0 b. 1 c. 3 d. none of these
10. Read the passage and answer the following questions:-
Suppose a lot of n objects having n1 objects one kind, n2 objects of second kind, n3 objects of
third kind,....,nk objects of kth kind satisfying the condition n1+n2+….+nk = n, then the number
of possible arrangements / permutations of m objects out of this lot is the coefficient of xm in the
­ ni xȜ ½
expansion of m! – ®¦ ¾
¯ Ȝ 0 Ȝi ¿
i. The number of permutations of the letters of the word AGAIN taken three at a time is
a. 48 b. 24 c. 36 d. 33
ii. The number of permutations of the letters of the word EXAMINATION taken 4 at a time is
a. 136 b. 2454 c. 2266 d. none of these
iii. The number of permutations of the letters of the word EXERCISES taken 5 at a time is

225
a. 2250 b. 30240 c. 226960 d. none of these
iv. The number of ways in which an arrangement of 4 letters of the word PROPORTION can be
made is
a. 700 b. 750 c. 758 d. none of these
v. The number of permutations of the letters of the word SURITI taken 4 at a time is
a. 360 b. 240 c. 216 d. none of these

ANSWERS

1. d 2. b 3. b 4. c

5. a 6. b 7. b 8. a

9. b 10. (i) d, (ii) b, (iii) a, (iv) c, (v) d

226
PERMUTATIONS & COMBINATIONS - III
Simple Applicationsr on nP and
r
n
C
Important Results
1 Total number of selections of one or more objects from n different objects
= nC1+nC2+....+nCn=2n–1
2 Total number of selections of any number of things from n identical things
­(n  1);whenselectionof zerothingsisallowed
= ®
¯n;whenatleastonethingistobeselected.
3 Total number of selections from p like things, q like things of another type and r distinct things
­°(p  1)(q  1)2 r – 1(if atleastonethingtobeselected)

°̄(p  1)(q  1)2 r – 2(if noneorallcannotbeselected)
4 Total number of selections of r things from n different things when each thing can be repeated unlimited
number of times = n+r–1Cr–1
5 Total number of ways to divide n identical things among r persons = n+r–1Cr–1
6 Results on distribution
Distribution of n things to r boxes
Given Condition Number of ways
n distinct things Empty boxes are allowed rn
r distinct boxes Empty boxes are not allowed coefficient of xn in n!(ex–1)r
n identical things Empty boxes are allowed n+r–1
C r–1
r distinct boxes Empty boxes are not allowed n–1
C r–1
7 Division of items into groups
(i) Groups of unequal size.
x Number of ways in which (m+n+p) items can be divided into unequal groups containing
(m  n  p)!
m, n, p items is
m!n!p!
x Number of ways to distribute (m+n+p) items among 3 persons in the group containing
m,n & p items is (m  n  p)! 3!
m!n!p!
(ii) Groups of equal size
x Number of ways in which (mn) different items can be divided equally into m groups
each containing n objects
­ (mn )!
° (n!) m m!;if orderof gorupsisnotimpor tan t
°
= ®
° (mn )! ; if order of groups is impor tan t
°¯ (n!) m       

Note
(i) If there are m items of one kind, n items of another kind, then the number of ways of

227
choosing r items out of these = coefficient of xr in (1+x+x2+...........+xm)(1+x+x2+.......+xn)
(ii) If there are m items of one kind n items of another kind, then the number of ways of choosing
r items such that at least one item of each kind is included
= coefficient of xr in (x+x2+ ......+xm) (x+x2+....+xn)
8 Results related with points, Lines, Rectangle, Polygon, Circle, etc.
(i) If these are n points in the plane, number of line segments nC2
(ii) Number of points n, then the number of triangles nC3
(iii) Number of diagonals in a regular polygon having n sides = nC2–n
(iv) Number of parallelograms when a parallelogram is cut by two sets of m lines parallel to
its sides = m+2C2 m+2C2

­ n

°
°
Number of
  rec tan gles ¦r 1
r3
(v) ® n
n
° Number of s
  TXDUHV ¦ r
2
°¯ r 1
n

­ m1 n 1 mn
°° Numberof rec tan gles C2 C2 (m  1)(n  1)
4
® n n
(vi) ° Numberof sTXDUHV ¦ (m – r  1)(n – r  1)
°¯ r 1
m
n

(vii) Maximum number of parts in which a plane can be divided by n straight lines = 1+ ¦ r
r 1

(viii) Maximum number of points of intersection of n straight Lines = 1× C2 n

(ix) Maximum number of points of intersection of n circles = 2×nC2


(x) Maximum number of points of intersection of n parabolas = 4×nC2
De-arrangement
Number of arrangement of m things in a row so that none of them occupies its original place is
­ 1 1 1 m 1 ½
m! ®1 –  –  .......  (–1) ¾
¯ 1! 2! 3! m!¿
Exponent of prime p in n!
ªn º ª n º ª n º ªnº
Ep(n!) = « » + « 2 » + « 3 » +..........+ « k » where k is the largest positive integer such that
¬ p ¼ ¬p ¼ ¬p ¼ ¬p ¼
pk d n<pk+1
Solved Examples
1 The number of zeroes at the end of 100! is
(a) 23 (b) 24 (c) 25 (d) None of these
Solution :
ª100 º ª100 º ª100 º
«¬ 5 »¼ + «¬ 52 »¼ + «¬ 53 »¼ +....................

228
= 20+4+0
= 24
Ans (b)
2 The total number of integral solutions of the triplet (x,y,z) for the equation xyz=24 is
(a) 30 (b) 60 (c) 120 (d) None of these
Solution :
3!
24.1.1 o = 3
2!
12.2.1 o 3! = 6
6.4.1 o 3! = 6
8.3.1 o 3! = 6
3!
6.2.2 o = 3
2!
4.3.2 o 3! = 6
? Total = 30
30 positive integral solutions
Total number of integral solutions with negative integers included is 30×4 = 120
Ans (c)
3 The total number of squares in a chess board is
(a) 64 (b) 65 (c) 204 (d) None of these
Solution :
8(8  1)(16  1)
12+22+32+................+82 = = 204
6
Ans (c)
4 20 lines pass through a given plane. The maximum number of parts in which the plane can be
divided is
(a) 210 (b) 211 (c) 212 (d) None of these
Solution :
Use 1+ ¦ n
20.21
= 1+ ¦ 20 = 1+ = 2111
2
Ans (b)
5 The number of quadrilaterals that can be formed using 10 points in a plane out of which 4 are
collinear is
(a) 210 (b) 209 (c) 185 (d) None of these
Solution:
10
C4–4C4–4C3. 6C1 = 185
Ans (c)
p
6 The total number of distinct rational numbers x such that 0<x<1 and x = where
q
p,q  {1,2,3,4,5,6} is

229
(a) 15 (b) 13 (c) 11 (d) None of these
Solution :
Values of p Possible rational numbers
1 1 1 1 1
1 , , , ,
2 3 4 5 6
2 2 2 2
2 , , ,
3 4 5 6
3 3 3
3 , ,
4 5 6
4 4
4 ,
5 6
5
5
6
Out of 15 possible rational numbers, only 11 are distinct.
Ans (c)
7 The sum of 5 digit number in which only odd digits occur without repetition is
(a) 277775 (b) 555550 (c) 1111100 (d) None of these
Solution :
Sum of n digit numbers
(10 n – 1)
= (Sum of digits) (n–1)!
10 – 1
(10 5 – 1)
= (1+3+5+7+9) (5–1)!
10 – 1
= 25 × 11111 × 24
= 6666600
Ans (d)
PRACTICE QUESTIONS
1 An n digit number is a positive number with exactly n digits. Nine hundred distinct n-digit numbers
are to be formed using only the three digits 2, 5 and 7. The smallest value of n for which this is
possible is
(a) 6 (b) 7 (c) 8 (d) 9
2 Match the following :
Consider all possible permutations of the letters of the word ENDEANOEL
Column I Column II
(a) The number of permutations containing the word ENDEA is (p) 5!
(b) The number of permutations in which the letter E occurs in
the first and last position is (q) 2×5!
(c) The number of permutations in which none of the letters
D, L, N occurs in the last five positions is (r) 7×5!
(d) The number of permutations in which the letters A, E, O
occur only in odd positions is (S) 21×5!

230
3 Five balls of different colors are to be placed in 3 boxes of different sizes. Each box can hold all 5
balls. The number of ways we can place the balls so that no box is empty, is
(a) 116 (b) 126 (c) 144 (d) 150
4 A student is allowed to select atmost n books from a collection of (2n+1) books. If number of
ways in which he can select atleast one book is 63, then n =
(a) 3 (b) 4 (c) 6 (d) 5
5 A rectangle with sides 2m–1, 2n–1 is divided into squares of unit length by drawing lines parallel to
sides of a rectangle. The number of rectangles with odd side length is
(a) (m+n+1)2 (b) mn(m+1)(n+1) (c) m2n2 (d) 4m+n–1
6 Out of 5 apples, 10 mangoes and 15 oranges, the number of ways of distributing 15 fruits each to
two persons, is
(a) 56 (b) 64 (c) 66 (d) 72
7* Match the following
Column I column II
(a) The number of positive integral solutions of the
equation x1x2x3x4x5 = 1050 is O , then O is divisible by (p) 3
(b) Let y be the element of the set A = {1,2,3,5,6,10,15,30} (q) 4
and x1, x2, x3 be integers such that x1x2x3 = y.If O be (r) 5
the number of integral solutions of x1x2x3 = y, then O
is divisible by (s) 8
(c) Let a be a factor of 120. If O be the number of positive
integral solutions of x1x2x3=a, then O is divisible by (t) 16
8 The maximum number of points into which 4 circles & 4 straight lines intersect is
(a) 26 (b) 50 (c) 56 (d) 72
9 A is a set containing n elements. A subset P1 is chosen and A is reconstructed by replacing the
elements of P1. The same process is repeated for subsets P2, .......Pm with m>1. The number of ways
of choosing P1, P2, .......,Pm, so that P1 ‰ P2 ‰ ...... ‰ Pm = A is
(a) (2m–1)mn (b) (2n–1)m (c) m+nCm (d) None of these
10 Number of points having position vector aî  bˆj  ck̂ where a,b, c  {1,2,3,4,5,} such that
2a+3b+5c is divisible by 4 is
(a) 70 (b) 140 (c) 210 (d) 280
11 Read the passage and answer the following questions.
A is a set containing n elements. A subset P of A is chosen and the set A is reconstructed by
replacing the elements of P. A subset Q of A is chosen again. Find the number of ways of
choosing P & Q when
(i) Q is subset of P is
(a) 3n (b) 2n (c) n.3n–1 (d) None of these
(ii) P & Q contain just one element is
(a) 2n (b) 3n (c) n.3n–1 (d) None of these
(iii) P = Q is
(a) 2n (b) 3 n (c) n.3n–1 (d) None of these
Note:* Questions with more than one option is correct.

231
Answers
1. b 2. a o p; b o s; c o q; d o q 3. d 4. a 5. c
6. c 7. a o p,r; b o q,s,t; c o q,s,r,t
,t 8. b 9. d
10. a 11. (i) a (ii) c (iii) a

232
PERMUTATIONS AND COMBINATIONS - IV
Simple Applications on nPr and nCr

Important Results
1. Sum of digits in the unit place of all numbers formed by a1, a2.........an taken all at a time is given by
(n–1)! (a1+a2+......an) if repetition of digits is not allowed.
2. Sum of all the numbers which can be formed using the digits a1, a2.........an (repetition not allowed)

=
n
(n–1)! (a1+ a2+........+an) 10 – 1
9
=
(n–1)! (sum of digits) 11n.......
times
1


3. Number of whole number solutions x i t 0i (non-negative) of x1+x2+..... xr = n is n–1
Cr–1
Number of solutions of D  2E  3J ..... qT n is


­°coefficient of x n in 1 – x –1 1 – x 2 –1 1 – x 3 –1....... 1 – x q –1 if zeroisincluded
®
–1

°̄coefficient of x n inx1 2...... q 1 – x –1 1 – x 2 ....... 1 – x q if zeroisnotincluded
–1

4. Number and sum of divisors


Let N = ap bq cr where a, b, c are primes & p, q, r Z .
i. Number of divisors of N = (p+1) (q+1) (r+1)
Sum of divisors of N = (1+a+a 2+....ap) (1+b+b2+....bq) (1+c+c2+....cr)
ii. Number of ways in which N can be resolved as a product of two factors is

­1 ½
°° 2 a  1 b  1 c  1 if Nisnotperfectsquare °°
® ¾
°­® 1 a  1 b  1 c  1  1½¾if Nisaperfectsquare°
°¯¯ 2 ¿ °¿
5. Number of ways in which a composite number N can be resolved into two factors which are
relatively prime (or co prime) to each other is 2 n–1 where n is the number of different prime
factors is N.
Multinomial Theorem:
Coefficient of x r in (1–x)–n = n+r–1Cr. Number of ways of making a selection from m+n+p = N
things where p are alike of one kind, m alike of second kind and n alike of third kind taken r at
a time is given by coefficient of xr is expansion of
(1+x+x2+…….xm) (1+x+x2+…….xn) (1+x+x2+…….xp)

Example: Number of selection of 4 letter words from the letters for the ward PROPROTION
is
^PP, RR, OOO, T, I, N`
Coefficient of x4 is (1+x+x2) (1+x+x2) (1+x+x2+ x3) (1+x) (1+x) (1+x)
Condition for divisibility of a number

233
A number abcde will be divisible
1. by 4 if 2d+e is divisible by 4
2. by 8 if 4c+2d+e is divisible by 8
3. by 3 if a+b+c+d+e is divisible by 3
4. by 9 if a+b+c+d+e is divisible by 9
5. by 5 if e = 0 or 5
a
 c 
e – bN d
6. by 11 if Sum of digits sum digit
of
is divisible by 11
1
 
atoddplaces atevenplaces

7. by 6 if e = even and a+b+c+d+e is divisible by 3


8. by 18 if e = even and a+b+c+d+e is divisible by 9

Solved examples
1. The number of divisors of (6!)3! is
a. 364 b. 9100 c. 2275 d. 75
Solution: (6!)3! = (24.32.51)6
= 2 24.3 12.5 6
no.of divisors = 25x13x7 = 2275
Ans: c
2. The number of ways in which three district number in AP can be selected from
1,2,3--------,24 is
a. 132 b. 572 c. 264 d. 150
12.11
Ans: 12C2+12C2 = .2 132 (First and last number should either be both even or both odd
2 .1
and the middle number is average of the two)
3. If x,y,z are integers and x t 0 , y t 1 , z t 2 and x+y+z = 15, then the number of ordered
triplets (x,y,z) is
a. 91 b. 455 c. 17
C2 d. none of these
Solution: x t 0 , y t 1 , z t 2 Put y-1=Y,z-2=z 1 , x+y+z=12. so, we can say
12 objects ( alike) are to be distriuted among 3 persons. (distribution of alike objects)
Apply n+r–1Cr–1 when n = 12, r = 3
14 x13
? 12+3–1
C3–1 = 14C2 = = 91
2
Ans: a
4. a,b,c,d are odd natural numbers such that a+b+c+d = 20, then number of quadrapulets (a,b,c,d)
is
a. 165 b. 455 c. 310 d. 255
Solution: Let a = 2p+1, b = 2q+1, c = 2r+1, d = 2s+1
Ÿ p+q+r+s = 8
? 8+4–1
C4–1 = 11C3 = 165 (distribution of alike objects)
Ans: a
5. The number of positive integral solutions of x+y+z d 10 is ________________.

234
Solution: Let x+y+z+a = 10 wherex t 1, y t 1, Z t 1, a  z & a t 0 )
Required number = n+r–1Cr–1
= 7+4–1
C4–1 = 10C3
10.9.8
= 120
1 .2 .3

PRACTICE QUESTIONS
1. Number of divisors of the form (4n+2); n t 0 of the integer 240 is
a. 4 b. 8 c. 10 d. 13
2. If r,s,t are prime numbers and p,q are the positive integers such that LCM of p,q is r 2s4t2, then
the number of ordered pairs (p,q) is
a. 252 b. 254 c. 225 d. 224
3. The number of seven digit integers, with sum of the digits equal to 10 and formed by using the
digits 1,2 and 3 only, is
a. 55 b. 66 c. 77 d. 88
4. Let n and k be positive integers such that n t k 1 C2 . The number of solutions (x 1,x 2,....x k);
x1 t 1 t x 2 t 2,.......x k t k all integers satisfying x1+x2+......+xk = n is
k k k k
n– n –1– n –1– n 1–
a. 2
Ck b. 2
Ck c. 2
C k –1 d. 2
C k –1
5. The number of divisors of the form 4n+1, n t 0 of the number 1010111111313 is
a. 750 b. 840 c. 924 d. 1024
ªxº ª x º
6. The number of positive integer solution of the equation « » «¬101»¼ is
¬ 99 ¼
a. 2500 b. 2499 c. 1729 d. 1440
IV
7. Let N be natural number. If its first digit (form the left) is deleted , it gets reduced to . The
57
sum of all the digits of N is
a. 15 b. 18 c. 24 d. 30
1 1 1
8. The number of positive integral pairs (x,y) such that  , x  y is
x y 2007
a. 5 b. 6 c. 7 d. 8
9. The number of ordered triplets of positive integers which satisfy the inequality 15 d x  y  z d 45
is
a. 45
C 2– 14C 2 b. 45
C 3–14C 3 c. 46
C 3– 15C 3 d. none of these

235
10.* Match the following:
Column I Column II
a. Total number of functions f{1,2,3,4,5} p. divisible by 11
o {1,2,3,4,5} that are into and f(i) z i is q. divisible by 7
b. If x, x2 x3 = 2.5.7 2 then the number of r. divisible by 3
solution sets for (x1, x2, x3) where
x i  N, x i ! 1is s. divisible by 4
c. Number of factors of 3780 are divisible
by either 3 or 2 or both is
d. Total number of divisors of n = 2 5.34.510
that are of the form 4Ȝ  2, Ȝ t 1 is
11. Read the passage and answer the following questions
Five balls are to be placed in 3 boxes. Each can hold all the five balls. In how many ways can
we place the balls so that no box remains empty, when
i. Balls and boxes are all different
a. 150 b. 6 c. 50 d. 2
ii. balls are identical but boxes are different
a. 150 b. 6 c. 50 d. 2
iii. balls are different but boxes are identical
a. 150 b. 6 c. 50 d. 2
iv. balls as well as boxes are identical.
a. 150 b. 6 c. 50 d. 2

Note: * Questions with more than one option is correct

ANSWERS

1. a 2. c 3. c 4. c 5. c 6. b 7. a 8. c

9. b 10. a o p,s; b o q,r; c o p,s; d o r 11. (i) a (ii) b (iii) c (iv) d

236
BINOMIAL THEOREM - I
Principle and simple applications
Binomial Theorem for Positive Integral Index
If x and y are real, then for all n  N
(x+y)n = nC0xn + nC1x n–1y+nC2x n–2y2 + ....... + nCrx n–ryr + ....+nCn–1x1yn–1+nCnyn
n
= 6 nCrxn–ryr
r 0

n–1
(x–y)n = nC0xn – nC1xn–1y1 + nC2xn–2y2......nCr(–1)rx n–ryr+.... n Cn –1(–1) x 1yn–1+nCn(–1)nyn

(x+y)n + (x–y)n=2{nC0xn + nC2x n–2y2 + nC4x n–4y4+.......]


(x+y)n – (x–y)n=2{nC1xn–1 a1+nC3xn–3a3+nC5x n–5y5+........]
(1+x)n = nC0+nC1x+nC2x2 +........nCrxr+......nCnxn
(1–x)n=nC0– nC1x+nC2x 2 –........nCr(–1)rxr+......+nCn(–1)nxn
Properties of Binomial Expansion
(i) The number of terms in the expansion of (x+y)n where n  N is (n+1).
(ii) The sum of exponents of x & y in (x+y)n is equal to n, the index of the expansion.
(iii) Since nCr=nCn–r; r=0, 1, 2, ......n, the binomial coefficients equidistant from the begin
ning and the end are equal.
i.e. nC0=nCn, nC1 = nCn–1 and so on.
(iv) The general term is the expansion of (x+y)n is given by
Tr+1 = nCr x n–r yr
(v) Coefficient of (r+1)th term in the expansion of (1+x)n is nCr=coefficient of xr.
(vi) If n is odd, then {(x+y) n+(x–y)n} and {(x+y)n–(x–y)n} have same number of terms equal to
§ n  1·
¨ ¸.
© 2 ¹
If n is even, then

§n ·
{(x+y)n+(x–y)n} has ¨  1¸ terms and
©2 ¹

§n·
{(x+y)n–(x–y)n} has ¨ ¸ terms.
©2¹
(vii) Middle term

§n ·
If n is even then in the expansion of (x+y)n, ¨  1¸ th terms is the middle term.
©2 ¹
237
§ n  1· § n  3·
If n is odd natural number, then ¨ ¸ th and ¨ ¸ th are the middle terms in the
© 2 ¹ © 2 ¹
expansion of (x+y)n.
(viii) Let S = (x+y)n = nC0xn+nC1x n–1y+.......+nCn–1xyn–1+nCnyn where n  N
n
= 6 nCrxn–ryr
r 0

Replacing r by n–r we get,


n
S = 6 nCn–r x ryn–r
r 0

= n
C nyn+nCn–1yn–1x+......+ nC n–1yx n–1+nC nx n.
i.e. By replacing r by n–r, we are writing the binomial expansion in the reverse order
Properties of Binomial Coefficient
(i) Sum of two binomial coefficients, nCr+nCr–1=n+1Cr

n
(ii) n
Cr = n–1
Cr–1
r
n
Cr n – r 1
(iii) n =
C r –1 r
(iv) n
Cr = nCs Ÿ either r = s or r+s = n
Multinomial Theorem (For a positive integral index)
n!
6 n1 n 2 nk
(x1+x2+........+xk) = n
n 1! n 2 !.....nk! x1 x 2 ......x k ,

Where n1+n2+......+nk=n and 0 < n1, n2, .......nk< n

n!
• The greatest coefficient in this expansion is where q is the quotient
(q!) k –r
((q  1)!) r
and r is the remainder when n is divided by k.
Eg. Find the greatest coefficient in (x+y+z+w)15
15!
n=15, k=4 we have 15=4×3+3 i.e. q=3, r=3 greatest coefficient =
(3!)1 (4!)3
• Number of distinct terms in the expansion is n+k–1Ck–1 (Total number of terms).
• Number of positive integer solutions of x 1+x2+....+xk=n is n–1Ck–1.
• Number of non negative integer solutions of x 1+x2+....+xk=n is n+k–1Ck–1.
• Sum of all the coefficients is obtained by setting x1=x2=.....xk=1.

238
Greatest Coefficient and Greatest term
Consider the binomial expansion of (x+y)n. where n  W. For a given value of n,
Maximum value of nCr is nCn/2 if n is even
n n
Maximum value of nCr is C n –1 = C n 1 if n is odd.
2 2

To find the greatest term in the expansion of (x+y)n,

n 1
(i) Find m =
x
1
y

(ii) If m is an integer we have mth and (m+1)th terms as greatest terms.


(iii) If m is not an integer, then ([m]+1)th term is the greatest term where [.] denote the greatest
integer < m.
Divisibility Problems
From the expansion
(1+a)n = 1+nC1a+nC2a2+.....+nCnan, we can see that
(i) (1+a)n–1 is a mutliple of a = M(a)
(ii) (1+a)n–1–na is a mutliple of a2 = M(a2)

n ( n – 1) 2
(iii) (1+a)n–1–na– a is a mutliple of a3 and so on.
2
For example
• (1+8)50–1 = 9n–1 is M(8)
• (1+8)50–1 –50×8= 9n–399 is M(8 2) = M(64)

50 u 49 2
• (1+8)50–1 –50×8– 8 = M(83) = M(512) and soon
2
Binomial Theorem for any index (for negative or fractional index)

n ( n – 1) 2 n ( n – 1)(n – 2) 3
If nHQ, then (1+x)n=1+nx+ x+ x +..... f provided ~x~< 1.
2! 3!
• For any index n, the general term in the expansion of
n (n – 1).........(n – r  1) r
i. (1+x)n is Tr+1= x
r!
(–1) r n (n  1).........(n  r – 1) r
ii. (1+x)–n is Tr+1= x
r!

239
(–1) r n (n – 1).........(n – r  1) r
iii. (1–x)n is Tr+1= x
r!

iv. (1–x)–n is Tr+1= n (n  1).........(n  r – 1) xr


r!
• The following expansions should be remembered (for ~x~< 1).
i. (1+x)–1=1–x+x 2–x 3+ ........ f
ii. (1–x)–1=1+x+x 2+x 3 + ........ f
iii. (1+x)–2=1–2x+3x 2–4x 3+ ........ f
iv. (1–x)–2=1+2x+3x 2+4x 3+ ........ f
• Note : The expansion in ascending powers of x is valid if x is small. If x is large (i.e.
1
~x~> 1), then we may find it convenient to expand in powers of , which then will be
x
small.
Exponential series
x x2 x3
• e = 1+ 
x   .....f
1! 2! 3!
1 1 1
• e = 1+    .....f (e a 2.72 )
1! 2! 3!
§ 1 1 1 ·
• e +e–1= 2 ¨1     .....f ¸
© 2! 4! 6! ¹
§1 1 1 1 ·
• e –e–1= 2 ¨     .....f ¸
© 1! 3! 5! 7! ¹
Logarithmic series
For –1 < x d 1
x 2 x3 x 4
loge(1+x)=x–  –  .....f
2 3 4
x 2 x3 x 4
• loge(1–x)= –x– – –  .....f , –1 d x < 1
2 3 4

§ 1  x · §¨ x  x  x  .....f ·¸
3 5

• log ¨ ¸ =2 ¨ ¸ , –1 < x < 1


©1– x ¹ © 3 5 ¹
1 1 1
• loge 2=1–  –  .....f | 0.693
2 3 4
Solved Examples
1. If C0, C1, C2,......Cn denote the coefficients in the binomial expansion of (1+x) n, prove that :
C1+2C2+3C3+.....+nCn=n.2n–1

240
n
i.e. ¦ r.Cr=n.2n–1
r 1

Solution :
We have
C1+2C2+3.C3+.....+nC n
n
= ¦ r.Cr
r 1

n
= ¦ r..nCr [' Cr = nCr]
r 1

n n ª n n n –1 º
= ¦ r.. Cr–1 «¬' C r . C r –1 »
n–1
r 1 r r ¼
n
= ¦ n–1Cr–1
r 1

= n(n–1C0+n–1C1+....+ n–1C n–1)=n(1+1) n–1 >' x 1@


= n.2n–1
2. If C0, C1, C2,......Cn denote the coefficients in the binomial expansion of (1+x) n, prove that:
C0+3C1+5C2+.....+(2n+1)C n=(n+1).2n.
Solution
We have,
C0+3C1+5C2+.....+(2n+1)C n
n
= ¦ (2r+1)Cr
r 0

n
= ¦ (2r+1)nCr [' Cr=nCr]
r 0

n
= ¦ (2r..nCr+nCr)
r 0

n n
= ¦ 2r..nCr+ ¦ nCr
r 0 r 0

n
n n ª n n n –1 º
= 2 ¦ r.. .n –1 C r –1 + ¦ nCr «' Cr r
. Cr –1 »
r 1 r r 0 ¬ ¼

241
n n
= 2n ¦ n–1Cr–1+ ¦ nCr
r 1 r 0

= 2n.2n–1+2n
= n.2n+2n=(n+1)2n
3. If C0, C1, C2,......Cn denote the coefficients in the binomial expansion of (1+x) n, prove that:
12.C 1+2 2.C 2+3 2.C 3+.....+n 2.C n=n(n+1)2 n–2
Solution
We have,
12.C 1+2 2.C 2+3 2.C 3+.....+n 2.C n
n
= ¦ r2.Cr
r 1

n
= ¦ r2.nCr
r 1

n
= ¦ [r(r–1)+r] nCr
r 1

n n n – 1n –2 n n
.
= ¦ r(r–1). r r – 1 C r – 2  ¦ r. n –1
C r –1
r 1
r 1 r

§ n · §n ·
n (n – 1)¨ ¦ n – 2 C r – 2 ¸  n ¨ ¦ n –1 C r –1 ¸
©r 2 ¹ ©r 1 ¹

= n(n–1)(n–2C 0+n–2C 1+....+ n–2C n–2)


+n(n–1C0+n–1C1+....+n–1Cn–1)
= n(n–1).2n–2+n.2n–1
= n(n–1+2)2 n–2
= n(n+1)2n–2
4. If C0, C1, C2,......Cn denote the coefficients in the binomial expansion of (1+x) n, prove
that:13.C1+23.C2+33.C3+.....+n3.Cn=n2(n+3)2n–3
Solution
We have,
13.C 1+2 3.C 2+3 3.C 3+.....+n 3.C n
n
= ¦ r3.Cr
r 1

n
= ¦ r3.nCr
r 1

242
n
= ¦ [r(r–1)(r–2)+3r(r–1)+r] nCr
r 1

n n n
= ¦ r(r–1)(r–2)nCr+ ¦ 3r (r–1)nCr+ ¦ r..nCr
r 1 r 1 r 1

n
n n – 1 n – 2 n –3
= ¦ r(r–1)(r–2). . . . Cr –3
r 3 r r –1 r – 2
n n n – 1 n–2 n n
+ ¦ 3r(r–1) .  C r –2  ¦ r. n –1 C r –1
r 2 r r –1 r 1 r

§n · § n · §n ·
n (n – 1)(n – 2)¨ ¦ n – 3 C r –3 ¸  3n (n – 1)¨ ¦ n – 2 C r – 2 ¸  n ¨ ¦ n –1 C r –1¸
©r 3 ¹ ©r 2 ¹ ©r 1 ¹

= n (n–1) (n–2) { n–3C0+n–3C1+.......n–3Cn–3


+3n(n–1) (n–2C0+n–2C1+......n–2Cn–2
+ n {n–1C0 +n–1C1 +......+n–1Cn–1}
= n (n–1) (n–2). 2 n–3+3n(n–1).2n–2 +n.2n–1
= {(n–1) (n–2) +6(n–1) +4} n2 n–3
= n (n2+3n) 2n–3
= n2 (n+3) 2n–3
5. If C0,C1,C2.....,Cn–1,Cn denote the binomial coefficients in the expansion of (1+x) n, prove
C1 C C C n ( n  1)
that:  2. 2  3. 3  ......  n. n
C0 C1 C2 C n –1 2
C1 C C C
Solution: We have,  2. 2  3. 3  ......  n. n
C0 C1 C2 C n –1
n
Cr
= ¦r C
r 1 r –1

n n
Cr
= ¦ r.
r 1
n
C r –1
n
§ n – r  1· ª n Cr n – r  1º
= ¦ r.¨© r ¸¹ «' n
¬ C r –1 r ¼
»
r 1

= ¦ n – r  1
r 1

= ¦ ^ n  1 – r`
r 1

243
n

= n n  1 – ¦ r
r 1

n (n  1)
= n n  1 –
2
n ( n  1)
=
2
6. If C0,C1,C2.....,Cn–1,Cn denote the binomial coefficients in the expansion of (1+x) n, prove
C 0C1C 2 .....C n –1 (n  1) n
that: (C 0+C1) (C1+C2) (C2+C3) (C3+C4)......(C n–1+Cn) =
n!
Solution:
We, have (C0+C1) (C1+C2) (C2+C3)......(C n–1+Cn)

§ C ·§ C · § C ·
= C 0C 1C 2.....C n–1 ¨1  1 ¸ ¨1  2 ¸ .....¨1  n ¸
© C0 ¹ © C1 ¹ © C n –1 ¹
­ n n
C ½
= (C 0C 1....C n–1) – ®1  n r ¾
r 1 ¯ C r –1 ¿

n – r  1½
(C 0C 1....C n–1) – ­®1 
n
= ¾
r 1 ¯ r ¿
n  1·
(C 0C 1....C n–1) – §¨
n
= ¸
r 1
© r ¹

= (C 0C 1....C n–1) –
n
n  1 n
n!
r 1

7. If C0,C1,C2,.....Cn denote the binomial cofficients in the expansion of (1+x) n, prove that
C C C 2 n 1 – 1
C 0  1  2  ......  n
2 3 n 1 n 1

Solution: we have, C 0  C1  C 2  ......  C n


2 3 n 1
n
Cr
= ¦ r 1
r 0

n
1
= ¦ r 1. C n
r
r 0

n
1 n 1
= ¦ n  1. r  1 . C n
r
r 0

244
1 n n 1 n
= ¦ . Cr
n 1 r 0 r 1

1 n n 1 ª n 1 n 1 n º
= ¦ Cr 1
n 1 r 0 «¬ C r 1 r 1
. Cr »
¼

=
1
n 1
> n 1
C1  n 1 C 2  n 1 C3  ....n 1 C n 1 @
=
1
n 1
> n 1
C 0  n 1 C1  ....n 1 C n 1 – n 1
C0 @
=
1
n 1
>
2 n1 – 1 @
PRACTICE QUESTIONS
1 1 1
1. The sum    .......... ________
1!(n – 1)! 3!(n – 3)! 5!(n – 5)!
2. If the coefficient of x n in (1+x)101 (1–x+x2)100 is non-zero, then n cannot be of the form
a, 3r+1 b. 3r c. 3r+2 d. none of these
3. The coefficient of x ; 0 d r d n – 1 , is the expansion of (x+3) +(x+3) (x+2) +(x+3)n–2
r n–1 n–2

(x+2)2+......(x+2)n–1 are
a. n
Cr (3r – 2n) b. n
Cr (3n–r – 2n–r) c. n
Cr (3r – 2n–r) d. none of these
4. The number of real negative terms in the binomial expension of (1+ix) , n  N, x > 0 is
4n–2

a. n b. n+1 c. n–1 d. 2n
5. (n+2) C0 2 – (n+1) C1 2 +n. C2 2 ......is equal to
n n+1 n n n n–1

a. 4 b. 4n c. 4(n+1) d. 2(n+2)
f k –1
§ 1·
6. ¦
k 1
k ¨1  ¸
© n¹
a. n(n–1) b. n(n+1) c. n2 d. (n+1)2
7. The sum of rational term in 10

2  3 3  6 5 is equal to
a. 12632 b. 1260 c. 126 d. none of these
8. Last two digit of (23)14 are
a. 01 b. 03 c. 09 d. none of these
9.* n

If 4  15 = I+f, where n is an odd natural number, I is an interger and 0<f<1, then
a. I is a natural number b. I an even integer
c. (I+f) (1–F) = 1 d. 1–f = 4  5
n

10. The number of rational numbers lying in the interval (2002,2003) all whose digits after the
decimal point are non-zero and are in decreasing order is
9 10

a. ¦ 9
Pi b. ¦ 9
Pi c. 29–1 d. 210–1
i 1 i 1

245
11. Match the following:
Column I Column II
a. m
C1 nCm – mC2 2nCm + mC3 3nCm..... p. The coefficent of xm in the expansion of
+(–1)m–1mCmmnCmis ((1+x)n–1)m
1  x n 1
b. n
Cm + n–1
Cm + n–2
Cm +......+ Cm is
m
q. The coefficent of x in
m
x
c. C0Cn+ C1Cn–1+......CnC0 is r. The coefficent of x in (1+x)2n
n

d. 2k nC0 – 2k–1 nC1 n–1Ck–1+(–1)k nCk s. The coefficent of xk in the expansion


n–k
C0 is (1+x)n

Note:* Question with more than one option is correct

ANSWERS

2 n –1
1. 2. c 3. b 4. a
n!
5. c 6. c 7. d 8.

9. a,c,d 10. c 11. a o p; b o q; c o r; d o s

246
BINOMIAL THEOREM - II
Applications of Binomial Coefficients
1 Bino-geometric series
n
C o  n C1 x  n C 2 x 2 ............  n C n x n =(1+x)n
eg.nC0+nC1.3+nC2.32+.............+ nCn3n = (1+3)n = 4n
2 Bino-arithmetic series
a n C o  (a  d ) n C1  (a  2d ) n C 2  .................  (a  nd ) n C n
This series is the sum of the products of corresponding terms of
n
C o , n C1 , n C 2 ,.............n C n (binomial coefficients) and a, a+d, a+2d, ............,a+nd
(arithmetic progression)
Such series can be solved either by
(i) eliminating r in the multiplier of binomial coefficient from the (r+1)th terms of the series
n
(i.e. using r C r = n n–1 C ) or r–1
(ii) Differentiating the expansion of xa(1+xd)n or (If product of two or more numericals occur,
then differentiate again and again till we get the desired result)
eg. Prove that nC0+2 nC1+3nC2+..............+(n+1) nCn = (n+2). 2n–1
n
C0+2.nC1+.....+(n+1) nCn
n
= ¦ (r  1) nCn
r 0

n n
n n
= ¦ r Cr + ¦ Cr
r 0 r 0

n n n
n
= ¦ r. n–1
Cr–1 + ¦ C r
r 1 r r 1
= n.2 +2 = (n+2)2n–1
n–1 n

OR
Consider the expansion
n
C0+nC1 x+nC2 x 2+..........+nCn x n = (1+x)n
Multiply by x
n
C0x+nC1 x 2+nC2 x3+.........+nCn x n+1 = x(1+x)n
Differentiate w.r.t.x
n
C0+nC1 2x+nC23x 2+..........+nCn(n+1)x n = x n(1+x)n–1+(1+x)n
Put x = 1
n
C0+2nC1+3nC2+............+(n+1) nCn = n2n–1+2n = (n+2)2n–1
3. Bino-harmonic series
n
C 0 n C1 n
C2 n
Cn
+ + +.........+
a a  d a  2d a  nd
This series is the sum of the products of corresponding terms of
n
C o , n C1 , n C 2 ,.............n C n (binomial coefficients) and

247
1 1 1 1
, , ,................ (harmonic progression)
a a  d a  2d a  nd
Such seris can be solved either by
(i) eliminating r in the multiplier of binomial coefficient from the (r+1)th term of the series
1 nC 1 n+1
(ie using r = Cr+1) or
r 1 n 1
(iii) integrating suitable expansion
Note
(i) If the sum contains C0, C1, C2............Cn are all positive signs, integrate between limits 0
to 1
(ii) If the sum contains alternate signs (i.e.+ & –) then integrate between limits –1 to 0
(iii) If the sum contains odd coefficients (i.e. C o, C2, C4,.......) then integrate between –1 to
+1.
(iv) If the sum contains even coefficient (i.e. C1, C3, C5,........) the find the difference between
(i) & (iii) and then divide by 2
(v) If in denominator of binomial coefficient is product of two numericals then integrate
two times first time take limits between 0 to x and second time take suitable limits
2 n 1 – 1
n n n n
C0 C1 C2 Cn
eg: prove that + + +..................+ =
1 2 3 n 1 n 1
n n n n
C1 C2 Cn n C
n
C0 + + +..............+ = ¦  r
2 3 n 1 r 0 r 1

n
1 n Cr n
= ¦  Cr
n 1 r 0 r 1
1 n n 1
= ¦ C r 1
n 1 r 0
1
= (2n+1–n+1C0)
n 1
1
= (2n+1–1)
n 1
OR
Consider the expansion
(1+x)n = nC0+nC1x+nC2x 2+...........+nCnx n
Integrate between limit to 0 to 1
1 1
ª (1  x ) n 1 º ªn n x2 n x3 n
C n x n 1 º
« » = « C 0 x  C1  C2  ........  »
«¬ n  1 »¼ 0 «¬ 2 3 n  1 »¼
0

§ 2 n 1 1 ·¸ n
C1 n
C2 n
Cn
¨ –
¨ n  1 n  1 ¸ = C 0+ + +...........+
n
© ¹ 2 3 n 1

248
n n n
C1 C2 Cn 1
? C0+
n
+ +...........+ = (2n+1–1)
2 3 n 1 n 1
4. Bino-binomial series
n
C 0 n C r  n C1 n C r 1  n C 2 n C r  2  ...........  n C n – r n C n or
m
C 0 n C r  m C1n C r –1  m C 2 n C r – 2  ...........  m C r n C 0
Such series can be solved by multiplying two expansions, one involving the first factors as
coefficient and the other involving the second factors as coefficients and finally equating
coefficients of a suitable power of x on both sides.

Prove That
n–1
C0 nC1+n–1C1nC2+n–1C2nC3+..........+n–1Cn–1nCn = 2n–1Cn–1
We have
(nC0xn+nC1xn–1+nC2xn–2+.......+nCn–1x+nCn) (n–1C0+n–1C1x+n–1C2x2+.........+n–1Cn–1xn–1) = (1+x)n
(1+x)n–1
Ÿ (nC0xn+nC1xn–1+nC2xn–2+....+nCn–1x+nCn) (n–1C0+n–1C1x+n–1C2x2+......+n–1Cn–1xn–1) = (1+x)2n–1
Equate the coefficients of xn–1 on both sides, n–1C0nC1+n–1C1nC2+..............+n–1Cn–1nCn = 2n–1Cn–1
Note : For the sake of convenience, the coefficients nC0, nC1,.........nCr,......nCn are usually denoted
by C0, C1,..........Cr,.......Cn respectively

Use of complex numbers in Binomial Theorem


We know (cos T +sin T )n = cosn T +sinn T .
Expand and the binomial and then equating the real and imaginary parts, we get
cosn T = cosn T –nC2cosn–2 T sin2 T +nC4cosn–4 T sin4 T +................
sinn T = nC1cosn–1 T sin T –nC3cosn–3 T sin3 T +nC5cosn–5 T sin5 T +...............
C1 tan T – n C1 tan 3 T  n C5 tan 5 T...............
n

Ÿ tann T = 1 – n C tan 2 T  n C tan 4 T – n C tan 6 T  .....


2 4 6

Solved Examples
1. If C0, C1, C2, C3,...........Cn–1, Cn denote the binomial coefficients in the expansion of (1+x)n, prove
that

C0C1+C1C2+C2C3+.......+C n–1Cn
( 2n )!
= (n – 1)!(n  1)!

1.3.5(2n – 1)
= .n.2 n
(n  1)!

Solution :
Using binomial expansion, we have
(1+x)n = C0+C1x+C2x2+........+Crxr+.....+Cnxn.......(A) and

249
(1+x)n = C0xn+C1xn–1+C2xn–2+.........+Crxn–r+.............+.........+Cn–1x+Cn.....(B)
Multiplying (A) and (B), we get
(1+x)2n
= (C0+C1x+C2x2+.....+Crxr+.....+Cnxn)×(C0xn+C1xn–1+C2xn–2+.....+Crxn–r+......+Cn–1x+Cn)
or (C 0 +C 1 x+C 2 x 2 +.......+C rx r+.......+C nx n)(C 0 x n+C 1x n–1 +C2 n–2 +......+C rx n–r+......+C n–
1
x+Cn)=(1+x)2n..................(C)
Equating the coefficients of xn–1 on both sides of (C), we get General Term
C0C1+C1C2+.......+C n–1Cn= Cn–1 2n TS1+ 1= 2n c x S
S

( 2n )!
Ÿ C0C1+C1C2+.......+Cn–1Cn=2nCn–1 =
(n  1)!(n – 1)!
Now,
( 2n )!
(n  1)!(n – 1)!
1.2.3.4.5.6..........(2n – 2)(2n – 1)(2n )
= (n  1)!(n – 1)!

^1.3.5.......(2n – 1)`^2.4.6........2n`
= (n  1)!(n – 1)!

^1.3.5.......(2n – 1)`QQ
=
(n  1)!(n – 1)!

1.3.5.......(2n – 1) 2 QQ (Q –  ) 


= (n  1)! (n – 1)!

1 (  Q –) n.2 n



= (n  1)!
Hence,
( 2n )!
C0C1+C1C2+C2C3+.........+Cn–1Cn = (n – 1)!(n  1)!

n
1 (  Q – )n .2

= ( n  1)!
2. If C0, C1, C2, C3,...........Cn–1, Cn denote the binomial coefficients in the expansion of (1+x)n, prove
that
(2n )!
C02+C12+C22+..........Cn2 =
( n! ) 2

1.3.5....(2n – 1) n
= 2
n!
Solution :
Using binomial expansion, we have

250
(1+x)n = C0+C1x+C2x2+........+Crxr+.....+Cnxn.......(A) and
(1+x)n = C0xn+C1xn–1+C2xn–2+.........+Crxn–r+.............+.........+Cn–1x+Cn.....(B)
Multiplying (A) and (B), we get
(1+x)2n
= (C0+C1x+C2x2+.....+Crxr+.....+Cnxn)×(C0xn+C1xn–1+C2xn–2+.....+Crxn–r+......+Cn–1x+Cn)
or (C 0 +C 1 x+C 2 x 2 +.......+C rx r+.......+C nx n)(C 0 x n+C 1x n–1 +C2 n–2 +......+C rx n–r+......+C n–
1
x+Cn)=(1+x)2n..................(C)
Equating the coefficients of xn on both sides of (C), we get
General Term
C02+C12+C22+.......Cn2=2nCn
TS+1 = 2n c x S
S
(2n )!
Ÿ C0 +C1 +C2 +.......C =
2 2 2 2
n! n!
n

Now
(2n )! 1.2.3.4.5..........(2n – 2)(2n – 1)(2n )
=
n! n! n! n!
^1.3.5.......(2n – 1)`^2.4.6........(2n – 2)(2n )`
=
n! n!

^1.3.5.......(2n – 1)`u 2 n u ^1.2.3........(n – 1).n`


=
n! n!

^1.3.5.......(2n – 1)`2 n n!
=
n! n!
1.3.5.......(2n – 1) n
= 2
n!
(2n )!
Hence, C02+C12+C22+.......Cn2 =
n! n!
1.3.5.......(2n – 1) n
= 2
n!
3. If C0,C1,C2,.......Cn denote the binomial fulfillments in the expansion of (1+x)n, prove that ;
C02–C12+C22–C32+......+(–1)n Cn2
­0, if nisodd
=® n/2 n
¯(–1) . C n / 2 , if niseven
Solution : We have,
(1+x)n = (C0+C1x+C2x2+....+Cnxn) .........(i)
Also,
(1+x)n = (C0xn+C1xn–1+..........+Cn–1x+Cn)...........(ii)
Replacing x by –x in (i), we get
(1–x)n = C0–C1x+C2x2–C3x3+.......+(–1)nCnxn........(iii)
Multiplying (ii) and (iii), we get
(C0–C1x+C2x2–C3x3+.....+(–1)nCnxn)×(C0xn+C1xn–1+C2n–2+........+Cn–1x+Cn) = (1+x)n(1–x)n
or (C 0 –C 1x+C 2x 2–C 3 x 3 +.........+(–1) nC nx n)×(C 0 x n+C 1x n–1 +C 2x n–2 +........+C n– ) = (1–
x2)n........(iv)

251
Equating coefficients of xn on both sides of (iv), we get
C02–C12+C22–C32+......+(–1)nCn2 = Coeffecient of xn in (1–x2)n.............(v)
Clearly, RHS of (v) contains only even powers of x when it is expanded with the help of binomial
theorem. Therefore,
? Coefficient of xn in (1–x2)n = 0, if n is an odd natural number..
If n is even, suppose (r+1) the term in the binomial expansion of (1–x2 )n contains xn. We have,
Tr+1 = nCr(–1)r(x2)r = nCr(–1)rx2r
For this term to contain xn, we must have,
2r = n Ÿ r = n/2
? Coeff. of xn = nCn/2(–1)n/2
Hence,
C02–C12–C22–C32+.....+(–1)nCn2
­0, if nisodd
=® n/2 n
¯(–1) . C n / 2 , if niseven
n n
4. If (1+x)n = C0+C1x+C2x2+....+Cnxn prove that ¦ ¦ (C r  C s ) = (n+1)
r 0 s 0
Sloution: We have,
n n
¦ ¦ (C r  C s )
r 0 s 0

n n n n
= ¦ ¦ Cr  ¦ ¦ Cs
r 0 s 0 r 0 s 0

n
§ n · n § n ·
= ¦
r 0
¨ ¦ C r ¸  ¦ ¨ ¦ Cs ¸
©s 0 ¹ r 0 ©s 0 ¹
n n
n n
= ¦2 + ¦2
s 0 r 0
= (n+1)2n+(n+1)2n
= 2(n+1)2n
= (n+1)2n+1
n n

5. If (1+x)n = C0+C1x+C2x2+....+Cnxn prove that ¦ ¦C C


r 0 s 0
r s = 22n

Solution: we have,
n n
¦ ¦ C r Cs
r 0 s 0

n § n ·
= ¦ ¨ C r ¦ Cs ¸
r 0 © s 0 ¹
n
n
= ¦ 2 .C r
r 0

252
§ n ·
= 2n ¨ ¦ Cr ¸
©r 0 ¹
= 2n.2n = (2n)2 = 22n
n n § n ·§ n ·
ALITER ¦ ¦ C r C s = ¨ ¦ C r ¸ ¨ ¦ C s ¸ = 2n.2n = 22n
r 0 s 0 ©r 0 ¹ ©s 0 ¹
6. If (1+x) = C0+C1x+C2x2+....+Cnxn prove that
n

¦ ¦ (C +C ) = n.2n
0d r  s d n
r s

Solution: We have, Cr
C0 C1 C2 ...... Cn
Cs
¦ ¦ C r  C s
n n n
¦ ¦ (C r  C s ) = ¦ C r  C s +2
r 0 s 0 r 0 0d r  s d n C0 2C0 C0+C1 C0+C2 ...... C0+Cn

§ ·
¦ ¦ C r  C s
n

Ÿ (n+1)2n+1 = 2 ¨ ¦ C r ¸ +2 C1 C1+C0 2C1 C1+C2 ...... C1+Cn


©r 0 ¹ 0d r  s d n

Ÿ (n+1)2n+1 = 2 .2 +2
n ¦ ¦ C r  C s C2 C2+C0 C2+C1 2C2 ...... C2+Cn
0d r  s d n ..
Ÿ n.2 n+1
=2 ¦ ¦ C r  C s .. .
0d r  s d n .

Ÿ 0dr¦ ¦ C r  C s = n.2n Cn Cn+C0 Cn+C1 Cn+C2 ...... 2Cn


s d n

7. If (1+x)n = C0+C1x+C2x2+....+Cnxn prove that ¦ ¦ C C = 1 (22n–2nC )


0d r  s d n r s 2 n

Solution: We have,
n n
§ n ·
¦ ¦C C
r 0 s 0
r s
©r 0
2
= ¨ ¦ C r ¸ + 2 0dr¦ ¦ C r Cs
sd n¹
Cr
¦ ¦ C r Cs C0 C1 C2 ...... Cn
Ÿ 2 = Cn+2 0dr sd n
2n 2n
Cs

C0 C 02 C0C1 C0C2 ...... C0Cn


Ÿ 0dr¦ ¦ C r C s = 1 [22n–2nC ]
sd n 2 n

ALITER We have, C1 C1C0 C12 C1C2 ...... C1Cn

2
§ n · § n 2· § · C2 C2C0 C2C1 C22 ...... C2Cn
¨ ¦ C r ¸ = ¨ ¦ C r ¸ +2 ¨ ¦ ¦ C r C s ¸
©r 0 ¹ ©r 0 ¹ © 0 d r s d n ¹ ..
.. .
§ · .
Ÿ (2n)2 = 2nCn+2 ¨ ¦ ¦ C r C s ¸
© 0 d r s d n ¹ Cn CnC0 CnC1 CnC2 ...... Cn 2
1 2n 2n
Ÿ 0dr¦ ¦ C r Cs = [2 – Cn]
sd n 2

253
PRACTICE QUESTIONS
1 If n is an integer between 0 and 21, then the minimum value of n!(21–n)! is attained for n =
(a) 1 (b) 10 (c) 12 (d) 20
2 404
C4–4C1303C4+4C2202C4–4C3101C4 is equal to
(a) (401)4 (b) (101)4 (c) 0 (d) (201)4
1 1 1
3 If (3+x2008+x2009)2010 = a0+a1x+a2x2+.......+anxn, then the value of a0– a1– a2+a3– a4–
2 2 2
1
a +a .......is
2 5 6
(a) 32010 (b) 1 (c) 22010 (d) None of these
4
2
­n

® C 0  C3  ........... –
n 1 n
2

½ 3
C1  n C 2  n C 4  n C5 ......... ¾ + n C – n C  n C – n C  ....... 2=
¯ ¿ 4 1 2 4 5

(a) 3 (b) 4 (c) 2 (d) 1


20
5 Value of ¦ r (20 – r ) (20Cr)2 is equal to
r 0
(a) 400 39C20 (b) 400 40C19 (c) 400 39C19 (d) 400 38C20
6 If for z as real or complex , (1+z +z ) = C0+C1z +C2z +............+C16z , then
2 4 8 2 4 32

(a) C0–C1+C2–C3+............+C16 = 1 (b) C0+C3+C6+C9+C15 = 37


(c) C2+C5+C8+C11+C14 = 36 (d)
C1+C4+C7+C10+C13+C16 = 37
7 Read the passage and answer the following questions
Any complex number in polar form can be an unpleasing in Euler’s form as cos T +sin T = ei T which
n n

is useful is finding the sum of series ¦ C r cos ș  i sin ș ¦ C r cos rș  i sin rș


n r n

r 0 r 0

= ¦ C r e = (1+ei T )n
n ișr

r 0

Also we know that the sum of binomial series does not change if r is replaced by n–r.
100
100
(i) Value of ¦ C r sin(rx0) is .....
r 0

(a) 2100 cos


( ) sin (50x)
x
2
(b) 2100sin50x cos
x
2

cos (50x) sin ( )


x
(c) 2101 2 (d) 2101sin100 50xcos50x

50
50
ii. In triangle ABC, the value of ¦ C r ar bn–r cos(rb–(50–r)A) is equal to (a,b,c are sides
r 0
opposite to A, B, C & S in semi perimeter)
(a) c49 (b) (a+b)50 (c) (25–a–b)50 (d) None of these

254
50
50
¦ C r sin 2rx
r 0
(ii) If f(x) = 50
50 , then f ( S /8) is
¦ C r cos 2rx
r 0

(a) 1 (b) –1 (c) irrational value (d) None of these


8 Match the following
Column I Column II

¦¦
10
C i  10 C j 2 20 – 20 C10
(a) iz j
(p)
2
10
(b) ¦ ¦ C i  10 C j (q) 220 – 20C10
0di d jd n

(c) ¦¦ 10
Ci 10C j (r) 220
0d i  jd n

10
¦ ¦
10
10
C i  10 C j 2 20  20 C10
(d) (s)
i 0 j 0 2
9 The coefficient of O n P n is the expansion of (1+ O )n (1+ P )n ( O + P )n is

(a) ¦
n

r 0
C
n
r
2
(b)
n
¦
r 0
n
C r –2
2
(c)
n
¦
r 0
n
C r 3
2
(d) ¦
n

r 0
C
n
r
3

10 If C1,C2..........Cn are binomial Coefficients, then the value of C –2C +3C –..........–2nC22n is 1
2
2
2
3
2

(a) n2 (b) (–1)n–1n (c) 2(–1) n Cn (d) –n2 n–1 2n–1

'Note : Questions with * have more than one correct option'

ANSWERS
1. b 2. b 3. 4. d

5. d 6. a,b,d 7. (i) a (ii) c (iii) a

8. a o q; b o s; c o p; d o r 9. d 10. c

255
BINOMIAL THEOREM - III
Principle and simple applications - Problem Solving
1. Binomial Theorem for positive integral index.
If x, y H R and n HN
n

(x+y) = Cox + C1x y+ C2x y +.........+ Crx y +....+ Cny = ¦ nCrx n–ryr
n n n n n–1 n n–2 2 n n–r r n n
r 0

Properties
• Number of terms of the above expansion is (n+1)
• The binomial coefficients equidistant from the beginning and the end in a binomial expansion
are equal.
• General term = Tr+1=nCr xn–ryr
th
§n ·
n is even : only one middle term ¨  1¸ term .
©2 ¹
Middle term
th th
§ n 1· § n 3·
n is odd : Two middle terms ¨ ¸ and ¨ ¸ term .
© 2 ¹ © 2 ¹
n
n is even Cn .
2

Greatest coefficient
n n
n is odd C n2–1 and C n21 .
Greatest Term
To find numerically greatest term in the expansion of (1+x)n
x n  1
i. Calculate m = x 1
ii. If m is an integer, then Tm and Tm+1 are equal and both are greatest term.
iii. If m is not an integer then T[m]+1 is the greatest term.
n
§ y·
Note : To find the greatest term in the expansion of (x+y) , find the greatest term in ¨1  ¸ and n
© x¹
n
§ y·
then multiply by x (since (x+y) =x ¨1  ¸
n n n
© x¹
2. Multinomial Theorem (for a positive integral index)
n!
(x 1+x 2+x 3+.....x k)n = ¦ n !n !n !.....n ! x 1
n1 n n
x 2 2 x 3 3 .....x k
nk

1 2 3 k

where niH{0, 1, 2,....n}, n 1+n2+.....+nk=n

256
n!
• The greatest coefficient in this expansion is where q is the quotient and
(q!) k –r
((q  1)!) r
r is the remainder when n is divided by k.
Eg. Find the greatest coefficient in (x+y+z+w)15
15!
n=15, k=4 we have 15=4×3+3 i.e. q=3, r=3 greatest coefficient =
(3!)1 (4!)3
• Number of distinct terms in the expansion is n+k–1Ck–1 (Total number of terms).
• Number of positive integer solutions of x 1+x2+....+xk=n is n–1Ck–1.
• Number of non negative integer solutions of x 1+x2+....+xk=n is n+k–1Ck–1.
• Sum of all the coefficients is obtained by setting x1=x2=.....xk=1.
3. Binomial Theorem for Negative or Fractional Indices
If nHQ, then
n ( n – 1) 2 n ( n – 1)(n – 2) 2
(1+x)n=1+nx+ x+ x +..... f provided ~x~< 1.
2! 3!
• For any index n, the general term in the expansion of
n ( n – 1).........( n – r  1) r
i. (1+x)n is Tr+1= x
r!
(–1) r n (n  1).........(n  r – 1) r
ii. (1+x)–n is Tr+1= x
r!
(–1) r n (n – 1).........(n – r  1) r
iii. (1–x)n is Tr+1= x
r!

iv. (1–x)–n is Tr+1= n (n  1).........(n  r – 1) xr


r!
• The following expansions should be remembered (for ~x~< 1).
i. (1+x)–1=1–x+x 2–x 3+ ........ f
ii. (1–x)–1=1+x+x 2+x 3 + ........ f
iii. (1+x)–2=1–2x+3x 2–4x 3+ ........ f
iv. (1–x)–2=1+2x+3x 2+4x 3+ ........ f
• Note : The expansion in ascending powers of x is valid if x is small. If x is large (i.e. ~x~> 1),
1
then we may find it convenient to expand in powers of , which then will be small.
x
4. Exponential series
x x2 x3
• e = 1+ 
x   .....f
1! 2! 3!
1 1 1
• e = 1+    .....f (e a 2.72 )
1! 2! 3!
§ 1 1 1 ·
• e +e–1= 2 ¨1     .....f ¸
© 2! 4! 6! ¹

257
§1 1 1 1 ·
• e –e–1= 2 ¨     .....f ¸
© 1! 3! 5! 7! ¹
5. Logarithmic series
For –1 < x d 1
x 2 x3 x 4
loge(1+x)=x–  –  .....f
2 3 4
x 2 x3 x 4
• loge(1–x)= –x– – –  .....f , –1 d x < 1 or x  1
2 3 4

§ 1  x · §¨ x  x  x  .....f ·¸
3 5

• log ¨ ¸ =2 ¨ ¸ , –1 < x < 1


©1– x ¹ © 3 5 ¹
1 1 1
• loge 2=1–  –  .....f | 0.693
2 3 4
Solved Examples
n

1. Let (1+x) = n
¦a x
r 0
r
r
,

§ a1 ·§ a 2 ·§ a 3 · § an ·
then ¨¨1  a ¸¸¨¨1  a ¸¸¨¨1  a ¸¸.......¨¨1  a ¸¸ is equal to
© 0 ¹© 1 ¹© 2 ¹ © n –1 ¹

n  1 n 1 n  1 n n n –1
a. b. c. d. None of these
n! n! (n – 1)!
Solution :
(1+x)n = nC0+nC1x+nC2x2+.........+ nCnx n
= a0+a1x +a2x2 +...........+anxn (given)
Comparing a0=nC0, a1=nC1, a2=nC2,.......... and so on.
§ n
C1 ·§ n C 2 ·§ n
C3 · § n
Cn ·
¨
?¨ 1  ¸¨
¨ 1  ¸
¸¨
¨ 1  ¸
¸.......¨
¨ 1  ¸
©
n ¸
C 0 ¹© n
C1 ¹© n
C2 ¹ ©
n
C n –1 ¸¹

§ n – 0 ·§ n – 1 ·§ n – 2 · § n – n 1·
= ¨1  ¸¨1  ¸¨1  ¸.......¨1  ¸
© 1 ¹© 2 ¹© 3 ¹ © n ¹

1 n 1 n 1 n
. . ........
1 n n  1 n
=
1 2 3 n n!
Ans. b
n n
1 r
2. If a = ¦ n , then ¦ n equals
n
r 0 Cr r 0 Cr
1
a. (n–1).an b. nan c. na d. None of these
2 n
258
Solution :
n
r
Let S = ¦
r 0
n
Cr
0 1 2 3 n
ŸS n n  n  ..........  n
n
C0 C1 C2 C3 C n ....(1)
n n –1 n – 2 n – 3 0
Also, S  n  n  n  ..........  n
n
C0 C1 C2 C3 C n ....(2)
Adding (1) and 2
n n n n
2S n
n n  .........  n
C0 C1 C2 Cn

§ 1 1 1 1 ·
2S n ¨¨ n  n  n  .........  n ¸¸
© C0 C1 C2 Cn ¹

1
Ÿ2S=nan ŸS= na
2 n
Ans. c
3. If (1+x)10=a0+a1x+a2x 2+a3x 3+.......+a10x 10, then (a0–a2+a4–a6+a8–a10)2 + (a1–a3+a5–a7+a9)2 is
equal to
a. 310 b. 2 10 c. 29 d. none of these
Solution :
Put x=i and x = –i
Ÿ (1+i)10= (a0–a2+a4–a6+a8–a10) + i(a1–a3+a5+a5–a7+a9) ....(1)
Also, (1–i)10= (a0–a2+a4–a6+a8–a10) – i(a1–a3+a5–a7+a9) ....(2)
Multiply (1) and (2)
((1+i)(1–i)10=(a0–a2+a4–a6+a8–a10)2 + (a1–a3+a5–a7+a9)2
210=(a0–a2+a4–a6+a8–a10)2 + (a1–a3+a5–a7+a9)2
Ans. b
4. If (1+x+2x 2)20=a0+a1x+a2x 2+a3x3+.......+a40x40, then a0+a2+a4+........+a38 is equal to
a. 219(2 20–1) b. 220(2 19–1) c. 219(2 20+1) d. none of these
Solution :
Put x=1 and x = –1 and adding we get 4 20+220=2(a0+a2+a4+....+a38+a40)
Ÿ 239+219=a0+a2+.....+a38+220 ' a 40 2 20
? a0+a2+a4+.....+a 38=239+2 19–2 20
= 219(220+1–2)
= 219(220–1)
Ans. a
5. The coefficient of x13 in the expansion of (1–x)5 (1+x+x2+x3)4 is
a. 4 b. –4 c. 0 d. none of these
Solution :
Coefficient of x 13 in = (1–x)5 (1+x+x2+x3)4 = (1–x)5 ((1+x)(1+x 2))4

259
= (1–x) {(1–x)(1+x)(1+x 2)}4
= (1–x) {(1–x4)4
= (1–x) (1–4x4+6x8–4x12+x16)
? coefficient of x13 is –1×–4=4
Ans. a
6. The sum 20C0+20C1+20C2+.............. + 20C10 is equal to
20! 1 20! 20!
a. 220+ b. 219– c. 219+ d. none of these
(10!) 2 2 (10!) 2 (10!) 2
Solution :
In the expansion of (1+x)20, put x=1
220 = 20C0+20C1+20C2+.............. + 20C9+ 20C10+....... 20C20
= 2(20C0+20C1+.............. + 20C10)– 20C10 (' nCr=nCn–r)
2 20  20 C10 20
Ÿ = C0+20C1+.............. + 20C10
2
1 20!
= 219 + =20C0+20C1+.............. + 20C10
2 (10!) 2
Ans. d
PRACTICE QUESTIONS
10
§x 3 ·
1. The coefficient of x in ¨ – 2 ¸ is
4
©2 x ¹
405 405 450
a. b. c. d. None of these
256 259 263
2. Let Tn denotes the number of triangles which can be formed using the vertices of a regular
polygon of n sides. If Tn+1–Tn=21, then n equals
a. 5 b. 7 c. 6 d. 4
m
§10 ·§ 20 · §p·
3. The sum ¦ ¨¨ i ¸¸¨¨ m – i ¸¸ , where ¨¨ q ¸¸ =0 if p>q, is maximum when m is
i 1 © ¹© ¹ © ¹
a. 5 b. 10 c. 15 d. 20
4. Coefficient of t24 in (1+t2)12 (1+t12) (1+t24) is
a. 12
C 6+3 b. 12
C 6+1 c. 12
C6 d. 12
C 6+2
5. If Cr=(k –3) Cr+1, then k belongs to
n–1 2 n

a. – f,–2@ b. >2, f c. [– 3 , 3 ] d. 3 ,2
6. If (1+ax) =1+8x+24x +......., then a = __ and n = ____.
n 2

20
§ 1 ·
7. The greatest term in the expansion of 3 ¨1  ¸ is
© 3¹

§ 20 · 1 § 20 · 1 1 § 20 ·
¨¨ ¸¸ ¨¨ ¸¸ ¨ ¸
9 ¨© 9 ¸¹
a. b. c. d. none of these
© 7 ¹ 27 © 6 ¹ 81

260
8. If x= 6
2  1 , then the integral part of [x] is
a. 98 b. 197 c. 196 d. 198
9. The greatest integer m such that 5 divides 7 +2 .3 for nHN is
m 2n 3n–3 n–1

a. 0 b. 1 c. 2 d. 3
1
10. If =a +a +a x 2....., then an =
(1 – ax )(1 – bx ) 0 1 2

a n 1 – b n 1 b n 1 – a n 1 bn – a n a n – bn
a. b. c. d.
b–a b–a b–a b–a
11. Read the passage and answer the following questions.
If n is a positive integer and a1, a2, a3 .....am H C, then
n!
(a1+a2+a3+.....+am)n = ¦ n !n !n !......n ! .a 1 .a 2 .a 3 .......a m where n1, n2, n3..... nm are all
n1 n2 n3 nm

1 2 3 m

non-negative integers subject to the condition n1+n2+n3+....+nm=n.


i. The number of distinct terms in the expansion of (x1+x2+x3+.....+xn)4 is
a. n+1
C4 b. n+2
C4 c. n+3
C4 d. n+4
C4
ii. The coefficient of x y z in the expansion of (1+x–y+z)
3 4 9 is

a. 2320 b. 2420 c. 2520 d. 2620


iii. The coefficient of a b c in the expansion of (bc+ca+ab) is
3 4 5 6

a. 40 b. 60 c. 80 d. 100
iv. The coefficient of x39 in the expansion of (1+x+2x2)20 is
a. 5.2 19 b. 5.2 20 c. 5.2 21 d. 5.2 23
v. The coefficient of x in (1–x+x ) and in (1+x–x ) are respectively a and b, then
20 2 20 2 20

a. a=b b. a>b c. a<b d. a+b=0


12*. Match the following :
Column I Column II
a. If n be the degree of the polynomial (p) 2
§ 7

(3x 2  1) ¨ x  (3x  1) – x – (3x  1) ¸ ,
©
2 2
7
·
¹
then n is divisible by (q) 4
b. In the expression of (x+a) there is only one middle term
n
(r) 8
for x=3, a=2 and seventh term is numerically (s) 16
greatest term, then n is divisible by (t) 32
c. The sum of the binomial coefficients in the expansion of
(x–3/4+nx5/4)m, where m is positive integer lies between
200 and 400 and the term independent of x is equals 448.
Then n5 is divisible by
'Note : Questions with * have more than one correct option'
ANSWERS
1. a 2. b 3. c 4. d 5. d 6. a=2, n=4 7. a
8. b 9. c 10. b 11. (i) c, (ii) c (iii) b (iv) c (v) b
12. ao p, q, r; bo p, q, r, s; co p, q, r, s, t

261
BINOMIAL THEOREM
For Positive Integral Index - Problem Solving (Lecture-04)

Summation of Series (involving binomial coefficients)


1 Bino-geometric series
n
C o  n C1 x  n C 2 x 2 ............  n C n x n =(1+x)n
2 Bino-arithmetic series
a n C o  (a  d ) n C1  (a  2d ) n C 2  .................  (a  nd ) n C n
This series is the sum of the products of corresponding terms of
n
C o , n C1 , n C 2 ,.............n C n (binomial coefficients) and a, a+d, a+2d, ............,a+nd
(arithmetic progression)
Such series can be solved either by
(i) eliminating r in the multiplier of binomial coefficient from the (r+1) th terms of the
n
series (i.e. using r C r = n n–1 C ) r–1
or
(ii) Differentiating the expansion of x (1+xd)n or (If product of two or more numericals occur,
a

then differentiate again and again till we get the desired result)
3 Bino-harmonic series
n
C 0 n C1 n
C2 n
Cn
+ + +.........+
a a  d a  2d a  nd
This series is the sum of the products of corresponding terms of
n
C o , n C1 , n C 2 ,.............n C n (binomial coefficients) and
1 1 1 1
, , ,................ (harmonic progression)
a a  d a  2d a  nd
Such seris can be solved either by
(i) eliminating r in the multiplier of binomial coefficient from the (r+1) th term of the
series
1 nC 1 n+1
(ie using r = Cr+1) or
r 1 n 1
(iii) integrating suitable expansion
Note
(i) If the sum contains C 0, C1, C2............C n are all positive signs, integrate between
limits 0 to 1
(ii) If the sum contains alternate signs (i.e.+ & –) then integrate between limits –1 to 0
(iii) If the sum contains odd coefficients (i.e. C o, C2, C4,.......) then integrate between –1
to +1.
(iv) If the sum contains even coefficient (i.e. C1, C3, C5,........) then find the difference between
(i) & (iii) and then divide by 2

262
(v) If in denominator of binomial coefficient is product of two numericals then integrate two
times first time take limits between 0 to x and second time take suitable limits
4 Bino-binomial series.
n
C 0 n C r  n C1 n C r 1  n C 2 n C r  2  ...........  n C n – r n C n or
m
C 0 n C r  m C1n C r –1  m C 2 n C r – 2  ...........  m C r n C 0
Such series can be solved by multiplying two expansions, one involving the first factors as
coefficient and the other involving the second factors as coefficients and finally equating
coefficients of a suitable power of x on both sides.
Binomial coefficients
1 C0+C1+C2+C3+............2n
2 C0–C1+C2–C3+............=0
3 C0–C1+C2–C3+...........+Cr(–1)r= n–1Cr(–1)r ;r<n
4 Co+C2+C4+C6+............=2n–1
5 C1+C3+C5+C7+............=2n–1
nS
6 C0–C2+C4–C6+..........= 2 n cos 4
nS
7 C1–C3+C5–C7+.......= 2 n sin 4
8 C0+C4+C8+C12+.......=
1 § n –1

¨2
n

nS ·
 2 cos ¸
4 ¹

9 C1+C5+C9+C13+.......=
1 § n –1

¨2  2 nsin n4S ·¸¹
1§ n nS ·
10 C0+C3+C6+C9+.......= ¨ 2  2 cos ¸
3© 3 ¹
11 C1+2C2+................= ¦ rC r n.2 n –1

12 C1–2C2+3C3................= ¦ (–1) r –1
rC r 0
13 12C1+22C2+..................=n(n+1)2 2n–2
14 12C1–22C2..............= 0
15 C02+C12+C22+.............= 2nCn
­0if nisodd
16 C0 –C1 +C2 –C3 .............= ®
2 2 2 2
n/2 n
¯(–1)  C n / 2 if niseven

17 ¦ CiCj = 22n–1 – 2n–1Cn


0di jd n

18 ¦ (Ci–Cj)2 = (n+1) 2nCn–22n


0di jd n

263
Note : Consider the equation x 1+x 2+...........+x r = n,. n  N.
Number of positive integral solutions = n–1Cr–1
Number of non negative integral solutions = n+r–1Cr–1
Solved Examples
n

The value of ¦
2n
1 C r r is
r 1

(a) n.22n–1 (b) 22n–1 (c) 2n–1+1 (d) None of these


Solution:
n
2n
¦r r
2n–1
Cr–1
r 1
n

2n ¦
2 n –1
= C r –1
r 1

= 2n(2n–1C0+2n–1C1+2n–1C2+......2n–1Cn–1)
2 2 n –1
= 2n. = n.22n–1 [' 22n–1 = C0+C1+.........+C2n–1 Ÿ 22n–1= 2(C0+C1+C2+......+ Cn–1)]
2
Ans (a)
2 The coefficient of x5 in the expansion of (1+x)21+(1+x)22+.........+(1+x)30 is
(a) 31C5–21C5 (b) 31C6–21C6 (c) 30C6–20C6 (d) None of these
Solution:
Co-efficient of x5 in (1+x)21+(1+x)22+............+(1+x)30

= Co-efficient of x in
(1  x ) 21 (1  x )10 – 1
5
^ `
(1  x ) – 1
Ÿ coefficient of x in (1+x) – (1+x)21 is 31C6 – 21C6
6 31

Ans (b)
3 The number of distinct terms in the expansion of (x+y–z)16 is
(a) 136 (b) 153 (c) 16 (d) 17
Solution :
Apply n+r–1Cr–1 to get number of terms 16+3–1C3–1 = 18C2 = 153
Ans (b)
4 If I is the integral part of (2+ 3 )n and f is the fractional part. Then (I+f) (1–f) is equal to
(a) 0 (b) 1 (c) n (d) None of these
Solution :
Let (2+ 3 )n = I+f..........................(1) (0 d f<1)
and (2– 3 )n = F.............................(2) (0<F<1)
(1) + (2) gives (adding 0 <f+F<2)
2(nC0.2n+nC2.2n–2( 3 )2+........) = I+f+F
Ÿ I+f+F is an even integer
Ÿ I+F is an integer

264
Ÿ f+F = 1 (' 0 <f+F<2)
F = 1–f
? (I+F)(1–f) = (2+ 3 )n (2– 3 )n = (4–3)n = 1
Ans (b)
5 If the middle term of (1+x)2n (n  N) is the greatest term of the expansion, then the interval in
which x lies is
ªn 1 n  2º ª n – 1 n  1º ª n n  1º
(a) « n , n » (b) « n , n » (c) «n 1, n » (d) None of these
¬ ¼ ¬ ¼ ¬ ¼
Solution :
Tn d Tn+1 & Tn+1 t Tn+2
Ÿ 2nCn–1 xn–1 d 2nCn.xn & Cn.xn t 2nCn+1xn+1
2n

2n 2n
C n –1 Cn
Ÿ dx & x.
2n
Cn 2n
C n 1 t

n n 1
xt & xd
2n – n  1 2n – n
ª n n  1º
Ÿ x « , »
¬ n 1 n ¼
Ans (c)
6 If C0, C1, C2, ...................Cn are the binomial coefficients in expansion of (1+x)n, n being even, then
C0+(C0+C1)+(C0+C1+C2)+..........+(Co+C1+......+Cn–1) is equal to
(a) n.2n (b) n.2n–1 (c) n.2n–2 (d) n.2n–3
Solution
? C0+(C0+C1)+.........+(C0+C1+.......Cn–2)+(C0+C1+..........Cn–1)
= (Cn)+(Cn+Cn–1)+.......+(C0+C1+.........+Cn–2)+(C0+C1+.....+Cn–1)
n
= 2n+2n+2n+... times (Adding the terms equidistant from the begining and the end)
2
n n
= . 2 = n.2n–1
2
Ans (b)
100
§ 3 1 ·
7 The number of terms in the expansion of ¨ x  3  1¸ is
© x ¹
(a) 201 (b) 200 (c) 300 (d) 100c3
Solution
100 2 100
§ 3 1 · § 3 1 · § 3 1 · § 3 1 ·
¨ x  3  1¸ = C0+C1 ¨ x  ¸
3 +C2 ¨ x  3 ¸ +............+C100 ¨
x  3¸
© x ¹ © x ¹ © x ¹ © x ¹
1 1 1
gives terms of x3, x6, ..............x300, 3 , 6 ,.......... 300 and a constant term
x x x

265
? 201 terms
Ans (a)
Exercise
1 If Cr stands for Cr, then the sum of the series
n

§n· §n·
¨ ¸!¨ ¸!
2 © 2 ¹ © 2 ¹ (C02–2C12+3C22–..........+(–1)n (n+1)Cn2) when n is an even positive integer, is equal
n!
to
(a) (–1)n/2 (n+2) (b) (–1)n (n+1) (c) (–1)n/2 (n+1) (d) None of these
§ 30 · § 30 · § 30 · § 30 · § 30 · § 30 ·
2 ¨¨ ¸¸ ¨¨ ¸¸ – ¨¨ ¸¸ ¨¨ ¸¸ +.........+ ¨¨ ¸¸ ¨¨ ¸¸ is equal to
©0¹ © 10 ¹ ©1¹ © 11 ¹ © 20 ¹ © 30 ¹
(a) 30
C11 (b) 60C10 (c) 30C10 (d) C55
65

3 If r = 0, 1, 2, ........10, let Ar, Br and Cr denote, respectively, the coefficent of xr in the expansions
10

of (1+x)10, (1+x)20 and (1+x)30. Then ¦A


r 1
r (B10 Br – C10Ar) is equal to

(a) B10 – C10 (b) 2 –C A )


A10 ( B10 10 10

(c) 0 (d) C10–B10


§n· §n· §n· § n – 1· §n· § n – 2· §n· §n – k·
4 Value of 2k ¨¨ ¸¸ ¨¨ ¸¸ –2k–1 ¨¨ ¸¸ ¨¨ ¸¸ + 2k–2 ¨¨ ¸¸ ¨¨ ¸¸ – .......+(–1)k ¨¨ ¸¸ ¨¨ ¸¸ is
©0¹ ©k¹ ©1¹ © k – 1¹ © 2¹ © k – 2¹ ©k¹ © 0 ¹
§n· § n – 1·
(a) ¨¨ ¸¸ (b) ¨¨ ¸¸ (c) 1 (d) None of these
©k¹ © k – 1¹
n § 1 3r ·
7 r 15 r
5 ¦ (–1) r
n ¨
Cr ¨ r  2r
 r
 4 r  .........uptomtems) ¸¸ =
r 0 ©2 2 23 2 ¹

2 mn – 1 2m – 2n
(a) (b) (c) 1 (d) None of these
2 mn (2 n – 1) m–n
2n

¦a
2n
6 If
r 0
r (x–2)r = ¦b
r 0
r
(x–3)r and ak = 1 for all k t n, then bn =

(a) nCn (b) 2n+1Cn+1 (c) 2n+1Cn (d) None of these


7 If (1+x) = C0+C1x+C2x +..........+Cnx , then the sum of the products of the Ci’s taken two at a
n 2 n

time represented by ¦¦ Ci C j (0 d i<j d n) is equal to


(2n )! (2n )!
(a) 22n–1 (b) 2n – (c) 22n–1 – (d) None of these
2(n!) 2 2(n!) 2

266
2 n
q  1 § q  1· § q  1·
8 Given sn = 1+q+q +..........+q and Sn = 1+
2 n
+ ¨ ¸ + .........+ ¨ ¸ , q z 1 then
2 © 2 ¹ © 2 ¹
n+1
C1+n+1C2s1+n+1C3s2+.........+n+1Cn+1sn =
Sn
(a) 2n Sn (b) Sn (c) (d) None of these
2n
n
§n· 1
9
lim
nof ¦ ¨¨ r ¸¸ r =
r 0© ¹ (r  3)n
(a) e (b) e–1 (c) e+1 (d) e–2
2
§ x 2 x 4 x 6 x8 ·
10 The coefficient of x is the expansion of ¨¨1 
8    ¸ is
© 2! 4! 6! 8! ¸¹

1 2 1 1
(a) (b) (c) (d)
315 315 105 210
11 Match the following :
Column I Column II
(a) The sum of binomial coefficients of terms containing
power of x more than x20 in (1+x)41 is divisibile by (p) 239
(b) The sum of binomial coefficients of rational terms in
the expansion of (1+ 2 )42 is divisible by (q) 240
21
§ 1 2 1 ·
(c) If ¨ x   x  2 ¸ = a0x–42+a1x–41+a2x–40+........+a82x40, (r) 241
© x x ¹
then a0+a2+.....+a82 is divisible by
(d) The sum of binomial coefficients of positive real terms
in the expansion of (1+ix)42 (x>0) is divisible by (s) 238
12 Read the passage and answer the questions that follow:
An equation a0+a1x+a2x2+.......+a99x99+x100 = 0 has roots 99C0, 99C1 ,99C2,........99C99.
(i) The value of a99 is
(a) 298 (b) 2 99 (c) –299 (d) None of these
(ii) The value of a98 is
2198 –198 C 99 2198 198 C 99
(a) (b) (c) 2 99 – 99 C 49 (d) None of these
2 2
(iii) The value of (99C0)2+(99C1)2+............+(99C99)2 is
2 2 2 2 –2a
(a) 2a 98 – a 99 (b) a 98 – a 99 (c) a 99 98
(d) None of these
Answers
1. a 2. c 3. d 4. a 5. a 6. b
7. c 8. a 9. d 10. a 11. a o p, b o r, c o r,, d o q
12. (i) c (ii) a (iii) c

267
TRIGONOMETRY - I
Trigonometric Functions

1. a. Measurement of angles. There are three systems of measurement of angles.


i. Sexagesimal system
Here 1 right angle = 90° (degrees)
1° = 60' (minutes)
1' = 60" (seconds)
ii. Centrimal system
Here 1 right angle = 100 g (grades)
1g = 100' (minutes)
1' = 100" (seconds)
iii. Circular system. Here an angle is measured in radians. One radian corresponds
to the angle subtended by arc of length ‘r’ at the centre of the circle of radius
r. It is a constant quantity and does not depend upon the radius of the circle.
b. Relation between the three systems:
ʌ c = 180° = 200 g = 2rt ‘s .
c. If ș is the angle subtended at the centre of a circle of radius ‘r’, by an arc of length A
A
then ș.
r
Note that here A , r are in the same units and ș is always in radians.

CONVENTION FOR PERPENDICULAR AND BASE IN A RIGHT TRIANGLE


Side opposite to 90° is called hypotenuse and side opposite to angle ș considered for T--
ratios is known as perpendicular and third remaining side is base.

2. T-RATIOS (or Trigonometrical functions)


p b p
sin ș , cos ș , tan ș ,
h h b
h
cos ecș ,
p

h b
sec ș and cot ș
b p
‘p’ perpendicular; ‘b’ base and ‘h’ stands for hypotenuse.

3. DOMAINS AND RANGES OF TRIGONAL METRICAL FUNCTIONS


Function Domain Range
sinx R [–1,1]
cosx R [–1,1]
tanx R– ^ 2n  1 ʌ / 2 : n  Z` R

268
cotx R– ^nʌ : n  Z` R
secx R– ^ 2n  1 ʌ / 2 : n  Z` – f,–1@ ‰ >1, f
cosecx R– ^nʌ : n  Z` – f,–1@ ‰ >1, f
4. Signs of trigonometrical functions in different quadrants:
i. I quadrant: All t-ratios are positive t
ii. II quadrant : sin and cosec are positive and all others are negative.
iii. III quadrant: tan and cot are positive and all others are negative.
iv. IV quadrant: cos and sec are positive and all others are negative.

5. Values of t-ratios of some standard angles:

6. a. Trigonometric functions of 2 nʌ + ș , n  Z will be same as of ș


i.e., sin (2 nʌ + ș ) = sin ș , cos(2 nʌ + ș ) = cos ș , etc.
b. Trigonometrical functions of – ș , for all values of ș
sin(– ș ) = –sin ș , cos(– ș ) = cos ș ,
tan(– ș ) = –tan ș
cot(– ș ) = –cot ș , sec(– ș ) = sec ș ,
cosec(– ș ) = –cosec ș
c. The values of t-ratios of any angle can be expressed in terms of an angle in first
quadrant.
ʌ ʌ
Let A = n. r ș where n  Z,0 d ș  . Then
2 2
§ ʌ ·
i. sin ¨ n r ș¸ r sin ș if n is even
© 2 ¹
= +cos ș, if n is odd
§ ʌ ·
ii. cos ¨ n r ș¸ r cos ș if n is even
© 2 ¹
= -sin ș , if n is odd
§ ʌ ·
iii. tan ¨ n r ș¸ r tan ș if n is even
© 2 ¹
= r cot ș , if n is odd

269
§ ʌ ·
iv. cot ¨ n r ș¸ r cot ș if n is even
© 2 ¹
= r tan ș , if n is odd
§ ʌ ·
v. sec ¨ n r ș¸ r sec ș if n is even
© 2 ¹
= r cosec ș , if n is odd
§ ʌ ·
vi. cosec ¨ n r ș¸ r cos ecș if n is even
© 2 ¹
= r sec ș , if n is odd
The sign R.H.S. is decided from the quardrant in which A lies.

7. IDENTITIES
1
1. sin ș .cosec ș =1 or cosec ș =
sin ș
1
2. cos ș .sec ș =1 or sec ș =
cos ș
1
3. tan ș .cot ș =1 or cot ș =
tan ș
sin ș
4. tan ș =
cos ș
cos ș
5. cot ș =
sin ș
6. sin2 ș +cos2 ș =1 or sin2 ș =1 – cos2 ș
or cos2 ș =1 – sin2 ș
7. sec2 ș –tan2 ș =1 or sec2 ș =1+ tan2 ș
or tan2 ș = sec2 ș –1
8. cosec2 ș – cot2 ș =1 or cosec2 ș =1+ cot2 ș
or cot2 ș = cosec2 ș –1
9. sin(A+B) = sinA cosB + cosA sinB
10. sin(A–B) = sinA cosB – cosA sinB
11. cos(A+B) = cosA cosB – sinA sinB
12. cos(A–B) = cosA cosB + sinA sinB
tan A  tan B
13. tan(A+B) =
1 – tan A tan B
cot A cot B – 1
14. cot(A+B) =
cot A  cot B

270
tan A – tan B
15. tan(A–B) =
1  tan A tan B
cot A cot B  1
16. cot(A–B) =
cot B – cot A
17. 2sinA cosB = sin(A+B) + sin(A–B)
18. 2cosA sinB = sin(A+B) – sin(A–B)
19. 2cosA cosB = cos(A+B) + cos(A–B)
20. 2sinA sinB = cos(A–B) – cos(A+B)
§ C  D· § C – D·
21. sinC + sinD = 2sin ¨© ¸ cos¨ ¸
2 ¹ © 2 ¹

§ C  D· § C – D·
22. sinC – sinD = 2cos ¨© ¸ sin ¨ ¸
2 ¹ © 2 ¹

§ C  D· § C – D·
23. cosC + cosD = 2cos ¨© ¸ cos¨ ¸
2 ¹ © 2 ¹

§ C  D· § C – D· ,
24. cosC – cosD – 2sin ¨ ¸ sin ¨ ¸
© 2 ¹ © 2 ¹
2 tan ș
25. sin 2ș = 2sin ș cos ș =
1  tan 2 ș
26. cos 2ș = cos2 ș – sin2 ș = 2cos2 ș –1
1 – tan 2 ș
= 1–2sin ș = 2
1  tan 2 ș

1 cos ș
27. 1+ cos 2ș = 2cos2 ș or cos ș =
2

1 – cos 2ș
28. 1– cos 2ș = 2sin2 ș or sin ș =
2

1 – cos 2ș sin 2ș 1 – cos 2ș


29. tan ș = =
sin 2ș 1  cos 2ș 1  cos 2ș
2 tan ș
30. tan 2ș =
1 – tan 2 ș
31. sin 3ș = 3sin ș – 4sin3 ș
32. cos 3ș = 4cos3 ș – 3cos ș

271
3 tan ș – tan 3 ș
33. tan 3ș =
1 – 3 tan 2 ș
34. sin2A – sin2B = sin(A+B).sin(A–B)
= cos2B – cos2A
35. cos2A– sin2B = cos(A+B).cos(A–B)= cos 2 B- sin2A
36. sin(A+B+C) = sinA cosB cosC + sinB cosA cosC
+ sinC cosA cosB – sinA sinB sinC
= cosA cosB cosC [tanA + tanB +tanC – tanA tanB tanC]
37. cos(A+B+C) = cosA cosB cosC – sinA sinB cosC
– sinA cosB sinC – cosA sinB sinC
= cosA cosB cosC [1–tanA tanB – tanB tanC – tanC tanA]
tan A  tan B  tan C – tan A tan B tan C
38. tan(A+B+C) =
1 – tan A tan B – tan B tan C – tan C tan A
39.
sin Į + sin Į  ȕ + sin Į  2ȕ +.......sin Į  n – 1ȕ
­ § n – 1· ½ § nȕ ·
sin ®Į  ¨ ¸ ȕ ¾ sin¨ ¸
¯ © 2 ¹ ¿ © 2¹
=
§ ȕ·
sin¨ ¸
© 2¹

40.
cos Į + cos Į  ȕ +cos Į  2ȕ +.......cos Į  n – 1ȕ
­ § n – 1· ½ § nȕ ·
cos®Į  ¨ ¸ ȕ¾ sin¨ ¸
¯ © 2 ¹ ¿ © 2¹
=
§ ȕ·
sin¨ ¸
© 2¹

0 0 0
1 1 1
8. Some t-ration of 18 0,720,360,540,150, 22 ,67 ,7 , 90,810,270,630 etc.
2 2 2
5 –1
1. sin180 = cos 72 0
4
5 1
2. cos360 = sin 540
4

10  2 5
3. cos180 = sin 720
4

10 – 2 5
4. sin360 = cos 540
4

272
3 1 0
5. sin75 = 2 2 cos15
0

3 1
6. tan750 = cot 150
3 –1
3 –1
7. sin150 = 2 2 cos 750

3 –1
8. tan150 = 2– 3 = cot 750
3 1

§ 1 0· § 1 0·
9. tan ¨ 22 ¸ 2 – 1 cot ¨ 67 ¸
© 2 ¹ © 2 ¹

§ 1 0· § 1 0·
10. tan ¨ 67 ¸ 2  1 cot ¨ 22 ¸
© 2 ¹ © 2 ¹

§ 1 0·
11. cot ¨ 7 ¸
© 2 ¹
6 4 3 2= 3 2 2 1 § 1 0·
tan ¨ 88 ¸
© 2 ¹

§ 1 0·
12. tan ¨ 7 ¸
© 2 ¹
6– 4 – 3 2 = 3– 2
2 –1
§ 1 0·
cot ¨ 82 ¸
© 2 ¹

3 5  5 – 5
13. sin90 = cos 810
4

3 5  5 – 5
14. cos90 = cos 810
4

5 5 – 3– 5
15. sin270 = cos 630
4

5 5  3– 5
16. sin630 = cos 27 0
4
17. Į cos ș  b sin ș will always lie in the interval {– a 2  b 2 , a 2  b 2 }, i.e. the maximum

and minimum value of a cos ș  b sin ș is a 2  b 2 – a 2  b 2 repectively..


18. For 0 < ș < ʌ , minimum value of a sin ș  b cos ecș is 2 ab
19. For – ʌ / 2  ș  ʌ / 2 , minumum value of a cos ș +b sec ș is 2 ab

273

20. For 0  ș  ʌ/2 or ʌ  ș  , Minimum value of a tan ș + b cot ș is 2 ab
2
21. Periods of sinx, cosx, secx, cosecx, is 2ʌ and period of tanx and cotx is ʌ .
22. If a function is periodic with period Į , i.e. f(x+ Į ) = f(x), then period of the
function
§p · q
f ¨ x¸ is Į
©q ¹ p
23. If f(x) and g(x) are periodic functions with periods Į and ȕ repectively, then period of
f (x )
the function f(x) r g(x), f(x).g(x) or is L.C.M of Į and ȕ
g( x )
LCM of numerators
24. LCM of rational numbers is
HCF of denominatiors
25. Expression of sin(A/2) in terms of sinA
2
§ A A·
¨ sin  cos ¸ 1  sin A
© 2 2¹
2
§ A A·
and ¨ sin – cos ¸ 1 – sin A
© 2 2¹
A A
so that sin  cos r 1  sin A ......(1)
2 2
A A
and sin – cos r 1 – sin A ......(2)
2 2
By addition and subtraction, we have
A
2 sin r 1  sin A r 1 – sin A ......(3)
2
A
and 2 cos r 1  sin A B 1 – sin A ......(4)
2
The ambiguities of sign in relation (1) and (2) is determined by the following
diagram.

274
Y

sin(A/2) + cos(A/2) +
S Q
sin(A/2) – cos(A/2) +
S/4< A/2<3S/4

sin(A/2)+ cos(A/2) – sin(A/2) + cos(A/2) –

X X

sin(A/2) – cos(A/2) + sin(A/2) – cos(A/2) –


3S/4<A/2<5S/4 –S/4<A/2<S/4

sin(A/2) + cos(A/2)+
P sin(A/2) – cos(A/2) – R
5S/4<A/2 <7S/4
V

1. i. If sin ș1 + sin ș 2 +............ sin ș n = n, then


sin ș1 = sin ș 2 =............= sin ș n =1
ii. If cos ș1 + cos ș 2 +........... cos ș n = n, then
cos ș1 = cos ș 2 =........... cos ș n = 1
2. i. sin ș + cos ecș = 2 Ÿ sin ș =1
ii. cos ș + sec ș = 1 Ÿ cos ș =1
3
3. i. si n150 + cos15 0 = sin75 0 + cos75 0 =
2
1
ii. cos15 – sin15 = sin75 – cos75 =
2
1
4. i. tan 150 + cot 150 = tan 750 + cot 750 =
2
ii. cot 150 – tan 150 = tan 750 – cot 750 = 2 3
5. i. 0

cos ș – cos 60  ș – cos 60 – ș = 0
0

cos ș + cos 120  ș + cos 120 – ș = 0
0 0
ii.
cos ș + cos 240  ș + cos 240 – ș = 0
0 0
iii.
sin ș – sin 60  ș + sin 60 – ș = 0
0 0
iv.
sin ș + sin 120  ș – sin 120 – ș = 0
0 0
v.
sin ș + sin 240  ș – sin 240 – ș = 0
0 0
vi.

275
6. i.
tan 450  ș tan 450 – ș =1
ii. cot 450
 ș cot 45 0
– ș =1
7. i. If A+B = 450 then (1+ tanA) (1+ tanB) = 2
ii. If A+B = 1350 then (1– tanA) (1– tanB) = 2
iii. If A+B = 450 then (1– cotA) (1– cotB) = 2
iv. If A+B = 135 0 then (1+ cotA) (1+ cotB) = 2
xy b
8. i. If cos x + cos y = a, sin x + sin y = b, then tan
2 a
xy –a
ii. If cos x – cos y = a, sin x – sin y = b, then tan
2 b
x–y a
iii. If cos x – cos y = a, sin x + sin y = b, then tan
2 b
x–y b
iv. If cos x + cos y = a, sin x – sin y = b, then tan
2 a

9. i. 0 0

1
sin ș sin 60  ș sin 60 – ș = sin 3ș
4

ii. 1
sin ș sin 120  ș sin 120 – ș = sin 3ș
0 0
4

10. i. 0
0

1
cos ș cos 60  ș cos 60 – ș = cos 3ș
4

ii. 1

cos ș cos 120  ș cos 120 – ș = cos 3ș
0 0
4
tan ș tan 60  ș tan 60 – ș = tan 3ș
0 0
11. i.
tan ș tan 120  ș tan 120 – ș = tan 3ș
0 0
ii.
cot ș cot 60  ș cot 60 – ș = cot 3ș
0 0
12. i.
cot ș cot 120  ș cot 120 – ș = cot 3ș
0 0
ii.

PRACTICE QUESTIONS
1
1 If cot D +tan D = m and – cos D = n, then
cos D
(a) m(mn2)1/3–n(nm2)1/3 = 1 (b) m(m2n)1/3–n(mn2)1/3 = 1
(c) n(mn ) –m(nm ) = 1
2 1/3 2 1/3
(d) n(m2n)1/3–m(mn2)1/3 = 1
Solution
Given that
cot D +tan D = m
1+tan2 D = mtan D
sec2 D = mtan D

276
1 – cos 2 D
also =n
cos D
sin 2 D
=n
cos D
tan2 D = n sec D
squaring
tan4 D = n2 sec2 D
= n2m tan D
tan3 D = n2m
tan D = (n2m)1/3
sec2 D = m(n2m)1/3
sec D –tan2 D = 1
2

m(n2m)1/3–(n2m)2/3 = 1
m(n2m)1/3–(n4m2)1/3 = 1
m(n2m)1/3–n(nm2)1/3 = 1
Correct option is ‘a’
3 cos 2E – 1 tan D
2 If cos2 D = , then equals
3 – cos 2E tan E
(a) 1 (b) –1 (c) 2 (d) – 2
Solution
3 cos 2E – 1
cos2 D =
3 – cos 2E

3(1 – tan 2 E)
–1
1 – tan 2 D 1  tan 2 E
=
1  tan 2 D 1 – tan 2 E
3–
1  tan 2 E

3 – 3 tan 2 ȕ – tan 2 ȕ – 1
=
3  3 tan 2 ȕ – 1  tan 2 ȕ

2 – 4 tan 2 E
=
2  4 tan 2 E

1 – tan 2 D 1 – 2 tan E
2

=
1  tan 2 D 1  2 tan E
2

By componendo and dividendo we get,


2 2
=
– 2 tan D
2
– 4 tan 2 E

277
tan 2 D
2=
tan 2 E
tan D
= r 2
tan E
correct options are ‘c’ & ‘d’
3 cot15°+cot75°+cot135°–cose30° is equal to
(a) –1 (b) 0 (c) 1 (d) None of these
Solution
cot15°+cot75°+cot135°–cosec30°
= cot15°+tan15°–tan45°–cosec30° [cot15°+tan15° = cos15° + sin 15°
2 sin 15° cos15°
= –1–2 2
sin 30q =
sin 15° cos15°
=4–3 2
=1 = sin 30q ]
correct option is c
4 If sin(y+z–x), sin(z+x–y), and sin(x+y–z) are in A.P then tanx, tany and tanz are in
(a) A.P (b) GP (c) HP (d) None of these
Solution
sin(y+z–x), sin(z+x–y) and sin(x+y–z) are in AP
? sin(z+x–y)–sin(y+z–x) = sin(x+y–z)–sin(z+x–y)
2coszsin(x–y) = 2cosxsin(y–z)
sinx cosz cosy–cosx siny cosz = cosx siny cosz–cosx cosysinz
Divide by cosx cosy cosz we get
tanx–tany = tany–tanz
tanx+tany = 2tany
? tanx,tany,tanz are in A.P
option a is correct
5 If D + E = 90, then the maximum value of sin D sin E is
(a) 1 (b) 1/2 (c) 3/2 (d) None of these
Solution
D + E = 90
sin D sin E = sin D sin(90– D )
= sin D cos D
2
= sin D cos D
2
sin 2D
=
2
– 1 sin 2D 1
We knew that – 1 d sin2 D d 1 or
2 d 2 d2

278
1
? maximum value of sin D sin E =
2
correct option is ‘b’
EXERCISE
1. The least value of secA+secB+secC in an acute angle triangle is
a. 3 b. 6 c. 2 d. none of these
2. The sumof maximum and minimum values of cos2T–6sinTcosT+3sin2T+2 is

a. 2 10 b.  c.  d. none of these
3. Let f(T)=sinT(sinT+sin3T). Then f(T) is
a. t 0 only when Tt 0 b. d 0 only real T
c. t 0 for all real T d. d 0 only when Td 0

S
4. Let TH (0, ) and t1=(tanT)tanT t2=(tanT)cotT t3=(cotT)tanTand t4=(cotT)cotT then
4
a. t 1> t 2 > t 3 > t 4 b. t 4> t 3 > t 1 > t 2 c. t 3> t 1 > t 2 > t 4 d. t 2> t 3 > t 1 > t 2
5. For a positive integer n, let

§T·
fn(T)=tan ¨ 2 ¸ (1+secT) (1+sec2T) (1+sec4T)......(1+sec2nT). Then
© ¹

§S· § S· §S· § S ·
a. f2 ¨ 16 ¸ =1 b. f3 ¨ 32 ¸ =1 c. f4 ¨ 64 ¸ =1 d. f5 ¨ 128 ¸ =1
© ¹ © ¹ © ¹ © ¹
6. The maximum value of (cosD1)(cosD2) (cosD3)........(cosDn) under the restrictions
S
0 dD1,D2 ...Dn d and (cotD1), (cotD2)......(cotDn)=1 is
2
1 1 1
a. n/2 b. n c. d. 1
2 2 2n
7. If cos4T+D, sin4T+Dare the roots of the equaton x 2+b(2x+1)=0 and cos2T+E,sin2T E are the
roots of the equation x 2+4x+2=0, then b is equal to
a. 1 b. –1 c. 2 d. –2
8. If in 'ABC, tanA+tanB+tanC=6 and tanAtanB=2, then sin A: sin B:sin C is
2 2 2

a. 8:9:5 b. 8:5:9 c. 5:9:8 d. 5:8:9


tan3A
9. If k(k z 1)
cosA

279
cos A k2 –1 sin 3A 2k 1
a. b. c. k d. k>3
cos 3A 2k sin A k –1 3
T I
10. If (x–a) cos T + y sin T = (x–a) cos I + y sin I = a and tan – tan = 2b, then
2 2
T 1
a. y2 = 2ax – (1– b2)x 2 b. tan = (y + bx)
2 x
I 1
c. y2 = 2ax – (1– a2)x 2 d. tan = (y – bx)
2 x
11. Passage
Increasing product with angles are in GP

­ sin2 n Į ½
° n if Į z nʌ °
° 2 sinĮ °
° 1 ʌ °
cos D cos2 D cos22 D .......cos2n D = ® n , if Į n  I¾
°2 2 1
n
°
° –1 ʌ °
° 2 n , if Į 2 n – 1 °
¯ ¿

On the basis of above infromation, answer the following questions.


2ʌ 4ʌ 6ʌ
i. The value of cos cos cos is
7 7 7
–1 1 1 1
a. b. c. d.
2 2 4 8

S 6

ii. If D =
13
then the value of – cos rD is
r 1

1 –1 1 1
a. b. c. d. –
64 64 32 8
S 3S 5S 7S 9S 11S 13S
iii. The value of sin sin sin sin sin sin sin is
14 14 14 14 14 14 14
1 1 1
a. 1 b. c. d.
8 32 64
S 5S 7S
iv. The value of sin sin sin is
18 18 18

280
1 1 1
a. b. c. d. –1
16 8 8
S S S S S
v. The value of 64 3 sin cos cos cos cos is
48 48 24 12 6
a. 8 b. 6 c. 4 d. –1
12. Matrix Match Type
Column I Column II
(a) In triangle ABC, 3sinA+4cosB=6 and
3cosA+4sinB = 1 then ‘ c can be (p) 60°
(b) In any triangle if(sinA+sinB+sinC)(sinA+sinB–sinC)
= 3sinAsinB then the angle c (q) 30°
(c) If 8sinxcos5x–8sin5xcosx = 1 then x = (r) 165°
(d) ‘O’ is the centre of the inscribed circle in a 30°–60°–90°
triangle ABC with right angled at c. If the circle is tangents
to A B at D then the angl e ‘ COD is (s) 75°
aoq ; bop ; cos ; dor
13. Assertion and Reason type questions
A : Both A and R individually true and R is the correct explanation of A
B : Both A and R individually true and R is not the correct explanation of A
C : A is true but R is false
D : A is false but R is true
If A+B+C = S , then
(i) Assertion (A) : cos2A+cos2B+cos2C has its minimum value 3/4
Reason (R) : Maximum value of cosAcosBcosC is 1/8
(a) A (b) B (c) C (d) D
(ii) Assertion (A) : sin S /18 is a roots of 8x –6x+1 = 0
3

Reason (R) : For any T  R, sin3 T = 3sin T –4sin3 T


(a) A (b) B (c) C (d) D
(iii) In any ' ABC

§ A B C· A B C
Assertion (A) : A n ¨ cot  cot  cot ¸ = An cot  An cot  An cot
© 2 2 2¹ 2 2 2


Reason (R) : A n 1  3  2  3 = A n1+ A n 3 + A n 2  3
Answers
1. b 2. c 3. c 4. b 5. a, b, c, d 6. a 7. b, c 8. b, d 9. a, b, c, d 10. a, b, d 11. (i) d
(ii) a (iii). d(iv). b (v). b 12.a o q ; b o p ; c o s ; d o r 13. (i) a (ii) a (iii) b

281
TRIGONOMETRY - II
Trigonometric Functions
1 sin47° + sin61°–sin11°–sin25° is equal to
(a) sin36° (b) cos36° (c) sin7° (d) cos7°
Solution
sin47+sin61°–sin11°–sin25°
(2sin54°cos7°) – (2sin18°cos7°)
= 2cos7°(sin54°–sin18°)
= 2cos7°(cos36°–sin18°)
§ 5 1 § 5 –1··
¨ ¸
= 2cos7° ¨ 4 – ¨¨ 4 ¸¸ ¸
© © ¹¹

§1·
= 2cos7° ¨ ¸
©2¹
= cos 7°
correct option is ‘d’
x y z
2 If = = , then x+y+z is equal to
cos T § 2S · § 2S ·
cos¨ T – ¸ cos¨ T  ¸
© 3 ¹ © 3 ¹
(a) 1 (b) 0 (c) –1 (d) none of these
Solution
We have
x y z
= =
cos T § 2S · § 2S ·
cos¨ T – ¸ cos¨ T  ¸
© 3 ¹ © 3 ¹

sum of Numerator
we know that each ratio is equal to
sum of denominator
x y z xyz
? = = =
cos T § 2S · § 2S · § 2S · § 2S ·
cos¨ T – ¸ cos¨ T  ¸ cos T  cos¨ T – ¸  cos¨ T  ¸
© 3 ¹ © 3 ¹ © 3 ¹ © 3 ¹

§ 2S · § 2S ·
now, cos T  cos¨ T – ¸  cos¨ T  ¸
© 3 ¹ © 3 ¹
2S
=cos T +2cos T cos (cos(A+B)+cos(A–B)=2cosAcosB)
3
§ 1·
= cos T +2cos T ¨ – ¸
© 2¹

282
=0
? x+y+z = 0
correct option is ‘b’
§ S· § 3S · § 5S · § 7S ·
3(a) The value of ¨1  cos ¸ ¨1  cos ¸ ¨1  cos ¸ ¨1  cos ¸ is
© 8¹ © 8 ¹© 8 ¹© 8 ¹

1 S 1 1 2
(a) (b) cos (c) (d)
2 8 8 2 2
Solution
7ʌ § S· S
cos = cos ¨ S – ¸ = – cos
8 © 8 ¹ 8

3S §S S· S
cos = cos ¨ – ¸ = sin
8 ©2 8¹ 8
5S § ʌ ʌ· S
cos = cos ¨©  ¸¹ = – sin
8 2 8 8
§ S· § S· § ʌ· § S·
= ¨1  cos ¸ ¨1  sin ¸ ¨©1 – sin ¸¹ ¨1 – cos ¸
© 8¹ © 8¹ 8 © 8¹

§ 2 S· § 2 S·
= ¨1 – cos ¸ ¨1 – sin ¸
© 8¹ © 8¹
S S
= sin2 cos2
8 8
2 2
§ S S· § S·
¨ 2 sin cos ¸ ¨ sin ¸ 1
= © 8 8¹ = © 4¹ =
8
4 4
correct option is ‘c’
3 If x 1 ,x 2 ...x n are in AP whose common difference is D , then the value of
sin D (secx1secx2+secx2secx3+.......+secxn–1secxn) is
sin( n – 1)D sin nD
(a) cos x1 cos x n (b) cos x1 cos x n
(c) sin(n–1) D cosx1cosxn (d) sinn D cosx1cosxn
Solution
We have
sin D secx1secx2+sin D secx2secx3+............+sin D secxn–1secxn
sin( x 2 – x1 ) sin( x 3 – x 2 ) sin( x n – x n –1 )
= cos x cos x + cos x cos x +............+ cos x cos x
1 2 2 3 n –1 n

283
sin x 2 cos x1 – cos x 2 sin x1 sin x 3 cos x 2 – cos x 3 sin x 2
= cos x 1 cos x 2 + cos x 2 cos x 3 +.............

sin x n cos x n –1 – cos x n sin x n –1


+ cos x n –1 cos x n
= tanx2–tanx1+tanx3–tanx2+........+tanxn–tanxn–1
= tanxn–tanx1
sin x n cos x1 – cos x n sin x1
= cos x n cos x1
sin( x n – x 1 )
= cos x cos x
n 1

sin( n – 1)D
= cos x cos x (' xn = x1+(n–1) D xn–x1 = (n–1) D )
n 1

? correct option is ‘a’

4 Let ƒ(n) = 2cosnx  n  N, then ƒ(1) ƒ(n+1) –ƒ(n) is equal to


(a) f(n+3) (b) f(n+2) (c) f(n+1)f(2) (d) f(n+2)f(2)
Solution
We have f(n) = 2cosnx  n  N
f(1) = 2cosx
f(n+1) = 2cos(n+1)x
f(1) f(n+1) = 4cosx cos(n+1)x
f(1)f(n+1) – f(n) = 4cosx cos(n+1)x–2cosnx
= 2[2cosxcos(n+1)x–cosnx]
= 2[cos(n+2)x+cosnx–cosnx]
= 2cos(n+2)x
= f(n+2)
option ‘b’ is correct
5 The ratio of the greatest value of 2–cosx+sin2x to its least value is
(a) 1/4 (b) 9/4 (c) 13/4 (d) None of these

Solution
2–cosx+sin2x
= 2–cosx+1–cos2x
= –(cos2x+cosx)+3
2
§ 1· 1
= – ¨ cos x  ¸ + +3
© 2¹ 4
2
§ 1 · 13
= – ¨ cos x  ¸ +
© 2¹ 4

284
2
13 § 1·
= – ¨ cos x  ¸
4 © 2¹
13 1
Maximum value occurs at cosx = –
4 2
Minimum value occurs at cosx = 1
2
13 § 1 ·
? – ¨1  ¸
4 © 2¹
13 9 4
– = =1
4 4 4
13
? The ratio of greatest to the least is
4
option ‘c’ is correct
6 If sin(120– D ) = sin(120– E ), 0< D , E < S , then find the relation between D and E .
Solution
If sinA = sinB
Then A = B or A = S –B
Here, sin(120–° D ) = sin(120–° E)
? 120– ° D = 120–° E or 120–° D = S –(120– °E)
D = E or 120– ° D = 60+° E D + E = 60°
sin x – sin z
7 If x,y,z are in AP, then is equal to
cos z – cos x
(a) tany (b) coty (c) siny (d) cosy
Solution
We have x,y,z are in AP
? x+z = 2y
xz §x–z·
2 cos sin ¨ ¸
sin x – sin z 2 © 2 ¹
Here, =
cos z – cos x §xz· §x–z·
2 sin ¨ ¸ sin ¨ ¸
© 2 ¹ © 2 ¹

285
cos y
=
sin y
= coty
correct option is ‘b’

PRACTICE QUESTIONS
a cos I  b
1. It cos T , then tan T 2 equals
a  b cos I

ab I ab I ab I


a. tan b. cos c. sin d. none
ab 2 ab 2 ab 2
2. If acos2T + bsin2T = c has D and E as its solution, then tanD + tanE equals

2a 2b 2c
a. b. c. d. none
bc ca ab

1 3
3. The value of  equals
sin 10 cos 10 0
0

a. 1 b. 4 c. 2 d. 0

S 3S 5S 7S
4. cos 4  cos 4  cos 4  cos 4 equals
8 8 8 8

1 3
a. b. 1 c. d. 2
2 2

S 3S 5S
5. sin  sin  sin  ............ n terms equals
n n n

n
a. 1 b. 0 c. d. none
2

1  sin 4T  1
6. If x then one of the values of x is
1  sin 4T  1

a. – tanT b. cotT
c. tan S 4  T
d.  cot S 4  T
7. The value of cos12 0 + cos840 + cos1560 + cos1320 is

1 1 1
a. b. 1 c. – d.
2 2 8

286
8. sin60 – sin660 + sin780 – sin 420 is

1 1
a. –1 b. – c. d. 1
2 2

S 2S 4S 8S 16S 32S
9. cos cos cos cos cos cos equals
65 65 65 65 65 65

1 1 1 1
a. b. c. d.
8 16 32 64
10. sin360 sin720 sin1080 sin1440 equals

5 3 1
a. b. c. d. none
16 16 16
11. Passage
If A, B, C be the angles of a triangle, then

a) ¦ sin2A = 4sinA sinB sinC

b) ¦ cos2A = 1 – 4cosA cosB cosC

c) ¦ sinA = 4 cos A cos B cos C


2 2 2

d) ¦ cosA = –1 + 4 sin A sin B sin C


2 2 2

e) ¦ tanA = tanA tanB tanC ie., S 1 = S3

f) ¦ tan A tan B = 1 or ¦ cot A = cot A cot B cot C


2 2 2 2 2 2
Answer the following questions based upon above passage.

sin A  sin B  sin C


i) In a triangle ABC equals
sin A  sin B  sin C

a. tan A 2 cot B 2 b. cot A 2 tan B 2 c. cot A 2 cot B 2 d. tan A 2 tan B 2

ii) sin2A + sin2B + sin2C – 2cosAcosBcosC equals


a. 1 b. 2 c. 3 d. 4
iii) I n a 'ABC, whose angles are acute and positive such that A+B+C= S and

cot A cot B cot C = K then


2 2 2

a. K < 3 b. K < 3 3 c. K > 3 3 d. none

287
iv) tanA, tanB, tanC are the roots of the cubic equation x3 – 7x2+11x –7 = 0 then A + B + C equals
S
a. b. S c. 0 d. none
2
12. Matchning type question :-
Column I Column II
A B C
a) sin(B+C–A) + sin(C+A–B) + p. 1– 2 cos cos cos
sin(A+B–C)= 2 2 2

A B C
b) sin 2  sin 2  sin 2 = q. 1
2 2 2

A B C
c) If tan
2
 tan 2  tan 2 = K r. 4sinA sinB sinC
2 2 2
then K >
Assertion and reason type questions
A : Both A and R are individually true and R is the correct explaination of A
B : Both A and R are individually true and R is not the correct explaination of A
C : A is true but R is false
D : A is false but R is true
13. Assertion (A) : tanT + 2tan2T + 4tan4T + 8tan8T + 16cot16T = cotT
Reason (R) : cotT – tanT = 2cot2T
a. A b. B c. C d. D
4xy
Assertion (A) : sec T
2
14. Ÿx = y
( x  y) 2
Reason (R) : secT > 1
a. A b. B c. C d. D
15. Assertion (A) : If A, B, C, D be the angles of a cyclic quadrilateral then
cosA + cosB + cosC + cosD = 0
Reason (R) : sinA + sinB + sinC + sinD = 0
a. A b. B c. C d. D
Answers :-
1. a 2. b, c 3. b 4. c 5. b
6. a,b,c,d 7. c 8. b 9. d 10. a
11. i-c, ii-b, iii-c, iv-b 12. a-r, b-p, c-q 13. a
14. a 15. c

288
TRIGONOMETRY - III
Trigonometric Functions - Problem Solving
Trigonometrical ratios and Identities

1 An angle is positive, if it is measured in anti clockwise direction and is negative if it is measured


in clock wise direction
lengthof thearc
2 Angle in radian =
Radius of the circle
A
i.e; T =
r
3 System of Measurement of Angles

Sexagesimal system (D) Centesimal system (G) Circular system (C)


S
1 right angle = 90° (90 degrees) 1 right angle = 100g (100 grades) 1 right angle = radians
2
1° = 601 (60 minutes) 1g = 1001 (100 minutes) 180° = S
11 = 6011(60seconds) 11 = 10011(100 seconds)
D G 2C
Note : = = OR (see the graph below)
90 100 S
4 Basic trigonometrical identities
(i) sin2 T + cos2 T = 1 Sc

Ÿ |sin T | d 1 and |cos T | d 1


Ÿ – 1 d sin T d 1 and – 1 d cos T d 1
(ii) sec T – tan2 T = 1
2

Ÿ |sec T | t 1
Two right
Ÿ sec T d –1 or sec T t 1 angles

tan T may take any real value


(iii) cosec2 T –cot2 T = 1 180° 200g
Ÿ |cosec T | t 1
Ÿ cosec T d –1 or cosec T t 1
cot T may take any real value.
5 Sign of Trigonometrical Ratio : To find the sign of a trigonometrical ratio, remember the sentence
Add Sugar To Coffee
A S T C
1 quadrant
st
2nd quadrant 3rd quadrant 4th quadrant.
Where A stands for all ratios are positive in 1st quadrant
S stands for sin & its reciprocal are positive in 2nd quadrant
T stands for tan & its reciprocal are positive in 3rd quadrant
C stands for cos & its reciprocal are positive in 4rd quadrant

289
6 Domain & Range
Function Domain Range
sin T R [–1,1]
cos T R [–1,1]
­ S ½
tan T R– ®(2n  1) ,n  Z¾ R
¯ 2 ¿
cot T R– ^nS,n  Z` R
cosec T R– ^nS,n  Z` R–(–1,1)

­ S ½
sec T R– ®(2n  1) ,n  Z¾ R–(–1,1)
¯ 2 ¿

7 Trigonometric ratios in terms of each of the other

sin T cos T tan T


sin T
sin T sin T 1 – sin 2 T 1 – sin 2 T

1 – cos2 T
cos T 1 – sin T 2 cos T
cos T
tan T 1
tan T tan T
1  tan T 2
1  tan 2 T

1 cos ec 2 T – 1 1
cosec T
cos ecT cos ecT cos ec 2 T – 1

sec 2 T – 1 1
sec T sec 2 T – 1
sec T sec T
1 cot T 1
cot T
1  cot T 2
1  cot T 2
cot T

8 Sum and Difference formula.


(i) sin (A r B) = sin A cos B r cosA sin B
(ii) cos (A r B) = cos A cos B  sin A sin B
tan A r tan B
(iii) tan (A r B) = 1  tan A tan B


S 1 r tan A
(iv) tan ( r A) = 1  tan A sd
4 

290
cot A cot B1
(v) cot (A r B) =
cot B r cot A
(vi) sin(A+B)sin(A–B) = sin2A – sin2 B = cos2B–cos2A
(vii) cos(A+B)cos(A–B) = cos2A – sin2 B = cos2B–sin2A
9 Multiple and half angles

­2 sin T cos T
°
(i) sin2 T = ® 2 tan T
°̄1  tan 2 T

Also, 1 r sin 2T = |cos T r sin T |

­cos 2 T – sin 2 T
°
°2 cos T – 1
2

°
(ii) cos2 T = ®1 – 2 sin 2 T
°
°1 – tan T
2

¯°1  tan 2 T

Also 1+cos2 T = 2cos2 T


1–cos2 T = 2sin2 T
2 tan T
(iii) tan2 T =
1 – tan 2 T
1 – cos 2T
Also = tan T
sin 2T
1 – cos 2T
= tan2 T
1  cos 2T

­3 sin T – 4 sin 3 T
°
(iv) sin3 T = ® §S · §S ·
°4 sin¨ 3 – T ¸ sin T sin ¨ 3  T ¸
¯ © ¹ © ¹

3 sin T – sin 3T
or sin3 T =
4

­4 cos3 T – 3 cos T
°
(v) cos3 T = ® §S · §S ·
°4 cos¨ 3 – T ¸ cos T cos¨ 3  T ¸
¯ © ¹ © ¹

cos 3T  3 cos T
or cos3 T =
4

291
­ 3 tan T – tan 3 T
° 1 – 3 tan 2 T
°
(vi) tan3 T = ®
°tan §¨ S – T ·¸ tan T tan §¨ S  T ·¸
°¯ © 3 ¹ ©3 ¹
10 Transformation formulae
§CD· §C–D·
(i) sin C + sin D = 2sin ¨ ¸ cos ¨ ¸
© 2 ¹ © 2 ¹
§CD· §C–D·
sin C – sin D = 2cos ¨ ¸ sin ¨ ¸
© 2 ¹ © 2 ¹
§CD· §C–D·
cos C + cos D = 2cos ¨ ¸ cos ¨ ¸
© 2 ¹ © 2 ¹
§CD· §C–D·
cos C – cos D = –2sin ¨ ¸ sin ¨ ¸
© 2 ¹ © 2 ¹
(ii) 2 sin A cos B = sin(A+B) + sin (A–B)
2 cos A sin B = sin(A+B) – sin (A–B)
2 cos A cos B = cos(A+B) + cos (A–B)
2 sin A sin B = cos(A–B) – cos (A+B)
sin( A r B)
(ii) tan A r tan B =
cos A cos B
sin( B r A )
cot A r cot B =
sin A sin B
(iv) cot A – tan A = 2cot2A
1
cotA+tan A = 2cosec2A =
sin A cos A
cos( AB)
(v) 1 r tan A tan B =
FRV$ FRV%
§S · §S ·
(vi) cosA r sinA = 2 sin ¨ r A ¸ = 2 cos ¨© 4 A ¸¹
©4 ¹
11 Three angles
sin(A+B+C) = cosA cosB cosC (tanA+tanB+tanC–tanAtanBtanC)
cos(A+B+C) = cosAcosBcosC(1–tanAtanB–tanBtanC–tanCtanA)
tan A  tan B  tan C – tan A tan B tan C
tan(A+B+C) =
1 – tan A tan B – tan B tan C – tan C tan A

292
12 Trigonometrical series

§ (n – 1)d · § nd ·
sin ¨ A  ¸ sin ¨ ¸
© 2 ¹ © 2 ¹
(i) sinA+sin(A+d)+sin(A+2d)+...............+sin(A+(n–1)d) =
§d·
sin ¨ ¸
©2¹

§ (n – 1)d · § nd ·
cos¨ A  ¸ sin ¨ ¸
© 2 ¹ © 2 ¹
(ii) cosA+cos(A+d)+cos(A+2d)+..........+cos(A+(n–1)d) =
§d·
sin ¨ ¸
©2¹

sin(2 n A)
(iii) cosA. cos2A. cos2 A...........................cos 2 A = n
2 n–1
2 sin A
2 cos 2 n T  1
(iv) (2cos T –1) (2cos2 T –1) (2cos22 T –1)..........(2cos2n–1 T –1) = ; n N
2 cos T  1
(v) tan T +2tan2 T +22tan22 T +23tan23 T +............+2n tan2n T +2n+1cot2n+1 T = cot T ;  n  N

(vi) 2  2  2  ........  2  2 cos 2 n T = 2 cos T , n  N where there are n square root signs
on left hand side.
13 Greatest and least value of asin T r bcos T is a 2  b 2 and – a 2  b 2 respectively

i.e. –a 2  b 2 d asin T r bcos T d a 2  b 2


Also sin2 T + cosec2 T t 2
cos2 T + sec2 T t 2
tan2 T + cot2 T t 2
14 Method of componendo and dividendo.
p a p–q a –b pq ab
If = then by componendo and dividendo we can write = or =
q b pq ab p–q a –b
Periodicity
All the six trigonometric functions are periodic sin T , cos T , cosec T , sec T are periodic with
period 2 S where tan T and cot T are period c with period S

293
Solved Examples
1 If ƒ( T )= sin4 T +cos4 T +1, then the range of ƒ( T ) is
ª3 º ª 3º
(a) « 2 ,2 » (b) «1, 2 » (c) [1 ,2] (d) None of these
¬ ¼ ¬ ¼
Solution :
ƒ( T ) = (sin2 T +cos2 T )2 –2sin2 T cos2 T +1
sin 2 2T
=2 –
2
1 3 1
? ƒ( T ) min = 2 – (1) = and ƒ( T ) max = 2– (0) = 2
2 2 2
Ans : (a)
2 If 4n D = S , then the value of
cot D .cot2 D .cot3 D ..............cot(2n–1) D is
(a) 1 (b) –1 (c) f (d) None of these
Solution :
§S ·
cot D .cot(2n–1) D =cot D .cot(2n D – D )=cot D .cot ¨ – D ¸ =cot D .tan D =1
©2 ¹
Product of terms equidistant from the beginning and end is 1
S
The middle term is cotn D = cot =1
4
Given expression = 1.1.1......................n times = 1
Ans : (a)
D E
3 If tan and tan are roots of the equation 8x2–26x+15 = 0 then cos( D + E ) =
2 2
627 627
(a) – (b) (c) 1 (d) None of these
725 725
Solution :
D E 26
tan  tan
§D E· 2 2 8 – 26
tan ¨  ¸ = = =
© 2 2 ¹ 1 – tan D tan E 1 – 15 7
2 2 8

§ DE· § 676 ·
1 – tan 2 ¨ ¸ 1– ¨ ¸
© 2 ¹ © 49 ¹ = – 627
? cos( D + E ) = =
§ DE· § 676 · 725
1  tan 2 ¨ ¸ 1 ¨ ¸
© 2 ¹ © 49 ¹
Ans (a)

294
y
4 If cos(x–y), cosx, cos(x+y) are in H.P, then cos x. sec equals
2
(a) 1 (b) 2 (c) 2 (d) None of these
Solution :
2 cos( x – y) cos( x  y)
cosx =
cos( x  y)  cos( x – y)

2(cos2 x – sin 2 y)
Ÿ cosx =
2 cos x cos y
2cos2x cosy = 2cos2x–2sin2y
2sin2y = 2cos2x(1–cosy)
y y y
2(4sin2 cos2 ) =2cos2x. (2sin2 )
2 2 2

cos 2 x y
Ÿ = 2 Ÿ cosx.sec = r 2
y 2
cos 2
2
Ans (c)
S 15S S 7S
5 If tan , x, tan are in A.P and tan , y, tan are in A.P, then
9 18 9 18
(a) 2x = y (b) x = y (c) x = 2y (d) None of these
Solution :
2x = tan20° +tan50° 2y = tan20°+tan70°
sin 20q. cos 50q  cos 20q. sin 50q sin 20q. cos 70q  cos 20q. sin 70q
2x = 2y =
cos 20q. cos 50q cos 20q. cos 70q
sin 70q sin 90q
2x = 2y =
cos 50q. cos 20q sin 20q. cos 20q
cos 20q
2x =
cos 50q. cos 20q
1 1 2
Ÿ 2x = = .......................(1) 2y = .....................(2)
cos 50q sin 40q sin 40q
From (1) and (2)
2x = y
Ans : (a)
S 3S 5S
6 The value of sin + sin + sin +..............n terms is
n n n
S
(a) 1 (b) 0 (c) (d) None of these
2

295
Solution :

­ 2.S .2S ½
 (n – 1)
nS ° n n °
sin sin ® ¾
2n ° 2 S §S S·
° sin sin ¨  S – ¸
¯ ¿ n ©n n¹
Given Series = =
S S
sin sin
2n 2n

S
sin . sin S
= n =0
S
sin
2n
Ans : (b)
n –1
rS
7 ¦ cos
r 1
2

n
=

n n 1 n
(a) (b) – (c) – 1 (d) None of these
2 2 2 2
Solution : (c)
n –1
rS n —1
(1  cos 2rS/n) 1 n –1
1 n –1
cos 2rS
¦ cos 2
r 1 n
= ¦r 1 2
=
2
¦1 +
r 1 2
¦
r 1
( )
n

n –1 1 § 2S 4S (n – 1)S ·
=  ¨ cos  cos  ..........  cos 2 ¸
2 2© n n n ¹

­ 2S 2(n – 1)S ½
2S °  °
sin cos ® n n
¾
n –1 1 2n ° 2 °
=  . ¯ ¿
2 2
S
sin
n

S
n – 1 1 sin n . cos S n –1 1 n
=  . = – = –1
2 2 S 2 2 2
sin
n
Ans : (c)
PRACTICE QUESTIONS
§ S·
1 Let T  ¨ 0, ¸ and t1 = (tan T )tan T , t2 = (tan T )cot T , t3 = (cot T )tan T and t4 = (cot T )cot T , then
© 4¹
(a) t1>t2>t3>t4 (b) t4>t3>t1>t2 (c) t3>t1>t2>t4 (d) t2>t3>t1>t4

296
§ T·
2* For a positive integer n, let ƒn( T ) = ¨ tan ¸
© 2¹
(1+sec T )(1+sec2 T )(1+sec22 T )...........(1+sec2n T ), then
§S· §S· §S· § S ·
(a) ƒ2 ¨ ¸ =1 (b) ƒ3 ¨ ¸ =1 (c) ƒ4 ¨ ¸ =1 (d) ƒ5 ¨ ¸ =1
© 16 ¹ © 32 ¹ © 64 ¹ © 128 ¹
3 Two rays are drawn through a point A at an angle 30°. A point B is taken on one of them at a distance
‘a’ from the point A. A perpendicular is drawn from the point B to the other ray, and another
perpendicular is drawn from its foot to A B to meet AB at another point from where the similar
process is repeated indefinitely. The length of the resulting infinite polygonal line is
A B3 B1
30°

B4

B2

(a) a(2+ 3 ) (b) a(2– 3 ) (c) a (d) None of these


4 If cos6 D +sin6 D +ksin22 D =1(0< D < S /2), then k is
(a) 3/4 (b) 1/4 (c) 1/3 (d) 1/8
5 In an acute angled triangle ABC, the least value of secA+secB+secC is
(a) 6 (b) 8 (c) 3 (d) none of these
S 1 2S 1 3S
6 The sum to infinite tems of the series cos + cos + cos +................ is
3 2 3 3 3
(a) 0 (b) 2 (c) 1 (d) 6
2S
7 If D = , then the value of tan D tan2 D +tan2 D tan4 D +tan4 D tan D is
7
13
(a) 0 (b) –7 (c) (d) 2
4
8 If sin(y+z–x), sin (z+x–y) sin(x+y–z) be is A.P., then tanx, tany, tanz are in
(a) A.P. (b) G.P (c) H.P. (d) None of these
3  cot 76q cot 16q
9* =
cot 76q  cot 16q
(a) tan16° (b) cot76° (c) tan46° (d) cot44°
D E
10 If x cos D +ysin D = 2a, xcos E +ysin E = 2a and 2sin sin =1, then
2 2
2ax 2a 2 – y 2
(a) cos D +cos E = 2 (b) cos D cos E = 2
x  y2 x  y2
297
(c) y2=4a(a–x) (d) cos D +cos E =2cos D cos E
11 Match the following :-
Column I Column II
(a) The maximum value of y = cos(2A+ T )+cos(2B+ T ) (p) 2sin(A+B)
where A&B are constants is
(b) The maximum value of y = cos2 A+cos2 B where
§ S·
A+B is a constant & A,B ¨ 0, ¸ is (q) 2sec(A+B)
© 2¹
(c) The minimum value of y = sec2A+sec2B where
§ S·
A+Bis a constant & A,B ¨ 0, ¸ is (r) 2cos(A+B)
© 4¹
(d) The minimum value of (s) 2cos(A–B)
y = tan T  cot T – 2 cos 2(A  B) where A, B are

§ S·
constants and T  ¨ 0, ¸ is
© 2¹
12 Let A0 A1 A2 A3 A4 A5 be a regular hexagon inscribed in a circle of unit radius. Then the product
of the lengths of the line segments A0 A1, A0A2 and A0 A4 is
3 3 3
(a) (b) 3 3 (c) 3 (d)
4 2
13 If A, B, C, D are the smallest positive angles in ascending order of magnitude which have their
sines equal to the positive quantity k, then the value of
A B C D
4sin +3sin +2sin +sin is
2 2 2 2
(a) 2 1 – k (b) 2 1  k (c) 2 k (d) None of these
14 Read the paragraph and answer the following questions .
§ S·
If D , E , J , G are the solutions of the equation tan ¨ T  ¸ = 3tan3 T , no two of which have
© 4¹
equal tangents, then
(i) The value of tan D +tan E +tan J +tan G is
(a) 1/3 (b) 8/3 (c) –8/3 (d) 0
(ii) The value of tan D tan E tan J tan G is
(a) –1/3 (b) –2 (c) 0 (d) None of these
1 1 1 1
(iii) The value of + + + is
tan D tan E tan J tan G
(a) –8 (b) 8 (c) 2/3 (d) 1/3

298
15* Which of the following is / are correct ?
log e (sin x )
(a) (tan x ) log e (sin x ) > (cot x ) ,  x  (0, S /4)
(b) 4 log e (cos ecx ) < 5
log e (cos ecx )
,  x  (0, S /2)
log e (cos x ) log e (cos x )
§1· §1·
(c) ¨ ¸ <¨ ¸ ,  x  (0, S /2)
©2¹ ©3¹
log (sin x )
(d) 2 log e (tan x ) < 2 e ,  x  (0, S /2)

'Note : Questions with * have more than one correct option'

Answers
1. b 2. a, b, c 3. a 4. a 5. a 6. a 7. b
8. a 9. c, d 10. c 11. a o s ; b o r ; c o q ; d o p 12. c
13. b 14. (i) d (ii) a (iii) 8 15. a, b, c, d

299
TRIGONOMETRY - I
Trigonometric Functions - Problem Solving

1 Values of trigonometrical ratios of some particular angles

1 4– 2 – 6
(i) sin 7 q =
2 2 2

1 4 2  6
cos 7 q =
2 2 2
1
tan 7 q =
2
3– 2
2 –1

1
cot 7 q =
2
3 2 2 1
3 –1
(ii) sin15° = cos75° =
2 2

3 1
cos15° = sin75° =
2 2
tan15° = cot75° = 2 – 3
cot15° = tan75° = 2  3
1 1
(iii) sin22 q = 2– 2
2 2
1 1
cos22 q = 2 2
2 2
1
tan22 q = 2 – 1
2
1
cot22 q = 2  1
2
5 –1
(iv) sin18° = cos72° =
4

10  2 5
cos18° = sin72° =
4

10 – 2 5
sin36° = cos54° =
4

300
5 1
cos36° = sin54° =
4

(v) cos9° =
1
2
1  sin 18q  1 – sin 18q
(vi) cos27° =
1
2

1  cos 36q  1 – cos 36q
2 Conditional identities
If A, B, C are angles of a triangle (i.e. A+B+C= S ) then
x tanA+tanB+tanC= tanAtanBtanC
x cotAcotB+cotBcotC+cotCcotA = 1
A B B C C A
x tan tan +tan tan +tan tan =1
2 2 2 2 2 2
A B C A B C
x cot +cot +cot = cot cot cot
2 2 2 2 2 2
x sin2A+sin2B+sin2C = 4sinAsinBsinC
x cos2A+cos2B+cos2C = – 1 – 4cosAcosBcosC
A B C
x sinA+sinB+sinC = 4cos cos cos
2 2 2
A B C
x cosA+cosB+cosC = 1 + 4sin sin sin
2 2 2
3 Trigonometric ratios of sum of more than three angles.
x sin(A1+A2.................+An) = cosA1cosA2.............cosAn(S1–S3+S5–...............)
x cos(A1+A2.................+An) = cosA1cosA2.............cosAn(1–S2+S4–S6+..............)
S1 – S3  S5 – .......
x tan(A1+A2.................+An) = 1 – S  S – S  .....
2 4 6

where S1 = ¦ tanA1 = sum of tangents of angles

S2 =¦ tanA1tanA2 = sum of tangents taken two at a time etc.


In particular, if A1=A2=..................An= A, then
S1 = n tanA ; S2 = nC2tan2A ; S3=nC3tan3A etc.
sin nA = cosnA (nC1tanA–nC3tan3A+nC5tan5A–.....................)
cos nA = cosnA (1–nC2tan2A+nC4tan4A–.....................)
C1 tan A – n C 3 tan 3 A  n C5 tan 5 A – ...........
n

tan nA =
1 – n C 2 tan 2 A  n C 4 tan 4 A – ......................

301
Solved Examples
cot x 5S
1 If ƒ(x) = and D + E = , then the value of ƒ( D ).ƒ( E ) is
1  cot x 4
1 1
(a) 2 (b) – (c) (d) None of these
2 2
Solution :
cot D cot E 1 1
ƒ( D ).ƒ( E ) = . = .
1  cot D 1  cot E 1  tan D 1  tan E
1 1 1 1
= . = ×
1  tan D § S · 1  tan D 1 – tan D
1  tan ¨ S  – D ¸ 1
© 4 ¹ 1  tan D

1 1  tan D 1
= =
1  tan D 2 2
Ans : (c)
2 The value of tan81°–tan63°–tan27°+tan9° equals
(a) 1 (b) 2 (c) 3 (d) 4
Solution :
(tan81°+tan9°)–(tan63°+tan27°)
=(cot9°+tan9°)–(cot27°+tan27°)
1 1
= –
sin 9q cos 9q sin 27q cos 27q
2 2 2u 4 2u 4
= – = –
sin 18q sin 54q 5 –1 5 1

=
^
8 5 1 – 5 1
=
`
8u 2
=4
5 –1 4
Ans : (d).
3 The number of integral values of k for which the equation 7cosx+5sinx = 2k+1 has a unique
solution is
(a) 4 (b) 8 (c) 10 (d) 12
Solution :
7 5 2k  1
. cosx+ sinx =
74 74 74
2k  1
Ÿ sin(x+ D ) =
74
2k  1
Now –1 d d1
74

302
– 74 – 1 74 – 1
Ÿ dkd
2 2
Ÿ –4.8 d k d 3.8
Ÿ k = –4, –3, –2, –1, –0, 1, 2, 3
i.e. 8 values.
Ans : (b)
sin x 1 cos x 3 § S·
4 If and where x,y ¨ 0, ¸ then tan (x+y) =
sin y 2 cos y 2 © 2¹
(a) 13 (b) 14 (c) 17 (d) 15
Solution :
sin2x+cos2x=1
1 2 9
Ÿ sin y+ cos2y= 1
4 4
3 5
Ÿ cosy= and tany=
2 2 3

5 5
Also sinx = and tanx =
4 2 3 3

5 5

tan x  tan y 3 3 3
? tan(x+y)= 1 – tan x. tan y =
5 5
1– .
3 3 3

5 3 5
= 9–5
3

4 5
= × 3 = 15
4
Ans : (d)
S
5 If D + E = and E + J = D , then tan D is equal to
2
(a) 2(tan E +tan J ) (b) tan E +tan J (c) tan E +2tan J (d) 2tan E +tan J
Solution :
J = D –E
tan D – tan E
Ÿ tan J = tan( D – E ) =
1  tan D. tan E

303
tan D – tan E
Ÿ tan J =
§S ·
1  tan D. tan¨ – D ¸
©2 ¹
tan D – tan E
Ÿ tan J =
11
2tan J = tan D –tan E
Ÿ tan D = tan E +2tan J
Ans : (c)
7
rS
6 ¦ tan
r 1
2

16
=

(a) 34 (b) 35 (c) 37 (d) None of these


Solution : Given series can be simplified to
§ 2 S S· § 2S 2S · § 2 3S 3S ·
¨ tan  cot 2 ¸ + ¨ tan 2  cot 2 ¸ + ¨ tan  cot 2 ¸ +1
© 16 16 ¹ © 16 16 ¹ © 16 16 ¹
Ÿ General pattern is tan T +cot T
2 2

sin 4 T  cos 4 T 1 – 2 sin 2 T cos 2 T 4


= = = –2
sin T cos T
2 2
sin T cos T
2 2
sin 2 2T
4u 2 8
= –2 = –2
1 – cos 4T 1 – cos 4T

§ · § · § ·
¨ 8 ¸ ¨ 8 ¸ ¨ 8 ¸
¨ – 2¸ ¨ – 2¸ ¨ – 2¸
?¨ S ¸ + ¨ 1 – cos S ¸+ ¨ 1 – cos 3S ¸ +1
¨ 1 – cos ¸ ¨ ¸ ¨ ¸
© 4 ¹ © 2 ¹ © 4 ¹

8 2 8 2
= –2+8–2+ –2+1
2 –1 2 1

8 2 8 2
= + –6+8+1
2 –1 2 1
16  8 2  16 – 8 2
= +3 = 32+3 = 35
2 –1
Ans : (b)

304
Exercise
§ 1 – p0
2 ·
1 ¨ ¸
1 If pn+1= (1  p n ) , then cos ¨ p p p .......f ¸ is equal to
2 © 1 2 3 ¹
1
(a) 1 (b) –1 (c) p0 p0(d)
2 If A, B, C are acute positive angles such that A+B+C= S and cotA cotBcotC= k, then
1 1 1 1
(a) k d (b) k t (c) k< (d) k>
3 3 3 3 9 3
xy
3 If ¦ xy = 1, then ¦ 1 – xy
=

1 4
(a) (b)
xyz xyz
(c) xyz (d) None of these
4 The value of cot16°cot44°+cot44°cot76°–cot76°cot16° is
1 –1
(a) 3 (b) (c) (d) –3
3 3
5 The number of solutions of tan(5 S cos T ) = cot(5 S sin T ) for T in (0,2 S ) is
(a) 28 (b) 14 (c) 4 (d) 2
6 If cos x = tan y, cos y = tan z and cos z = tan x, then a value of sin x is equal to
(a) 2cos18° (b) cos18°
(c) sin18° (d) 2sin18°
n

7 Let n be an odd integer. If sinn T = ¦


r 0
br sinr T ,  T , then
(a) b0 = 1, b1=3 (b) b0 =0, b1=n
(c) b0= –1 b1= n (d) b0=0, b1=n2–3n+3
S
8 If e – S / 2 < T < , which is larger, cos log e T or log e (cos T)
2
(a) cos log e T (b) log e (cos T)
(c) both are equal (d) None of these
n –1
2rS
9 ¦ (n – r) cos
r 1 n
for n t 3 is ____________

n
(a) (b) n
2
(c) (n–3) (d) None of these

305
10* Match the following :-
Column I Column II
(a) In an acute angled ' ABC, the least values (p) O–P = 2
of ¦ sec A & ¦ tan 2 A are O and P respectively, then
(b) In ' ABC, the least values of ¦ cos ec(A / 2) (q) P– O= 3
& ¦ sec (A / 2) and O & P respectively then
2
(r) O–P = 4
§A· § B· §C·
(c) In ' ABC, the least values of cosec ¨ ¸ cosec ¨ ¸ cosec ¨ ¸ (s) 3 O –2 P =0
©2¹ ©2¹ ©2¹
& ¦ cos ec A are O & P
2
respectively, then (t) 2 O –3 P = 0

sin A
11 In any ' ABC, the minimum value of ¦ is
sin B  sin C – sin A
(a) 3 (b) 0 (c) 4 (d) None of
these
S 3S 5S
12 If cos , cos , cos , are the roots of the equation 8 x3–4x2–4x+1 = 0.
7 7 7
On the basis of above information, answer the following questions :-
S 3S 5S
(i) The value of sec +sec +sec is
7 7 7
(a) 2 (b) 4 (c) 8 (d) None of
these
S 3S 5S
(ii) The value of sin sin sin is
14 14 14
1 1 7 7
(a) (b) (c) (d)
4 8 4 8
S 3S 5S
(iii) The value of cos cos cos is
14 14 14
1 1 7 7
(a) (b) (c) (d)
4 8 4 8
S 3S 5S
(iv) The equation whose roots are tan2 , tan2 , &tan2 , is
7 7 7
(a) x –35x +7x–21=0
3 2
(b) x –35x2+21x–7=0
3

(c) x –21x +35x–7=0


3 2
(d) x3–21x2+7x–35=0
3
§ 2r – 1 · 3 2 § 2r – 1 ·
the value of ¦ tan ¨ ¸ ¦ cot ¨
2
(v) ¸ is
r 1 © 7 ¹ r1 © 7 ¹
(a) 15 (b) 105 (c) 21 (d) 147

306
S 5S 7S
13 If a = sin sin sin , and x is the solution of the equation y = 2[x]+2 and y = 3[x–2], then
18 18 18
a=
1
(a) [x] (b) (c) 2[x] (d) [x]2
>x @
14 If tan D , tan E , tan J are the roots of x3–px2–r =0, then the value of (1+tan2 D )(1+tan2 E ) (1+tan2 J )
is equal to
(a) (p–r)2 (b) 1+(p–r)2 (c) 1–(p–r)2 (d) None of
these
15 If tan D is an integral solution of 4x2–16x+15<0 and cos E is the slope of the bisector of the
angle in the first quadrant between the x & y axes, then the value of sin( D + E ): sin( D – E ) is
equal to
(a) –1 (b) 0 (c) 1 (d) 2

'Note : Questions with * have more than one correct option'

Answers
1. c 2. a 3. a 4. a 5. a 6. d 7. b
8.a 9. a 10. a o q,s ; b o p,t ; c o r 11. a
12. (i) b (ii) b (iii) d (iv) c (v) b 13. b 14. b 15. c

307
TRIGONOMETRY - II
Trigonometric Equations
1. TRIGONOMETRIC EQUATION. An equation involving one or more trigonometrical ratios
of unknown angles is called a trigonometrical equation.
2. SOLUTION OF TRIGONOMETRIC EQUATION. A value of the unknown angle which
satisfies the given equation is called a solution of the equation.
(a) Principal solution. The smallest numerical value, positive or negative for the angle
of a trigonometrical equation is called its principal solution. If a positive angle as well as a
negative angle of smallest value (and equal) are available, then we take the positive value
as the principal values.

ª S Sº
(i) The principal solution of sin T k, k d 1 is the value of T in the interval « – , »
¬ 2 2¼
which satisfy the equation.

(ii) The principal solution of cos T k, k d 1 is the value of Tin the interval [0, S]
which satisfy the equation.

§ ʌ ʌ·
(iii) The principal solution of tan T k, k  f the value of T in the interval ¨© – , ¸¹ ,
2 2
which satisfy the equation.
(b) General solution. Since all the trigonometric functions are many one onto, therefore, there
are infinite values of ș for which trigonometric functions have the same value. Therefore,
all such possible values of ș for which trigonometric ratios are same is known as general
solutions of the variable angle ș .
3. GENERAL SOLUTIONS OF TRIGONOMETRIC RATIOS
(i) If sin T= 0, then T = nS,n Z (set of integers)
(ii) If cos T = 0, then T = (2n+1)S,n Z
(iii) If tan T= 0, then T = nS,n Z
ª S Sº
(iv) If sin T = sinD then T= nS + (–1)n D, n  Z, D  « – , »
¬ 2 2¼
(v) If cos T = cos D, then T= 2nS r D, n  Z, D [0, S]
ª S Sº
(vi) If tan T = tan D, then T = nS + D, n  Z D  « – , »
¬ 2 2¼
S
(vii) If sin T = 1, then T = 2nS + ,n Z
2
(viii) If cos T = 1, then T = 2nSn  Z
(ix) If sin2 T = sin2 D or cos2 T = cos2 D or tan2 T = tan2 D then T = nS± D, n  Z

308
§ Į – b·
(x) If sin (aT + b) = sin D, then TznS+(–1)n ¨© ¸ . First of all we have to write the
a ¹
general solution as such for aT + b and then, find T as aT + b = nS + (–1)n.D

1
ŸT= [nS+(–1)n D–b]
a
(xi) For n  Z, sin nS = 0 and cos nS = (–l)n
sin(nS+T)=(–1)n sinT
cos(nS + T) = (–l)n cosT
(xii) If n is an odd integer, then

nS n –1 nS
sin (–l) 2 cos 2 
2

§ nS ·
sin ¨  T ¸ (–l) 2 cosT
n –1

© 2 ¹
n1
§ nʌ ·
cos¨  ș¸ (–1) 2
sin ș
© 2 ¹

4. IMPORTANT POINTS TO REMEMBER


(i) For equations of the type sin T = k or cos T = k, one must check that |k| d 1.
(ii) Avoid squaring the equation, if possible, because it may lead to extraneous solutions.
(iii) Do not cancel the common variable factor from the two sides of the equations which are
in a product because we may loose some solutions.
(iv) The answer should not contain such values of T which make any of the terms undefined or
infinite.
(v) Check that denominator is not zero at any stage while solving equations.

S
(vi) (a) If tan T or sec T is involved in the equation, T should not be odd multiple of .
2
(b) If cot T or cosec T is involved in the equation, T should not be a multiple of S or 0.
(vii) If two different trigonometric ratios, such as, tan T and sec T are involved then after
solving we cannot apply the usual formulae for general solution, because periodicity
of the functions are not same.
(viii) If L.H.S. of the given trigonometric equation is always less than or equal to k and RHS is
always greater than k, then no solution exists. If both the sides are equal to k for same
value of T, then solution exists and if they are equal for different value of T, then solution
does not exist.

309
EXAMPLES
1
1 The general solution of the equation ¦ cos(r x) sin(rx) = 2
2
is

S (4m  1) S
(a) 2m S + , m  I (b) m I
6 n (n  1) 2 
(4m – 1) S
(c) ,m I (d) None of these
n (n  1) 2 
Solution
n
1
¦ cos(r x ) sin(rx ) =
r 1
2

2
n

= ¦ 2 cos(r x) sin(rx)
r 1
2
=1

= ¦ [sin(r
r 1
2
 r ) x – sin( r 2 – r ) x ] = 1

= ¦ [sin r (r  1) x – sin r(r – 1)x ] = 1


r 1

= sin2x–sin0+sin6x–sin2x+sin12x–sin6x+.............+sin(n(n+1)x)–sin(n(n–1)x) = 1
= sin[n(n+1)x]–sin0 = 1
= sin(n(n+1)x) = 1
S
= sinn(n+1)x = sin
2
sin(nS+T) = sinT
§ S·
n(n+1)x = ¨ 2mS  ¸ , m  I
© 2¹
4mS  S
x= , mI
2n (n  1)
(4m  1) S
= , mI
n (n  1) 2
option ‘b’ is correct
2 If [sinx] + [ 2 cosx] = –3, x  [0,2 S ] [ [.] denotes the greatest integer function], then x belongs
to
§ 5S · ª 5S º § 5S · ª 5S º
(a) ¨ S, ¸ (b) «¬S, 4 »¼ (c) ¨ ,2S ¸ (d) «¬ 4 ,2S»¼
© 4 ¹ © 4 ¹
Solution
' [sinx] + [ 2 cosx] = –3

310
[sinx] = –1 [ 2 cosx] = –2
? – 1 d sinx<0 and –2 d 2 cosx<–1
–1
– 2 d cosx<
2
1
–1 d sinx<0 and –1 d cosx<– ( –1 cosx d 1)
2 ' d
since both sinx and cosx are negative
§ 3S 5S ·
i.e. x  ( S ,2 S ) and x  ¨ , ¸
© 4 4 ¹
§ 5S ·
? x  ¨ S, 4 ¸
© ¹
option ‘a’ is correct
3 The number of solutions of the equation sinx+2sin2x = 3+sin3x in the interval [0, S ] is
(a) 0 (b) 1 (c) 2 (d) 3
Solution
We have
sinx+2sin2x = 3+sin3x
sin3x–sinx–2sin2x+3 = 0
2cos2xsinx–4sinxcosx+3 = 0
2sinx(cos2x–2cosx)+3 = 0
2sinx(2cos2x–1–2cosx)+3 = 0
sinx(4cos2x–4cosx–2)+3 = 0
sinx{(2cosx–1)2–3}+3 = 0
sinx(2cosx+1)2+3(1–sinx) = 0
' 0dxd S
' 0 d sinx d 1
1–sinx t 0
sinx(2cosx–1)2 t 0
? each term is equal to zero
1–sinx = 0 Ÿ sinx = 1
? cosx = 0
sinx(2cosx–1)2 z 0
? no solution
§ S S·
4 The number of values of T in the interval ¨ – , ¸ suggesting the equation
© 2 2¹

3 sec2 T
= tan4 T +2tan2 T is
(a) 1 (b) 2 (c) 3 (d) None of these
solution

311
We have

3 sec2 T
= tan4 T +2tan2 T
= (1+tan2 T )2 –1
= (sec2 T )2 –1
let sec2 T = x ? x t 1

3 x
= x2–1

graph of y = 3 x
& y = x2–1 y y ( 3 )x

intersect at one point


When x = 2 ; y = 3 2
y=x –1
(0,1)
? sec2 T = 2 X
O
sec T = r 2 (0,–1)

§ S S·
? T takes two values in ¨ – 2 , 2 ¸
© ¹
option ‘b’ is correct
2 2
5 If 0 d x d S and 81sin x + 81cos x
= 30 then x is equal to
S S S
(a) (b) (c) S (d)
6 2 4
Solution
we have
2 2
81sin x + 81cos x
= 30
2 2
81sin x + 811– sin x
= 30
2 81
81sin x + 2 = 30
81sin x

2
let 81sin x = y
y2–30y+81 = 0
y2–27y–3y+81 = 0
(y–27)(y–3) = 0
2
81sin x
= 27 or 3
34 sin
2
x = 33 or 31
2
? 4 sin x = 3 or 1
3 1
sin2x = or
4 4
3 1
sinx = r or r
2 2

312
3 1
' 0 d x d S ? sinx = or
2 2
S 2S S 5S
? , , ,
3 3 6 6
6 The equation a sinx+bcosx = c
where |c| > a 2  b 2 has
(a) one solution (b) two
(c) no solution (d) infinite number of solution
Solution
We have
asinx+bcosx = c
a b c
sinx + cosx =
a b
2 2
a b
2 2
a  b2
2

c
sinx cos D +cosx sin D =
a  b2
2

|c|
sin(x+ D ) = sin E >1 ' |c| > a 2  b 2 i.e. >1
a 2  b2
not possible
correct option is c
PRACTICE QUESTIONS
§ 1 ·
1. If cos 2ș =
2  1 ¨© cos ș ¸ ,then ș =

ʌ ʌ
a. 2nʌ b. 2 nʌ r c. 2 nʌ r d. none
4 3
2. It 3cos2 ș – 2 3 sin ș cos ș – 3sin2 ș = 0 then ș equals
nʌ ʌ nʌ ʌ nʌ ʌ nʌ ʌ
a.  b. – c.  d. –
2 6 2 6 2 3 2 3
3. The general solution of
sinx – 3sin2x + sin3x = cosx – 3cos2x + cos3x is
ʌ nʌ ʌ
a. nʌ  b. 
8 2 8
§ nS · S §3·
c. (–1) n ¨ ¸  d. 2nS  cot –1 ¨ ¸
© 2 ¹ 8 ©2¹

313
–ʌ ʌ
4. The number of points of intersection of 2y = 1 and y = cosx in d x d is
2 2
a. 1 b. 2 c. 3 d. 4
5. The number of values of x in the internal >0,5S@ satisfying the equation 3sin 2x – 7sinx + 2
= 0 is
a. 0 b. 5 c. 6 d. 10
§ʌ · §ʌ · 1
6. The sum of all the solutions of the equation cos ș cos¨  ș¸ cos¨ – ș¸ = , ș >0,6ʌ@ is
©3 ¹ ©3 ¹ 4
100 ʌ
a. 15ʌ b. 30ʌ c. d. none
3

7. The number of values of ș >0,4ʌ@ satisfying the equation 3 cos x – sin x t 2 is


a. 0 b. 2 c. 4 d. 8
1
8. If sin ș, cos ș, tan ș are in G.P., then ș equals
6
ʌ ʌ ʌ ʌ
a. 2 nʌ r b. 2 nʌ r c. nʌ  (–1) n d. nʌ 
3 6 3 3
9. The number of solutions of the equation tanx + secx = 2cosx lying in the internal
>0,2ʌ@ is
a. 0 b. 1 c. 2 d. 3
10. The general solution of equation sin 2 ș sec ș + 3 tan ș = 0 is
ʌ
ș = nʌ  (–1)
n1
a. b. ș nʌ
3

ʌ nʌ
c. ș nʌ  (–1) n1 d. ș
6 2
11. Passage
Solution of equations asinx r bcosx = c.
General value satisfying two equations.
acos ș r bsin ș = c, ș satisfying two equations.
a. The equation acos ș r bsin ș = c be first converted to r cos ș r Į = c where
a = rcos Į ,b = rsin Į
b. ș satisfying two equations
Find the common value of ș lying between 0 and 2ʌ and then add 2nʌ
Answer the following questions based upon above passage
(i) The number of intergral values of k for which the equation 7cosx + 5sinx = 2k+1 has a
solution is
a. 4 b. 8 c. 10 d. 12

314
§ 7S ·
(ii) If cos3x + sin ¨ 2 x – ¸ = – 2 then x equals
© 6 ¹
ʌ ʌ ʌ
a. 6m – 1 b. 6m  1 c. 2m  1 d. none
3 3 3
(iii) The values of x such that – ʌ  x  ʌ and satisfying the equation are given by
>1 cos x  cos x  cos x .....tof @ = 43
2 3
x equals
8
ʌ 2ʌ –ʌ – 2ʌ
a. 3 b. c. d.
3 3 3
(iv) The number of solution of the equaiton e – e
sinx –sinx
–4 = 0 is
a. 1 b. 2 c. 4 d. 0
Answers
1. b, c 2. a 3. b 4. b 5. c 6. b 7. c 8. a
9. c 10. b 11. (i). b (ii). b (iii), a, b, c, d (iv). b

315
TRIGONOMETRY - III
Trigonometric Equations
1. The number of values of x in the internal [0, 3S] satisfying the equation 2sin2x+5sinx–3=0 is
a. 6 b. 1 c. 2 d. 4.
Solution :
We have 2sin2x+5sinx–3=0 Y
2sin x+6sinx–sinx–3=0
2

(2sinx–1) (sinx+3)=0 1
1 1
y
2 2
1 o S 3S 2S S X
(' –1 dsin x d 1)
S 5S
sinx= (' sin xz3) 2 2 2
2 –1

1
y=sinx and y= intersect in 4 points
2
? number of values of x in the internal [0, 3S] is 4.

§ S S· nS
2. The number of values of T in the internal ¨ – , ¸ such that T z for n=0, r 1, r2 and tanT
© 2 2¹ 5
= cot5Tas well as sin 2T=cos4T is _______.
Solution :
Given, tanT = cotT

§S ·
tanT=tan ¨ – 5T ¸
©2 ¹

S
–5T=nS+ T
2

S
6T= –nS
2

S nS
T= –
12 6
Also cos4T=sin2T

§S ·
cos4T=cos ¨ – 2T ¸
©2 ¹

§S ·
4T=2nS r ¨ – 2T ¸
©2 ¹

316
nS § S T ·
T= r¨ – ¸
2 ©8 2¹
Taking positive Taking negative

3T nS S T nS S
 –
2 2 8 2 2 8

S S
3T=nS+ T=nS–
4 4

4 nS  S S
T= T=(4n–1)
12 4

S
T=(4n+1)
12

§ S S·
These values of T gives only 3 common solution lies in the interval ¨ – , ¸ .
© 2 2¹
3. The number of all possible values of T, where 0 < T < S, for which the system of
equaitons(y+z)cos3T=(xyz)sin3T

2 cos 3T 2 sin 3T
xsin3T= 
y z
and (xyz) sin3T=(y+2z)cos3T+ysin3T
have a solution (x 0, y0, z0) with y0z0z0 is
Solution :

yz
Given equations are xsin3T= yz cos3T

cos 3T cos 3T
xsin3T– – = 0 .....(1)
y z

2 cos 3T 2 sin 3T
xsin3T– – = 0 ....(2)
y z

1 2 1
xsin3T– cos3T– cos3T– sin3T  ....(3)
z y z
from (2) and (3) we get
2sin3T= cos3T+sin3T

317
sin3T=cos3T

S
tan3T=1=tan
4

S 5S 9S S 5S 9S
3T= , , or T= , ,
4 4 4 12 12 12
4. The solution of the equation 4sin4x+cos4x=1 is
a. x=2nS b. x=nS+1 c. x=(n+2)S d. none of these
Solution:
Given that 4sin4x+cos4x=1
4sin4x+(cos2x–1) (cos2x+1)=0
4sin4x–sin2x(cos2x+1)=0
sin2x[4sin2x–cos2x–1]=0
sin2x[5sin2x–2]=0

2
sin2x=0 or sin2x=
5

2
sinx=0 or sinx= r
5

2
x=nS or x=nSDwhere sinD  r
5
?option d is correct.
5. The solution of the equation [sinx+cosx] 1+sin2x=2, –Sdx dSis

S S 3S
a. b. S c. d.
2 4 4
Solution : [sinx+cosx]1+sin2x
1 sin 2 x
§ §S ··
¨¨ 2 sin ¨  x ¸ ¸¸
© ©4 ¹¹

S
1 sin 2
S § § S S ··
2 =2
4
at x= , ¨¨ 2 sin ¨  ¸ ¸¸ =
2
4 © © 4 4 ¹¹
? Option c is correct.

318
1
6. The most general solutions 2sinx+2cosx=21– 2 are

S S S S
a. nS– b. nS+ c. nS+(–1)n d. 2nS r
4 4 4 4
1
Solution : We have 2sinx+2cosx=21– 2

AM t GM

2sin x  2cos x
2 t 2sin x 2cos x

t 2 2sin x2 cos x (Equality holds when sinx=cosx)

t 2 2sin x 2cos x

But the minimum value of sinx+cosx is – 2

§S ·
? sinx+cosx= 2 sin ¨  x ¸
©4 ¹

§S ·
and–1 d 2 sin ¨  x ¸ d 1
©4 ¹

§S ·
– 2 d 2 sin ¨© 4  x ¸¹ d 2

? sinx=cosx

tanx=1=tan S
4

x=nS  S
4
? option b is correct.
7. The set of values of T satisfying the inequation 2sin2T–5sinT+2>0, where 0 < T < 2Sis

a. § S · § 5S · b. ª S º ª 5S º
¨ 0, ¸ ‰ ¨ ,2S ¸ «0, 6 » ˆ « 6 ,2S»
© 6¹ © 6 ¹ ¬ ¼ ¬ ¼

c. ª S º ª 2S º d. none of these
«0, 3 » ‰ « 3 ,2S»
¬ ¼ ¬ ¼

319
Solution : Given inequation 2sin2T–5sinT+2>0
2sin2T–4sinT–sinT+2>0
Y
2sinT(sinT–2)–1(sinT–2)>0
(2sinT–1) (sinT–2) > 0 1 y = sinx 1
y
1 2
1 2
x
ŸsinT< or sinT> 2 o S 5S S S
2 6 6

but sinT> 2 not possible –1

1
?sinT<
2

If x H §¨ 0, S ·¸ ‰ §¨ 5S , S ·¸ for x  S
© 6¹ © 6 ¹

1
?sinT< ŸT  §¨ 0 S ·¸ ‰ §¨ 5S , S ·¸ for T  S .
2
© 6¹ © 6 ¹
correct option is ‘d’
8. Passage
Suppose equation is f(x)–g(x)=0 or f(x)=g(x)=y say. Then draw the graphs of y=f(x) and y=g(x).
If graphs of y=f(x) and y=g(x) cuts at one, two, three ...., no points, then number of solutions
are one, two, three, ... zero respectively.
On the basis of above information, answer the following questions.

x
1. The number of solution of sinx= is
10
a. 4 b 6 c. 8 d. none of these

5S
2. Total number of solutions of the equation 3x+2tanx= in x H[0, 2S] is equal to
2
a. 1 b 2 c. 3 d. 4
3. Total number of solutions of sin{x}=cos{x}, where {.} denotes the fractional part, in [0,
2S] is
a. 3 b. 5 c. 7 d. none of these

3 S
4. If 1–sinx= x – +a has no solution when a H R+ then
2 2

3 S
a. a  R+ b. a> +
2 3
320
§ 3 S · §3 3 S ·
c. a  ¨ 0, 2  ¸ d. a¨ 2 , 2  ¸
© 3¹ © 3¹
Solution :

x
1. Graphs of y=sinx and y = meet exactly six times. Hence no of solutions =6
10

x
y
10
1 y=sinx

–S –S –S –S –S o S S S S S X

5S
2. We have 3x+2tanx= in x H [0, 2S]
2
5S 3x
tanx = –
4 2
y

y=tanx

o 3S 2S
x
S S
2 2

5S 3x
y
4 2

5S 3S
Graphs of y = – and y = tanx meet exactly three times in [0, 2S]
4 2
Thus number of solution = 3

321
3. sin{x}=cos{x}
Graphs of y=sin{x} and y=cos{x} meet excatly 7 times in [0, 2S]

1 o o o o o o o

x
o S

4. slope of y=1–sinx is –cosx

3 S S 3
slope of y= x – +a for x > is
2 2 2 2

2
P

O S X
2

3 7S § 7S 3 ·
cosx=– Ÿx= Ÿ P = ¨ 6 , 2¸
2 6 © ¹

3 S
If y= x – +a passes throgh ‘P’ then
2 2

3 S
a= –
2 3

3 S
Ÿa> 2–
3

322
PRACTICE QUESTIONS
1. SinT+ 3 cosT=6x–x2–11, 0dTdS, x R, holds for
a. no values of x and T b. one value of x and two values of T
c. two values of x and two values of T d. two points of values of (x, T)

2 1 2
2. For 0 dx d2S, then 2
cos ec x
y – y  1d 2
2
a. is satisfied by exactly one value of y b. is satisfied by exactly two value of x
c. is satisfied by x for which cos x=0 d. is satisfied by x for which sin x=0
3. Let tanx–tan2x > 0 and ~2sinx~ < 1. Then the intersection of which of the following two sets
satisfies both the inequalities?

S
a. x > nS, n Z b. x > nS– ,n Z
6 

S S
c. x < nS – ,n Z d. x < nS  ,n Z
4  6 
4. The equation (cosp–1)x2+(cosp)x+sinp=0 in the varibale x has real roots. Then p can take any
value in the interval

§ S S·
a. (0, 2S) b. (–S, 0) c. ¨– , ¸ d. (0, S)
© 2 2¹
5. Let 2sin2x + 3sinx–2>0 and x2–x–2<0 (x is measured in radians). Then x lies in the inerval

§ S 5S · § 5S · §S ·
a. ¨ , ¸ b. ¨ – 1, ¸ c. (–1, 2) d. ¨ ,2 ¸
©6 6 ¹ © 6 ¹ ©6 ¹

3S
6. If (cosec2T–4)x 2+(cotT+ 3 )x + cos2 =0 holds true for all real x, then the most general
2
values of T can be given by (n HZ)

11S 5S 7S 11S
a. 2nS+ b. 2nS+ c. 2nS r d. nS r
6 6 6 6

S
7. If sin4x+cos4y+2=4 sinx cosy and 0 d x, y d then sinx+cosy is equal to
2

3
a. –2 b. 0 c. 2 d.
2

323
29
8. The solution of the equation sin10x+cos10x= cos42x is
16

nS S S
a. x= + ,nHI b. x=nS ,nHI
4 8 4

S
c. x=2nS ,nHI d. none of these
2
9. The most general values of x for which sinx+cosx=min {1, a2–4a+6},aHR are given by

S
a. 2nS, nHN b. 2nS , n HN
2

S S
c. nS+(–1)n – ,n HN d. none of these
4 4

1
10. Number of solutions of the equations y= [sinx+[sinx]] and [y+[y]]=2cosx where [.] denotes
3
the greatest integer function is
a. 0 b. 1 c. 2 d. infinite
3 1
sin 2 x – sin x 
11. If cos x 2 2 =1, then possible values of x are

S S S
a. nS or nS + (–1)n ,nHI b. nS or 2nS + or nS+(–1)n , n H Z
6 2 6

S
c. nS + (–1)n ,nHI d. nS , n H I
6

§ 1 ·
If ¨ cos x  cos 2 x ¸ (1+tan22y) (3+sin3z)=4, then
2
12.
© ¹
a. x may be multiple of S b. x cannot be an even multiple of S

S
c. z can be a multiple of S d. y can be a multiple of
2
13. Matrix match type
Column I Column II

S 5S
a. If 3cos2T–2 3 sinTcosT–3sin2T=0 than T = p ,
6 6

324
7S
b. If rsinT=3, r=4(1+sinT) q. 2nS+
6
where 0 dTdSthen T=

1 1 S S 5S 2S
c. If sinT= – and tanT= , then r. , , ,
2 3 6 3 6 3
the general value of T which satisfies both the equations is
nS S
d. If 0 dx dSand 81sin x  81cos
2 2
x =30 then x is s. 
2 6

If 3sin 2 x  2 cos 2 x + 31–sin 2 x  2 cos x =28, then the value of x are given by
2
14.
a. tanx=1 b. tanx=–1 c. cosx=0 d. none of these
cos x
15. If e{(sin
2
x  sin 4 x  sin 6 x ....f ) log e 2} satisfies the equation x2–9x+8=0, then value of ,
cos x  sin x

S
0 < x< is
2

a.
1
2
3 1 b.
1
2
3 –1 c.
1
2
d. 0

Integer type questions .


16. If D, E satisfy the equaiton
12sinD+5cosD=2E –8E+21, then E –2E, is ___________.
17. The value of x and y satisfy the equaiton tan2(x+y)+cot2(x+y)=1–2x–x2, then the value of x2–
3x+2 is ___________.

18. The integral value of p for which p cosx – 2sinx = 2  2 – p has a solution, is _______
Answers
1. b, d 2. a,b,c 3. a,d 4. d 5. d 6. a,b 7. c 8. a
9. c 10. a 11. b 12. a,d 13. a o s ; b o p ; c o q ; d o r ;
14. b,c 15. b 16. 0 17. 6 18. 2

325
TRIGONOMETRY - IV
Trigonometric Equations - Problem Solving
Equations involving trigonometric functions of unknown angles are called trigonometric equations.
Numerically least angle is called the principal value (solution).
Since, trigonometric functions are periodic, a solution is generalised by means of periodicity of the function.
The solution consisting of all possible solutions of a trigonometric equation is called its general solution .
Trigonometric Equations General solutions (n  Z)
sin T = 0 T= nS
cos T = 0 T = (2n+1) S /2
tan T = 0 T= nS
cot T = 0 T = (2n+1) S /2
sin T = sin D T = n S +(–1)n D
cos T = cos D T = 2n S r D
tan T = tan D T = n S+ D
sin2 T = sin2 D T=nS r D
cos2 T = cos2 D T=nS r D
tan2 T = tan2 D T=nS r D
Method of an auxiliary angle
Equations of the form a cos T r b sin T = c are equivalent to the elementary trigonometric equations
c b a
cos( T  I )= where sin I = , cos I =
a 2  b2 a 2  b2 a 2  b2
If |c| > a 2  b 2 , then a cos T r b sin T = c has no solution.

If |c| d a 2  b 2 , then put


|c|
= cos D so that
a 2  b2
cos( T  I ) = cos D
Ÿ T  I = 2n S r D
Ÿ T = 2n S r D r I ; nZ
Note :
(i) While solving, if you are squaring a trigonometric equation (which should be avoided as for as
possible), check the solution for extraneous roots.
(ii) Do not cancel terms containing unknown terms which are in product (it may cause loss of a valid
solution)
(iii) Remove all those values (angles) which make any of the terms undefined or infinite.
Solving trigonometric equations graphically
Sketch the graph of each side of the equation. Now look for all the points of intersection within the given
interval.

326
Solved Examples
1 The number of distinct solutions of
ª Sº
sin5 T cos3 T = sin9 T cos7 T in «0, » is
¬ 2¼
(a) 4 (b) 5 (c) 8 (d) 9
Solution :
2sin5 T cos3 T = 2sin9 T cos7 T
Ÿ sin8 T +sin2 T =sin16 T +sin2 T
Ÿ sin16 T –sin8 T = 0
Ÿ 2cos12 T .sin4 T = 0
sin4 T =0 or cos12 T = 0
nS S
gives T = or gives T = (2n+1)
4 24
S S S 3S 5S 7S 9S 11S
? T = 0, , or ? T= , , , , ,
4 2 24 24 24 24 24 24
? 9 possible values
Ans : (d)
2 The number of values of x in [0,5 S ] satisfying 3cos2x–10cosx+7=0 is
(a) 5 (b) 6 (c) 8 (d) 10
Solution : Y
3(2cos2x–1) –10cosx+7 = 0
3cos2x–5cosx+2= 0 Ÿ 3cos2x–3cosx+2= 0
2 (3cox–2) (cox–1) = 0 1 2
3

Ÿ cosx = , 1. S 2S S S S
X
3 o
–1

2
Clearly the lines y = , 1 intersect the graph at 8 points
3
Hence 8 values
Ans : (c)
3 If sin2 T –2sin T –1=0 is to be satisfied for exactly 4 distinct values of T  [0,n S ],  n  N ; then
the least value of n is
(a) 2 (b) 6 (c) 4 (d) 8
Solution : Y
sin T –2sin T –1=0
2

sin T = 1 r 2 1
S 2S S S S

Ÿ sin T = 1– 2 (as 1+ 2 is rejected) o


X

clearly four solutions lie in [0,4 S ] –1

? least value of n is 4

327
Also 5th solution lies in [0,5 S ]
? greatest value can be 5.
Ans : (c)
4 The most general solution of secx–1 =
2 – 1 tanx is
S S
(a) n S+ (b) 2n S , 2n S + (c) 2n S (d) None of these
8 4
Solution :
Given equation can be simplified to
(1–cosx) =
2 – 1 sinx

2sin2
x
2

2 – 1 2sin
x
2
x
cos = 0
2
x§ x x·
2sin ¨ sin – ( 2 – 1) cos ¸ = 0
2© 2 2¹
x x
Ÿ sin =0 or tan = 2 – 1
2 2
x x S
Ÿ = nS tan = tan
2 2 8
x S
x = 2n S ,n z = n S+
2 8
S ,n z
x = 2n S +
4
Ans : (b)
5 Sum of all the solutions of
§S · §S · 1
cosx.cos ¨  x ¸ . cos ¨ – x ¸ = , x  [0,6 S ] is
©3 ¹ ©3 ¹ 4
110S
(a) 15 S (b) 30 S (c) (d) None
3
of these
Solution :
1 1
cos3x =
4 4
2 nS
cos3x=1 Ÿ cos3x=cos0 Ÿ 3x=2n S Ÿ x=
3
2S 4S 6S 8S 18S
? x=0, , , , ,...............,
3 3 3 3 3
Adding we get the sum as 30 S

328
Ans : (b)
6 The number of solutions of cosx= |1+sinx| for x [0,3 S ], is
(a) 3 (b) 2 (c) 4 (d) None of these
Solution :
It is evident from the figure that, the two graphs intersect at 3 points.
Y

S S
X
o
–1

Ans (a)
7 The number of solutions of 16sin 2 x +16cos 2 x = 10 in x  [0,2 S ] is
(a) 8 (b) 6 (c) 4 (d) 2
Solution :
2 2
16sin x
+ 161–sin x
= 10
2 16
put 16sin x = y Ÿ y+ = 10
y
y2–10y+16 = 0 gives y = 8,2
1 3
? 16sin
2
x = 2, 8 = 16 4 & 16 4 1
o S X
2S
1 3
? sin2x = , –1
4 4
1 3
Clearly lines y = , intersect the graph in 8 points
4 4
Hence 8 possible solutions.
Ans : (a)
PRACTICE QUESTIONS
1 Let ^
P = T : sin T – cos T `
2 cos T and
Q = ^T : sin T  cos T 2 sin T` be two sets. Then
(a) P  Q and Q–P z I (b) QŒP (c) PŒQ (d) P=Q
§ – S S· nS
2 The number of values of T in the interval ¨ , ¸ such that T z for n= 0, r 1, r 2 and
© 2 2¹ 5
tan T =cot5 T as well as sin2 T = cos4 T is ___________
(a) 0 (b) 1 (c) 2 (d) 3

329
3 The positive integer value of n>3 satisfying the equation
1 1 1
= + is
§ S · sin§ 2S · sin § 3S ·
sin ¨ ¸ ¨ ¸ ¨ ¸
©n¹ ©n¹ © n ¹
(a) 5 (b) 6 (c) 7 (d) 8
§ – S S· 2
4* All values of T in the interval ¨ , ¸ satisfying (1–tan T )(1+tan T )sec2 T + 2 tan T = 0
© 2 2¹
S –S
(a) (b) (c) 0 (d) None of these.
3 3
5 Match the following :-
Column I Column II
(a) Number of roots of cos7x+sin4x= 1 (p) 1
in the interval [0,2 S ]
(b) Value of ‘a’ for which a2– 2a+sec2 S (a+x) = 0 (q) 0
has solution
(c) Number of solutions of |cosx| = 2[x] (r) 4
6 The solution set of |4sinx–1|< 5 , |x|< S is

§ 4S · § S S · § 9S · § – 9S – S · § 3S 7 S ·
(a) ¨ – S,– ¸ ¨ – , ¸ ¨ , S¸ (b) ¨ , ¸‰ ¨ , ¸
© 5 ¹ ‰ © 5 10 ¹ ‰ © 10 ¹ © 10 10 ¹ © 10 10 ¹
§ – 9S · § – S 3S · § 7 S · § – 7 S 9S ·
(c) ¨ – S, ¸‰¨ , ¸ ‰ ¨ , S¸ (d) ¨ , ¸
© 10 ¹ © 10 10 ¹ © 10 ¹ © 10 10 ¹
7 The number of points (x,y) inside the circle x2+y2=4 satisfying the equation tan4x+cot4x+1=3sin2y is
(a) 1 (b) 2 (c) 4 (d) 8
8 If sinx = 2sin T , cosx = tany, tanx=cosz and cosy = tanz, then T is
(a) 18° (b) 36° (c) 54° (d) 72°
9 Read the following and then answer the questions.
Consider the system of equations
sinxcos2y = (a2–1)2+1, cosxsin2y = a+1
(i) the number of values of a for which the system has a solution is
(a) 1 (b) 2 (c) 3 (d) infinite.
(ii) The number of values of x  [0,2 S ] when the system has solution for permissible values of a
is
(a) 1 (b) 2 (c) 3 (d) 4
(iii) The number of values of y [0,2 S ] when the system has solution for permissible values of a
(a) 2 (b) 3 (c) 4 (d) 5
10 The total number of solutions of sin{x} = cos{x} in[0,2 S ] is
(a) 5 (b) 6 (c) 8 (d) None of these

330
§ 1·
11 If sinx+cosx= ¨¨ y  ¸¸ , x  [0, S ] then
© y¹

S 3S
(a) x , y=1 (b) y = 2 (c) x (d) None of these.
4 4
12 If the inequatlity sin2x+acosx+a2 > 1+cosx hods for any x  R then the largest negative integral
value of ‘a’ is
(a) –4 (b) –3 (c) –2 (d) –1
13 The number of solutions of x  [0,2 S ] for which [sinx+cosx] = 3+[–sinx]+[–cosx] is
(a) 0 (b) 4 (c) infinite (d) 1
14 The arithmetic mean of the roots of the equation 4cos x–4cos x–cos(315 S +x) =1 in the interval
3 2

(0,315) is
(a) 50 S (b) 51 S (c) 100 S (d) 315 S
15 Values of x & y satisfying the equation
sin7y = |x3–x2–9x+9| + |x3–x2–4x+4| + sec22y+cos4y are
S
(a) x=1, y=n S , n  Z (b) x=1, y=2n S + , n Z
2
(c) x=1, y=2n S , n  Z(d) None of these

'Note : Questions with * have more than one correct option'

Answers
1. d 2. d 3. c 4. a, b 5. a o r ; b o p ; c o q
6. c 7. c 8. a 9. (i) a (ii) b (iii) d 10. b
11. a 12. b 13. a 14. b 15. b

331
TRIGONOMETRY - I
Properties of Triangles
1. SINE RULE
sin A sin B sin C
k
a b c
a b c
or Ȝ
sin A sin B sin C

2. COSINE RULE
b 2  c2 – a 2
i. cos A
2bc
c2  a 2 – b2
ii. cos B
2ca
a 2  b2 – c2
iii. cos C
2ab
3. PROJECTION FORMULAE
i. b cos C + c cosB = a
ii. c cos A + a cosC = b
iii. a cos B + b cosA = c
4. NAPIER’S ANALOGY
§ B – C· b–c A
i. tan¨ ¸ cot
© 2 ¹ bc 2

§ C – A· c–a B
ii. tan¨ ¸ cot
© 2 ¹ ca 2

§ A – B· a–b C
iii. tan ¨ ¸ cot
© 2 ¹ ab 2

332
sin
A s – b s – c
5. a. i.
2 bc

sin
B s – c s – a
ii.
2 ca

sin
C s – a s – b
iii.
2 ab

A s s – a
b. i. cos
2 bc

B s s – b
ii. cos
2 ca

C s s – c
iii. cos
2 ab

tan
A s – b s – c
c. i.
2 s s – a

tan
B s – c s – a
ii.
2 s s – b

tan
C s – a s – b
iii.
2 s s – c

A s s – a
d. i. cot
2 s – b s – c
B s s – b
ii. cot
2 s – c s – a
C s s – c
iii. cot
2 s – a s – b
2ǻ 2
e. i. sin A s s – a s – b s – c
bc bc
2ǻ 2
ii. sin B s s – a s – b s – c
ca ca
2ǻ 2
iii. sin C s s – a s – b s – c
ab ab

333
6. RADIUS OF THE CIRCUMCIRCLE ‘R’
a b c abc
R
2 sin A 2 sin B 2 sin C 4ǻ

7. RADIUS OF THE INCIRCLE ‘r’


ǻ
r s – a tan A s – b tan B = s – c tan C 4R sin
A B
sin sin
C
s 2 2 2 2 2 2

8. RADII OF THE EXCIRCLES r1,r2 and r3


ǻ A A B C
i. r1 s tan 4R sin cos cos
s–a 2 2 2 2
ǻ B A B C
ii. r2 s tan 4R cos sin cos
s–b 2 2 2 2
ǻ C A B C
iii. r3 s tan 4R cos cos sin
s–c 2 2 2 2

334
B C
a cos cos
r1 2 2 etc
iv. A
cos
2
v. r1+r2+r3 = r + 4r
1 1 1 1
vi.  
r1 r2 r3 r
vii. r1r2+r2r3+r3r1 = s2
viii. a cosA + b cosB + c cosC = 4R sinA sinB sinC
ix. a cotA + b cotB + c cotC = 2(R+r)
B C C A A B
a sin
sin b sin sin c sin sin
Also, r 2 2 2 2 = 2 2
A B C
cos cos cos
2 2 2
9. If length of the median AD, BE and CF are given, then sides can be determined by using
the formula

a2
4
3

2BE 2  2CF 2 – AD 2
Similarly b2
4
3

2CF 2  2AD 2 – BE 2
and c2
4
3

2AD 2  2BE 2 – CF 2
10. Distance of the orthocentre from the vertex A is 2R cosA.
11. PEDAL TRIANGLE
Triangle formed by joining the foot of perpendiculars drawn from vertices to opposite sides of
a given tri-angle is called pedal triangle.
Length of the sides of pedal triangle of given ǻABC are a cos A, b cos B and c cos C respectively..
Angles of the pedal triangle are 180° – 2A, 180° – 2B and 180° – 2C respectively.
12. EXCENTRIC TRIANGLE
If I1, I2, and I3 are the centres of excircles, then ǻI1I 2 I 3 is called excentric triangle. I2, A, I3 ;
I3, B, I1 and I2, C, 12 are collinear. ǻABC is the pedal ǻ of ǻI1I 2 I 3 . Incentre ‘I’ of
the ǻABC will be orthocentre of ǻI1I 2 I 3 .
13. If median AD inclined at angles Į, ȕ, Ȗ with BC, CA and AB respectively then
b sin C
sin Į
2 b 2  2c 2 – a 2
a sin C a sin B
sin ȕ and sin Ȗ
2 b 2  2c 2 – a 2 2 b 2  2c 2 – a 2

335
14. Distance between orthocentre and circumcentre
= R 1 – 8 cos A cos B cos C
15. Distance between circumcentre and incentre is
A B C
R 1 – 8 sin sin sin R R – 2r
2 2 2
16. Distance between circumcentre and centre of excircles are
A B C
OI 1 R 1  8 sin cos cos R R  2r1
2 2 2

A B C
OI 2 R 1  8 cos sin cos R R  2r2
2 2 2

A B C
OI3R 1  8 cos cos sin R R  2r3
2 2 2
17. Length of the angle bisectors
2bc A bc sin A
AD cos
bc 2 A
b  c sin
2
2ca B 2ab C
BE cos and CF cos
ca 2 ab 2
18. Area of quadrilateral ABCD, if sum of a pair opposite angle is 2Į is
s – a s – b s – c s – d – abcd cos 2 Į
where 2s = a+b+c+d
19. m-n THEOREM
(m+n) cot ș = m cot Į – n cot ȕ
(m+n) cot ș = n cotB – m cotC

336
SOME EXTRA TIPS
1. i. ¦a 3
sin B – C 0

ii. ¦a 3
cos B – C 3abc
2. i. If a cosB = b cosA, then the triangle is isosceles
ii. If a cos A = b cos B, then the triangle is isosceles or right angled
iii. If a2+b2+c2 = 8R2, then the triangle is right angled
iv. If cos2A + cos2B + cos2C = 1, then the triangle is right angled
sin B
v. If cosA = , then the triangle is isosceles
2 sin C
a b c
vi. If , then the triangle is equilateral
cos A cos B cos C
3
vii. If cosA+cosB+cosC = ,then the triangle is equilateral
2
3 3
viii. If sinA+sinB+sinC = , then the triangle is equilateral
2
ix. If tanA+tanB+tanC = 3 3 , then the triangle is equilateral
x. If cotA+cotB+cotC = 3 , then the triangle is equilateral
3. a. The circumcentre lies (i) inside an acute angled triangle (ii) outside an obtuse angled
triangle.
b. The circumcircle of a right angled triangle is the mid point of the hypotenuse.
c. The orthocentre of a right angled triangle is the vertex at the right angle.
4. Triangle is equilateral if any one of the following holds:
a. R = 2r b. r1 = r2 = r3 c. r:R:r1 = 1:2:3
5. Triangle is right angled if r : R : r 1 = 2:5:12
6. If r1,r2,r3 are in H.P. iff a,b,c are in A.P.
7. In an equilateral triangle:
3a 2 a R
a. area = b. R c. r
4 3 2
3R
d. r1 r2 r3 e. r R r1 1 : 2 : 3
2

337
EXAMPLES
§1 ·
1 In triangle ABC, 2ac sin ¨ (A – B  C ¸ is equal to
©2 ¹
(a) a2+b2–c2 (b) c2+a2–b2 (c) b2–c2–a2 (d) c2–a2–b2
Solution
§1 ·
Given that 2ac sin ¨ (A – B  C ¸
©2 ¹
§1 ·
= 2ac sin ¨ (S – 2B) ¸
©2 ¹
§§ S ··
= 2ac sin ¨¨ ¨ – B ¸ ¸¸
©© 2 ¹¹
= 2ac cos B
§ a 2  c2 – b 2 ·
= 2ac ¨¨ 2ac
¸¸
© ¹
= a +c –b
2 2 2

Correct option is ‘b’


S
2 Let ABC be a triangle such that ‘ ACB = and let a,b and c denote the lengths of the side
6
opposite to A,B and C, respectively. The values of x for which a = x2+x+1, b= x2–1 and
c = 2x+1 is (are)

(a) – 2  3 (b) 1+ 3 (c) 2+ 3 (d) 4 3
Solution
S
Given that ‘ ACB =
6
a 2  b 2 – c2
cos( ‘ ACB) = cos C =
2ab
S ( x 2  x  1) 2  ( x 2 – 1) 2 – (2x  1) 2
cos =
6 2( x 2  x  1)(x 2 – 1)

4 3
3 (x 2  3 x  2)(x 2 – x)  x 2 – 1) 2 3 2x  2 x – 3x 2 – 2x  1
= Ÿ =
2 2(x 2  x  1)(x 2 – 1) 2 2(x 2  x  1)(x 2 – 1)

x( x 2  3 x  2 )  x 2 – 1 2
3 = 3 ( 2x –- 2 x –1)( x 2– 1)
x2  x  1 = =
2 2(x 2  x  1)(x 2 – 1)
2
2x  2 x – 1
3 =
x2  x  1

338

3 – 2 x2 +
3 – 2 x+
3 1 = 0

2 – 3 r 3
2 3 – 2
x=

x = – 2  3 , 1+ 3
as x>0 ? x = 1+ 3
Correct option is ‘b’

3 In ' ABC, interval angle bisector of ‘ A meets side BC in D. DE A AD meets AC in E and AB


in F. Then
2bc A
(a) AE is HM of b and c (b) AD = cos
bc 2
4bc A
(c) EF = sin (d) ' AEF is isosceles
bc 2
Solution
In ' AFE, we get AF = AE
? ' AFE is an isosceles '
ar( ' ABC) = ar ( ' ABD)+ar( ' ADC)
A
1 1 A 1 A
= bc sinA = c ADsin + bADsin
2 2 2 2 2 A A
c 2 2
A A A b
2bcsin cos = ADsin (b+c)
2 2 2
E
A B
2bc cos D C
AD = 2 a
F
bc
A
Also AD = AE cos
2
A
A 2bc cos
AEcos = 2
2 bc
? AE is HM of b and c
A A
A 2.2bccos . tan
Again EF = 2DE = 2AD tan = 2 2
2 bc
A
4bc sin
= 2
bc
Hence option a, b, c, d are correct

339
A
4 In a triangle ABC with fixed base BC, the vertex A moves such that cosB+cosC = 4 sin2
2
If a, b and c denote the lengths of the sides of the triangle opposite to the angles A,B and C
respectively, Then
(a) b+c = 4a (b) b+c = 2a
(c) locus of point A is an ellipse (d) locus of point A is a pair of straight lines.
Solution
A A cos B  C
Given that cosB+cosC = 4sin2
2 sin = sin S - B  C =
2 2 2 2
BC B–C A --
2cos cos = 4sin2
2 2 2
§B– C·
cos¨ ¸
© 2 ¹ 2
§BC· =
cos¨ ¸ 1
© 2 ¹
Componendo and Dividendo
B–C BC
cos  cos
2 2 2 1
B–C BC =
cos – cos 2–1
2 2
B C
2 cos cos
2 2
B C =3
2 sin sin
2 2
C B 1
tan tan =
2 2 3
s–a 1
=
s 3
3s–3a=s
2s=3a
a+b+c = 3a
b+c = 2a Ÿ AB+AC>BC
? locus of A is an ellipse.
Hence options ‘b’ & ‘c’ are correct
PRACTICE QUESTIONS
A B
1 The roots of the equation 6x2–5x+1 = 0 are tan and tan where A, B, C are the angles of a
2 2
triangle, then
(a) a2+b2>c2 (b) a2+b2 = c2 (c) a2–b2= c2 (d) None of these

340
cos A cos B cos C
2 If = = and the side a = 2 then area of triangle is
a b c
3
(a) 1 (b) 2 (c) (d) 3
2
3 If in a triangle PQR, sinP, sinQ, sinR are in A.P then
(a) The altitudes are in A.P (b) The altitudes are in H.P
(c) The medians are in G.P (d) The medians are in A.P
1 1 1
4 If the sides a, b, c of ' ABC are in A.P then cot A, cot B, cot C are in
2 2 2
(a) A.P (b) G.P (c) H.P (d) None of these
5 If radius of the incircle of a triangle with sides 5p, 6p and 5p is 6, then p is equal to
(a) 4 (b) 6 (c) 8 (d) 10
ª sin 2 A  sin A  1º
6 In any ' ABC, ¦ « sin A » is always greater than
¬ ¼
(a) 9 (b) 3 (c) 27 (d) 10
7 If c = a +b , then 4s(s–a)(s–b)(s–c) =
2 2 2

(a) s4 (b) b2c2 (c) c2a2 (d) a2b2


a 2  b2  c2
8 If a, b, c, d be the sides of a quadrilateral, then the minimum value of is equal to
d2
1 1 1
(a) (b) (c) (d) 1
2 3 4
r1 r2 r3
9 + = =
bc ca ab
1 1 1 1
(a) – (b) 2R–r (c) r–2R (d) –
2R r r 2R
10 Passage
a abc
(a) = 2R, R =
sin A 4'
' A A B C
(b) r= = (s–a) tan = 4R sin sin sin
3 2 2 2 2
' A A B C
(c) r1 = = s tan = 4Rsin cos cos
s–a 2 2 2 2
(d) If P be the orthocenter of a ' ABC and its distances PA from the vertex A and PL from
the sides BC are PA = 2RcosA, PL = 2RcosBcosC
Answer the following questions based upon above passage
(i) If r1 = 2r2 = 3r3, then
a 4 a 5 a 3 a 5
(a) (b) (c) (d)
b 5 b 4 c 5 c 3

341
1 1 1 1
(ii) 2 + 2 +
r3
2 + 2 =
r1 r2 r

(a)
a 2  b2  c2
(b)
¦a 2

(c) 4R (d) 4r
s2 '2
r1 – r r2 – r
(iii) + =
a b
a b c
(a) r1 (b) r2 (c) r3 (d) None of these

S
(iv) In a triangle ABC, let ‘ c = . If r is the inradius and R is the circumradius of the
2
triangle then 2(r+R) is equal to
(a) a+b (b) b+c (c) c+a (d) a+b+c
Answers
1. b 2. d 3. b 4. a 5. a 6. c 7. d 8. b
9. d 10. (i).b,d (ii). b (iii). c (iv). a

342
TRIGONOMETRY - II
Properties of Triangles
EXAMPLES
1 In a ' ABC the inradius and three exradii are r, r1, r2 and r3 respectively. In usual notations the
value of r.r1.r2.r3 is equal to
abc
(a) 2' (b) '2 (c) (d) None of these
4R
Solution
' ' ' '
r.r1.r2 r3 = . .
s s–a s–b s–c
'4
=
'2
= '2
correct option is ‘b’
2 The distance between the circumcentre and the orthocenter of a triangle ABC is
(a) R 1 – 8cosAcosBcosC (b) R 1  8cosAcosBcosC
(c) R 1 – 4cosAcosBcosC (d) None of these
Solution
Let O & P be circumcenter and orthocentre respectively of ' ABC
OF A AB.
We have ‘ OAF = 90°–C (' ‘ AOF = ‘ C)
= ‘ PAE (In ' ADC)
‘ OAP = ‘ A– ‘ OAF– ‘ PAE AE A
= ‘ A–(90– ° ‘ C)–(90– ° ‘ C)
= ‘ A+2 ‘ C–180° F
= ‘ A+2 ‘ C–( ‘ A+ ‘ B+ ‘ C) c E
c O
= ‘ C– ‘ B
P
OA = R (circum radius) and PA = 2RcosA
In ' OAP B D C
OP = OA +PA –2(OA) (PA)COS(C–B)
2 2 2

= R2+4R2cos2A–4R2cosAcos(C–B)
= R2+4R2cosA(cosA–cos(C–B)
= R2+4R2cosA(–cos(B+C)–cos(C–B))
= R2–4R2cosA(cos(B+C)+cos(C–B))
= R2–4R2cosA(2cosBcosC)
= R2–8R2 cos AcosBcosC
OP2 = R2 (1–8cosAcosBcosC)
OP = R 1 – 8cosAcosBcosC
Hence ‘a’ is the correct option
3 The radii r1, r2, r3, of escribed circles of the triangle ABC are in HP. If its area is 24sq.cm and its
perimeter is 24cm, then the lengths of its sides are

343
(a) 4, 6, 8 (b) 3, 9, 11 (c) 6, 8, 10 (d) None of these
Solution
Given that r1, r2 r3 are in HP
s–a s–b s–c
i.e. , , are in A.P
' ' '
or s–a, s–b, s–c are in A.P
or a, b, c are in AP
a+c = 2b
a+b+c = 24
3b = 24
b=8
24
s= = 12
2
a+c = 16
c = 16–a
' = s(s–a)(s–b)(s–c)
2

24×24 = 12(12–a)(12–8)(12–(16–a))
24×24 = 12×4(12–a)(a–4)
12 = (12–a)(a–4)
a2–16a+60 = 0
a2–10a–6a+60 = 0
(a–10)(a–6) = 0
a = 10 or a = 6
a = 10,c = 6, b=8 or a= 6, b=8, c= 10
correct option is ‘c’
r1  r2
4 In ' ABC the value of is always equal to
1  cos c
2r 2 2R 2
(a) 2r (b) 2R (c) (d)
R r
Solution
We know
A B C B C A
r1+r2 = 4R sin cos cos +4Rsin cos cos
2 2 2 2 2 2
C§ A B A B·
= 4Rcos ¨ sin cos  cos sin ¸
2 © 2 2 2 2¹

C § § A  B ··
= 4Rcos ¨¨ sin ¨ ¸ ¸¸
2 © © 2 ¹¹
C§ C·
=4Rcos ¨ cos ¸
2 © 2¹
C
= 4Rcos2
2
= 2R(1+cosC) (1+cos2 T =2cos2 T )
344
r1  r2
? = 2R
1  cos C
5 Let ABCD be a quadrilateral with area 18 with side AB parallel to the side CD and AB = 2CD.
Let AD be perpendicular to AB and CD.If a circle is drawn inside the quadrilaterals ABCD
touching all the sides, then its radius is
3
(a) 3 (b) 2 (c) (d) 1
2
Solution
Given that AB = 2CD
Let CD = a ? AB = 2a
? B(2a,0)
and c(a,2r)
Let the centre of in circle be (r,r)
Where r is the radius of the circle
Since AB || A to CD
? ABCD is a trapezium
1
ar(ABCD) = h(sum of parallel sides)
2
1
= ×2r(a+2a)
2
18 = 3ar
ar = 6
Now BC is a tangent to the circle
(x–r)2+(y–r)2 = r2
2r
Equation of tangent is y = – (x–2a)
2
2rx+ay–4ar = 0
2r 2  ar – 4ar
?r =
4r 2  a 2

2r 2 – 3ar
r=
4r 2  a 2
squaring
4r4+a2r2 = 4r4+9a2r2–12ar3
12ar3 = 8a2r2
3r = 2a [? ar = 6]
3r2 = 2ar
3r2 = 12
r2 = 4
r=2

6 In a triangle ABC, a : b : c = 4 : 5 : 6. The ratio of the radius of the circumcircle to that of the
incircle is

345
15 11 16 16
(a) (b) (c) (d)
4 5 7 3
Solution
abc '
R= , r=
4' s
R abc s
= ×
r 4' '
abc u s
=
4'2
abc u s
= 4(s(s – a )(s – b)(s – c))

2abc
= (2s – 2a )(2s – 2b)(2s – 2c)

2abc
= (a  b – c)(c  a – b)(b  c – a )

2 .4 .5 .6
=
3 .5 .7
16
=
7
1 1 1 1
7 The value of r 2 + r 2 + r 2 + 2 is
1 2 3 r

a 2  b2  c2 '2 a 2  b2  c2
(a) 0 (b) (c) (d)
'2 a 2  b2  c2 '
Solution
1 (s – a ) 2 1 (s – b ) 2 1 (s – c) 2 1 s2
?r2 = ; r 2 = ; r2 = ; =
1 '2 2 '2 3 '2 r2 '2
1 1 1 1 s 2  (s – a ) 2  (s – b) 2  (s – c) 2
?r 2 + r 2 + r2 + 2 =
1 2 3 r '2
4s 2  a 2  b 2  c 2 – 2s(a  b  c)
=
'2
4s 2  a 2  b 2  c 2 – 4s 2
= (a+b+c = 2s)
'2
a 2  b2  c2
=
'2

346
PRACTICE QUESTIONS
a 3  b3  c 3
1 In ' ABC, a t b t c, if = 8, then the maximum value of a is
sin 3 A  sin 2 B  sin 3 c
1
(a) (b) 2 (c) 8 (d) 64
2
2 Sides of triangle ABC are in AP. If a <min{b,c}, then cosA may be equal to
3c – 4b 3c – 4b 4c – 3b 4c – 3b
(a) (b) (c) (d)
2b 2c 2b 2c
lin
3 – 4 sin A sin C
3 In a ' ABC, angles A,B,C are in AP. Then x o c is
|A –C|
(a) 1 (b) 2 (c) 3 (d) 4
4 In a triangle ABC, 2a +4b +c = 4ab+2ac, then the numerical value of cosB is equal to
2 2 2

3 5 7
(a) 0 (b) (c) (d)
8 8 8
5 If a, b, c be the sides of a ' ABC and if roots of the equation a(b–c)x2+b(c–a)x+c(a–b) = 0 are
A B C
equal, then sin2 , sin2 , sin2 are in
2 2 2
(a) AP (b) GP (c) HP (d) AGP
2 2 1 8a
6 In a ' ABC sides a, b, c are in AP and + + = then the maximum value
1! 9! 3! 7! 5! 5! (2b)!
of tanA tanB is equal to
1 1 1 1
(a) (b) (c) (d)
2 3 4 5
¦ tan A
7 If A, B, C, D are the angles of quadrilateral, then is equal to
¦ cot A
(a) S tanA (b) S cotA (c) 2
¦ tan A (d) 2
¦ cot A
bc ca ab
8 With usual notations, if in a ' ABC, = = , then cosA: cosB: cosC is equal
11 12 13
to
(a) 7 : 19 : 25 (b) 19 : 7 : 25 (c) 12 : 14 : 20 (d) 19 : 25 :20
§ sin2 A  sin A  1·
9 In and ' ABC, – ¨¨ sin A
¸ is always greater than
¸
© ¹
(a) 9 (b) 3 (c) 27 (d) None of these
10 The radius of the circle passing through the centre of incircle of ' ABC, and through the end
points of BC is given
§a· §a· §A· §a· §A·
(a) ¨ ¸ cosA (b) ¨ ¸ sec ¨ ¸ (c) ¨ ¸ sinA (d) asec ¨ 2 ¸
©2¹ ©2¹ ©2¹ ©2¹ © ¹

347
11 If in a ' ABC, a, b, c are in AP and p1, p2, p3 are the altitude from the vertices A, B, C respectively
then
(a) p1, p2, p3 are in AP (b) p1, p2, p3 are in HP
3R 1 1 1 3R
(c) p1+ p2 +p3 d (d) p + p + p d
' 1 2 3 '
12 If tanA, tanB are the roots of the quadratic abx –c x+ab = 0, where a, b, c are the sides of a
2 2

triangle, then
a b
(a) tanA = (b) tanB =
b a
c2
(c) cosC = 0 (d) tanA+tanB =
ab
13 If sin E is the GM between sin D and cos D , then cos2 E is equal to
§S · §S · §S · §S ·
(a) 2sin2 ¨ 4 – D ¸ (b) 2cos2 ¨ 4 – D ¸ (c) 2cos2 ¨ 4  D ¸ (d) 2sin2 ¨ 4  D ¸
© ¹ © ¹ © ¹ © ¹
14 Passage 1
If p1, p2, p3 are altitudes of a triangle ABC from the vertices A, B, C respectively and ' is the
area of the triangle and s is the semipermanent of the triangle. On the basis of above information,
answer the following questions :
1 1 1 1
(i) If p + p + p = then the least value of p1, p2, p3 is
1 2 3 2
(a) 8 (b) 27 (c) 125 (c) 216
cos A cos B cos C
(ii) The value of p1 + p2 + p 3 is

1 1 a 2  b2  C2 1
(a) (b) (c) (d)
r R 2R '

b 2 p1 c 2p 2 a 2p 3
(iii) The minimum value of + + is
c a b
(a) ' (b) 2 ' (c) 3' (d) 6'
(iv) The value of p1 +p2–2+p3–2 is
–2

¦ a 2 Sa 3 ¦a
2
Sa 2
(a) (b) (c) (d)
4 '2 8'3 4 '2 8'2
(v) In the triangle ABC, the altitudes are in AP, then
(a) a, b, c are in AP (b) a, b, c are in HP
(c) a,b,c are in GP (d) angles A, B, C are in AP
15 Passage II
In a triangle if the sum of two sides is x and this product is y (x t 2 y ) such that
(x+z) (x–z) = y where z is the third side of the triangle.

348
On the passes of above information, answer the following questions.
(i) Greatest angle of the triangle is
(a) 105 (b) 120 (c) 135 (d) 150
(ii) Cerium radius of the triangle is
(a) x (b) y (c) z (d) None of these
(iii) In radius of the triangle is
y z y 3 z 3
(a) 2(z  x )
(b) 2( x  y )
(c) (d)
zx xy
(iv) Area of the triangle is
y 3 x 3 z 3
(a) (b) (c) (d) None of these
4 4 4
(v) The sides of the triangle are
x r x 2 – 4y y r y 2 – 4z z r z 2 – 4x
(a) , z (b) ,z (c) ,z (d) None of these
2 2 2
Assertion and Reason
r1  r2  r3
16 Assertion (A) : In any ' ABC, the minimum value of is
r
Reason (R) : AM t GM
(a) A (b) B (c) C (d) D
17 Assertion (A) : If A, B, C D are angle of a cyclic quadrilateral then ¦ sin A = 0
Reason (R) : If A, B, C, D are angles of cyclic quadrilateral then ¦ cos A = 0
(a) A (b) B (c) C (d) D
§ 1– a2 ·
18 Assertion (A) : In any ' ABC, the square of the length of the bisector AD is bc ¨¨ (b c) 2 ¸¸
©  ¹
2bc A
Reason (R) : In any ' ABC, length of bisector AD is cos
bc 2
(a) A (b) B (c) C (d) D
Integer Type Questions
19 If the radius of the circumcircle of a triangle is 12 and that of the incircle is 4, then the square
of the sum of radii of the escribed cirle must be
§ A·
¨ ¦ a cos 2 ¸
¨ 2¸
20 In a ' ABC, the maximum value of 1000 ¨ a  b  c ¸ must be
¨ ¸
© ¹
21 Matrix Match Type
Column I Column II
(a) In a ' ABC, if 2a2+b2+c2 = 2ac+2ab, then (p) ' ABC is equilateral '
(b) In a ' ABC, if a2+b2+c2 = 2 b(c+a), then (q) ' ABC is right angled '
(c) In a ' ABC,if a2+b2+c2 = bc+ca 3 , then (r) ' ABC is scalene '

349
(s) ' ABC is scalene right
angled '
(t) Angles B, C, A are in AP

Answers
1. b 2. d 3. a 4. d 5. c 6. b 7. a 8. a
9. c 10. b 11. b,d 12. a, b, c, d 13. a,c 14. (i). b (ii). b
(iii). d (iv). c (v). b 15. (i). b (ii). d (iii).c (iv). a (v). a
16. b 17. d 18. a 19. 2704 20. 0750 21. a o p,t b o q,s c o q, r, t.

350
TRIGONOMETRY - III
Properties of Triangles - Problem Solving

Properties and Solutions of Triangles


Notation :
Vertices A,B,C
Sides a, b, c
Centroid G (Point of intersection of medians )
Orthocentre O (Point of intersection of altitudes)
Circumcentre S (Point of intersection of perpendicular bisectors of the sides)
Incentre I (Point of intersection of internal bisectors of the angles)
Excentres I1,I2,I3 (Point of intersection of internal bisector of an angle & external bisectors
of the other two angles).
Circumradius R (radius of circle with centre S and passing through the vertices)
Inradius r (radius of circle with centre I and touching the sides)
Exradii r1, r2, r3 ( radii of circles with centres I1,I2,I3 respectively and touching the sides)
abc
Semiperimeter s =
2
Area of triangle '
Concepts and Formula
1 Sine law
a b c
In any triangle ABC, = =
sin A sin B sin C
2 Cosine law
In any triangle ABC
b2  c2 – a 2
x cosA = or a2 = b2+c2–2bc cosA
2bc
c2  a 2 – b 2
x cosB = or b2 = c2+a2–2ca cosB
2ac
a 2  b 2 – c2
x cosC = or c2 = a2+b2–2ab cosC
2ab
3 Projection formula
x a = bcosC+ccosB
x b = ccosA+acosC
x c = acosB+bcosA
4 Napier’s analogy (Tangent rule)
B–C b–c A
x tan = cot
2 bc 2

351
C–A c–a B
x tan = cot
2 ca 2
A–B a–b C
x tan = cot
2 ab 2
5 Auxiliary formulae
(Trigonometrical ratios of half angles of a triangle)
A (s – b)(s – c) B (s – c)(s – a ) C (s – a )(s – b)
x sin = ; sin = ; sin = ;
2 bc 2 ca 2 ab

A s(s – a ) B s(s – b) C s(s – c)


x cos = ; cos = ; cos = ;
2 bc 2 ca 2 ab

A (s – b)(s – c) B (s – c)(s – a ) C (s – a )(s – b)


x tan = ; tan = ; tan = ;
2 s(s – a ) 2 s(s – b) 2 s(s – c)

§ ' · § ' · § ' ·


¨¨ ¸¸ ¨¨ ¸¸ ¨¨ ¸¸
© s(s – a ) ¹ © s(s – b) ¹ © s(s – c) ¹
6 Area of Triangle
x Area of triangle ABC ' = s(s – a )(s – b)(s – c) (Hero’s formula)
1 1 1
x ' = 2 absinC = 2 bcsinA = 2 casinB
2 2 2
1 a sin B sin C 1 b sin C sin A 1 c sin A sin B
x ' = = =
2 sin(B  C) 2 sin(C  A) 2 sin(A  B)
2' 2
7 x sinA = = s(s – a )(s – b)(s – c)
bc bc
2' 2
x sinB = = s(s – a )(s – b)(s – c)
ca ca
2' 2
x sinC = = s(s – a )(s – b)(s – c)
ab ab
sin A sin B sin C 2'
Also = = = A
a b c abc
D E
8 m – n Rule
In any triangle,
(m+n) cot T = m cot D – n cot E T
B m n C
= n cot B – m cot C
9 Circumcircle of a triangle.
Circle passing through the angular point of a ' ABC is called its circumcircle. Its radius is
denoted by R. The circum centre may lie within, outside or upon one of the sides of the

352
triangle. In a right angled triangle the circum centre is the mid-point of hypotenuse.
a b c
x R= = =
2 sin A 2 sin B 2 sin C
abc
x R=
4'
10 Incircle of a triangle
The circle which touches the sides is called inscribed circle or incircle. Its radius is denoted by
r.
'
x r=
s
A B C
x r = (s–a) tan = (s–b)tan = (s–c)tan
2 2 2
B C A C B A
a sin sin b sin sin c sin sin
2 2 2 2 2 2
x r= A = B = C
cos cos cos
2 2 2
A B C
x r = 4Rsin sin sin
2 2 2
11 Escribed circles of a triangle.
The circle which touches side BC and two sides AB & AC produced of ' ABC is called
escribed circle opposite to the angle A. Its radius is denoted by r1 .Similarly r2 & r3 denote the
radii of escribed circles opposite to angles B & C.
' ' '
x r1 = ; r2 = ; r3 =
s–a s–b s–c
A B C
x r1 = s tan ; r2 = s tan ; r3 = s tan
2 2 2

B C A C B A
a cos cos b cos cos c cos cos
2 2 2 2 2 2
x r1 = ; r = r3 =
A 2 B C
cos cos cos
2 2 2
A B C A B C A B C
x r1 = 4Rsin cos cos ; r2 = 4Rcos sin cos ; r3 = 4Rcos cos sin
2 2 2 2 2 2 2 2 2
12 Orthocentre and Pedal triangle
x The triangle MNP formed by joining the feet of the altitudes is called the pedal triangle
x The distance of orthocentre O from vertices A,B,C are 2RcosA, 2RcosB and 2RcosC
x Distance of O from sides are 2RcosBcosC, 2RcosCcosA and 2RcosAcosB.

353
x In ' MNP,, A

‘ M = S –2C; MN = acosA
N
‘ N = S –2B; NP = ccosC M
O
‘ P = S –2A; MP = bcosB
B P
C

i.e. sides of pedal triangles are a cosA(Rsin2A), bcosB(Rsin2B) and ccosC(Rsin2C).


x Circumradii of the triangles OBC, OCA & OAB are equal.
13 Note :
x Orthocentre of ' ABC is the incentre of pedal triangle MNP..
x Incentre I of ' ABC is the orthocentre of ' I1,I2I3.
x Centroid of ' ABC lies on the line joining the circumcentre to the orthocentre and divides it in
the ratio 1:2
x Circumcentre of the pedal triangle bisects the line joining the circumcentre of the triangle
to the orthocentre. I 3 A I 2

14 Excentral Triangle.
The triangle formed by joining the three excentres I1, I2, & I3 of I

' ABC is called the excentral or excentric triangle.


B C
Here AI1 A I2 I3 B
2

BI2 A I1 I3
& CI3 A I1 I2
I
So clearly ' ABC is pedal triangle of excentral triangle ' I1I2I3
1

A B C
x Sides of excentral trangle are 4R cos , 4R cos and 4Rcos and its angles are
2 2 2
S A S B S C
– , – , –
2 2 2 2 2 2
A B C
x II1 = 4Rsin ; II2 = 4Rsin ; II3 = 4Rsin
2 2 2
A
15 Nine point circle
Circle circumscribing the pedal triangle of a given triangle
bisects the sides of the given triangle and also the line
joining the vertices of the given triangle to the orthocentre F E
of the given triangle. This circle is known as nine point O
circle.
i.e. Nine point circle passes through the mid point of
the sides, feet of the perpendiculars and the mid points B L D C
of the line joining the orthocentre to the angular points.
16 Distance between special points
x Distance between excentre and circumcentre
A B C
OI1 = R 1 8 sin cos cos
2 2 2

354
A B C
OI2 = R 1 8 cos sin cos
2 2 2
A B C
OI3 = R 1 8 cos cos sin
2 2 2
x Distance between cirumcentre & orthocentre is 1 – 8 cos A cos B cos C

x Distance between cirumcentre & incentre is R 2 – 2Rr

x Distance between incentre & orthocentre is 2r 2 – 4R 2 cos A cos B cos C

x Perimeter (P) and area (A) of a regular polygon of n sides inscribed in a circle of radius r are
S 1 2S
given by P = 2nrsin and A = nr2sin .
n 2 n

x Perimeter (P) and area (A) of a regular polygon of n sides circumscribed about a given circle
S 1 2S
of radius r are given by P = 2nrtan and A = nr2tan
n 2 n
17 Length of medians (Apollonious rule)
b 2  c2 a2
m12 = –
2 4
c 2
 a 2
b 2
m2 =
2

2 4
a 2
 b 2
c 2
m32 = –
2 4
3 2 2 2
Also m12+m22+m32 = (a +b +c )
4
If A = 90°, then m22+m32 = 5m12
4
Area = m(m – m1 )(m – m 2 )(m – m 3 ) where 2m = m1+m2+m3.
3
18 Altitudes
2' 2' 2'
h1 = , h2 = , h3 = ,
a b c
1 1 § 1 1 ·§ 1 1 ·§ 1 1 · 2 1 1 1
Area is given by = 4 ¨¨ – ¸¸¨¨ – ¸¸¨¨ – ¸¸ where = h + h + h
' h © h h1 ¹© h h 2 ¹© h h 3 ¹ h 1 2 3
2bc A 2ca B
19 Length of internal bisectors of angles A,B,C are given by cos ; cos ;
bc 2 ca 2
2ab C
cos respectively
ab 2

355
20 Some useful results.

C
x r1+r2 = 4Rcos2
2
C
x r3–r = 4Rsin2 .
2
x r1+r2+r3–r = 4R
x rr1r2r3 = ' 2
1 1 1 1
x r1 + r2 + r3 = r

1 1 1 1 a 2  b 2  c2
x + r1
2 + 2 +
r2 r3
2 =
r2 '2

x s= r1r2  r2 r3  r3r1 = ¦r r 1 2

r1r2 r3 r1r2 r3
x ' = =
s
¦r r 1 2

r1r2 r3
x r=
¦ r1r2
(r1  r2 )(r2  r3 )(r3  r1 )
x R=
4¦ r1r2

r1 (r2  r3 ) r2 (r3  r1 ) r3 (r3  r2 )


a= ;b= ;c=
¦r r
1 2 ¦r r 1 2 ¦r r
1 2

21 Solution of triangles
(a) Solution of a general triangle.

Given To find Formulae used


a, b, c A, B, C x Find ' = s(s – a )(s – b)(s – c)
2' 2' 2'
x sinA = , sinB = , sinC = ,
bc ca ab
A '
or tan = etc.
2 s(s – a )

b 2  c2 – a 2
or cosA = etc.
2bc
(if a, b, c are sides)

356
AB C
a, b, C c, A, B x = 90° –
2 2
A–B a–b C
& tan = cot
2 ab 2
a sin C
x c= (sine rule)
sin A
or c2= a2+b2–2abcos C (cosine rule)
a, A, B C, a, b x C = 180° –(A+B)
a sin B a sin C
(or c, A, B) x b= ,c= (sine rule)
sin A sin A
c sin B c sin A
(or b= ,a= as the case
sin C sin C
may be)
b
a, b, A c, B,C x sinB = sinA ........................(i)
a
x C = 180° – (A+B)....................(ii)
a sin C
x c= ..............................(iii)
sin A
Now, the following cases arise.

A<90° A<90° A<90° A>90°


& a<bsinA & a= bsinA & a>bsinA (ambiguous case)
b sin A b sin A
Here sinB = sinB = 1 sinB = x If a d b, then B is
a a
also obtuse which
is not possible
Ÿ sin B>1not possible Ÿ B = 90° gives 2 angles x If a>b, then A>B & C
? No triangle is only one triangle such that will be an acute angle
possible is possible B1+B2 = 180° So solution exists
which is right two triangles
anlged at B are possible

357
(b) Solution of a right angled triangle
Let ‘ C = 90°
Given To find Formulae used
a,b A, B, c x c2 = a2+b2
a b
(two sides) x tanA= (or tanB = )
b a
x B = 90°–A(or A = 90°–B)
c,a A, B, b x b2 = c2–a2
a
(hypotenuse & x SinA =
c
one side) x b = ccosA or b=a cot A
x B = 90° – A
a, A B, b, c x B = 90° – A
(Side and angle) x b = a cot A
a
x c=
sin A
c,A B, a, b x B = 90° – A
(hypotenuse & angle) x a = c sin A
x b = c cos A
Solved Examples
4
1 If in ' ABC, a = 6, b = 3 and cos(A–B) = then its area is
5
(a) 8 (b) 9 (c) 6 (d) None of these
Solution : Using Napier’s analogy,
A–B a–b C §A –B· 1 – cos(A – B)
tan = cot and tan ¨ ¸ =
2 ab 2 © 2 ¹ 1  cos(A – B)

4
1–
1 6–3 C §A –B· 5 1
Ÿ = cot tan ¨ ¸ = 4 =
3 63 2 © 2 ¹ 1 3
5

C
Ÿ cot =1
2
C = 90°
1 1
? Area of ǻ = absinC = .6.3.sin90° = 9 square units
2 2
Ans (b)

358
2 If the angles A, B, C are the solutions of the equation tan3x–3k.tan2x–3tanx+k = 0, then the
' ABC is
(a) isosceles (b) equilateral (c) acute angled (d) None of these
Solution: tanA, tanB, tanC are roots of the given equation
? tanA+tanB+tanC = 3k, tanAtanB+tanBtanC+tanCtanA= –3 and tanA.tanB.tanC = – k
But tanA+tanB+tanC = tanAtanBtanC
Ÿ 3k = – k Ÿ 4k = 0 gives k= 0
? tanAtanBtanC = 0
Ÿ tanA = 0 or tanB = 0 or tanC = 0 which is not possible
Ans : (d)
3 In ' ABC
sin4A+sin4B+sin4C = sin2Bsin2C+2sin2Csin2A+2sin2Asin2B, then ‘ A =
S 5S S 5S 5S S
(a) , (b) , (c) , (d) None of these
6 6 6 3 6 3
Solution : Given equation is,
a4+b4+c4 = b2c2+2c2a2+2a2b2 (using sine rule)
Ÿ a4+b4+c4+2b2c2–2c2a2–2a2b2 = 3b2c2.
(b2+c2–a2)2 = 3b2c2
(b 2  c 2 – a 2 ) 2 3
Ÿ 2 2 =
4b c 4
2 2
§ b2  c2 – a 2 · § 3·
¨¨ ¸¸ = ¨ ¸
2 bc ¨ 2 ¸
© ¹ © ¹

3 3
cos2A = Ÿ cosA = r
4 2
S 5S
ŸA = ,
6 6
Ans (a)
4 If the median of a triangle through A is perpendicular to AB, then
(a) 2tanA+tanB = 0 (b) 2tanA–tanB = 0 (c) tanA–2tanB = 0 (d) tanA+2tanB=0
Solution :
tanA = – tan (180–A)
C
x
=–2
c a
2

§x· D
= – 2¨ ¸ 2x
b
©c¹ x 2
a

= – 2tanB
S–A
tanA+2tanB = 0 E c c B
A
Ans (d)

359
S S
5 In ' ABC, medians AD and BE are drawn. If AD = 4, ‘ DAB = and ‘ ABE = , then the area
6 3
of ' ABC is (in square units)
64 8 16 32
(a) (b) (c) (d)
3 3 3 3 3 3 3 3
Solution :
In ' ABG
A
S AG
cos = S

6 AB 6

2 AD E
3
= (? AD = 4)
2 3 AB G
S

3 2u 4
3

Ÿ = B D C
2 3AB
16
Ÿ AB =
3 3
1 S
Area of ' ABD = AB.AD. sin
2 6
1 16 1 16
= . .4. = units2
2 3 3 2 3 3
32
? Area of ' ABC = 2 area of ' ABD = units2
3 3
Ans (d)
6 In ' ABC, a2+b2+c2 = ac+ 3 ab, then the triangle is
(a) equilateral (b) isosceles (c) right angled (d) None of these
Solution :
a2+b2+c2–ac– 3 ab = 0

§ D · § 3a 2 ·
¨  c 2 – ac ¸ + ¨¨  b 2 – 3ab ¸¸ = 0
¨ ¸
©  ¹ © 4 ¹
2
§a ·
2 § 3a ·
Ÿ ¨ – c ¸ + ¨¨ – b ¸¸ = 0
©2 ¹ © 2 ¹

a 3
Ÿ =c & a =b
2 2

360
2b
Ÿ a = 2c & a =
3
2b
a = 2c = =O
3
O 3O
?a = O, c = ,b=
2 2
? a2 = b2 + c2
? right angled
Ans (c)
7 In ' ABC, if cotA+cotB+cotC = 3 , then show that the triangle is equilateral
Solution :
In ' ABC, tanA+tanB+tanC = tanAtanBtanC
Ÿ cotBcotC+cotCcotA+cotBcotA = 1 ..........................(1)
Let cotA=x, cotB=y, cotC=z
given that x+y+z = 3
Squaring,
x2+y2+z2+2xy+2yz+2zx =3×1
x2+y2+z2+2xy+2yz+2zx = 3 (xy+yz+zx) using (1)
i.e. x2+y2+z2–xy–yz–zx = 0
Ÿ 2x2+2y2+2z2–2xy–2yz–2zx = 0
Ÿ (x–y)2+(y–z)2+(z–x)2 = 0
Ÿ x=y & y = z & z = x
Ÿ x=y=z
Ÿ cotA = cotB = cotC
Ÿ A=B=C
Ÿ The triange is equilateral.
PRACTICE QUESTIONS
S S sin ‘BAD
1 In ' ABC, ‘ B = , ‘ C = , Let D divides BC internally in the ratio 1:3, then
3 4 sin ‘CAD
is equal to
1 1 1 2
(a) (b) (c) (d)
6 3 3 3
2 If in ' PQR, sinP, sinQ, sinR are in A.P., then
(a) The altitudes are in A.P (b) The medians are in G.P
(c) The altitudes are in H.P (b) The medians are in A.P
S §P· §Q·
3 In ' PQR, ‘ R = , if tan ¨ ¸ and tan ¨ ¸ are the roots of ax2+bx+c = 0 (a z 0), then
2 ©2¹ ©2¹
(a) a+b = c (b) b+c = a (c) c+a = b (d) b = c

361
( A – B  C)
4 In ' ABC, 2ac sin =
2
(a) a2+b2–c2 (b) c2+a2–b2 (c) b2–c2–a2 (d) c2–a2–b2
S
5 In ' ABC, let ‘ C = . If r is the inradius and R is the circumradius of the triangle, then
2
2(r+R) is equal to
(a) a+b (b) b+c (c) c+a (d) a+b+c
2S
6 In radius of a circle which is inscribed in an isosceles triangle one of whose angle is is 3 , then
3
area of triangle is
(a) 4 3 (b) 12–7 3 (c) 12+7 3 (d) None of these
7 If the angles A,B&C of a triangle are in A.P and if a, b and c denote the lengths of the sides
a c
opposite to A,B&C respectively, then the value of the expression sin2C+ sin2A is
c a
1 3
(a) (b) (c) 1 (d) 3
2 2
8 Read the following passage and answer the questions.
Consider the circle x2+y2 = 9 and the parabola y2 = 8x. They intersect at P & Q in the first and
fourth quadrants, respectively. Tangents to the circle at P & Q intersect the x–axis at R and
tangents to the parabola at P&Q intersect the x axis at S.
(i) The ratio of the areas of ' PQS and ' PQR is
(a) 1: 2 (b) 1:2 (c) 1:40 (d) 1:8
(ii) The radius of the circumcircle of the ' PRS is
(a) 5 (b) 3 3 (c) 3 2 (d) 2 3
(iii) The radius of the incircle of the triangle PQR is
8
(a) 4 (b) 3 (c) (d) 2
3
9* Internal bisector ‘A of ' ABC meets side BC at D.A line drawn through D perpendicular to AD
intersects the side AC at E & side AB at F. If a,b,c represent sides of ' ABC, then
2bc A
(a) AE is the HM of b & c (b) AD = cos
bc 2
4bc A
(c) EF = sin (d) ' AEF is isosceles
bc 2
10* A straight line through the vertex P of a ' PQR intersects the side QR at the point S and the
circumcircle of the triangle PQR at the point T. If S is not the centre of the circumcircle, then
1 1 2 1 1 2
(a) + < (b) + >
PS ST QS u SR PS ST QS u SR

362
1 1 4 1 1 4
(c) + < (d) + >
PS ST QR PS ST QR
A
11* In a ' ABC with fixed base BC, the vertex A moves such that cosB+cosC = 4sin2 . If a, b and c
2
denote the lengths of the sides of the triangle opposite to the angles A, B and C respectively, then
(a) b+c = 4a (b) b+c = 2a (c) locus of point A is an ellipse
(d) locus of point A is a pair of straight line
12 Match the following :
Column I Column II
cos A cos B cos C
(a) In ' ABC , if = = (p) 2 3  6
a b c
1
and the side a = 2, then area of the triangle is (q)
6
3 3 3
a b c
(b) In ' ABC, a t b t c, if = 7,
sin A  sin 3 B  sin 3 C
3

then the maximum possible value of a is (r) 3


7
(c) Two sides of a triangle are given by the roots of the
equation x2–2 3 x+2 = 0. the angle between the
S
sides is .The perimeter of the triangle is (s) 3
3
7 R
13 In ' ABC if cosA+CosB+cosC = , then =
4 r
4 3 2 3
(a) (b) (c) (d)
3 4 3 2
a
2 2 1 8
14 In ' ABC, sides a,b,c are in A.P and + + = then the maximum value of
1!9! 3!7! 5!5! (2b)!
tanAtanB is
1 1 1 1
(a) (b) (c) (d)
2 3 4 5
1 1 1
15 If A1A2A3......An be a regular polygon of n sides and A A = A A + A A , then
1 2 1 3 1 4

(a) n=5 (b) n=6 (c) n=7 (d) None of these

'Note : Questions with * have more than one correct option'


Answers
1. a 2. c 3. a 4. b 5. a 6. c
7. d 8 (i) c (ii) b (ii) d 9. a,b,c,d 10. b,d
11. b,c 12. a o s; b o r; c o p 13.a 14. b 15. c

363
TRIGONOMETRY - I
Inverse Trigonometric Functions
A ƒunction ƒ: A o B is invertible iff it is a bijection. The inverse of ƒ is denoted by ƒ–1 and is defined
as ƒ–1(y) = x œ ƒ(x)=y. Trigonometric functions are periodic and hence they are not bijective. But if
we restrict their domains and codomains they can be made bijective and we can obtain their inverses.
1 Sin–1 x:
The symbol sin–1x or arcsinx denote the angle T so that sin T = x. As a direct meaning, sin–1x is
not a function, as it does not satisfy the requirements for a rule to become a function. But by a
suitable choice [–1,1] as its domain and standardized set [– S /2, S /2] as its range, then rule sin–
1
x is a single valued function
Thus sin–1x is considered as a function with domain [–1,1] and range [– S /2, S /2]
The graph of y = sin–1x is as shown below which is obtained by taking the mirror image, of the
portion of the graph of y = sin x from x = – S /2 to x = S /2, on the line y = x

cos–1x:
By following the discussions, similar to above, we have cos–1 x or arccos x as a function with domain
[–1,1] and range [0, S ]
The graph of y = cos–1 x is similarly obtained as the mirror image of the portion of the graph of y =
cos x from x = 0 to x = S

tan–1x:
We get tan–1x or arctanx as a function with domain R and range (– S /2, S /2). Graph of y = tan–1x

364
cosec–1x:
cosec–1x or arccosec x is a function with domain R–(–1,1) and range [– S /2, S /2] – {0}. Graph of
y = cosec–1x

sec–1x:
sec–1x or arcsec x is a function with domain R–(–1,1) and range [0, S ] – { S /2}.Graph of y = sec–
1
x is

cot–1x:
cot–1x or arccot x is a function with domain R and range (o, S ). Graph of y = cot–1x is

Property : “–x”
The graphs of sin–1x, tan–1x, cosec–1x are symmetric about origin.
Hence we get sin–1(–x) = –sin–1x
tan–1(–x) = –tan–1x
cosec–1(–x) = –cosec–1x
Also the graphs of cos–1x, sec–1x, cot–1x are symmetric about the point (0, S /2). From this,
we get
cos–1(–x) = S –cos–1x
sec–1(–x) = S –sec–1x
cot–1(–x) = S –cot–1x

365
Notes :
S
(i) x2+y2 d 1 & x,y t 0 Ÿ d sin–1x + sin–1 y d and x2+y2 t 1 & x,y t 0
2
S
Ÿ sin–1x + sin–1y d S
2d
S S
(ii) xy<1and x,y t 0 Ÿ 0 d tan–1x +tan–1y< ; xy>1 and x,y t 0 Ÿ <tan–1x +tan–1y< S
2 2
(iii) For x<0 or y<0 these identities can be used with the help of property “–x”
i.e. change x or y to –x or –y which are positive
Domain & range of inverse trigonometric Functions
Function Domain Range (Principal value branch)
sin x–1
[–1,1] >– S /2 ,– S / 2 @
cos–1x [–1,1] [0, S ]
tan x–1
(– ff (– S /2, S /2)
cot x –1
(– ff (0, S )
cosec–1x ( –f–1] ‰ [1,f) [– S /2,0) ‰ (0, S /2]
sec x –1
( –f–1] ‰ [1,f) [0, S /2) ‰ ( S /2, S ]
Note : If no branch of an inverse trigonometric function is mentioned, then it means the principal value
branch of the function.
Properties of Inverse Trigonometric Functions
1
­ ʌ ʌ
°° – 2nʌ  x,2nʌ – 2 d x d 2nʌ  2 ,n  Z
(i) sin–1(sinx) = ®
° 2n  1 ʌ – x, 2n  1 ʌ – ʌ d x d 2n  1 ʌ  ʌ , n  Z
°¯ 
2 2

Y

y=
–2
S

x –x x
y= y= X
S

0 S 2S

Period = 2 S & it is an odd function.


­ – 2nS  x ,2nS d x d 2n  1 S,n  Z
(ii) cos–1(cosx) = ®
¯2nS – x ,(2n – 1)S d x d 2nS,n  Z

366
Y

y=

y=
+x

x
–x

y=

2S
2S
y=

–x
X
–2S –S 0 S 2S

Period=2 S and it is an even function


ʌ ʌ
(iii) tan–1(tanx) = – nʌ  x , nʌ –  x  nʌ  , n  Z
2 2
Y

x –x
y= y=
S

 S  S/2 0 S /2 S
+x
y=
S

Period = S
(iv) cot–1(cotx) = – nʌ  x ,nʌ  x  ( n  1) ʌ,n  Z
Y

+x x
y= x–
S

y=
S

y=
X
S 0 S

Period = S
­ S
°°x – 2nS,2nS d x d (2n  1)S,x z 2S  2
®
(v) sec–1(secx) = ° S
– x  2 n S, ( 2 n  1) S d x d 2 n S, x z ( 2 n S – n  Z
¯°
 
2

367
Y

y=
+x

y=

2S
y=
2S

–x
y=

  S   S  S  0 S  S 
–x X
S 

Period = 2 S

­ S S
°° – 2nS  [2nS – 2 d x d 2nS  2
(vi) cosec–1(cosecx) = ®° S S
°¯(2n  1)S – x ,(2n  1)S – 2 d x d (2n  1)S  2  [ z nS,n  Z

y= 2
S

x–
y=

–x
y=
S

–x

X
S 0 S S
x
y=

Period = 2 S
2. (i) sin(sin–1x) = x, –1 d x d 1 (ii) cos(cos–1x) = x, –1 d x d 1

Y Y

1 1

X X
–1 0 1 –1 0 1
–1 –1

(iii) tan(tan–1x) = x, x  R (iv) cot(cot–1x) = x, x  R

368
Y Y

1 1

X X
–1 0 1 –1 0 1
–1 –1

(v) sec(sec–1x) = x, x  R (–f–1] ‰ [1,f

X
–1 0 1
–1

(vi) cosce(cosce–1x)= x, x  R (–f–1] ‰ [1,f

X
–1 0 1
–1

3. (i) sin–1x+cos–1x= S 2 , –1 d x d 1

(ii) tan–1x+cot–1x = S 2 , x  R

(iii) sec–1x+cosce–1x = S 2 , x  R (–f–1] ‰ [1,f

1
4. (i) sin–1x=cosec–1 , –1 d x d 1
x

369
§1·
(ii) cos–1x=sec–1 ¨ ¸ , –1 d x d 1
©x¹

­cot –1 1 ,x ! 0
° x

(iii) tan–1x= ®
°̄ – S  cot 1 x ,x  0
–1

5 (i) sin– 1(–x) = –sin–1x, –1 d x d 1
(ii) cos–1(–x) = S –cos–1x, –1 d x d 1
(iii) tan (–x) = –tan x,
–1 –1
–fx<f
(iv) cot–1(–x) = S –cot–1x, –fx<f
(v) cosec–1(–x) = –cosec–1x, x d –1or x t 1
(vi) sec–1(–x ) = S –sec–1x, x d –1or x t 1
6. Conversions of inverse trigonometric functions

1
x
(i)
1 – x2

x
sin–1x = cos –1 1 – x 2  tan –1
1 – x2

–1 1 – x2 –1 1 –1 1
= cot   Cosec  sec
x x 1– x2

1– x2 1
(ii)
x

1 – x2
cos–1x = sin –1 1 – x 2  tan –1
x
x 1 1
cot –1   Cosec
–1
 sec
–1

= 1– x 2
1– x 2 x

1+ x2
x
(iii)
1

370
tan–1x = [ 1 1
sin –1 Cos–1 cot –1
2   2   x
1 x 1 x
1 x2
cos ec –1   Sec 1  x
–1 2

°

­sin –1 x 1 – y 2  y 1 – x 2 if – 1 d x , y d 1 & x 2  y 2 d 1
 
°°   orif xy  0&x  y ! 1
2 2

7. (i) sin –1 x  sin –1 y ® –1


2 2

°S – sin x 1 – y  y 1 – x if 0  x , y d 1 & x  y ! 1
2 2

°

°¯ – S – sin –1 x 1 – y 2  y 1 – x 2 if – 1 d x , y  0 & x 2  y 2 ! 1

°

­sin –1 x 1 – y 2 – y 1 – x 2 if – 1 d x , y d 1 & x 2  y 2 d 1
 
°°   orif xy ! 0&x 2  y 2 ! 1
(ii) sin –1 x – sin –1 y ® –1
2

°S – sin x 1 – y – y 1 – x if 0  x d 1,– 1  y d 0&x  y ! 1
2 2 2

°

°¯ – S – sin –1 x 1 – y 2 – y 1 – x 2 if – 1 d x  0,0  y d 1&x 2  y 2 ! 1

cos x  cos y

­°cos –1 xy – 1 – x 2 1 – y 2 if – 1 d x, y d 1&x  y t 0


–1 –1
8. (i) ®
°̄2S – cos –1 xy – 1 – x 2 1 – y 2 if – 1 d x , y d 1&x  y d 0


­°cos –1 xy  1 – x 2 1 – y 2 if – 1 d x , y d 1&x d y


(ii) cos –1 x – cos –1 y ®
°̄ – cos –1 xy  1 – x 2 1 – y 2 if – 1 d y d 0,0  x d 1&x t y

­ –1 § x  y ·
°tan ¨¨ ¸¸if x\  
° © 1 – xy ¹
°° § xy ·
9. (i) tan –1 x  tan –1 y ®S  tan –1 ¨¨ ¸¸if [ !  \ !  [\ ! 
° © 1 – xy ¹
° § xy·
° – S  tan –1 ¨¨ ¸¸if [   \   [\ ! 
¯° © 1 – xy ¹

­ –1 § x – y ·
°tan ¨¨ ¸¸if x\ ! –
° © 1  xy ¹
°° § x–y·
(ii) tan x – tan y ®S  tan –1 ¨¨
–1 –1
¸¸if [ !  \   [\  –
° © 1  xy ¹
° § x–y ·
°– S  tan –1 ¨¨ ¸¸if [   \ !  [\  –
°¯ © 1  xy ¹

371
Remark : If x 1, x 2 ,..........x n  R, then tan –1 x 1 +tan –1 x 2+.........+tan –1 x n
§ s1 – s 3  s 5 – s 7 ........ ·
= tan–1 ¨¨ ¸¸
© 1 – s 2  s 4 – s 6  ...... ¹
Where sk is the sum of the product of x1,x2,............ xn taken k at a time.
ie. s1 = x1 + x2+..........+xn =¦xi
s2 = x1x2+x2x3+.......+xn–1xn=¦x1x2.
s3 = ¦x1x2x3.................etc.

­ –1
–1

°sin 2x 1 – x , if 2 d x d 2
2 1
°
10. (i) 2 sin –1 x
° –1
2
®ʌ – sin 2x 1 – x , if
1
2
d x d1
°
°

°– ʌ – sin 2x 1 – x , if – 1 d x d
¯
–1 2 –1
2

­ –1
–1

°sin 3x – 4x , if 2 d x d 2
3 1
°
(ii) 3 sin –1 x
° –1
3 1
®ʌ – sin 3x – 4x , if  x d 1
2

°
° –1
3
–1
° – ʌ – sin 3x – 4x , if – 1 d x  2
¯

2 cos –1 x

­°cos –1 2x 2 – 1 , if 0 d x d 1
®
11. (i)

°̄2ʌ – cos –1 2x 2 – 1 , if – 1 d x d 0

­ –1 3
1

°cos 4x – 3x , if 2 d x d 1
°
(ii) 3 cos –1 x
° –1 3

®2ʌ – cos 4x – 3x , if
–1
2
dxd
1
2
°
° –1 3

–1
°2ʌ  cos 4x – 3x , if – 1 d x d 2
¯

­ –1 § 2x ·
°tan ¨ 1 – x 2 ¸, if – 1  x  1
° © ¹
12. (i) ° –1 § 2x ·
2 tan –1 x ®ʌ  tan ¨ 2 ¸
, if x ! 1
° ©1 – x ¹
° –1 § 2x ·
° – ʌ  tan ¨ 2 ¸
, if x  –1
¯ ©1– x ¹

372
­ –1 § 3x – x 3 · –1 1
°tan ¨¨ ¸, if
2 ¸
x
° © 1 – 3x ¹ 3 3
°
–1 § 3x – x ·
3
° 1
(ii) 3 tan –1 x ® ʌ  tan ¨
¨ 1 – 3x 2 ¸¸, if x ! 3
° © ¹
° § 3
·
° – ʌ  tan –1 ¨ 3x – x2 ¸, if x  1
¨ ¸
¯° © 1 – 3x ¹ 3

§ 2x · §1 – x2 · § 2x ·
Note :
–1
If |x|d 1 then 2 tan x sin –1 ¨ 2 ¸
cos –1 ¨¨ ¸
2 ¸
tan –1 ¨ 2 ¸ .
©1 x ¹ ©1 x ¹ ©1– x ¹
1
If |x|>1, change x to in the above.
x
Note : In cases of identities in inverse trigonometric functions, principal values are to be taken. As such
signs of x,y etc., will determine the quadrant in which the angles will fall. In order to bring the angles
of both sides in the same quadrant, adjustment by S is to be made.
1 Evaluate tan–1 tan(–6)
Solution:
S S
We know that tan–1 (tan T) = T if – < T
2 2
§ S S·
? (2 S –6)  ¨ – 2 , 2 ¸
© ¹
tan(2 S –6) = – tan6 = tan(–6)
Therefore tan–1 tan(–6) = tan–1 tan (2 S –6) = 2 S –6
2x
2 If 2tan–1x+sin–1 is independent of x, then
1 x2
(a) x  [1, f ) (b) x  [–1,1] (c) x  (– f ,–1] (d) None of these
Solution:

­S – 2 tan –1 x,x t 1
°
Sin–1 2x = ® – (S  2 tan x )x d –1
–1

°2 tan –1 x, x | 1
¯ _
? 2tan–1x+ S –2tan–1x = S when x  [1, f )
and 2tan x– S –2tan x = – S when x  (– f ,–1]
–1 –1

Ans a,c.
3 If 0<x<1, then 1 x 2 [{xcos(cot–1x)+sin(cot–1x)}2–1]1/2 =
x
(a) (b) x (c) x 1 x 2 (d) 1 x 2
1 x2

373
Solution:
1/ 2
ª­ x 1 ½
2
º
«®x cos cos –1
 sin sin –1
¾ – 1»
1 x 2 «¬¯ 1 x2 1 x2 ¿ »¼

1/ 2
ª­ x 1 ½
2
º
«® x  ¾ – 1»
1 x 2 «¬¯ 1  x 2 1 x2 ¿ »¼


1/ 2
ª 1 x2 2
– 1º»
1 x 2 «¬ = 1  x 2 .x
¼
Ans (c)
§ S S· § tan x · § 3 sin 2x ·
4 If x  ¨ – , ¸ , then the value of tan–1 ¨ ¸ + tan–1 ¨ ¸ is
© 2 2¹ © 4 ¹ © 5  3 cos 2 x ¹
x
(a) (b) 2x (c) 3x (d) x
2
Solution:

§ 6 tan x ·
¨ ¸
§
–1 ¨
tan x ·
–1 ¨ 1  tan 2
x ¸
tan ¸ + tan
© 4 ¹ ¨ 3(1 – tan 2 x ) ¸
¨5 ¸
© 1  tan 2 x ¹

§ tan x · § 6 tan x ·
tan–1 ¨ ¸ + tan–1 ¨ ¸
© 8  2 tan x ¹
2
© 4 ¹

§ tan x 3 tan x ·¸
¨ 
¨ 4 4  tan 2 x ¸ tan x 3 tan x
tan–1 ¨ 3 tan 2 x ¸ as . <1
¨ 1– ¸ 4 4  tan 2 x
© 4(4  tan 2 x ) ¹

§ 16 tan x  tan 3 x ·
tan ¨¨–1 ¸¸
© 16  tan x ¹
2

tan–1 (tanx)
x
Ans d.
5 An integral solution of the equation
1
tan–1x+tan–1 = tan–13 is
y
(a)(2,7) (b) (4,–13) (c) (5,–8) (d) (1,2)

374
Solution:
1
tan–1x+tan–1 = tan–1 3
y
§ 1·
¨x ¸ x

tan–1 ¨ = tan–1 3 if <1
¨ x¸ y
¨ 1– y ¸
© ¹

1
x
y
x =3
1–
y

1 3x
x+ = 3–
y y
3x  1
y=
3–x
which satisfied by options a, b, c and d
Ans a, b, c, d
6 Sum to the n terms of the series
cosec–1 10 +cosec–1 50 +cosec–1 170 +............+cosec–1 (n 2  1)(n 2  2n  2) is

S S
(a) 0 (b) f (c) tan–1(n+1) – (d) cot–1(n+1) –
4 4
Solution
Let T = cosec–1 ( n 2  1)( n 2  2n  2)
cosec2 T = (n2+1) (n2+2n+2)
= (n2+1)2 + 2n(n2 +1) +n2+1
1+cot2 T = (n2+n+1)2+1
cot T = n2+n+1
1 (n  1) – n
tan T = =
n  n  1 1  (n  1)n
2

T = tan–1(n+1) –tan–1n
Thus, sum to n terms of the series
= (tan–12–tan–11) + (tan–13–tan–12)+...........+(tan–1(n+1)–tan–1n)
= tan–1(n+1)–tan–11
S
= tan–1(n+1) –
4

375
Ans. c
7 In a ' ABC, if A = tan–12, B = tan–13 then C is equal to
S S S
(a) (b) (c) (d) None of these
3 4 6
Solution:
A+B+C = S
C = S –(A+B)
= S –(tan–12+tan–13)
­ 5 ½
= S – ®S  tan
–1
¾ ' xy>1
¯ 1 – 6¿
= S – S +tan–11
S
=
4
Ans. b
PRACTICE QUESTIONS
1 If x satisfies the inequation x –x–2>0, then a value exits for
2

(a) sin–1x (b) sec–1x (c) cos–1x (d) None of these


2 If [sin x] +[cos x] = 0, where x is a non negative real number and [.] denotes the greatest
–1 –1

integer function, then complete set of values of x is


(a) (cos1,1) (b) (–1,cos1) (c) (sin1,1) (d) (cos1,sin1)
3 –1 –1 –1 S
If cos x+cos y+cos z = 3 , then xy+yz+zx is
(a) –3 (b) 0 (c) 3 (d) S
2S
4 If sin–1x+sin–1y = then cos–1x+cos–1y =
3
2S S S
(a) (b) (c) (d) S
3 3 6
5 The greatest of tan1, tan–11, sin–11, sin1, cos1 is
(a) sin1 (b) tan1 (c) tan–11 (d) None of these
6 The value of cos (cos12)–sin–1(sin12) is
–1

(a) 0 (b) S (c) 8 S –24 (d) None of these


7 cot–1
cos D –tan–1
cos D = x, then sin x =
D D D
(a) tan2 (b) cot2 (c) tan D (d) cot
2 2 2
8 If sin x = 2sin a has a solution for
–1 –1

1 1
(a) |a| t (b) |a| d
2 2
1
(c) all real values of a (d) |a|<
2

376
y
9 If cos–1x–cos–1 = D , then 4x2–4xycos D +y2 is equal to
2
(a) 2sin2 D (b) 4 (c) 4sin2 D (d) –4sin2 D
§x· §5· §S·
10 If sin–1 ¨ ¸ +cosec–1 ¨ ¸ = ¨ ¸ , then the value of x is
©5¹ ©4¹ ©2¹
(a) 4 (b) 5 (c) 1 (d) 3
§ –1 5 2·
11 The value of cot ¨ cos ec  tan –1 ¸ is
© 3 3¹
6 3 4 5
(a) (b) (c) (d)
17 17 17 17
Assertion | Reasoning
§ 2x ·
12 Let f(x) = sin–1 ¨ 2 ¸
©1 x ¹
2
Statement-1 : f c (2) = – and
5
§ 2x ·
Statement-2 : sin–1 ¨ 2 ¸ = S – 2tan x,  x>1.
–1
©1 x ¹
(a) Statement-1 is True, statement-2 is True ;statement-2 is a correct explanation
for statement-1
(b) Statement-1 is True, statement -2 is True ; statement - 2 is NOT a correct explanation for
statement - 1
(c) Statement -1 is True, statement - 2 is False
(d) Statement -1 is False, statement - 2 is True.
Comprehension (Q.No. 13 to 15)

­2 tan –1 x ,| x |d 1
§ 2x · °
Given that tan–1 ¨ 2 ¸ = ® – S  2 tan
–1
x,x ! 1
© 1 – x ¹ °
¯S  2 tan x ,x  –1
–1

­2 tan –1 x ,| x |d 1
°
§ 2 x · ®S – 2 tan –1 x,x ! 1
–1 ¨
sin 2 ¸ =
© 1 – x ¹ ° – (S  2 tan –1 x ), x  –1
¯ 

S
and sin–1x+cos–1x = for –1 d x d 1.
2

377
§ 4x · § x·
13 sin–1 ¨ 2 ¸ +2tan–1 ¨ – ¸ is independent at x then :
©x 4¹ © 2¹
(a) x  [1, f ) (b) x [1,1] (c) [–2,2] (d) x  [–3,4]
§1 –1 2 x ·
14 If (x–1) (x2+1)>0, then sin ¨ tan 2
– tan –1 x ¸ =
©2 1– x ¹
1
(a) –1 (b) 1 (c) (d) None of these
2
§ 6x · S
15 If cos–1 ¨ 2 ¸ =– + 2 tan–1 3x then x 
© 1  9x ¹ 2

§ 1 1· §1 ·
(a) (– f ,–1) (b) ¨– , ¸ (c) ¨ ,f¸ (d) None of these
© 3 3¹ ©3 ¹
16 Match the following
Column I Column II
(a) If cos–1a+cos–1b+cos–1c = 3 S then ab+bc+ca is (p) 2n
10 10
S
(b) ¦ cos –1 x i = 0, then
i 1
¦x
i 1
i (q) sin–1x –
6
2n 2n

(c) ¦ sin
i 1
–1
x i = n S , then ¦ x i is
i 1
(r) 3

­ 3 1 2
½ 1
(d) f(x) = sin ® –1x – 1 – x ¾ , – d x d 1 is (s) 10
¯ 2 2 ¿ 2

Answers
1. b 2. d 3. c 4. b 5. b 6. c 7. a 8. b
9. c 10. d 11. a 12. a 13. c 14. a 15. c
16. a o r, b o s, c o p, d o q

378
TRIGONOMETRY II
Inverse Trigonometric Functions
Hyperbolic functions
(i) sinh(–x) = – sinhx odd function
cosh(–x) = coshx even function
tanh(–x) = –tanhx odd function
(ii) Function Domain Range
sinh–1x R R
cosh–1x (0,f) (1,f)
tanh x
–1
R (–1,1)
coth–1x R–{0} R–[–1,1]
sech x
–1
(0,f) (0,1)
cosech–1x R–{0} R–{0}
(iii) sinh (sinh x) = x
–1
sinh–1(sinhx) = x
cosh (cosh–1x) = x cosh–1(coshx) = x
tanh (tanh x) = x
–1
tanh–1(tanhx) = x
sinh (sin –1 x)=xn sinh (sinh–1xn) = xn
(iv)
sinh–1x = loge x  x2 1

cosh–1x = loge x  x 2 – 1
1 § x 1·
tanh–1x = loge ¨ ¸ x > 1, x< –1
2 © x –1¹
1 § x –1·
coth–1x = loge ¨ ¸ x > 1, x< –1
2 © x 1 ¹

§1 1 – x2 ·
¨ ¸
sech x = loge ¨
–1
x ¸ 0 < x d1
© ¹

­ §1 1 x2 ·
°log e ¨ ¸if x ! 0
°° ¨ x ¸
© ¹
®
cosech–1x = ° §1– 1 x2 ·
log ¨ ¸if x  0
° e¨ ¸
°¯ © x ¹

§1·
(v) sinh–1x = cosech–1 ¨ ¸
©x¹
sinh–1x = cosh–1 x 2  1
cosh–1x = sinh–1 x 2 – 1
sinh (cosh–1x) = x2 –1

379
nS 2
1 Total number of positive integral values of n sothat the equation cos –1x+(sin–1y)2 = and
4
S2
(sin y) –cos x =
–1 2
are consistent, is equal to
–1
16
(a) 1 (b) 4 (c) 3 (d) 2
Solution
(4n  1)S 2 On adding the two given equations
we have 2(sin–1y)2 =
16
(4n  1)S 2 2S 2 1 7
0d d Ÿ – dnd
16 4 4 4
(4n – 1)S 2
Also 2cos–1x = On subtracting the two given equations
16
(4n – 1)S 2 1 8 1
0d d 2S Ÿ 4 d nd +
S 4
16
? n=1
Ans (a)
2 The minimum value of (sin–1x)3+(cos–1x)3 is equal to
S3 5S3 9S3 11S3
(a) (b) (c) (d)
32 32 32 32
Solution
Let y = (sin–1x)3+(cos–1x)3
= (sin–1x+cos–1x) {(sin–1x)2+(cos–1x)2–sin–1x.cos–1x}
S
= {(sin–1x+cos–1x)2–3sin–1x.cosx}
2

S ­ S – 3 sin –1 x§ S – sin –1 x ·½
2

= ® ¨ ¸¾
2¯4 ©2 ¹¿

S ­ 3S –1 S2 ½
= ®3(sin –1
x ) 2
– sin x  ¾
2 ¯ 2 4¿

S ­ 3S –1 3.S 2 3S 2 S2 ½
= ®3(sin –1
x ) 2
– sin x  –  ¾
2 ¯ 2 16 16 4¿

S ­° § –1 S · S 2 ½°
2

= ®3¨ sin x – ¸  ¾
2 °̄ © 4 ¹ 16 °¿

380
2
S S2 S3 § –1 S·
Minimum value of y = . = since ¨ sin x – ¸ t 0
2 16 32 © 4¹
Ans: (a)
§1· §3·
3
If A = 2tan–1 2 2 – 1 and B = 3sin–1 ¨ ¸ +sin–1 ¨ ¸ . then
©3¹ ©5¹
(a) A = B (b) A<B (c) A>B (d) None of these
Solution

A = 2tan–1 2 2 – 1 = 2tan–1(1.828)>2tan–1 3
2S
A>
3

1 § 1 § 1 ·3 · § 4 · 23
¨ ¸
3sin–1 = sin–1 ¨ 3. 3 – 4¨ 3 ¸ ¸ = sin–1 ¨1 – ¸ = sin–1 = sin–1(0.852)
3 © © ¹ ¹ © 27 ¹ 27

1 § 3·
3sin–1 <sin–1 ¨¨ 2 ¸¸
3 © ¹

§3· § 3·
sin–1 ¨ ¸ = sin–1(0.6)< sin–1 ¨¨ 2 ¸¸
©5¹ © ¹
S S 2S
? B< + =
3 3 3
Hence A>B
Ans (c)
4 The complete solution set of sin–1(sin5)>x2–4x is
(a) |x–2| < 9 – 2S (b) |x–2| > 9 – 2S (c) |x| < 9 – 2S (d) |x| > 9 – 2S
Solution
sin–1sin5>x2–4x
sin–1sin(5–2 S )>x2–4x
ª S Sº
5–2 S >x2–4x (5–2 S )  « – , »
¬ 2 2¼
x2–4x+2 S –5<0
(x–2)2<9–2 S
|x–2|< 9 – 2S
Ans (a)
S
5 Let (x,y) be such that sin–1(ax)+cos–1y+cos–1bxy = .
2
Match the statements in column I with statements in column II
Column I Column II
(a) If a = 1 and b = 0, then (x,y) (p) Lies on the circle x2+y2 = 1
(b) If a = 1 and b = 1, then (x,y) (q) Lies on the (x2–1) (y2–1) = 0

381
(c) If a = 1 and b = 2, then (x,y) (r) Lies on y = x
(d) If a = 2 and b = 2, then (x,y) (s) Lies on the (4x2–1) (y2–1) = 0
Solution
S
cos–1y+cos–1bxy = – sin–1 ax
2
cos–1y+cos–1bxy = cos–1ax
Let cos–1y = A, cos–1bxy = B and cos–1ax = C
A+B = C
B = A–C
cos(A–C) = cosB
cosAcosC+sinAsinC = cosB
y ax+sinA sinC = bxy
sin A sin C = bxy–axy
sin2A sin2C = (b–a)2x2y2
(1–a2x2)(1–y2) = x2y2(b–a)2
(a) Put a = 1 and b = 0
(1–x2)(1–y2) = x2y2
x2+y2 = 1
(b) Put a = 1 and b = 1
(1–x2)(1–y2) = 0
(x2–1)(y2–1) = 0
(c) Put a = 1 and b = 2
(1–x2) (1–y2) = x2y2
x2+y2 = 1
(d) Put a = 2 and b = 2
(1–4x2)(1–y2) = 0
(4x2–1)(y2–1) = 0
(a) o (p), (b) o (q), (c) o (p), (d) o (s)
§ x2 x3 · § 2 x4 x6 ·
6 ¨
If sin ¨ x –  .......... ¸
¸ –1 ¨ x –
+ cos ¨  ..........¸¸ = S for 0<|x|<
2 , then x equals
–1
© 2 4 ¹ © 2 4 ¹ 2
1 1
(a) (b) 1 (c) – (d) –1
2 2
Solution
§ x 2 x3 · S § 2 x 4 x6 ·
¨
sin ¨ x
–1–  .......... ¸
¸ –1 ¨ x –
= – cos ¨  ..........¸¸
© 2 4 ¹ 2 © 2 4 ¹
§ x2 x3 · § 2 x4 x6 ·
¨
sin ¨ x
–1–  .......... ¸
¸ = sin–1 ¨ x –
¨  ..........¸¸
© 2 4 ¹ © 2 4 ¹

or sin–1T+ cos –1T = 1/2


so,
x4 x6 x 2 x3
x2 –  .......... = x –  ..........
2 4 2 4
382
x2 x
§ x ·
2
§ x· a
1 – ¨¨ – ¸¸ = 1 – ¨ – ¸ ' Sf  r  1
© 2¹ 1– r
© 2 ¹
2x 2 2x
2 =
2 x 2x
2x2+x3 = 2x+x3
2x(x–1) = 0
x = 0, 1
x=1 (0<|x|< 2 )
Ans (b)
7 Let a, b, c be positive real numbers. Let
a ( a  b  c) b( a  b  c ) c( a  b  c)
T = tan–1 + tan–1 + tan–1 , then tan T =
bc ca ab
S S
(a) (b) (c) S (d) None of these
4 2
Solution
Let a+b+c = u
ªAlso you can use tan –1x  tan –1 y  tan –1z º
au bu cu « »
T = tan–1 +tan–1 +tan–1 « tan –1 ª x  y  z – xyz º »
bc ca ab « «1 – xy – yz – zx » »
¬ ¬ ¼ ¼

§ au bu ·
¨  ¸
–1 ¨ bc ca ¸ cu au bu u abc ab
T = S + tan ¨ ¸ + tan ab ,
–1
= =
c c
=
c
+1>1
au bc bc ca
¨1– ¸
© bc ac ¹

§ (a  b ) u ·
¨ ¸
¨ abc ¸ cu
T = S + tan–1 ¨ u ¸ + tan –1
ab
¨ 1– ¸
© c ¹

§ ( u – c) u c · cu
= S + tan–1 ¨¨ . ¸ + tan–1
¸
© abc (c – u ) ¹ ab

cu
= S – tan–1 uc + tan–1
ab ab
= S
Ans (c)

383
PRACTICE QUESTIONS
S
1 If tan–1x+tan–1y+tan–1z = S or then
2
(a) x+y+z = 3xyz (b) x+y+z = 2xyz (c) xy+yz+zx = 1 (d) None of these
2 If [cos–1x] + [cot–1x] = 0 , where x is a nonnegative real number and [.] denotes the greatest integer
function then complete set of values of x is
(a) (cos1,1) (b) (cot1,1) (c) (cos1,cot1) (d) None of there
3 Range of the function f(x) = cos–1(–{x}), where {.} is fractional part function is
§S · §S º ªS · § Sº
(a) ¨ , S¸ (b) ¨ , S» (c) «¬ 2 , S ¸¹ (d) ¨ 0, »
©2 ¹ ©2 ¼ © 2¼
3S
4 The sum of solutions of the equation 2sin–1 x2  x 1 +cos–1 x 2  x = is
2
(a) 0 (b) –1 (c) 1 (d) 2
5* Which of the following is a rational number
§ –1 –1 1 · §S –1 3 ·
(a) sin ¨ tan 3  tan ¸ (b) cos ¨ – sin ¸
© 3¹ ©2 4¹

§ § 1 –1 63 · · §1 5·
¨ ¸¸
(c) log2 ¨ sin ¨¨ 4 sin 8 ¸¸ (d) tan¨¨ cos –1
2 3
¸
¸
© © ¹¹ © ¹
Assertion | Reasoning
§ 1 · § 1 ·
6 Statement 1 : sin–1 ¨ ¸ >tan–1 ¨ ¸
© e¹ © S¹
Statement 2 : sin–1x > tan–1y for x>y,  x,y (0,1)
(a) Statement -1 is True, Statement -2 is True, Statement -2 is a correct explanation for
Statement- 1.
(b) Statement -1 is True, Statement-2 is True, statement-2 is NOT correct explanation for
Statement - 1
(c) Statement-1 is True, Statement -2 is False
(d) Statement 1 is False, Statement -2 is True.
Comprehension (Q.No. 7 to 9)
ª 1 º
It is given that A = (tan–1x)3+(cot–1x)3 where x>0 and B = (cos–1t)2+(sin–1t)2 where t  «0,
¬ 2 »¼
S S
and sin–1x+cos–1x = for –1 d x d 1 and tan–1x+cot–1x = for all x  R.
2 2
7 The interval in which A lies is
ª S3 S3 · § S3 S3 · ª S3 S3 ·
(a) « , ¸¸ (b) ¨¨ , ¸¸ (c) « 32 , 8 ¸¸ (d) None of these
¬7 2¹ © 40 10 ¹ ¬ ¹
8 The maximum value of B is

384
S2 S2 S2
(a) (b) (c) (d) None of these
8 16 4 § m – SM ·
9 If least value of A is m and maximum value of B is M then cot–1cot ¨ ¸=
© M ¹
7S 7S S S
(a) – (b) (c) – (d)
8 8 8 8
Single Integer Answer type Question
y § 3 ·
10 The number of all positive integral solutions of the equation tan–1x+cos–1 1  y 2 = sin–1 ¨ ¸,
© 10 ¹
are .....................
11 If cos–1(4x3–3x) = a+b
1
cos–1x, for –1<x<– then [a+b+2] is .............
2
12 Match the statement of column I with values of column II
Column I Column II
(a) Absolute difference of greatest and least value
S
of 2 (sin2x–cos2x) (p)
4
(b) Absolute difference of greatest and least value
S
of x2–4x+3, x  [1,3] is (q)
6
1 – x
(c) Greatest value of tan–1 , x  [0,1], is (r) 4
1 x
(d) Absolute difference of greatest and least
ª 1 1 º
value of cos–1x2, x  « – , , is (s) 1
¬ 2 2 »¼
13* If a d tan–1x+cot–1x+sin–1x d b, then
S S
(a) a = (b) a = 0 (c) b = (d) b= S
4 2
14 cot–1(2.12)+cot–1(2.22)+cot–1(2.32)+..............upto f is equal to
S S S
(a) (b) (c) (d) S
4 3 2 1
§ 1 · § · § 2 ·
15 Number of solutions of the equation tan–1 ¨ ¸ +tan–1 ¨ ¸ = tan–1 ¨ 2 ¸ is
© 2 x 1 ¹ © 4 x 1 ¹ ©x ¹
(a) 1 (b) 2 (c) 3 (d) 4

'Note : Questions with * have more than one correct option'


Answers
1. c 2. c 3. c 4. b 5. a, b, c 6. a 7. c 8. c 9.d
10. 2 11. –2 12. a o r, b o s, c o 0, d o q 13. b,d 14. a 15. a

385
TRIGONOMETRY - III
Inverse Trigonometric Functions - Problem Solving
A ƒunction ƒ: A o B is invertible iff it is a bijection. The inverse of ƒ is denoted by ƒ–1 and is
defined as ƒ–1(y) = x œ ƒ(x)=y. Trigonometric functions are periodic and hence they are not
bijective. But if we restrict their domains and codomains they can be made bijective and we
can obtain their inverses.
Domain & range of inverse trigonometric Functions
Function Domain Range (Principal value branch)
sin–1x [–1,1] >– S /2 ,– S / 2 @
cos–1x [–1,1] [0, S ]
tan x–1
(– ff (– S /2, S /2)
cot–1x (– ff (0, S )
cosec x –1
( –f–1] ‰ [1,f) [– S /2,0) ‰ (0, S /2]
sec x–1
( –f–1] ‰ [1,f) [0, S /2) ‰ ( S /2, S ]
Note : If no branch of an inverse trigonometric function is mentioned, then it means the principal
value branch of the function.
Properties of Inverse Trigonometric Functions
1
­ ʌ ʌ
°° – 2nʌ  x,2nʌ – 2 d x d 2nʌ  2 ,n  Z
(i) sin–1(sinx) = ®
° 2n  1 ʌ – x, 2n  1 ʌ – ʌ d x d 2n  1 ʌ  ʌ , n  Z
°¯ 
2 2

Y

y=
–2
S

x –x x
y= y= X
S

0 S 2S

Period = 2 S & it is an odd function.


­ – 2nS  x ,2nS d x d 2n  1 S,n  Z
(ii) cos–1(cosx) = ®
¯2nS – x ,(2n – 1)S d x d 2nS,n  Z
Y
y=

y=
+x

x
–x

y=

2S
2S
y=

–x

X
–2S –S 0 S 2S

Period=2 S and it is an even function

386
ʌ ʌ
(iii) tan–1(tanx) = – nʌ  x , nʌ –  x  nʌ  , n  Z
2 2
Y

x –x
y= S y=
 S  S/2 0 S /2 S
+x
y=
S

Period = S
(iv) cot–1(cotx) = – nʌ  x ,nʌ  x  ( n  1) ʌ,n  Z
Y

+x x
y= x–
S

y=
S

y=
X
S 0 S

Period = S
­ S
°°x – 2nS,2nS d x d (2n  1)S,x z 2S  2
(v) sec–1(secx) = ®° S
°¯ – x  2nS,(2n  1)S d x d 2nS,x z (2nS – 2 n  Z

Y
y=
+x

y=

2S
y=
2S

–x
y=

–x

X
  S   S  S  0 S  S  S 

Period = 2 S

387
­ S S
°° – 2nS  [2nS – 2 d x d 2nS  2
(vi) cosec–1(cosecx) = ®° S S
°¯(2n  1)S – x ,(2n  1)S – 2 d x d (2n  1)S  2  [ z nS,n  Z

y= 2

S
x–

y=
–x =
S

–x
y
X
S 0 S S
x
y=

Period = 2 S
2. (i) sin(sin–1x) = x, –1 d x d 1 (ii) cos(cos–1x) = x, –1 d x d 1

Y Y

1 1

X X
–1 0 1 –1 0 1
–1 –1

(iii) tan(tan–1x) = x, x  R (iv) cot(cot–1x) = x, x  R

Y Y

1 1

X X
–1 0 1 –1 0 1
–1 –1

388
(v) sec(sec–1x) = x, x  R (–f–1] ‰ [1,f

X
–1 0 1
–1

(vi) cosce(cosce–1x)= x, x  R (–f–1] ‰ [1,f

X
–1 0 1
–1

3. (i) sin–1x+cos–1x= S 2 , –1 d x d 1

(ii) tan–1x+cot–1x = S 2 , x  R

(iii) sec–1x+cosce–1x = S 2 , x  R (–f–1] ‰ [1,f

1
4. (i) sin–1x=cosec–1 , –1 d x d 1
x
§1·
(ii) cos–1x=sec–1 ¨ ¸ , –1 d x d 1
©x¹

°
­cot –1 1 ,x ! 0
x
(iii) tan–1x= ®

°̄ – S  cot 1 x ,x  0
–1

5 (i) sin– 1(–x) = –sin–1x, –1 d x d 1


(ii) cos–1(–x) = S –cos–1x, –1 d x d 1
(iii) tan–1(–x) = –tan–1 x, –fx<f
(iv) cot–1(–x) = S –cot–1x, –fx<f

389
(v) cosec–1(–x) = –cosec–1x, x d –1or x t 1
(vi) sec–1(–x ) = S –sec–1x, x d –1or x t 1
6. Conversions of inverse trigonometric functions

1
x
(i)
1 – x2

x
sin–1x = cos –1 1 – x 2  tan –1
1 – x2

1 – x2 –1 1 1
= cot –1   cosec  sec
–1

x x 1– x2

1– x2 1
(ii)
x

1 – x2
cos x
–1
= sin –1 1 – x 2  tan –1
x
x 1 1
cot –1   cosec
–1
 sec
–1

= 1– x 2
1– x 2 x

1+ x2
x
(iii)
1

tan–1x = [ 1 1
sin –1 –1
cot –1
2   cos 2   x
1 x 1 x
1 x2
cos ec –1   sec 1  x
–1 2

°

­sin –1 x 1 – y 2  y 1 – x 2 if – 1 d x , y d 1 & x 2  y 2 d 1
 
°°   orif xy  0&x 2  y 2 ! 1
7. (i) sin –1 x  sin –1 y ®

–1 2

°S – sin x 1 – y  y 1 – x if 0  x , y d 1 & x  y ! 1
2 2 2

°

–1 2

¯° – S – sin x 1 – y  y 1 – x if – 1 d x , y  0 & x  y ! 1
2 2 2

390
°

­sin –1 x 1 – y 2 – y 1 – x 2 if – 1 d x , y d 1 & x 2  y 2 d 1
 
°°   orif xy ! 0&x 2  y 2 ! 1
(ii) sin –1 x – sin –1 y ®
–1 2

°S – sin x 1 – y – y 1 – x if 0  x d 1,– 1  y d 0&x  y ! 1
2 2 2

°

°¯ – S – sin –1 x 1 – y 2 – y 1 – x 2 if – 1 d x  0,0  y d 1&x 2  y 2 ! 1

cos –1 x  cos –1 y

­°cos –1 xy – 1 – x 2 1 – y 2 if – 1 d x, y d 1&x  y t 0
8. (i) ®

°̄2S – cos –1 xy – 1 – x 2 1 – y 2 if – 1 d x , y d 1&x  y d 0


­°cos –1 xy  1 – x 2 1 – y 2 if – 1 d x , y d 1&x d y


–1 –1
(ii) cos x – cos y ®
°̄ – cos –1 xy  1 – x 2 1 – y 2 if – 1 d y d 0,0  x d 1&x t y

­ –1 § x  y ·
°tan ¨¨ ¸¸if x\  
° © 1 – xy ¹
°° § xy ·
9. (i) tan –1 x  tan –1 y ®S  tan –1 ¨¨ ¸¸if [ !  \ !  [\ ! 
° © 1 – xy ¹
° § xy·
° – S  tan –1 ¨¨ ¸¸if [   \   [\ ! 
°¯ © 1 – xy ¹

­ –1 § x – y ·
°tan ¨¨ ¸¸if x\ ! –
° © 1  xy ¹
°° § x–y·
(ii) tan –1 x – tan –1 y ®S  tan –1 ¨¨ ¸¸if [ !  \   [\  –
° © 1  xy ¹
° § x–y ·
°– S  tan –1 ¨¨ ¸¸if [   \ !  [\  –
¯° © 1  xy ¹

Remark : If x 1 , x 2 ,..........x n  R, then tan –1 x 1 +tan –1 x 2+.........+tan –1 x n


§ s1 – s 3  s 5 – s 7 ........ ·
= tan–1 ¨¨ ¸¸
© 1 – s 2  s 4 – s 6  ...... ¹
Where sk is the sum of the product of x1,x2,............ xn taken k at a time.
ie. s1 = x1 + x2+..........+xn =¦xi
s2 = x1x2+x2x3+.......+xn–1xn=¦x1x2.
s3 = ¦x1x2x3.................etc.

391
­ –1
–1

°sin 2x 1 – x , if 2 d x d 2
2 1
°
10. (i) 2 sin –1 x
° –1
2
®ʌ – sin 2x 1 – x , if
1
2
d x d1
°
°

°– ʌ – sin 2x 1 – x , if – 1 d x d
¯
–1 2 –1
2

­ –1
–1

°sin 3x – 4x , if 2 d x d 2
3 1
°
(ii) 3 sin –1 x
° –1
3 1
®ʌ – sin 3x – 4x , if  x d 1
2

°
° –1
3
–1
° – ʌ – sin 3x – 4x , if – 1 d x  2
¯

–1
2 cos x

­°cos –1 2x 2 – 1 , if 0 d x d 1
®
11. (i)

°̄2ʌ – cos –1 2x 2 – 1 , if – 1 d x d 0

­ –1 3
1

°cos 4x – 3x , if 2 d x d 1
°
(ii) 3 cos –1 x
° –1 3

®2ʌ – cos 4x – 3x , if
–1
2
dxd
1
2
°
° –1 3

–1
°2ʌ  cos 4x – 3x , if – 1 d x d 2
¯

­ –1 § 2x ·
°tan ¨ 1 – x 2 ¸, if – 1  x  1
° © ¹
12. (i) ° –1 § 2x ·
2 tan –1 x ®ʌ  tan ¨ 2 ¸
, if x ! 1
° © 1 – x ¹
° –1 § 2x ·
° – ʌ  tan ¨ 2 ¸
, if x  –1
¯ ©1– x ¹

­ –1 § 3x – x 3 · –1 1
°tan ¨¨ ¸, if
2 ¸
x
° © 1 – 3x ¹ 3 3
°
–1 § 3x – x ·
3
° 1
(ii) 3 tan –1 x ® ʌ  tan ¨
¨ 1 – 3x 2 ¸¸, if x ! 3
° © ¹
° § 3
·
° – ʌ  tan –1 ¨ 3x – x2 ¸, if x  1
°¯ ¨ ¸
© 1 – 3x ¹ 3

392
§ 2x · §1 – x2 · § 2x ·
Note :
–1
If |x|d 1 then 2 tan x sin –1 ¨ 2 ¸
cos –1 ¨¨ ¸
2 ¸
tan –1 ¨ 2 ¸ .
©1 x ¹ © 1  x ¹ ©1– x ¹
1
If |x|>1, change x to in the above.
x
Note : In cases of identities in inverse trigonometric functions, principal values are to be taken. As
such signs of x,y etc., will determine the quadrant in which the angles will fall. In order to
bring the angles of both sides in the same quadrant, adjustment by S is to be made.
13. Hyperbolic functions
(i) sinh(–x) = – sinhx odd function
cosh(–x) = – coshx even function
tanh(–x) = tanhx odd function
(ii) Function Domain Range
sinh x
–1
R R
cosh x
–1
(0,f) (1,f)
tanh–1x R (–1,1)
coth–1x R–{0} R–[–1,1]
sech–1x (0,f) (0,1)
cosech x R–{0}
–1
R–{0}
(iii) sinh (sinh x) = x
–1
sinh–1(sinhx) = x
cosh (cosh x) = x
–1
cosh–1(coshx) = x
tanh (tanh–1x) = x tanh–1(tanhx) = x
sinhn (sinh x)=x
–1 n
sinh (sinh–1xn) = xn
(iv)
sinh–1x = loge x  x 2  1

cosh–1x = loge x  x 2 – 1
1 § x 1·
tanh–1x = loge ¨ ¸ x > 1, x< –1
2 © x –1¹
1 § x –1·
coth–1x = loge ¨ ¸ x > 1, x< –1
2 © x 1 ¹

§1 1 – x2 ·
¨ ¸
sech x = loge ¨
–1
x ¸ 0 < x d1
© ¹

­ §1 1 x2 ·
°log e ¨ ¸ if x ! 0
°° ¨ x ¸ 
© ¹
®
cosech x = °
–1 §1– 2 ·
1 x ¸
¨ if x  0
°log e ¨ ¸ 
°¯ © x ¹

393
§1·
(v) sinh–1x = cosech–1 ¨ ¸
©x¹
sinh–1x = cosh–1 x 2  1
cosh–1x = sinh–1 x 2 – 1
sinh (cosh–1x) = x2 –1
Solved Examples
1. The sum to infinite terms of the series
1 1 1
tan–1 + tan–1 + tan–1 +............................ is
3 7 13
S S S
(a) (b) (c) (d) None of these
6 4 3
Solution : By method of difference
1
Tn = tan–1
1 n  n2
1 n 1 – n
Tn = tan–1 1  n (n  1) = tan–1 n (n  1) = tan–1 (n+1) – tan–1n

? Tn = tan–1 (n+1) – tan–1n


T1 = tan–1 2– tan–11
T2 = tan–13 – tan–12
T3 = tan–14 – tan–13
.
.
.
Tn = tan–1 (n+1) – tan–1n
Adding, Sn = T1 + T2 +............ + Tn
= tan –1 (n  1) – tan–11
S S S
? Sf = tan–1 f –tan–1 1= –
2 4 4
Ans : b
2 The sum to infinite terms of the series
1 1 1
tan–1 2 + tan
–1
2 + tan
–1
+.................. f is
2.1 2. 2 2. 3 2
S S S
(a) (b) (c) (d) None
4 3 2

394
Solution :
1 2 2n  1 – 2n – 1
Tn = tan –1
= tan –1
= tan 1  2n  1 2n – 1
–1
2n 2 4n 2
Ÿ Tn = tan–1 (2n+1) – tan–1 (2n–1)
? T1 = tan–1 3– tan–11
T2 = tan–1 5– tan–13
T3 = tan–1 7– tan–13
.
.
.
Tn tan –1 (2n  1) – tan –1 (2n – 1)
Adding, Sn = T1+T2+.......Tn = Tan–1(2n+1)–tan–11
S S S
? Sf = tan–1 f –tan–1 1 = –
2 4 4
Ans : a
3 The value of
c1x – y c 2 – c1 c3 – c 2 1
tan–1 c y  x +tan–1 1  c c + tan–1 1  c c +......................+ tan–1 c is
1 1 2 2 3 n

x y
(a) tan–1 (b) tan–1 (c) tan–1 x–tan–1y (d) None
y x
Solution Write the series as

x 1 1 1 1 1 1 1
– – – –
y c1 c1 c 2 c c3 c n –1 c n 1
tan–1 + tan–1 + tan–1 2 + ........+ tan –1 1 1 + tan –1
x 1 1 1 1 1 cn
1 1 1 1
y c1 c1 c 2 c2 c3 c n –1 c n

§ –1 x 1· § 1 1· § 1 1·
Ÿ ¨¨ tan – tan–1 ¸¸ + ¨¨ tan–1 – tan–1 ¸¸ + ¨¨ tan–1 – tan–1 ¸¸ +
© y c1 ¹ © c1 c2 ¹ © c2 c3 ¹

§ –1 1 1· 1 x
.................. + ¨¨ tan – tan–1 ¸¸ + tan–1 = tan –1
y
© cn –1 cn ¹ cn
Ans : a
y
4. The number of positive integral solutions of the equation tan x + cos –1 –1
1  y2

3
= sin–1 is
10
(a) 1 (b) 2 (c) 3 (d) None

395
1
Solution : tan–1 x + tan–1 y = tan–1 3

1
tan–1 y = tan–1 3 – tan–1x

1 3– x
Ÿ =
y 1  3x
1 3x
y=
3– x
Put x = 1, then y = 2 Put x = 2, then y = 7
? (1,2) & (2,7) are two sets.
Ans : b
§n· S
5. If cot–1 ¨ ¸ ! ; n N, then the maximum value of n can be
©S¹ 6
(a) 4 (b) 5 (c) 6 (d) None
n S
Solution : < cot ( ' cot–1x is a decreasing function)
S 6
Ÿ n< S 3
n < 5.43 Ÿ n = 5 (max)
Ans : b
6. The value of
­ § · ·½
° ¨ –1 §¨ 2 – 3 ¸ 12 ¸°
–1
sin ®cot ¨ sin ¸  cos –1
 sec –1
2 ¸¾
¨ 4 ¹ 4 ¸°
° ¨© © ¹¿
¯
S S
(a) 0 (b) (c) (d) None
4 2

2– 3 4–2 3
Solution : ' sin –1 sin –1
4 8

–1 3 1 – 2 3
= sin
2 2 2

2
§ 3 –1·
= sin
–1
¨ ¸
¨ 2 2 ¸
© ¹

–1 3 –1
= sin
2 2

396
–1 S S
= sin sin
12 12

–1 ­ § S S S ·½ S
? sin ®cot ¨ 12  6  4 ¸ ¾ = sin–1 cot
¯ © ¹¿ 2
= sin 0 = 0
–1

Ans : a
7. The greatest value of (tan–1x)2 + (cot–1x)2 is .......................
Solution :
(tan–1x)2 + (cot–1x)2 = (tan–1x+cot–1x)2 – 2tan–1xcot–1x
S2 §S –1 ·
= – 2 tan–1x ¨ – tan x ¸
4 ©2 ¹

S2 §S ·
Let tan x= y, then LHS=
–1
– 2y ¨ – y ¸
4 ©2 ¹

S2 § 2 Sy S2 · 2.S 2 S 2
– S y +2y = 2 ¨¨ y –  ¸– 
2 16 ¸¹ 16
= 2
4 © 4
2
§ –1 S · S2
= 2¨ tan x – ¸ 
© 4¹ 8

S2
? Minimum value is
8
S2
Ans :
8

397
PRACTICE QUESTIONS
* More than one options questions

5S 2
1. If (tan–1x)2 + (cot–1 x)2 = , then x equals
8
(a) –1 (b) 1 (c) 0 (d) None of these
§ x2 x3 · § 2 x4 x6 ·
2. ¨
If sin ¨ x –  – ......... ¸
¸ –1 ¨ x –
+ cos ¨  – .........¸¸ = S for 0<|x|<
2 , then
–1
© 2 4 ¹ © 2 4 ¹ 2
x equals
1 –1
(a) (b) 1 (c) (d) –1
2 2
3. Match the conditions / expressions in column I with statement in column II.
S
Let (x,y) be such that sin–1(ax)+cos–1y + cos–1(bxy)=
2
Column I Column II
(a) If a=1 & b = 0, then(x,y) (p) lies on the circle x2+y2=1
(b) If a=1 & b = 1, then(x,y) (q) lies on (x2–1) (y2–1) = 0
(c) If a=1 & b = 2, then(x,y) (r) lies on y = x
(b) If a=2 & b = 2, then(x,y) (s) lies on (4x2–1) (y2–1) = 0
4 Sum to n terms of the series
cosec–1 10 +cosec–1 50 + cosec–1 170 + ..........+cosec–1 n 1 n
2 2

 2n  2 is
S S
(a) 0 (b) f (c) tan–1 n  1 – (d) cot–1 n  1 –
4 4
5 Match the following.
–1 –1 –1 –1
Let t1 = (sin–1x)Sin x ,t2=(sin–1x)Cos x ,t3 =(cos–1x)Sin x ,t4 = (cos–1 x)cos x
Column I Column II
(a) x  (0,cos1) (p) t1>t2>t4>t3
§ 1 ·
(b) x  ¨ cos1 , ¸ (q) t4>t3>t1>t2
© 2¹

§ 1 ·
(c) x  ¨ , sin1 ¸ (r) t1>t2>t4>t3
© 2 ¹

(c) x  sin1 ,1 (s) t3>t4>t1>t2
6. Read the passage & answer the following questions
S
If tan–1x : tan–1y = 1:4 (where |x|< tan ) then
6
(i) The value of y as an algebraic function of x will be

398
4 x (1  x 2 ) 4 x (1 – x 2 )
(a) (b)
x 4 – 6x 2  1 x 4 – 6x 2  1
4 x (1  x 2 )
(c) (d) None of these.
x 4  6x 2  1
(ii) The root of the equation x4 – 6x2+1=0 is
S S
(a) tan (b) tan
12 4
S S
(c) tan (d) tan
8 16
7 If a sin x – b cos x=c, then a sin x + b cos x is
–1 –1 –1 –1

Sab  c( b – a )
(a) 0 (b)
ab
S Sab  c(b – a )
(c) (d)
2 ab
n § r – r –1 ·
8. ¦ Sin –1 ¨ ¸
¨ r ( r  1) ¸ is
r 1 © ¹
S S
(a) tan–1 n – (b) tan–1 n  1 –
4 4
(c) tan–1 n (d) tan–1 n  1
9 If [cot–1x] + [cos–1x] = 0, then complete set of values of x is
(a) (cos1, 1] (b) (cot1,cos1) (c) (cot1, 1)
(d) None of these
10*. If (sin–1x+sin–1w) (sin–1y+sin–1z) = S 2 , then

x N1 y N2
D wher N1,N2,N3,N4  W
z N3 w N4
(a) has a maximum value of 2 (b) has a minimum value of 0
(c) 16 different D are possible (d) has a minimum value of –2.
11. The value of k (k>0) such that the length of the longest interval in which the function
S
ƒ(x)=sin–1 |sinkx|+cos–1(coskx) is constant is is / are
4
(a) 8 (b) 4 (c) 12 (d) 16
12*. Match the following
Column I Column II
S2
(a) (sin x) + (sin y) =
–1 2 –1 2
(p) 1
2
Ÿ x +y =
3 3

399
(b) (cos–1x)2 + (cos–1y)2 = 2 S 2 (q) –2
Ÿ x5+y5=
S4
(c) (sin x) + (sin y) =
–1 2 –1 2
(r) 0
4
Ÿ |x–y| =
(d) |sin–1x–sin–1y| = S (s) 2
Ÿ xy =
Answers
1 a 2 b
3 a o p, b o q, c o p, d o s 4 c
5 a o q, b o s, c o r, d o s 6 (i) b (ii) a
7 d 8 c
9 c 10 a, c, d
11 b 12 a o q,r, s , b o q , c o r,s d o p

___________________
_______________
_________

400
Mathematical Reasoning
Logical Statements, Tautology and Contradiction
Mathematical Logic
Statement (Proposition): A sentence which is either true or false but not both is called a statement.
They are denoted by p, q, r, s,...............A true statement is called a valid statement. If a statement
is false we call it invalid statement.
Truth Table
It is a tabular device to obtain the truth value of a compound statement or to check the
validity of a simple or compound statement
Number of horizontal lines in a truth table depends on the number of substatements present
in it.
If the problem involves n simple statements then number of rows is 2 n.
i.e.
No. of statements No.of rows
1 21 = 2
2 22 = 4
3 23 = 8
4 24 = 16
Logical Connectives
(i) Conjunction ( š )
A compound statement joining two statements by “and” is called a conjunction and is
denoted by š .
i.e. the conjunction of two statements p and q is denoted by p š q. p š q is true only if
both the components p and q are true otherwise it is false.
(ii) Disjunction ( › )
A compound statement joining two statements by “or” is called a disjunction and is
denoted by › .
i.e. the disjunction of two statements p & q is true or both p and q is true otherwise p › q
is false.
We can express both p › q and p › q is tabular form as under. T stands for true and F
stands for false.
p q pšq p› q
T T T T
T F F T
F T F T
F F F F
Quantifiers : phrases like “there exists” (  ) and “for all” (  ) are called quantifiers.
(iii) Implications ( Ÿ )
p Ÿ q. One way implication
Here p is called antecedent or hypothesis or premise and q is called consequence or
conclusion.
p Ÿ q is the same for each of the following
(i) If p then q

401
(ii) P is sufficient for q
(iii) q is necessary condition for p
(iv) p only if q
(v) q if p
(vi) q follows from p
(vii) q is consequence of p
Since a true statement cannot imply a false statement, p Ÿ q is always true except when
p is true and q is false . It may also be useful to note that p Ÿ q is equivalent to ~p › q
Note : The contrapositive of a conditional statement is formed by negating both the
hypothesis and the conclusion and then interchanging the resulting negations.
In otherwards, the contrapositivse negates and switches the parts of the sentence. It
does both the jobs of the INVERSE and the CONVERSE.
IMPORTANT : Contrapositive has the same truth value as the original conditional
statement.
Note :
Statement Converse Inverse Contrapositive Negation
pŸq qŸp ~p Ÿ ~q ~q Ÿ ~p ~(p Ÿ q)
The inverse and the converse of a conditional statement are logically equivalent to each other,
just as the conditional and its contrapositive are logically equivalent to each other.
(iv) Two way implication ( œ )
p œ q, “p implies and implied by q” or “p if and only if q”
p œ q is true if both p & q are true or both false and false when one of the statements is true
and the other is false.
p q pŸq qŸp pœq
T T T T T
T F F T F
F T T F F
F F T T T
(v) Negation
If p is a statement then negation p is written as ~ p.
If p is true then ~ p is false. If p is false then ~ p is true
p ~p
T F
F T
Note :
x ~ (~p) = p
x ~ (p š q) = ~ p › ~q
x ~ (p › q) = ~ p š ~q
x ~ (p Ÿ q) = (p š ~q)
x ~ (p œ q) = (p š ~q) › (~p š q)
(vi) NOR ( p )
Let p & q be two statements. Then “p p q” is called Joint Denial or “NOR” statement

402
(combination of NOT and OR) and read as “Neither p nor q”. “p p q” can also be written as
(p › q) or ~ (p › q).
Joint Denial is true only when p and q both are false.
p q p p q = ~ (p š q)
T T F
T F F
F T F
F F T

(vii) NAND ( n )
Let p & q be two statements. Then “p n q” is called NAND statement (combination of
NOT and AND) and is written as “p n q”
“p n q” is also written as (p š q) or ~ (p š q) This statement is false only if both p & q
are true.
p q p n q = ~ (p › q)
T T F
T F T
F T T
F F T
Here is a table that shows a commonly used Precedence of logical operators
Operator Precedence
~ 1
š 2
› 3
Ÿ 4
œ 5
Use of brackets
(i) If negation (i.e or ~) is repeated in the same statement then there is no need of
bracket.
(ii) If in a statement, the connectives of same type are present, then brackets are applied
from left.
(iii) If different connectives are used in a statement, then we remove the bracket of lower
order connective. But we cannot remove the bracket of higher order connective.
For example:
(i) p Ÿ (q š r) = p Ÿ q š r (order of š is less than order of Ÿ )
(ii) p › (q Ÿ r) z p › q Ÿ r

403
Table of Symbols
(i) If p then q pŸq
(ii) p if q qŸp
(iii) p only if q qop
(iv) p unless q ~q Ÿ p
(v) p is a sufficient condition for q pŸq
(vi) p is a necessary condition for q qŸp
(vii) A sufficient condition for p is q qŸp
(viii) A necessary condition for p is q pŸq
(ix) In order that p is sufficient that q qŸp
(x) In order that p is necessary that q poq
(xi) p if and only if q Pœq
(xii) p is a necessary and sufficient condition for q pœ q
Tautology and Contradiction (Fallacy)
A statement whose truth value is always T (i.e. True) is called a tautology and the statement
whose truth value is always F(i.e.False) is called a contradiction. Negation of tautology is a
contradiction and vice versa.
Logical equivalence
Two compound statements are said be logically equivalent if both have same truth values for all
possible assignments given to the variables.
Duality
Two compounds are said to be dual of each other if either can be obtained from the other by
interchanging š and › provided both remain valid.
For e.g. the dual of (p › q) š r is (p š q) › r..
Algebra of Statements
Commutative laws (i) (p › q) œ (q › p)
(ii) (p š q) œ (q š p)
Associative laws (i) p › (q › r) œ (p › q) › r
(ii) p š (q š r) œ (p š q) š r
Distributive laws (i) p š (q › r) œ (p š q) › (p š r)
(ii) p › (q š r) œ (p › q) š (p › r)
Idempotent laws (i) (p › p) œ p
(ii) (p š p) œ p
Abosorption laws (i) p › (p š q) œ p
(ii) p š (p › q) œ p
De Morgan’s laws (i) ~ (p › q) œ (~ p š ~q)
(ii) ~ (p š q) œ (~ p › ~q)
Detachment law ((p Ÿ q) š p) Ÿ q
Chain law ((p Ÿ q) š (q Ÿ p)) Ÿ (p Ÿ r)
Identity laws (t is tautology and (i) p š t = t š p = p
f is contradiction) (ii) p › f = f › p = p
Compliment laws (i) p › ~p = t
(ii) p š ~p = t

404
Solved Examples
1 If p Ÿ (q › r) is false, then the truth values of p,q,r are respectively
(i) T, F, F (b) F, T, T (c) T, T, F (d) F, F, F
Solution
p Ÿ q is false only when p is true and q is false
p Ÿ (q › r) is false when p is true and q › r is false and q › r is false when both q and r are false
Hence, truth values of p, q, r are respectively T, F, F.
Ans: (a)
2 Test the validity of the argument (S1, S2; S), where
S1 ; p › q, S2 : ~ p and S : q.
Solution
In order to test the validity of the argument (S1, S2; S), we first construct the truth table
for the conditional statement.
S1 š S2 o S i.e [(p › q) š ~p] o q
The truth table is as given below:
p q S1 = p › q S2= ~ p S1 š S2 S=q S1 š S2 o S
i.e. S1 š S2 o q
T T T F F T T
T F T F F F T
F T T T T T T
F F F T F F T
We observe that the last comumn of the truth table for S1 š S2 o S contains T only. Thus, S1 š S2 o
S is a tautology.
Hence, the given argument is valid.
3 If (p š ~q) š (~q š q) is
(a) a contradiction
(b) a tautology
(c) neither a tautology nor a contradiction
(d) both a tautology and a contradiction
Solution
(p š ~q) š (~p š q) { (p š ~p) š (~q š q)
{ (f š f)
{f
(f o false)
(By using associative laws and commutative laws)
? (p š ~q) š (~p š q) is a contradiction.
Ans : (a)
4 Which of the following is logically equivalent to ~ (~ p Ÿ q) ?
(a) ~ p š q (b) p š q (c) ~ p š ~ q (d) p š ~ q
Solution
Since ~ (p Ÿ q) = p š ~ q
~ (~ p Ÿ q) = p š ~ q
Ans : (c)

405
5 If p { He is a carpenter and q = He is making a table.
Then write down the following statement into symbols:
(i) He is a carpenter and making a table.
(ii) He is a carpenter but is not making a table.
(iii) It is false that he is a carpenter or making a table.
(iv) Neither he is a carpenter nor he is making a table.
(v) He is not a carpenter and he is making a table.
(vi) It is false that he is not a carpenter or is not making a table.
(vii) He is a carpenter or making a table.
Solution
The solution of above compound statements in terms of p and q are given below :
(i) pšq (ii) p š q (iii) (p › q) (iv) pš q
(v) pšq (vi) ( p › q) (vii) p › q
6 Write in words the converse, inverse, contrapositive and negation of the implication “If 2 is
less then 3, than 1/3 is less than 1/2.
Solution
Let p { 2 is less than 3,q { 1/3 is less than 1/2.
Then implication is p Ÿ q :
(i) Converse of p Ÿ q is q Ÿ p. In words q Ÿ p means “If 1/3 is less than 1/2, then 2
is less than 3”.
(ii) Inverse of p Ÿ q is p Ÿ q. Thus in words, p Ÿ q means “ If 2 is not less than 3,
then 1/3 is not less than 1/2”.
(iii) Contrapositive of p Ÿ q is q Ÿ p. Thus in words q Ÿ p means “If 1/3 is not less
than 1/2, then 2 is not less than 3”.
(iv) Negation of p Ÿ q is (p Ÿ q). Thus in words (p Ÿ q means “It is false than p implies
q”
7 The statement p o (q o p) is equivalent to
(a) p o (p o q) (b) p o (p › q) (c) p o (p š q) (d) p o (p š q)
Solution
p o (q o p) = ~ p › (q o p)
= ~ p › (~q › p) since p › ~ p is always true
= ~ p › p › q = p o (p › q)
Ans : (b)
8 Statement - 1:
~ (p l ~ q) is equivalent to p l q.
Statement - 2:
(~ p l ~q) is a tautology..
a. Statement - 1 is True, Statement - 2 is True ; Statement - 2 is a correct explanation for
Statement - 1
b. Statement - 1 is True, Statement - 2 is True ; Statement - 2 is NOT a correct explanation
for Statement - 1
c. Statement - 1 is True, Statement - 2 is False
d. Statement - 1 is False, Statement - 2 is True

406
Solution :
p q p lq ~p pl ~ q ~(p l ~ q) ~q ~p l ~ q
T T T F F T F T
T F F F T F T F
F T F T T F F F
F F T T F T T T
Ans (c)
Exercise
1 Are the following statements equivalent:
“If the traders do not reduce the price then the government will take action against them”. “It
is not true that the traders do not reduce the prices and government does not take action
against them”
2 Which of the following is false?
(a) (p Ÿ q) œ (~q Ÿ ~ p) is a contradiction
(b) (p › ~ p) is a tautology
(c) ~(~ p) œ p is a tautology
(d) (p š ~ p) is a contradiction
3 If each of the statement p Ÿ ~ q ; q Ÿ r ; ~ r is true, then
(a) p is false (b) p is true (c) q is true (d) None of these
4 Which of the following is true?
(a) ~(p › (~ q)) { (~ p) š q (b) (p š q) š (~q) is a tautology
(c) ~(p š (~ p)) is a contradiction (d) None of these
5 Which of the following is the contrapositive of ‘If two triangles are identical, then these are similar’?
(a) If two triangles are not similar, then these are not identical
(b) If two triangles are not identical, then these are not similar
(c) If two triangles are not identical, then these are similar
(d) If two triangles are not similar, then these are identical
6 The contrapositive of the converse of p Ÿ ~ q is
(a) ~ q Ÿ p (b) p Ÿ q (c) ~ q Ÿ ~ p (d) ~ p Ÿ ~ q
7 ~(p › q) › (~p š q) is equivalent to
(a) q (b) p (c) ~ p (d) ~ q
8 Negation of the compound proposition.
If the examination is difficult, then I shall pass if I study hard.
(a) The examination is difficult and I study hard but I shall not pass
(b) The examination is difficult and I study hard and I shall pass
(c) The examination is not difficult and I study hard and I shall pass
(d) None of these
9 If p is true, q is false and r is false, then which of the following is true?
(a) (p › q) Ÿ r (b) p š ~ (q › r) (c) (p › q) š r (d) p Ÿ ~(q Ÿ r)
10 If p { “she goes to market” and q { “She buys some fruits”.
Then choose the correct symbol for the given statements :
(i) Either she goes to market or she buys some fruits:
(a) p › q (b) p š q (c) ~ p › q (d) p › ~ q

407
(ii) If she goes to market, then she buys some fruits:
(a) ~ pšq (b)p Ÿ q (c) p š q (d) ~ p› q
(iii) Neither she go to market nor she buy some fruits:
(a) ~ p› ~ q (b) p Ÿ q (c) p › q (d) ~ pš~ q
(iv) She does not go to market and she buys some fruits:
(a) ~ pšq (b) ~ (p › q) (c) p š ~ q (d) pŸq
(v) She does not go to market unless she buys some fruits :
(a) pŸq (b) q Ÿ p (c) ~ q Ÿ ~ p (d) p› q
Answers
1. Yes 2. a 3. a 4. a 5. a
6. a 7. c 8. a 9. b 10. (i)
a (ii) b (iii) d (iv) a (v) c

408
STATISTICS
Measures of Central Tendency (Averages)
1 n
i. Arithmetic Unclassified data x ¦ xi
n i1
Mean or ungrouped frequency
n
1
Mean x distribution x
Ni1
¦ x if i ; N ¦ fi
i 1

§1 n ·
Step deviation method x A  h ¨ ¦ f i u i ¸
©N i 1 ¹
where A is assumed mean, h class interval
xi – A
and ui =
h
n1x1  n 2 x 2  ......n k x k
Combined mean x
Mathametical Averages

n1  n 2  ......  n k
where x1 , x 2 .....x k are means of k
groups having n1,n2......nk members.
n

¦w x
i 1
i i
Weighted Arithmetic (i) xw n if wi be the weight of
¦w
i 1
i

Mean variable xi
n

¦w d i i
(ii) Short cut method x w Aw  i 1
n

¦w
i 1
i

Aw = assumed mean, di = deviations


wi = weights
ii. Geometric Unclassified data G = x 1x 2 ....x n 1 / n or
1§ n ·
Mean G = antilog ¨ ¦ log x i ¸
n©i 1 ¹

x
n
Frequency distribution G f1
1
f2
x 2 ......x n f n 1/ N
where N ¦f i or
i 1

§1 n ·
G = antilog ¨ ¦ f i log x i ¸
©N i 1 ¹
iii. Harmonic Unclassified data n
H n
1
¦
i 1 xi
n
N
Mean Freqency distribution H n
fi
; N ¦f i

¦
i 1 xi
i 1

409
th
§ n  1·
Median Unclassified data a. n is odd ¨ ¸ value.
© 2 ¹
b. n is even
th th
§ n· §n ·
A.M. of ¨ ¸ and ¨  1¸ value
© 2¹ ©2 ¹
Ungrouped frequency The value of x for which the cumulative
N
distribution frequency is just greater than .
2
N
– cf
Grouped frequency M A 2 uh
f
distribution A = lower limit of median class
f = frequency of median class
h = width of median class
Position Average

cf = cumulative frequency of the class just


before the median class.
Mode Unclassified data Value which appears most frequently in the
distribution.
Ungrouped frequency Value of x which has greatest frequency.
distribution
f 0 – f –1
Grouped frequency M0 A uh
2 f 0 – f –1 – f 1
distribution A = lower limit of modal class
f0 = frequency of modal class
f–1 = frequency of pre-modal class
f1 = frequency of post modal class
h = length of modal class
If 2f0 – f–1 – f1 = 0,
f 0 – f –1
Then M 0 A uh
f 0 – f –1  f 0 – f1
Note: Like median, the other partition values quartiles, deciles, percentiles etc can be deter-
mined.
iN
– cf
i quartile Qi is given by Qi = A 
th 4 u h ; i = 1, 2, 3
f
Where symbols have same meanings as in median. Clearly, the second quartile Q 2 is median.
First quartile is called lower quartile and the third quartile is upper quartile.

410
Measures of Dispersion
The degree to which numerical data tend to spread about an average value is called variation or
dispersion of the data. It measures the scatterendness of various observation about some central
value.
Range Difference of the largest and smallest
values
Q 3 – Q1
Quartile Deviation
2
Q 3 – Q1
Coefficient of quartile Q 3  Q1 where Q1, Q3 are respectively
deviation the first & third quartiles.
1 n
Mean Devialion į unclassified data ¦ x – a where a = A.M,
n i1
Median or Mode as the case may be
ungrouped frequency M.D =
1 n
¦ fi x i – a where a = A.M,
n i1
distributon Median or Mode as the case may be &
(M.D is least when
n

measured from the N ¦f i


i 1
median)
1 n
Grouped frequency M.D = ¦ fi x i – a where a = A.M,
n i1
distribution Median or Mode as the case may be &
n
N ¦f i
i 1
2
§n ·
1 n 2 ¨¦
¨ xi ¸
1 n
Standard Unclassified data ı ¦ xi – x 2 ¦xi –¨ i 1n ¸¸
ni1 ni1
¨ ¸
© ¹
1 n
¦ f i x i – x
2
deviation ( ı ) Ungrouped frequency ı
Ni1
2
§ n ·
n ¨ ¦ fi x i ¸
1
distribution ı ¦ f i x i2 – ¨ i 1 ¸ where
N i1 ¨ N ¸
¨ ¸
© ¹

n
N ¦f i
i 1

411
2
§ n ·
n ¨ ¦ fiu i ¸
1
ı h ¦ f i u i2 – ¨ i 1 ¸
Grouped frequency N i1 ¨ N ¸
¨ ¸
© ¹
n
xi – A
distribution where, N ¦f i & ui
h
i 1

A o assumed mean
h o class interval
1 n
Root mean square Unclassified data s ¦ x i – a 2 ; a is asumed mean
n i1
n

deviation (s) Ungrouped frequency s


1 n
¦ f i x i – a 2 ; N ¦f i
Ni1 i 1

distribution
n

Grouped frequency s
1 n
¦ f i x i – a ; N
2
¦f i
Ni1 i 1

distribution

Note that s2= ı 2 + d2 where d = x – a


Clearly s is least when d = 0 i.e. x – a
i.e. root mean square devian is least when deviations are taken from x .
Note: median can be determined from graph also. It is the abscissa of the point of
intersection of “less than” ogive and “more than” ogive.
Note: i) The algebraic sum of the deviations of all the values of the variable from their mean
is zero.
i.e. ¦ x i – x 0 , for ungrouped distribution and ¦ f i x i – x 0 , for grouped
distribution.
ii. The sum of the squares of the deviations of the variable is minimum when taken about
arithmetic mean.
iii. Let x & y be two variables, b,c two constants and u = bx+cy. Then u bx  cy where
x & y are A.M.S of x i’s &yi’s, when b = c = 1, u = x+y & u x  y
i.e. The mean of sum of two variables is equal to sum of their means (it is true for more
than two variables also)
Note:
i. Standard deviation is independent of shift of origin but depends upon change of scale.
x ıx
i.e. if y , then ı y h
h
ii. Square of S.D., ie. ı 2 is called the variance.

412
ı
iii. Coefficient of variation c.v. = × 100.
x
The distribution for which the coefficent of variance is less is more consistent.
iv. S.D. ı of the combined group of two groups having means x1 , x 2 ; standard
deviations ı1 , ı 2 and number of elements n1, n2 is given by

ı2
1
n1  n 2

n1 ı12  d12  n 2 ı 22  d 22 where d = x – x and d = x – x
1 1 2 2

n1x1  n 2 x 2
x
n1  n 2 (combined mean)
Also ı 2 d Range 2

Symmetric and skew Distribution


In a symmetrical distribution, mean, median and mode coincide. Here frequencies are
symmetrically distributed on both sides of the central value. (ie. same number of frequencies
are distributed at the same linear distance on either side of mode).
The frequency curve is bell-shaped and mean = median = mode

Normal
Distributioon

In a skew distribution, the variation doesnot have symmetry.

Positive Negative
Skewness Skewness

Note: In a moderately skewed distribution, Mean – Mode = 3(Mean – Median)

413
Solved Examples

1. In any disrete series when all values are not same, the relation between M.D about mean and
S.D is

a. M.D = S.D b. M.D t S.D c. M.D < S.D d. M.D ! S.D


Solution:

For a distribution (x i,fi) i = 1,2,.....n

1
ı 2x
N
¦ f i x i – x 2 & M.D 1
N
¦ fi x i – x
2
1 §1 ·
ı – M.D ¦ fidi 2 – ¨ ¦ f i d i ¸ where di = x i – x
2 2
? x
©N ¹
N

= ı d2 ! 0

Ÿ ı 2x ! M.D.
2

Ÿ S.D. ! M.D.

Ans: c

2. The A.M. of n observation is x . If the sum of n–5 observations is a, then the mean of remaining
5 observation is

nx  a nx – a
a. b. c. nx  a d. none of these
5 5

Sloution:

If m is the mean of 5 observations, then

a
n – 5  5m § n1x1  n 2 x 2 ·
x n –5 ¨' x ¸
n – 5 5 © n1  n 2 ¹

Ÿ nx = a+5m

nx – a
Ÿ m
5

Ans: b

414
3. If x1 and x 2 are means of two distributions such that x1 < x 2 and x is the combined mean, then

x1  x 2
a. x  x1 b. x ! x2 c. x d. x1  x  x 2
2

Solution:

If n1 & n2 are the number of items in two distributions having means x1 & x 2 .

n1x1  n 2 x 2
x
n1  n 2

n1x1  n 2 x 2 n x – x1
? x – x1
n1  n 2
– x1 = 2 2
n1  n 2
!0 ' x 2 ! x1

Ÿ x ! x1

n1 x1 – x 2
Similary, x – x 2 0
n1  n 2

Ÿ x  x2

? x1  x  x 2

Ans: d

50 50
C 0 50 C 2 50 C 4 C50
4. The mean of , , ...... is
1 3 5 51

2 50 2 49 2 49
a. b. c. d. none of these
51 51 39 x17

Solution:

(1+x)50 + (1–x)50 = 2 ^50


C 0  50 C 2 x 2 ......  50 C 50 x 50 `
Integrating with limits 0 to 1

x 51 ½ 1  x 1  x 51 º
1 51 1
­ x3
2® 50 C 0 x  50 C 2  ....  50 C50 ¾ = – »
¯ 3 51 ¿0 51 51 ¼ 0

415
50 50
C2 C 50 1 251 2 50
50
C0   ........  .
3 51 2 51 51

250 2 49
Mean = =
51x 26 39x17

Ans: c

PRACTICE QUESTIONS
1. Mean of n terms is x . If these x items are successively increased by 2,2 2,23,........2n, then the
new mean is

2 n 1 2 n 1 2 2n
a. x b. x  c. x d. none of these
n n n n

2. The weighted A.M. of first n natural number whose weights are equal is

n 1 2n  1 2n  1 2n  1 n  1
a. b. c. d.
2 2 3 6

3. If G is the G.M. of the product of k sets of observations, with G.M.’s G1,G2......Gk respectively,
then G is equal to

a. logG1+logG2+........+logGk b. logG1+logG2........+logGk

c. G1G2......Gk d. none of these

4. The mean square deviation of n observations x 1,x2,......x n about –2 and 2 are 18 and 10
respectively. Then, S.D of the given set is

a. 1 b. 2 c. 3 d. 4

5. A car owner buys petrol at `7.50, `8.00 and `8.50 per litre for the 3 successive years. If he
spends `4,000 each year, then the average cost per litre of petrol is

a. `8 b. `8.25 c. `7.98 d. none of these

6. The mean of the values 0,1,2,.....,n with the corresponding weights nC0,nC1,.....nCn repectively
is

2n 2n 1 n 1 n
a. b. c. d.
n 1 n n  1 2 2

416
7. The quartile deviation of daily wages (in Rs.) of 7 persons is given below:

12,7,15,10,17,17,25 is

a. 14.5 b. 3.5 c. 9 d. 4.5

8. If a variable x takes values x i such that a d x i d b , for i = 1,2,....,n, Then

a. a d var x d b b. a 2 d var x d b 2

a2
c. d var x d. b – a 2 t var x
4

9. For (2n+1) observations x 1,–x1,x 2,–x2,.....,xn,–xn and 0, where xi’s are different, let S.D. and
M.D. denote standard deviation and mean deviaiton about median, then which is true

a. S.D.< M.D. b. S.D > M.D.

c. S.D. = M.D. d. nothing can be said in general

10. The AM and variance of 10 observations are 10 and 4 respectively. Later it is discovered that
one observation was incorrectly read as 8 instead of 18. Then, the correct value of mean and
variance are

a. 20,9 b. 20,14 c. 11,9 d. 11,5

11. In a frequency distribution, the mean and median are 21 and 22 respectively, then its mode is
approximately

a. 25.5 b. 24.0 c. 22.0 d. 20.5

12. For a symmetrical distribution Q1 = 20 and Q3 = 40, the median of the data is

a. 20 b. 30 c. 40 d. 10

13. If the mean deviation of 1,1+d,1+2d,......,1+100d from their mean is 255, then d =

a. 20.0 b. 10.1 c. 20.2 d. 10.0

30 30 30
C 0 30 C 2 C 20 30 C 21 C 30
14. The mean value of , ......, , ,....... is
1 3 21 22 31

2 31 431 213
a. b. c. d. none of these
31 31 31

417
15. Read the paragraph and answer the questions that follow:

If x1, x2 ,x3 are n values of the variable x, then mathematical averages. Arithmetic Mean (A.M.),
Geometric Mean (G.M.) and Harmonic Mean (H.M.) are count by the following formula’s.

x1  x 2  x 3  .....  x n 1­ n ½
A.M. = ®¦ x i ¾x i ! 0
n n ¯i 1 ¿

G.M. = x 1.x 2 ....x n 1/ n , if each xi (i=1,2,.....,n) is positive and

n 1
H.M. =
1 1 1 1 § 1·
n

x1 x 2
 .......
xn ¦¨ ¸
n i 1 © xi ¹

* In case of frequency distribution x i/fi (i = 1,2,.....n) where fi is the frequency of the variable x i,
n

then the claculation of A.M. is counted as A.M.= ¦ f i x i / N where N = f1+f2.....+fn


i 1

If w1,w2.....,wn be the weight assigned to the n values x1,x2,......xn then weighted A.M. is counted
n

¦w x
i 1
i i
by n

¦w
i 1
i

* If G1,G2 are the G.M’s of two series of sizes n 1 & n2 respectively, then the geometric mean
n1 log G1  n 2 log G 2
(G.M.) of the combined series is counted by log(G.M.)= . On the basis of
n1  n 2
above information answer the following questions.

1. If the mean of a set of observations x1,x2......,x2......,xn is x then the mean of observations xi+4i,
i = 1,2,3,......, n is
a. x  2 n  1 b. x  4 n  1 c. x  4n d. xn
2. The A.M. of n numbers of series is x . If the sum of first (n–1) terms is m then n th number is
a. x–m b. nx – m c. x – mn d. nx – nm
3. The mean of a set of n numbers is x . If each number is divided by 4 then new mean is
x
a. x b. x4 c. d. 4x
4

418
4. The weighted A.M. of first n natural numbers whose weights are squares the corresponding
numbers, is equal to
n n  1 3 2n  1 3 3 n n  1
a. b. c. n n  1 d.
2 2 n n  1 2 2 2n  1
5. Consider the series 1,4,16,64,256,......4 n, then which of the following is not true?
4 n 1 – 1
a. A.M.= b. G.M. = 2n
3 n  1

3.4 n n  1
c. H.M. = d. A.M=G.M.=H.M.
4 n 1 – 1
6. Let G be the G.M. of the product of (r+1) sets of observation with, G.M., G 1,G2,......Gr,Gr+1
respectively, then the vaue of G is
r 1 r 1 r 1

a. ¦ log G i b. –G
i 1
i c. – log G
i 1
i d. none of these
i 1

20
C 0 20 C 2 20 C 4 20 C 6 20
C 20
7. The mean value of , , , ,......... , equals
1 3 5 7 21
2 20 219 2 20 219
a. b. c. d.
3x 77 3x 77 21 21
8. Let x be the variate which assumes the values 0,1,2,3,4,......n with frequencies
qn,nC1pqn–1,nC1p2qn–1..........pn such that p+q = 1,then mean value of the frequency distribution
is
a. npq b. np c. npq d. n2p2q2
9. The mean of the divisors of 360 which are odd is
a. 15 b. 11 c. 13 d. 9
10. The ratio of the mean of the cubes of first n natural numbers to the means of the cubes of first
(n+1) natural nubers is given by
a. (n+1) : (n+2) b. (n+1)2 : n+2 c. n(n+1) : (n+2)2 d. none of these

'Note : Questions with * have more than one correct option'

ANSWERS

1. b 2. a 3. c 4. c

5. c 6. d 7. b 8. d

9. b 10. b 11. b 12. b

13. b 14. b

15. (1) a (2) b (3) c (4) d (5) d (6) b (7) a (8) b (9) c (10) c

419

Vous aimerez peut-être aussi